Download as pdf or txt
Download as pdf or txt
You are on page 1of 490

Problem Books in Mathematics

Edited by P. R. Halmos
Problem Books in Mathematics
Series Editor: P.R. Halmos

Polynomials
by Edward J. Barbeau

Problems in Geometry
by Marcel Berger, Pierre Pansu, Jean-Pic Berry, and Xavier Saint-Raymond

Problem Book for First Year Calculus


by George W. Blwnan

Exercises in Probability
by T. Cacoullos

An Introduction to Hilbert Space and Quantum Logic


by David W. Cohen

Unsolved Problems in Geometry


by Hallard T. Croft, Kenneth J. Falconer, and Richard K. Guy

Problems in Analysis
by Bernard R. Gelbaum

Problems in Real and Complex Analysis


by Bernard R. Gelbaum

Theorems and Counterexamples in Mathematics


by Bernard R. Gelbaum and John M.H. Olmsted

Exercises in Integration
by Claude George

Algebraic Logic
by S.G. Gindikin

Unsolved Problems in Number Theory


by Richard K. Guy

An Outline of Set Theory


by James M. Henle

(continued aJter index)


Bernard R. Gelbaum

Problems in Real and


Complex Analysis

With 18 Figures

Springer Science+Business Media, LLC t


Bernard R. Gelbaum
Department of Mathematics
State University of New York (SUNY)
Buffalo, NY 14214-3093
USA
Series Editor
Paul R. Halmos
Department of Mathematics
University of Santa Clara
Santa Clara, CA 95053
USA
Mathematics Subject Classification (1991): 26-01, 28-01, 30-01, 31-01

Library of Congress Cataloging-in-Publication Data


Gelbaum, Bernard R.
Problems in real and complex analysis/Bernard R. Gelbaum.
p. cm. - (Problem books in mathematics)
Includes bibliographical references and index.
ISBN 978-1-4612-6949-6 ISBN 978-1-4612-0925-6 (eBook)
DOI 10.1007/978-1-4612-0925-6
1. Mathematical analysis-Problems, exercises, etc. I. Title.
II. Series.
QA301.G46 1992
515-dc20 91-43908

Printed on acid-free paper.

© 1992 Springer Science+Business Media New York


Originally published by Springer-Verlag Berlin Heidelberg New York in 1992
Softcover reprint of the hardcover 1st edition 1992
Ali rights reserved. This work may not be translated or copied in whole or in part without the
written permission of the publisher Springer-Science+Business Media, LLC, except for brief
excerpts in connection with reviews or scholarly analysis. Use in connection with any form of
information storage and retrieval, electronic adaptation, computer software, or by similar or
dissimilar methodology now known or hereafter developed is forbidden.
The use of general descriptive names, trade names, trademarks, etc., in this publication, even if
the former are not especially identified, is not to be taken as a sign that such names, as understood
by the Trade Marks and Merchandise Marks Act, may accordingly be used freely by anyone.

Production managed by Francine Sikorski; manufacturing supervised by Vincent Scelta.


Photocomposed copy prepared using LaThX.

987654321
Preface

In the pages that follow there are:


A. A revised and enlarged version of Problems in analysis (PIA) . (All
typographical, stylistic, and mathematical errors in PIA and known to
the writer have been corrected.)
B. A new section COMPLEX ANALYSIS containing problems distributed
among many of the principal topics in the theory of functions of a
complex variable.
C. A total of 878 problems and their solutions.
D. An enlarged Index/Glossary and an enlarged Symbol List.
Notational and terminological conventions are to be found for the
most part under Conventions at the beginnings of the chapters. Spe-
cial items not included in Conventions are completely explained in the
Index/Glossary.
The audience to which the current book is addressed differs little from
the audience for PIA. The background of the reader is assumed to include a
knowledge of the basic principles and theorems in real and complex analysis
as those subjects are currently viewed. The aim of the problems is to
sharpen and deepen the understanding of the mechanisms that underlie
modern analysis.
I thank Springer-Verlag for its interest in and support of this project.

State University of New York at Buffalo B. R. G.

v
Contents

The symbol alb under Pages below indicates that the Problems for the
section begin on page a and the corresponding Solutions begin on page b.
Thus 3/139 on the line for Set Algebra indicates that the Problems in Set
Algebra begin on page 3 and the corresponding Solutions begin on page
139.

Preface v

REAL ANALYSIS

Problems Pages

1 Set Algebra and Function Lattices

1.1 Set Algebra 1.1 - 1.19 3/139


1.2 Function Lattices 1.20 - 1.36 7/146

2 Topology, Limits, and Continuity

2.1 Topology 2.1 - 2.52 12/153


2.2 Limits 2.53 - 2.72 21/169
2.3 Continuity 2.73 - 2.114 24/176

3 Real- and Complex-valued Functions

3.1 Real-valued Functions 3.1 - 3.60 30/192


3.2 Complex-valued functions 3.61 - 3.75 37/219

4 Measure and Topology

4.1 Borel Measures 4.1 - 4.32 40/225


4.2 Haar Measure 4.33 - 4.57 44/241

5 Measure Theory

5.1 Measure and Integration 5.1 - 5.147 48/248


5.2 Probability Theory 5.148 - 5.156 66/296
5.3 Ergodic Theory 5.157 - 5.163 68/300

'T'I'I
viii Contents

Problems Pages

6 Topological Vector Spaces

6.1 The Spaces LP(X, J-L), 1 ~ p ~ 00 6.1 - 6.68 70/305


6.2 Hilbert Space SJ 6.69 - 6.109 79/331
6.3 Abstract Topological 6.110 - 6.162 83/347
Vector Spaces
6.4 Banach Algebras 6.163 - 6.172 89/365

COMPLEX ANALYSIS

7 Elementary Theory

7.1 Geometry in C 7.1 - 7.15 95/371


7.2 Polynomials 7.16 - 7.21 99/376
7.3 Power Series 7.22 - 7.34 99/377

8 Functions Holomorphic in a Disc

8.1 General Results 8.1 - 8.25 102/381


8.2 Applications of Mobius 8.26 - 8.31 105/387
Transformations

9 Functions Holomorphic in a Region

9.1 General Regions 9.1 - 9.4 106/390


9.2 Regions nContaining D(O, 1) 9.5 - 9.10 106/391
9.3 Other Special Regions 9.11 - 9.33 107/392

10 Entire Functions

10.1 Elementary Theory 10.1 - 10.8 110/398


10.2 General Theory 10.9 - 10.29 110/399
10.3 Order of Growth 10.30 - 10.49 112/404

11 Analytic Continuation

11.1 Analytic Continuation of Series 11.1 - 11.7 115/414


11.2 General Theory 11.8 - 11.27117/417
Contents ix

Problems Pages

12 Singularities

12.1 General Theory 12.1 - 12.7 121/424


12.2 Meromorphic Functions 12.8 - 12.13 122/426
12.3 Mittag-Leffler, Runge, 12.14 - 12.22 123/427
and WeierstraB Theorems

13 Harmonic Functions

13.1 Basic Properties 13.1 - 13.12 124/430


13.2 Developments 13.13 - 13.21 125/433

14 Families of Functions

14.1 Sequences of Functions 14.1 - 14.5 127/437


14.2 General Families 14.6 - 14.19 128/438
14.3 Defective Functions 14.20 - 14.28 129/442
14.4 Bergman's Kernel 14.29 - 14.34 130/444
Functions
15 Convexity Theorems

15.1 Thorin's Theorem 15.1 - 15.6 132/447


15.2 Applications 15.7 - 15.10 134/448

Bibliography 451
Symbol List 455
Glossary /Index 463
REAL ANALYSIS:

PROBLEMS
1
Set Algebra and Function Lattices

1.1. Set Algebra


Conventions

When n is a cardinal number, the phrase "n objects" signifies n pairwise


different objects.
The following are the notations for the standard sets of numbers: N (the
set of natural numbers), Z (the set of integers), Q (the set of real rational
numbers), R (the set of real numbers), e (the set of complex numbers),
1r (the set of complex numbers of absolute value 1), and !HI (the set of
quaternions). Specialized notations are:

lllR (the set of real irrational numbers);


II (the set of irrational numbers);
AIR (the set of real algebraic numbers);
A (the set of algebraic numbers).

When S is a set of real numbers, S+ ~ S n [0,00), e.g.,

Z+ = {O, 1, 2, ... } = NLJ{O}.


When -00 ~ a ~ b ~ 00, there are the following one-dimensional in-
tervals:
(a, b) ~ {x a<x<b}, an open interval;
[a,b)~{x a~x<b}, a half-open interval;
(a,b] ~f {x a<x~b}, a half-open interval;
[a,b] ~f {x a~x~b}, a closed interval.
More generally, an interval I in R n is either 0 or the Cartesian product
of n one-dimensional intervals each of which has a nonempty interior. In
R n a half-open n-dimensional interval is a set of the form X~=l [ak' bk ).
If bk - ak is k-free, the half-open n-dimensional interval is a half-open n-
dimensional cube. When n > 1, elements of R n or en are regarded as
vectors and are denoted by boldface letters: x, 0, ....

3
B. R. Gelbaum, Problems in Real and Complex Analysis
© Springer-Verlag New York, Inc. 1992
4 1. Set Algebra and Function Lattices: Problems

When X is a set and E is a subset of 2x, R (E), resp. aR (E), resp.


A (E), resp. aA (E) is the intersection of the (nonempty) set of rings, resp.
a-rings, resp. algebras, resp. a-algebras containing E.
The cardinality of X is # (X), e.g., # (N) ~ ~o, # CJR) ~ c. The
ordinal number of the well-ordered set of equivalence classes of well-ordered
countable sets is n.
For a topological space X (cf. Chapter 2), O(X) resp. F(X) resp.
K(X) is the set of open resp. closed resp. compact subsets of X. [When
misinterpretation is unlikely, the notations 0 resp. F resp. K resp. 5/3, resp.
5>, serve for O(X) resp. F(X) resp. K(X) resp. S/3(X) resp. 5>, (JR n ).]
When E is a subset of a set X, the characteristic function of E is
XE ( x )= o ifxEE
def{l
otherwise'
= { En } nEN of sets there are:
For a sequence £ def

-def-.
£ = hmn-->ooEn =
def nu
00 00

Em;

U n Em·
n=lm=n
00 00

f:. =
(' def .
hmn-->ooEn =
def

n=lm=n

When £ =
-.
e.,
hmn-->oo En = £.
def-

A subset M of 2x is monotone if it is closed with respect to the for-


mation of limits of monotone sequences of sets in M. For a subset E of 2x,
M (E) is the monotone class generated by E, i.e., M (E) is the intersection
of all monotone classes containing E.
For any map f in yX, im(f) ~f f(X).
When X and Yare topological spaces, C (X, Y) is the set of continuous
maps from X to Y: C (X, Y) ~f {f : f E yX, f continuous}. When X
is locally compact Co (X, q is the set of continuous functions vanishing
at infinity; Coo (X, q is the set of continuous functions having compact
support: Coo(X,q ~f {f : f E eX, supp(f) E K(X) }.
The symbol (X, d) denotes a metric space X endowed with the metric
d.
The measure situation (X, 5, p,) consists of a set X, a a-ring 5 in 2x,
and a count ably additive set function p, : 5 :3 E I-> P, (E) E IC. When the
range of p, is a subset of [0,00], p, is a positive measure and (X, 5, p,) is
positive; when the range of p, is a subset of (-00,00] or of [-00,00), p, is a
signed measure and (X, 5, p,) is signed; when the range of p, is a subset of
e, p, is a complex measure and (X, 5, p,) is complex. [The abuse oflanguage
just exemplified is rather general: an adjective applied to p, is applied as
well to (X, 5, p,).] Unless otherwise qualified, p, in (X, 5, p,) is to be taken
as positive.
Problems 1.1-1.8 5

When J.L(E) < 00 for every E in 5, (X, 5, J.L) is finite; when X E 5 and
J.L(X) < 00, (X, 5, J.L) is totally finite.
For (X, 5, J.L), a set that is the countable union of sets of finite measure
is a-finite. When every E in 5 is a-finite, (X, 5, J.L) is a-finite; when X is
a-finite (whence X E 5), (X, 5, J.L) is totally a-finite.
When X is a topological space, '" in (X, 5,B, J.L) is a Borel measure; in
particular, J.L(K) < 00 if K E K(X).
To minimize the use of imperatives, e.g., "show, prove," etc., most
Problems are given as assertions to be demonstrated.

1.1. If M is a monotone subset of 2x and if M is closed with respect to the


formation of finite unions and intersections, then M is closed with respect
to the formation of countable unions and intersections.
1.2. If M is monotone, R is a ring of sets, and M :) R then M :) aR (R).
1.3. For (X, d), if 2x :) M :) O(X) and M is monotone then M contains
F(X).
1.4. If 21R :) M :) 0 (JR) and M is monotone then

M :) aR (0) and aR (0) = aR (F) = aR (K) .

1.5. If 5 is a a-ring then # (5) #- ~o.


1.6. a) If E c 2x and # (E) ~ 2 then # (aR (E)) ~ (# (E)/~o. b) What is
# (5,B (JR n ))? c) For M, the set of all a-finite (JR n , 5,B, J.L) such that for each
x in JR n , J.L({x}) == 0, what is #(M)?

1. 7. If A E aR (E) then for some finite or countable subset Eo in E,


A E aR (Eo).
1.8. If E c 2x and for each sequence {p, q, r, ... } in N, 2x contains a
sequence 8 ~f {Ap, Apq , ApqT) ... } such that:

i. Ap = q Apq , Apq = Ur Apqr , ... ,


n
ii. for each 8 there is in N an m (8) such that each member of 8 with
more than m (8) indices is in E,
then the set A of countable unions of sets Ap is closed with respect to
the formation of countable unions and intersections of its members. If,
furthermore,
iii. {E E E} =} {X \ E E E} (E is closed with respect to complementation),
then A = aR (E).
6 1. Set Algebra and Function Lattices: Problems

1.9. For a sequence e~ {En}nEN: a)

£={X x is in infinitely many En } ,


~={x X is in all but finitely many En } ,

~c£;

b) if En C En+l resp. En ::::> En+!, n E N, then limn-+ co En is UnEN En


resp. nnEN En; c) X£ = limn-+coXEn , X~ = limn-+coXEn •

1.10. If U is a subalgebra of eX, 1 : X 3 x 1--+ 1 is in U, and U is


closed with respect to the formation of pointwise sequential limits then: a)
F~ {E : XE E U} is a a-algebra; b) if (X, d) is a locally compact space
and U ::::> Coo (X, IC) then aR (K (X)) c F.

For a set E of sets and F, the set of all maps

the operation A yields, for each f in F, the set

Mf ~f U nf (vh ,
vENN kEN

and for E, the set A (E) ~f {Mf : f E F}. It is convenient to denote


f (v)k by M n1 ,n2, ... ,nk and thereby to indicate not merely the kth element
of f (v) but also the first k terms of the sequence from which the kth element
is derived. The map f is regular iff for all v and all k, f (vh+! C f (vh.
1.11. By abuse of language, A is idempotent; by abuse of notation,
A2 = A, i.e., A (A (E)) = A (E).
1.12. A(E) is closed with respect to the formation of a) countable inter-
sections and b) countable unions of elements of A(E).
1.13. If E is closed with respect to the formation of finite intersections,
then for every f in F and every M in E there is a regular 9 such that
Mg = Mf.
1.14. If f is regular then: a)

(1.1)
Problems 1.15-1.19 7

b) UVENN UkEN M n l,n2, ... ,nk is the countable union of sets in E; c) M denot-
ing M n1 ,n2, ... ,nk when k = 0,

(1.2)

1.15. If E = F ([0, 1]) then Sf3 ([0,1]) C A (E), i.e., every Borel set in [0,1]
is an analytic (Suslin) set.
1.16. If E = F ([0, 1]), hE C ([0, 1] ,lR), and E E E then h (E) E A (F (lR)).
1.17. The set of Lebesgue measurable sets is invariant under A, i.e.,
A (5). ([0, 1])) = 5>. ([0, 1]).
1.18. If a is an infinite cardinal number, there is an E such that for each
Min E, # (A(M)) = a.
1.19. If E E Sf3 ([0, 1]) and h E C ([0, 1] ,lR) then h (E) is Lebesgue
measurable.

1.2. Function Lattices


Conventions

The next discussion provides the basis for the Daniell-Stone extension of a
nonnegative linear functional I defined on a function lattice L. Limiting
operations entail the use of extended lR-valued functions, i.e., functions that
may assume the "values" ±oo. Consequently the extended real number
-def -
system lR = lR U { -00 } U {oo} is introduced. In lR, °.
00 = 0.

For a set X let V be a function lattice in jRx, i.e., a)

{J, 9 E V} =* {J 1\ g, f V 9 E V}
(V is a lattice); b) when f, 9 E V and a E lR then af E V and, when f + 9 is
meaningful, i.e., when for each x, f(x) + g(x) is not ofthe form 00 + (-00)
or -00 + 00, then f + 9 is in V.
In what follows, the formulre f ± 9 signify that f(x) ± g(x) are mean-
ingful for all x.
Thus V fails to be a vector space only insofar as addition of functions
in V is not always defined.
For any triple a, b, c of real numbers, the middle number is

mid (a, b, c) ~f (a 1\ b) V (a 1\ c) V (b 1\ c) .

For functions f, g, h (in lR x), the middle function is


mid (f, g, h) (x) ~f mid (f (x), 9 (x), h (x)).
8 1. Set Algebra and Function Lattices: Problems

1.20. a) mid(a, b, C) = (aVb)l\(aVc)l\(bVc)j b) mid (a,b,c) E {a,b,c}j c)


for some functions f, g, h, mid (f, g, h) tJ. {f, g, h}j d) f V 9 + f 1\ 9 = f + g.
1.21. If V is a function lattice then V is mid-closed, Le.,

{J,g,h E V} =} {mid (f,g,h) E V}.

1.22. If a E JR and f,g,p,fn are in V then:

{a =I- O} =} {mid(af,g,h) = amid (f,~,~)} j (1.3)

p + mid(f, g, h) = mid(p + f,p + g,p + h)j (1.4)


{h ::; fz} =} {mid (h, g, h) ::; mid (fz, g, h)} j (1.5)
mid(h~fz,g,h) =mid(h,g,h) ~mid(fz,g,h)j (1.6)

mid ( lim fn, g, h) = lim mid (fn,g, h). (1.7)


n-+CX) n~oo

If g(x) ::; h(x) then

h 1\ f(x) - 9 1\ f(x)
h(x) - g(x) if mid(f,g, h)(x) = h(x)
{
= ~(x) - g(x) if mid(f,g, h)(x) = f(x) (1.8)
otherwise.
The Problems that follow are posed in the context of a set X, a
function lattice V contained in RX, and a nonnegative linear functional

J: V 3 f I-t J(f) E JR.

Thus: a) {J :::: O} =} {J(f) :::: O}j b) when a, bE JR and f, 9 E V, then

J(af + bg) = aJ(f) + bJ(g)


so long as the left member of the equation is meaningful.
A function f in RX is Daniell measurable, Le., f E M, iff

{V 3 g::; hE V} =} {mid(f,g,h) E V}.

1.23. If

{{fn i J} 1\ {J (fn) ::; M < 00, n E N}} =} {{J E V} 1\ {J(fn) i J(f)}}

then: a) {J E V} =} {If I E V}j b) M is a function lattice; c) M is a


monotone class of functions; d) M is closed with respect to lim, lim, and
Problems 1.24-1.28 9

lim applied to sequences in Mj e) if I E M and III E V then IE Vj f)


{f E M} {:} {{O ~ 9 E V} =} {mid (f, -g, g) E V}}.

A set E is a Daniell measurable subset of X iff XE EM. The set of


Daniell measurable subsets of X is denoted A.
1.24. The set A is a a-ring and if 1 E M then A is a a-algebra.

When I E JRx, 0: E JR, and 0 is one of <,~, >,~, =, #,

Eo(f,o:) ~ {x : x E X,/(x) oo:}.

1.25. a) 1- 1 (S.a(JR)) c A iff one of Ed/,o:), E-:::,(f,o:), E>(f,o:), and


E>(f,o:) is in A for all real O:j b) for some (X,S,JL) there is an I such that
fo~ all real 0:, E=(f,o:) E A and 1- 1 (S.a(JR)) rt A.
1.26. If 1 E M then IE M iff for all real 0:, E< (f,o:) E A, Le., I is
Daniell measurable iff I is A-measurable.

Let L be a function lattice in JRx and let I: L 3 I 1--+ l(f) E JR be a


Daniell functional, i.e., a nonnegative linear functional satisfying

{L 3 In 1 O} =} {f(fn) lO}.

The Problems 1.27 - 1.36 lead to the construction in iRx of a function


lattice V containing L and to which 1 can be extended as a nonnegative
linear functional, denoted J, J : V 1--+ R There emerges a context for 1.20-
1.26, a measure situation (X, S, JL) for which L1(X, JL) = V, and a formula
J(f) = Ix I(x) dJL(x).
For a monotone sequence {an} nEN in JR, limn --+ co an exists as an element
of iR. Let Lu be the set of iR-valued functions that are limits of monotonely
increasing sequences of functions in L.

[ Note 1.1: Thus Y: Lu E I 1--+ lim n --+ co 1 (fn) is independent of


the sequence {fn}nEN' For simplicity, the notation Yis dropped
in favor of I. For I in Lu, l(f) is iR-valued.]

1.28. If I, 9 E Lu and 0:, (3 ~ 0 then

0:1 + (3g E Lu and 1 (0:1 + (3g) = 0:1 (f) + (31 (g) .


10 1. Set Algebra and Function Lattices: Problems

1.29. If Lu :3 In i I then I E Lu and I (In) i 1(1).


Let Lui be the set of all limits of monotonely decreasing sequences
{gn}nEN such that each gn E L u , -00 < limn->oo I (gn), and I (gd < 00.
1.30. If I E Lu then for some p in Lu and some h in L, p 2: 0 and
I =p+h.
-x
For I E IR :

if {g : g E L u , g 2: I} = 0 .
1(1) ~f {:f {I(g) g E Lu ,g 2: I} otherwise '
I(I) ~ -1(-1).

1.31. The functional 1 is subadditive and


a) {I:::; g} => { {1(1) :::; 1(g)} 1\ {I(I) :::; I(gn} ;
b) VI {I(I) :::; 1(1)};
c) {f E Lu} => {1(1) = I(I) = I(I)} ;
d) {{O:::; In} 1\ {I ~f ~ In } } => {1(1) :::; ~ 1 (In) } ;
e) {O:::; In i J} => {1 (In) i J(I)}.

[ Note 1.2: The last inequality is a precursor of the monotone


convergence theorem of Lebesgue.]
Let L1 be {I : I E jRx, -00 < I (I) = 1 (I) < 00 } and for I in L1
let J (I) be the common value of I (I) and 1 (I). In what follows, L1 plays
the role of V in 1.23-1.31. In particular, for L1 there are the sets M, A,
etc.
A measure p, : A :3 E f-+ p,(E) E [0,00] is defined according to:

E = { 00 if XE 1- L1 .
P, ( ) J (XE) otherwise

1.32. a) L C {f : f E L u , I (I) < oo} U Lui eLl; b) IE L1 iff


for every positive E, Lu contains a g and a -h such that h:::; I :::; g and
I (g + (-h)) < E; c) L1 is a function lattice; d) J is a nonnegative linear
functional on L1 and JIL = I; e) if L1 :3 In i f then IE L1 iff for some M,
J (In) :::; M < 00 [and then J (I) = limn->oo J (In)]; f) p, is a measure on A,
i.e., (X, A, p,) is a measure situation; g) IE L1 iff I E M and III E L1; h)
{J E L1} => {J (I) = Ix
I(x) dp,(x)}.
Problems 1.33-1.36 11

1.33. An f is in L1 iff for some 9 in Lui and some nonnegative function


p in L1, J (p) = 0 and f = 9 - p.
1.34. A nonnegative f is in M iff: a) for all 9 in L1, f I\g ELl; b) iff for
all h in L, f 1\ hELl.

Customarily (X, S, JL) is enlarged, via the set

N ~f {N : N E S, JL (N) = O}

of null sets, to include all sets of the form (E \ N 1 ) U N 2, E E S, Nt, N2 E N.


The resulting completion 5 is again a a-ring (a-algebra) according as S is a
a-ring (a-algebra). For E in Sand JL ((E \ N 1 ) u N 2 ) ~f JL(E). The measure
situation (X, S, JL) is complete iff 5 = S.
When X is a locally compact topological space and L is Coo (X, R.) then
for each nonnegative linear functional I : L f-+ R. the Daniell approach can
be applied to L to derive a measure situation (X, A, JL):

For example, if X ~f R.n and I (f) is the Riemann integral of f then


A = SA (R. n ), the a-algebra of Lebesgue measurable sets in R.n and JL is
n-dimensional Lebesgue measure: JL = An.
Let two functions in L1 be regarded as equal iff they differ only on a
null set. Then

is a metric for L1.


1.35. In the context of the Daniell construction, L is dense in L1 metrized
by d as described above.
x def-
Define an outer measure JL* : 2 3 E f-+ JL* (E) = I (XE) E (0,00]. Call
a set E Caratheodory measurable iff for all A in 2x

JL* (A) = JL* (A n E) + JL* (A \ E)

(E JL*-additively splits every set A.)


1.36. If 1 E M then E in 2x is Daniell measurable iff E is Caratheodory
measurable.
2
Topology, Limits, and Continuity

2.1. Topology
Conventions

A topological space is a pair (X,1) (or simply X) consisting of a set X


and a subset T of 2x : T is the topology for X. The elements of T are the
open sets of X, 0, X E T, and Tis closed with respect to the formation of
arbitrary unions and finite intersections.
When T and T are topologies for X and T c T then T is stronger
than Twhile Tis weaker than T. The weakest or trivial topology is {0,X}
and the strongest or discrete topology is 2x.
For a topological space X, a subset B ~ {U>.hEA of Tis a base for T
iff every element (open set) in Tis the union of (some) elements of B.
If B is an arbitrary subset of 2 x then B is contained in the discrete
topology 2x , whence the set of all topologies containing B is nonempty.
The intersection of all topologies containing B is the topology for which B
is a base.
When (Xl> 1i) and (X2 ,72) are topological spaces and I E X;l then
I is:
I> continuous iff 1-1 (72) c 1i;
I> open iff I (1i) c 72·
The set of continuous maps in X;l is 0 (Xl, X 2 ). A"( in 0((0,1], X)
is a curve and "(* ~f "(((0,1]) is the corresponding curve-image. When
"((0)= "((1), the curve is closed. If, to boot, "( is injective on (0,1), the
curve is a simple closed (Jordan) curve.
When A c X and

A ~ {U : U open and U c A}
then AO is UUEA U, the (possibly empty) interior of A. For a nonempty
subset A of X, a neighborhood N(A) is a set such that A c N(At. For
the set F ~f N(A) of neighborhoods of a nonempty set A, the following
obtain:
i. F i- 0, 0 ¢. F;
ii. {F, F' E.F} ::::} {F n F' E .F};
iii. {{F E .F} 1\ {F c G}} ::::} {G E F}.

12
B. R. Gelbaum, Problems in Real and Complex Analysis
© Springer-Verlag New York, Inc. 1992
Conventions 13

A base of neighborhoods at a point x in a topological space X is a


subset B of the set N ({x}) [for simplicity, N( x) 1 and is such that every
N ( {x}) [for simplicity, N (x) 1is a union of elements of B.
For a set X,of' "I E r, XOYErXoy is the Cartesian product of the sets
Xoy, i.e., XOYErXoy ~f {I : I: r 3 "11-+ 1("1) E Xoy}. When, for some X,
Xoy == X, then XoyErXoy = Xr and the choice of notation is a matter of
taste.
A map I: XoyErXoy 1-+ XoyErYoy between Cartesian products is finitely
(countably) determined iff for some finite (countable) subset bd kEK in r,
def{} d ,def{,}
each x = Xoy oyEr an x = Xoy oyEr'

{J(X)OYk = I (X')OYk' kE K} =? {I(x) = I (x')} .


When b. is a subset of r and for each {j in b., Ao is a subset of Xo then
the pair (b., {AO}OE~) determines the cylinder

When each Xoy is a topological space, a basic neighborhood B for the


product topology of XOYErXoy is a cylinder determined by:

a finite subset b. ~f bi}l::;i::;n of r;


points xoy; in Xoy;, 1 ~ i ~ n;
neighborhoods Noy; (xoyJ , 1 ~ i ~ n;

whereupon
B ~f 3 ({NOYi (XoyJ}OY;E~) .
The set of all finite unions of such cylinders is closed with respect to the
formation of finite intersections. It follows that the set ~ of all basic neigh-
borhoods is a base for the product topology of XOYErXoy.
More generally, a base for a topology of XOYErXoy is a set 2: of cylinders
such that the set of all unions of elements of 2: is closed with respect to the
formation of finite intersections.
A partially ordered set (poset) is a pair (r ~f b}, -<) (or simply r) in
which the order -< is transitive and the relation "I -< "I' (also written "I' »- "I)
obtains for a (possibly empty) set of pairs Ct, "I'). The partially ordered set
r is directed iff for each pair Ct, "I') for some g" in r, "I" »- "I and "I" »- "I'.
A directed set r is a diset and a subdiset of r is a subset that is a diset
with respect to the partial order in r.
14 2. Topology, Limits, and Continuity: Problems

A map n : A :3 A l-t n(A) E X of a diset A ~f {A, -<} is a net.


The net n converges to a in X iff for each N(a) and some Ao(N(a)) in A,
{Ao(N(a)) -< A} =? {n(A) E N(a)}.
A point a is a limit point of a set A (a E Ae) iff every N(a) meets
A \ {a}; the closure A of A is AUAe; A is closed iff X\A is open iff A = A;
A is dense in itself or self-dense iff A cAe; the union of all the self-dense
subsets of A is the self-dense kernel of A; A is perfect iff A is closed and
dense in itself.
A point a in A is isolated iff for some N(a), N(a) n A = {a}.
Subsets A and B of X are separated iff (A n B) U (A n B) = 0. The
space X is connected iff X is not the union of a pair of separated and
nonempty sets, i.e., iff X contains no nonempty proper subset that is both
open and closed, i.e., iff X is not the union of two nonempty and disjoint
open subsets.
A point adheres to or is adherent to or is in the adherence of a net n
iff x is in the closure of the range of n: x E n (A).
A net n : A l-t X is eventually in A iff for some Ao,

{A ~ AO} =? {n(A) E A};

n is frequently in A iff for every A and some a A', A' ~ A and n (A') E A.
Two nets n : A :3 A l-t X and n ' : A' :3 A' l-t X are essentially equal
(n ~ n/) iff for some AO resp. A~ in A resp. A',

{{A ~ AO} 1\ {A' ~ A~}} =? {n(A) = n (A')};

~ is an equivalence relation.
When A is a diset and A' c A then A' is cofinal with A iff for each A in
A and some A'(A) in A', A'(A) ~ A. If A' is cofinal with A and n : A -+ X
is a net, the net nlA" the restriction of n to A', is cofinal with n.
In general, for a set X, a filter in 2x is a subset F ~f {F} conforming
to i-iii, page 12, (for the set of neighborhoods of a point). A filter :F'
refines or is a refinement of or is finer than a filter F iff :F' J F, in which
case one writes :F' ~ :F. When:F' is finer than F, F is coarser than :F'.
The set of all filters in 2x is partially ordered by ~ and for a given filter
F there is, by virtue of anyone of the equivalents (Hausdorff maximality
principle, Axiom of Choice, Well-ordering Axiom, etc.) of Zorn's lemma, a
maximal filter U such that U ~:F. A maximal filter U is an ultrafilter.
The filter consisting of the single set X is coarser than any filter. For
x in X, the filter consisting of all the supersets of {x} is an ultrafilter.
When B is a subset of 2x , the set of all supersets of elements of B is a
filter iff the intersection of every pair of elements of B is nonempty, in which
case B is a filter base. The intersection of all filters containing a filter base
is the filter generated by B. Every filter F is a filter base and F generates
Conventions 15

itself. For a map /: X t--t Y and a filter base 8 in 2x , {f(B)}BEB ~f /(8)


is a filter base in 2 y .
When X is a topological space, a filter F converges to x in X iff every
N(x) contains an element of F: F invades every N(x). Because N(x) is
itself a filter, F converges to x iff F refines N (x)).
A point x in X adheres to or is adherent to or is in the adherence of
the base 8 of a filter iff each N(x) meets every element of 8, i.e., iff x is in
the closure of each element of 8.
A filter F ~f {F} is a diset with respect to the partial order induced
by reversed inclusion among the elements of F: F' ~ F iff F' c F. A
map n : F 3 F t--t n(F) E F is thus a net corresponding to the filter F.
Conversely, when A is a diset and n : A 3 A t--t n(A) E X is a net, let B).. be
{ n(J.L) : J.L ~ A}. Then {B)..} )..EA is a filter base that generates the filter Fn
corresponding to the net n. The correspondence net ~ filter is injective.
The correspondence filter ~ net is not a map because there can be more
than one net corresponding to a given filter.
[ Note 2.1: Owing to the correspondences between nets and
filters, any notion formulable in terms of nets is equally formulable
in terms of filters.]
The boundary as of a set S in a topological space X is S n X \ S.
A topological group G is a Hausdorff space and a group for which the
map G 2 3 {x,y} t--t xy-l EGis continuous.
A uniformity for a set X is defined by a nonempty subset U of 2xxx.
The assumptions about U are:
i. {U, V E U} '* {3W {{W E U} A {W c Un V}n;
ii. {U E U} '* { U :) { (x, x) : x EX} ~f 6 };
iii. {UEU},*{U-l~f{(y,x): (X,y)EU}EU};
iv. WoW denoting {(x, z) : 3y {(x, y), (y, z) E W} },
{U E U} '*
3W {{W E U} A {W 0 W c Un.
(Occasionally it is assumed as well that:
v. 6 = nUEU U.)
Note that a uniformity is a filter base in 2xxX and generates a filter
with some added properties, iii, iv, and occasionally v.
A subset 8 of 2xxX is a base for a uniformity iff the set U (8) of all
supersets of elements of 8 is a uniformity.
A uniformity situation (X, U) is a set X and a uniformity U for X.
Two important examples of uniformity situations are: a) a metric
space (X, d) for which U = { {(a, b) : d(a, b) < E} : f > O}; b) a topolog-
ical group G in which U = { { (a, b) : ab- 1 E V} : V E N(e) }.
If (X, d) is a metric space, i-v obtain for the corresponding unifor-
mity U, which has a countable base. Conversely, if (X, U) is a uniformity
16 2. Topology, Limits, and Continuity: Problems

situation, U satisfies i-v, and U has a countable base then for some metric
d: X 2 :3 {x,y} ~ d(x,y) E [0,(0), U is the uniformity corresponding to d
[GiJe, RoDi, WeI].
When (X, d) is a metric space, a E X, and r 2: 0, the closed ball B(a, r)
centered at a and of radius r is {x : x E X, d(x, a) ::; r}; the corresponding
open ball is B(a, r)O ~ {x : x E X, d(x, a) < r} (which is empty if r = 0).
For a uniformity situation (X, U), a net n : A :3 >. ~ n(>.) E X is a
Cauchy net iff for each U in U and some >'(U),

{>', >.' ~ >'(U)} '* {(n(>'), n(>.')) E U}.

The set X in (X, U) is Cauchy complete iff every Cauchy net converges. If
X is not Cauchy complete, an analog of the Cantor completion of a metric
space leads to the Cauchy completion X of X.
For I in lRx , a in X and N(a),

limx=al(x) ~f inf sup I(x), limx=al(x) ~f sup inf I(x).


NEN(a) xEN NEN(a) xEN

According as limx=al(x) = I(a) resp. limx=al(x) = I(a) for all a I is


upper semicontinuous (usc) resp. lower semicontinuous (lsc).
When A, B are subsets of a group G,

AB = {ab : a E A, bE B}, A- 1 = {a- 1 a E A}.

The set

(2.1)

is the Cantor set Co, i.e., the complement in [0,1] of the union of groups
of "middle third" open intervals

(the nth group consists of 2n - 1 intervals).


If 0 ::; a < 1 and the intervals described above are replaced by similarly
situated intervals of lengths contracted by the factor 1- a, the complement
of their union is the Cantor-like set Ca. The Lebesgue measure of C a is a.
If each interval in the nth group is replaced by a similarly situated interval
of length an and I::=12 n - 1a n = 1 - a, the complement of the union of
the intervals is homeomorphic to Ca. It, like each Ca , is a nowhere dense
perfect subset of [0,1] and its Lebesgue measure is a.
Problems 2.1-2.14 17

When K is a compact subset of an open set V in a topological space


X, the notation K -< f -< V signifies that f is a continuous function, for
all x,°:s :s
f(x) 1, f(K) = {I}, and f (X \ V) = {o}.

2.1. a) a E A iff some net n with range in A converges to aj b) a E A- iff


some net n with range in A \ {a} converges to aj c) A is closed iff A = A
iff A- c Aj d) a is an isolated point of A iff a E A \ A-j e) A is self-dense
iff A c A-j f) A is perfect iff A = A- (= A).
2.2. If, for some x, the net n : A 1-+ X, is frequently in each N(x) then A
contains a subdiset r such that some net m : r 1-+ X converges to x.
2.3. A filter :F converges to x iff every net n corresponding to:F converges
to x.
2.4. If the filter :F corresponds to the net n then n converges to x iff :F
converges to x.
2.5. a) f is usc iff - f is lsc iff for all real a, Edf, a) is openj b) f is lsc
iff - f is usc iff for all real a, E> (f, a) is open.

2.6. If f E IR x , a E X, and limx=af(x) ~f ME IR or limx=af(x) then for


some net n converging to a, f 0 n converges to M.
2.7. When the set of all sequences, regarded as nets with the common
range V ~f {O, I} is divided into ~-equivalence classes, the cardinality of
the resulting set S of equivalence classes is c.
2.8. If X is a topological space then x is adherent to a filter :F in 2x iff
some filter F finer than :F converges to x.
2.9. A topological space X is compact iff every ultrafilter in 2x converges.
Similarly, X is compact iff for every net n : A 1-+ X, there is a convergent
cofinal net n IA' .
2.10. True or false: IR is the continuous image of [0, I)?
2.11. For some uncountable subset E in [0,1], (E - E)O = 0. [Note that
°
if )"(E) > then (E - E)O i= 0, cf. 4.34.]
2.12. With respect to the metric d : [0,1)2 3 {x,y} 1-+ Ix - yl, [0,1) is
not complete. Is there a second metric D for [0,1) so that ([0,1), d) and
([0,1), D) are homeomorphic and ([0,1), D) is complete?
2.13. If f : [0, 1] ~ A x B is a homeomorphism then one of A and B
consists of a single point.
2.14. a) For each a in Co the ternary representation specified in (2.1) is
uniquej b) V N and Co are homeomorphicj c) the maps
f n : Co 3 a 1-+ (-1) ~, n EN,
18 2. Topology, Limits, and Continuity: Problems

are continuousj d) CPo : Co 3 a 1----+ 2::=1 ~n 2- n E [0,1]' the Cantor func-


tion, is a continuous surjectionj e) Co + Co = [0,2], Co - Co = [-1, l]j f)
CPo is not injective.

For a topological space X the weight W of X is the least of the cardi-


nalities of the bases of X. A base B of X is minimal iff # (B) = W. Let V
denote the set {O, I} endowed with the discrete topology. For a set M, VM
with the product topology is a dyadic space.
2.15. a) If X is a compact Hausdorff space of weight Wand # (M) = W
then X is the continuous image of a closed subset of V M . b) If B is a closed
subset of the Cantor set Co there is a continuous surjection h : Co ~ B.
c) If W = ~o then X is the continuous image of VN.
2.16. When #(X) > ~o, Xo E X, and

T~f {A : Xo ¢'. A or #(X \ A) E Z+ }

then: a) (X, 1) is a compact Hausdorff spacej b) if x =I- Xo then {x} E T; c)


the weight of X is #(X).

2.17. If 0 ~f {0'Y} 'YEr is a set of pairwise disjoint open sets in the dyadic
space X ~f VM then r is empty, finite, or countable.
2.18. If {X>.hEA is a set of separable spaces then a set 0 of pairwise
disjoint open sets in X ~f X>.EAX>. is empty, finite, or countable.
2.19. For X in 2.16, if f : V M 1----+ X is continuous then f (VM) =I- X.
2.20. If a closed subset F of ~ is the union of half-open intervals:

F (~) 3 F = U(a>., b>.],


>'EA

then it is the union of countably many of those half-open intervals.


2.21. True or false: ~ metrized according to

d(x, y) ~f Iarctan x - arctanyl

is complete?

2.22. If S c (0,00), sup(S) ~ u < 1, and

{{x E S} A {y E S}} '* {xy-l E S}

then u E S.
Problems 2.23-2.34 19

2.23. True or false: lR \ IQ and (lR \ IQ) n (0,1) are homeomorphic?


2.24. If X is a complete metric space without isolated points and U is a
nonempty open subset of X then #(U) 2: c.
2.25. A nonempty, countable, and compact subset K of a complete metric
space X contains an isolated point.
2.26. If {FdkEN is a sequence of closed subsets of lRn and lRn = UkEN Fk
then U kEN FI: is dense in lRn.
2.27. If A is an uncountable subset of lRn then: a) for some a in A and
every N(a), # (N(a) n A) > ~o; b) 8 ~f {b : bE A, # (N(b) n A) > ~o}
is closed; c) #(8) > ~o; d) when n = 1, for some b resp. c in A,

resp.
{f> O}::::} {#(An (C-f,C)) > ~o}.

2.28. (Cantor-Bendixson) If X contains a countable dense subset D then


for some perfect subset P and some countable subset C for which the self-
dense kernel is empty, X = Cl:JP.
2.29. If 8 E A (F ([0,1])) then # (8) ::; ~o or # (8) = c.
When f : X 1-+ Y is a map between metric spaces (X, d) and (Y,8), f
is expansive, an expansion resp. contractive, a contraction iff for all {x, x'}
in X 2 , 8 (f(x) , f (x')) 2: d (x, x') resp. 8 (f(x), f (x')) ::; d (x, x').
2.30. For a compact metric space (X, d), if f in XX is expansive then:
a) f is an isometry, i.e., for all X,Y in X, d(f(x),f(Y)) = d(x,y); b) f is
an auteomorphism of X; c) if f is contractive and f(X) = X then f is an
isometry.
2.31. For some compact metrizable space X there is an auteomorphism
T : X a~o X that is an isometry for no metric d compatible with the
topology of X.
2.32. For a compact metric space (X, d) and an equicontinuous subset E
of C (X, X) metrized according to D(f, g) = sup {d (f(x), g(x)) : x EX},
E is D-compact.
2.33. If (X, d) and (Y,8) are compact metric spaces then f in yX is
continuous iff the graph 9(f) ~ {(x, f(x)) : x EX} of f is closed in
XxY.
2.34. If X is a separable space then # (0) = # (F) ::; c.
20 2. Topology, Limits, and Continuity: Problems

2.35. True or false: for A ~f C ([0, 1], [0, 1]) metrized according to

d(j,g) ~f sup I/(x) - g(x)1 ~f III - glloo,


xE[O,I]

Ai the set of injective elements of A, As the set of surjective elements of A,


and Ab the set of bijective elements of A (Ab ~f Ai n As), a) Ai is closed?
b) As is closed? c) Ab is closed? d) A is connected? e) A is compact?
2.36. For compact Hausdorff spaces X and Y and I
in C(X, Y), if 1- 1 (y)
is connected for every y in Y then B ~ 1-1 (A) is connected for every
connected subset A of Y. If Y is not a Hausdorff space and I is as described,
there can be a connected set A such that 1- 1 (A) is not connected.
2.37. For a set {X,,} "Er of compact Hausdorff spaces and

X =
def X"ErX",
if IE C (X, JR) and E > 0, then for some finitely determined gin C (X, JR),
SUPxEX I/(x) - g(x)1 < Eo Furthermore I is count ably determined.
2.38. If X is compact, I E JR x , and for each t in JR, 1-1 ([t, (0)) is closed
then for some Xo in X, I (xo) = sUPxEX I(x) < 00.
2.39. For K compact and U1 , U2 open subsets of a Hausdorff space, if
K c U1 U U2 then Ui contains a compact subset K i , i = 1,2, such that
K = Kl UK2 .

2.40. If #(r) ~ No then B ~f [0, l]r is separable.

2.41. Endowed with the product topology, X ~f [0,1][0,1] is not metrizable


but as the function space {I : I: [0, 1] ~ [0,1]}, X is metrizable.
2.42. If yn and Yare homeomorphic for all n in N, need yN and Y be
homeomorphic?
2.43. Show [0,1] is not the union of countably many pairwise disjoint and
nonempty closed sets.
2.44. If n ~ 2, F is a closed proper subset of JR n , and the boundary 8F
contains no nonempty perfect subset then F is empty, finite, or countable.
2.45. If n > 1 and'"Y : [0, 1] ~ JRn is rectifiable then '"Y ([0, 1]) is a null set
(An) [whence b ([0, l])t = 0].
2.46. For JR, the set of all half-open intervals (a, b] is a base for a topology
T stronger than the Euclidean topology £ for R Is T a separable topology
for JR?
2.47. True or false: Q is a Go?
Problems 2.48-2.52 21

2.48. a) For (X, U) and U in U, x E Vx(U) ~f {y (x, y) E U} and


V(x) ~f {Vx(U) : U E U} is a filter base. b) If Tis the topology for which
{V(x) : V(x) E V(x), x EX} is a base then Tis a Hausdorff topology for
n
X iff L::. = UEU U. c) If X is compact in the topology T then X is Cauchy
complete.

If X resp. Y is a uniform space with a topology induced by a uniformity


U resp. V a map f : X t--+ Y is uniformly continuous iff for every vicinity V
in V and some vicinity U(V) in U, (f(x), f(y)) E V whenever (x, y) E U(V).
(A uniformly continuous map is continuous.)
[ Note 2.2: Every uniform space X is completely regular. Fur-
thermore, if (X,7) is a compact Hausdorff space then there is a
unique uniformity U that induces T[Bou, WeI].]
For each U of a uniformity U, let Vx(U) be as in 2.48. Then for the
corresponding topology T, Ou ~f {Vx(U)O : x EX} is an open cover of
X.
On the other hand, for a given set X, a given topology T for X gives
rise to a set of open covers of X. The open covers of X are partially ordered
by refinement, i.e., 0' >- 0 iff each element of 0' is contained in an element
ofO.
2.49. If Tis a topology induced by a uniformity U for X then with respect
to >- as defined above, the set {Ou : U E U } of open covers is a diset.
[Note 2.3: The contents of 2.49 may be viewed as a motivation
for the interesting, alternative, and equivalent approach to the
notion of a uniformity discussed in [Tu, WeI].]

2.50. If X is a topological space then every open cover of X contains a


sub cover of cardinality not exceeding #(X).
2.51. If X is a Hausdorff space for which N is a base and for each N in
N, VN is an open set containing aN then D ~f X \ UNEN" VN is empty, a
single point, or totally disconnected.
2.52. If G is an unbounded open set in [0, (0) then

D ~ {x : x E (0,00), nx E G infinitely often}

is Go dense in [0,(0).

2.2. Limits
Conventions

The series 2:::=1 an mayor may not converge; when it does, its sum is
2:::=1 an· The degree of a polynomial pis deg(p).
22 2. Topology, Limits, and Continuity: Problems

For z E C, the function sgn(z) is defined so that

zsgn(z) == Izl, sgn(z)lzl = z, sgn(z)sgn(z) = {~ ~t~tw~se.

2.53. If an :::: an+l :::: 0, n E N, then EnEN an converges iff EnEN 2na2n
converges.

2.54. (Abel) If, for all n in N, < An ~f E~=1 ak and


~oo % . ~oo %
° E%:1 ak = 00
t hen L....k=1 Ak = 00 and If r > 1 then L....k=1 AI.; < 00.

2.55. If bn °
! and E:=1 bn = 00 then for some sequence {an} nEN in JR+,
limn oo :: = 1 and E:=I(-l)nan diverges.
--+

2.56. If an, bn E JR, (an + bn) bn =I- 0, n E N, and both E:=1 :: and

(an)2 an
converge then En=1 (an + bn ) converges.
00 00
En=1 bn

Let E be the dyadic space vN. An element E of E is a sequence {En} nEN


°
such that En = or En = 1, n E N. For E:=1 an and E in E, E:=IEnan is
a subsum of E:=1 an.
2.57.
i. If an > 0, n E N, E:=1 an = 1 then the set 8 of all subsums is [0,1]
iff for some autojection 71' : N 3 n ~ 7l'(n) EN,

{n E N} =} {a7l"(n) ~ f a7l"(m)}'
m=n+l

ia. If the hypothesis E:=1 an = 1 in i is dropped and the hypothesis


°
lim n--+ oo an = is added then 8 = [0, E:=1 an] iff there is an
autojection 71' as described.
ii. In the context of i, if an> E%:nH ak, n E N then 8 is homeomorphic
to the Cantor set Co.
iia. If r E (0,1), a = 1 - r, and an ~f ar n - 1, n E N, then 8 is a
Cantor-like set iff 0< r < ~, whereas 8 = [0,1] iff ~~r < 1.
iii. In the context of iia what is .A (8) ~f a as a function of r?
iv. If {an} nEN C (0,00) and limn--+ oo an = 0 then {an} nEN contains a
EN} l~p<P of subsequences such that
finite or infinite sequence {{ a pq }
q
{an}nEN = Ul~p<P {apQ}qEN and for all p, apq > E%:1 ap,q+k'
Problems 2.58-2.67 23

1
v. In the context of iia and iv, if 0 < r < "2 there is no analogous
decomposition for which apq ~ L:;:'=1 ap,q+k, 1 ~ P < P.

2.58. If {qn}nEN C lR how are the convergence/divergence of L:~=1 _1_


n qn
related to limn--+ooqn, limn--+00 qn , and, if it exists, limn --+ oo qn?
2.59. There are constants G, D such that if n :::: 2 then
00
GInn ~ L (1- (1- 2- k f) ~ Dlnn.
k=1

2.60. For {Pn} nEN a sequence of polynomials for which the degrees are
bounded: deg (Pn) ~ M < 00, n E N, if the sequence converges to a
function f at M + 1 distinct points then f is a polynomial. If M = 00 and
convergence is uniform on [0,1), need f be a polynomial?

2.61. a) XQ(x) = limm--+ oo [limn--+oo {cos (m!x7l")}n) ~f limm--+ oo 8 m ; b)


sgn(x) = lim n--+ oo ~71" arctan (nx); c) limn--+ oo nsin (271"en!) = 271" [whence e i
Q); d) 1- XQ(x) = limm--+ oo sgn (sin 2 (m!7I"x)).
2.62. F(x) = e T
def
Ix00 e-"2 dt 1 0 as x i 00.
.,2 ,2

2.63. If {rn}nEN C lR then limn--+oo 1000 e- X [sin (x + ~)r dx = O.

2.64. lim€--+o 1000 (1 - e(€X)2) e- sin4 x dx = O.


x3

2.65. limn--+ oo I~oo (1 - e-%) e- 1tl sin 3 t dt=?

2.66. If

f-
(xlnx)
if x E (0,1)
{ 1
f(x) = if x = 0
if x = 1

then 1o
1
f(x)dx = 1-
00
L 2( )"
n=2 n n-l
1

2.67. If
00 00
lim tmn
m~~
= 0, n E N, ~ Itmnl ~ M
~
< 00, mEN, lim ~ tmn
m~oo~
= 1,
n=1 n=1

[T ~f (tmn):,n=1 is a Toeplitz matrix], and limm--+ oo L::=1 t;'n = 0 then


eZ = limm--+oo I1nEN (1 + tmnZ), e.g., eZ = limm--+oo (1 + ~) m.
24 2. Topology, Limits, and Continuity: Problems

2.68. For the Toeplitz matrix T ~f (tmn):,n=I' such that tmn 2: 0 and
2::'1 tmn == 1, if Sn i S then limm -+ oo 2::=1 tmnsn ~f am ---- s.
2.69. (Kronecker) If {an}nEN C C and the series 2::-1
- n
an converges then

rImN-+oo 2:;;=1
N
an clef
=
r
N~oo
A
N
0
= .
2.70. If a E R., Uo = a, u n+l = sin Un, n E Z+, then lim n -+ oo Un = O.
2.71. If, for all x in (0,00), JI(x) = x, fn+1(x) = xfn(x), n 2: 2, and
· n -+ oo f n (X ) clef
1Im = a eXIsts
. mte t h en 0 < an1 = x :::; eo.
and'IS fi' 1

2.72. IfaER.and
ifn =0
(~) ~f {~(a-1)"~ia-n+1) ifn E N

then on (-1,1), 2::=0 (~)xn = (1 + x)" and if a > 0 the series converges
absolutely and uniformly on [-1,1].

2.3. Continuity
Conventions

For f : X 1--+ X the compositions f 0 f 0 •.• 0 f, n EN, are well-defined


"-v-'
n times
and are denoted f{ n} .
An ideal I in a ring R is a principal ideal iff for some x, I is the
intersection of all R-ideals containing x.
For (X, S, Jl.) and pin (0,00], LP (X, dJl.) is the set of (equivalence classes
of) measurable functions f for which Ilfll~ ~ Ix If(x)IP dJl. < 00. When f
is an essentially bounded function, Ilflloo is the essential supremum of f:
Ilflloo ~ inf {M : If(x)l:::; M a.e.} (= limp-+ oo IIfllp)·
The symbol for Lebesgue measure in R. n is An or, absent ambiguity,
simply A. For simplicity, to represent an integral over a (measurable) subset
E resp. E of R. resp. R. n , IE f(x)dx resp. IEf(x)dx is used rather than
IE f(x) dA(X) resp. IE f(x) dAn (x).
When E is a topological vector space over a topological field IK (usually
R. or C), [E, F] is the set of morphisms in the category TVS of topological
vector spaces and continuous linear maps among them; [E, IK] is the dual
space and is denoted E*.
In a normed vector space (E, II II) the norm of X is Ilxll. The norm
induces a metric d in E according to the formula: d(x,y) ~f Ilx - yll.
When E, Fare normed vector spaces, and T E [E, F],

IITII ~f sup {IITxll : x E B(O, 1)}


Problems 2.73-2.80 25

is the operator norm of T. A compact T in [E, F] maps each bounded set


B into a precompact set, i.e., T(B) is compact.
An extreme point x in a convex subset K of a vector space E is a
vector not in the convex hull of two other vectors in K, i.e., x is not of the
form ty + (1 - t)z, t E (0,1), y, z E K, x tJ- {y, z}.

2.73. If A is an FO' in JR and f E C (JR, JR) then f (A) is an Fa.


2.74. If f E C (JR, JR) and V is open in JR then f (V) is a Borel set.
2.75. If f E C('lI',JR) for some z in 'lI', f(z) = f(-z), i.e., for some x in
JR, f-l(X) contains two antipodal points.
2.76. If f,g E C ([0, 1],JR) and {f(a) = f(b)} =? {g(a) = g(b)} then there
is a sequence {Pn}nEN of polynomials such that Pn 0 f ~ g on [0,1].
2.77. For
3X+1. 1

j
- - If-1<x<--
2 - - 3
f(x) = °
. 1 1
If - - < x < -
3 - 3 '
3x -1 1
-- if-<x<1
2 3 -

i. if L E (C([0,1],C))*, L(r) = 0, n E N, g E C([-1, 1], C), and


g ([-~,~]) = 0 then L(g) = 0;
ii. if L E C([0,1],C)*, L(J{n}) = 0, n E N, hE C([-1,1],C), h is odd
(h(x) = -h( -x)), and h ([ -~, ~]) = {O} then L(h) = O.

2.78. For fn : [0,1] :3 x f-* enx , n E N, and AN ~ {fn : n ~ N}, the


linear span SN of AN is II 11<Xl-dense in C ([0,1]' C) for all N in No
2.79. Iff EC([0,1],JR) and

then f(x) == O.
2.80. For some {an}nEN contained in C, if f E C([O, 1],C) and f(O) =0
then there is a sequence {mk(J)}kEN for which

<Xl
26 2. Topology, Limits, and Continuity: Problems

2.81. If IE C((O,OO),JR), °
< a < b < 00, and, for all h in (a,b),
I(nh) -+ ° °
as n -+ 00 then I(x) -+ as x -+ 00.

2.82. If 10 E C([O,l),JR), In(x) ~f fox In-1(t)dt, n E N, and for each x in


°
[0,1) and some nx in N, In.,{x) = then: a) [0,1) contains a nonempty open
set on which 10 is 0; b) for every n and every b in (0,1], In has infinitely
many zeros in (0, b).
2.83. If IE COO((O, 1), iC) and for each x in (0,1) there is an nx such that
°
I(n",)(x) = then (0,1) contains a nonempty open interval (a,b) and for
some polynomial p, II(a,b)= p.

2.84. If 9 E C ([0,1]' [0,1)), g(O) = 1 - g(l) = 0, and for some m in N,


g{m}(x) = x, then g(x) x. =
2.85. True or false: a) if for all x in JR, limyjx I (y) = I (x), i.e., if I is
left-continuous, then I is bounded on every finite interval? b) if for all x
in [0,1]' limy=xl(Y) = I(x), i.e., if I is upper semicontinuous, then I is
bounded above?
2.86. If I E C ([0,1], JR) and 1(0) = °then the sequence S ~f {r} nEN is
equicontinuous iff 11/1100 < 1.
. f01 xn I(x) dx
2.87. If I E C ([0,1], iC) then hmn--+oo 1 = 1(1).
fo xndx

2.88. If A ~f {I : I E C3 ([0, 1], JR), 11/1100, Ill"lloo ~ 1} then

M ~f ~~~ (11/'1100 + Ill'lloo) < 00.

2.89. Let P be the statement: A sequence {Pn} nEN of polynomials con-


verges uniformly on X to I. Prove or disprove P if I E Co (X, iC) and a)
X=[O,l],b)X=[O,oo),orc)X=D(O,l)~f{z: ZEC, Izl~l}.
2.90. What is a useful necessary and sufficient condition that a set F
closed in [0,00) be such that every I in C([O,oo),JR) is uniformly approx-
imable on F by polynomial functions of x n , n E N?
2.91. True or false: if I is uniformly continuous on a bounded interval
(a, b) then I is bounded on (a, b)?

For I in C (T, iC) and a in T, the translate I[a] of I is the map

T 3 Z r-+ I(az).
Problems 2.92-2.99 27

The translation-invariant Haar measure r is defined on the a-algebra of


Borel subsets of T: r is normalized so that r(T) = 1, i.e., if 0 ::; tl < t2 < 211"
. def t2 - tl
then r ({ e't: tl::; t < t2 }) =~.

2.92. If I, 9 E C (T, q then

lim [ I(z)g (zn) dr(z) = [ I(z) dr(z) [ g(z) dr(z).


n--+oo In: In: iT
2.93. If I E C (T, q and A ~f {I[a] : a E T} then the II lloo-closure
Conv(A) of the convex hull of A contains precisely one constant function
CU). What is CU)?
2.94. Let X be a complete metric space and in C (X, q let F, a subset
of C (X, q, be such that for each x in X, sUP!EF II(x)1 < 00. There is a
positive M and in X a nonempty open subset V such that for all x in V,
SUP!EF II(x)1 ::; M.
2.95. For X a compact Hausdorff space, if Un} nEN is bounded in C (X, q
and lim n--+ oo In(x) exists for all x in X then

2.96. If X, Yare compact Hausdorff spaces and

A ~f { t ligi : {Ii, gil EC (X, q x C (Y, q, n EN}


then A is II lloo-dense in C (X x Y, q.
2.97. For (X,1), (Y, d), and I E C(X, Y): a) ContU) is a G6; b) if
Un}nEN C C (X, Y), and limn--+ oo In ~f I then

c) if X is a complete metric space then ContU) is *-dense in X.

2.98. What is the set of extreme points of B(O, 1) ~f B1(E) for: a)


E ~f C([O, 1], q? b) E ~ Co(X, 1R) when X is a connected locally compact
Hausdorff space?
2.99. If A is a closed subset of [0, 1J and for every compact metric space
X there is a continuous surjection Ix : A ~ X then: a) the cardinality of
28 2. Topology, Limits, and Continuity: Problems

the set C of components of A is c; b) there is a set P homeomorphic to the


Cantor set Co, a set D such that # (D) S; No, and an open set U such that
A = Pl:JDl:JU; c) for each compact metric space X there is a continuous
surjection gx : A \ AD 1--4 X.
2.100. True or false: if Un}:'o C C ([0, 1], C) and L (fn) ---> L (fo) for all
L in C ([0, 1], C)*, then In (xn) ---> 10 (xo) whenever Xn ---> xo?
2.101. True or false: if L E C ([0,1]' C)* and IILII = 1 then for some I in
C ([0,1]' C), 11/1100
S; 1 and L(f) = I?

2.102. a) A ~ C ([0, 1], C) n {I : I (Q) c Q} is a II lloo-dense Borel


subset ofC([O,I],R). b) What is AD?
2.103. If A is a closed subspace of C ([0,1], C), 9 E RIR, and 9 . A c A
then Mg : A :3 11--4 gl E A is linear and continuous.

2.104. True orfalse: for A ~f {I : I E C ([0, 1], C) , I G) = O}: a) A


is a principal ideal? b) for some go in A, go . C ([0, 1], C) = A?
2.105. If K is a compact subset of [0,1]'

AK ~ {I : IE C ([0, 1], C), f(K) = {O} }, and


BK def U {I: I E C ([0,1], C), f(V) = {O} }
KCVEO([O,lj)

then: a) BK = A K ; b) AK is an ideal but AK is not a principal ideal.

2.106. a) If SeC (X, R) then h ~f V {I : I E S} is lsc while


1\ {f : f E S} is usc. b) When X is locally compact a nonnegative resp.
nonpositive h in R X is lsc resp. usc iff for some subset S of Coo (X, R),

h= V{I : f E S} resp. h = A{I : I E S} .

2.107. For P, Q, R one of the predicates lsc, usc, semicontinuous (sc),


what combinations (if any) of the predicates make the following true:

a) {P(f)I\Q(g)}:::;.{R(f±g)};
b) {P(f) 1\ Q(g)} :::;. {R(f· g)};
c) {P(f)I\Q(g)}:::;.{R(fog)}?

2.108. If X and Y are topological spaces and IE yX then I is continuous


at x iff: a) for every filter F converging to x, f (.F) is the base of a filter
converging to I (x ); b) for every net n converging to x the net I (n) converges
to f(x).
Problems 2.109-2.114 29

2.109. If (X, U) and (Y, W) are uniform spaces and X is compact then
every map f in C (X, Y) is uniformly continuous.

A property P of objects in a category C with a zero object is a Quotient


Lifting (QL) property iff for objects A, B in C and a quotient object AlB,
{P(B) /\P(AIB)} * {P(A)}.
2.110. For each category listed below prove or disprove that the associated
property P is a QL property:
i. Q, the category of groups and coset spaces, P is countabilitYi
ii. Q, P is abelianity;
nt. lXQ, the category of locally compact groups and coset spaces of closed
subgroups, P is compactness;
iv. Mm, the category of metric topological groups and coset spaces of
closed subgroups, P is separability;
v. m, the category of topological groups and coset spaces of closed sub-
groups, P is separability.

[Note 2.4: In iv above, separability is equivalent to the existence


of a countable dense subset. In v separability means simply the
existence of a countable base for the topology.]

2.111. The set A of all nonempty open intervals U containing the real
number a is a diset when it is partially ordered by reversed inclusion, i.e.,
U' >- U iff U' c U. For f in ~IR, U in N(x), and a y in U, let n(U) be
f(y). Let U in ~2 be the uniform structure for which a typical vicinity is,
for some positive E, V(E) ~ {{u,v} : lu-vl < E}. a) L ~f limx--+af(x)
exists iff n is a Cauchy net; b) f is continuous at a iff the net n converges
to f(a).
2.112. Let A be the set of all complements C in N of finite subsets of
N. Partially ordered by reversed inclusion, i.e., C' >- C iff C' c C, A is a
diset. For f in ~N and C in A, let n(C) be f (inf(C)). Show {f(k)}kEN is
a Cauchy sequence iff n is a Cauchy net.
2.113. For a net n : A 3 >. ...... n(>') E ~,

lim(n) ~f inf sup n(>'), and lim(n) ~f sup inf n(>.),


p,EA >-'?-p, p,EA >-'?-p,

show: a) n is a Cauchy net iff lim(n) = lim(n); b) for f and A as in 2.111,


limk--+oof(k) = lim(n), and limk--+oof(k) = lim(n).
2.114. For a complete metric space (X, d) without isolated points, if each
fin C (X,~) is uniformly continuous then X is compact.
3
Real- and Complex-valued Functions

3.1. Real-valued Functions


Conventions

A map ¢ : K f-+ lR of a convex open subset K of a vector space V is convex


iff for all t in [0,1] and any x and y in K,

¢(tx + (1- t)y) :$ t¢(x) + (1- t)¢(y).

When K is an open interval in lR and a E K, a line 1 through (a, ¢(a» is a


supporting line iff llies below the graph of ¢: {(P,q) E l} => {q:$ ¢(P)}.
A topological vector space V is an abelian topological group that is
also a module over a topological field K (usually e
or lR). In particular
the maps V x V 3 {x, y} f-+ X + Y E V, K x V 3 {t, x} f-+ tx E V are
continuous.
For topological vector spaces V, W, [V, W] resp. [V, W] e is the set of
continuous homomorphisms resp. continuous epimorphisms from V to W j
[V] resp. [V]a is the set of continuous endomorphisms resp. continuous au-
tomorphisms of Vj the identity in [V] and [V]a is id.
When x ~ (Xl, .. . , xn) E lRn , IIxll is the Euclidean length of x:

For f in (lRm)Rn and Xo in lRn , when there is in [lRn , lRm] a T such that

lim Ilf (xo + h) - f (xo) - T(h) II = 0


h'l"O
IIhll-O
Ilhll '

T is the differential or derivative df (xo) of fat Xo. By induction there are


defined higher differentials dkf, k = 2,3, .... Note: df (Xo) E [lRn , lRm] and
d 2 f(xo) E [lRn , [lRn,lRm]], etc., while dOf(xo) = f(xo). Correspondingly,
for k in N, C k (lRn , lRm) is the set of maps f : lRn f-+ lRm such that dkf(x)
exists and is continuous for all Xj Coo (lRn , lRm) ~ n~o C k (lRn , lRm). Sim-
ilar definitions apply for C k (lRn , em) and Coo (lRn , em) and even more
generally when lRn is replaced by an open subset U of lRn.

30

B. R. Gelbaum, Problems in Real and Complex Analysis


© Springer-Verlag New York, Inc. 1992
Conventions 31

An f in (Rm )lRn is in Lip( Q) iff for some constant K and all x, y,

IIf(x) - f(y) II :5 Kllx _ yliCl.

An I in RR enjoys the intermediate value property iff whenever y is


between I(a) and I(b) there is, between a and b, an x such that I(x) = y.
For I,g in RIR,

1+ ~f I V0 = I ~ III, 1- ~ I A0 =I -; III

I = 1+ + 1-, III = 1+ - 1-
max{f,g}~ IVg=g+(f-g)+, min{/,g}~ IAg=g+(f-g)-.

The determinant of a square matrix M is det(M).


When G is a locally compact group, a E G, and I E yG then

I[al(x) ~ I(ax), Jla1(x) ~ I(xa).

When (Xi, di ) ,i = 1,2, are metric spaces, € > 0, and I E C (Xl, X2),
the modulus of continuity of I at x is

w(f,Xj€)~fsup{6: {d1(x,y) <6}=?{d2 (f(x)-/(y)) <€}}.

A function I in CR is real analytic iff for each a in R there is a positive


r( a) and in C a sequence {en (a)} such that

L en(a)(x -
00

I(x) = a)n, Ix - al < r(a).


n=O
When S is a subset of a ring, S[x] is the set of polynomials

When S is a ring then S[x] is also a ring.


For a finite subset {an}:=l in Rand
m
def def
So = 0, Sm = ~
L..J an, m = 1, ... , N,
n=l

a chain of indices is a sequence n, n+ 1, ... ,n+k. An index n is distinguished


iff for some n' greater than n, Sn' > Sn-l j a maximal chain of distinguished
indices is a block.
32 3. Real- and Complex-valued Functions: Problems

3.1. (F. Riesz) If {an};;=l CJR,D~f {nk}f=l' nl <n2 < ... <nK,isthe
(nonempty) set of distinguished indices, n is in a block, n# is the greatest
index in the block, and n# > n then Sn# > Sn-l (whence L:nED an > 0).

For f in JR(O,l) and bounded, an x in (0,1) is distinguished if for some


x' in (x, 1), limy=xf(y) < f (x').
3.2. (F. Riesz) The set D of distinguished elements in (0,1) is open and
if 0 =I- D ~f U~=l (an, bn ), 1 ~ n < M ~ 00, and x E (an, bn ) then
f(x) ~ limY=bj(y).

3.3. If {ar}~=l and {bS}~=l' S ~ R, are sequences in C then for some p


in C[x],

deg(p) ~ R +S - 1, p(r-l)(l) = ar , 1 ~ r ~ R, p(s-1)(2) = bs , 1 ~ S ~ S.

3.4. If an > 0, n E N, {bn } nEN C JR, and L:~=l an < 00 then for some
monotonely increasing function f, f (b n + 0) - f (b n - 0) = an, n E N, and
ContU) ~ JR \ {bn}nEN ~ S.

3.5. If -00 < Xl < X2 < ... < Xn < 00, s(O) ~ L:~=l IXi - 01, and
x.!.!±! if n is odd
m = { - 2
m I 'n f 'IS even an d x~ < _ X~+l then minoEIR s(O) = s(m).
- <
_ m

3.6. If f E JRIR and for each x and some positive 8(x),

{x - 8(x) < a < x < b < x + 8(x)} =? {J(a) ~ f(x) ~ f(b)}

then {p < q} =? {J(p) ~ f(q)}: if f is locally monotone then f is monotone.


3.7. If -00 ~ u < v ~ 00, then ¢ in JR(u,v) is convex iff

{u<x~x'<Y'~y<v}=?{(¢(Y)-¢(x)) ~ (¢(Y'~-¢,<x'))}.
y-x Y -x

What is the geometric meaning of the criterion?


3.S. If ¢ in JRIR is convex then in [-oo,ooJ there are p and q such that
p ~ q and such that on:

( - 00, p) ¢ is monotonely decreasing;


{ [p, qJ ¢ is constant;
(q,oo) ¢ is monotonely increasing.

Give examples of convex functions for which

a) - 00 = p = q, b) - 00 = p < q < 00, c) - 00 = p < q = 00,


d) p = q = 00, e) - 00 < p < q = 00, f) - 00 < P < q < 00.
Problems 3.9-3.18 33

3.9. If ¢ in lR(u,v) is convex then ¢ E Lip(l), I has left- and right-hand


derivatives ¢f( x) and ¢~ (x) for all x in (u, v), and I is differentiable almost
everywhere on (u,v).
3.10. '!rue or false: if ¢, 'IjJ E IRIR and both are convex then so is ¢ 0 'IjJ
convex?
3.11. If ¢" exists on (u, v) then: a) ¢ is convex if ¢" > °on (u, v); b) if
°
¢ is convex then ¢" ~ on (u,v).
3.12. a) If ¢ E 1R1R, ¢ > 0, and In(¢) is convex then ¢ is convex; b) the
converse assertion is false.
3.13. '!rue or false:
°
i. if g E IR(O,l), g ~ 0, and g(x) ~ 00 as x ~ then for some convex
function ¢ in IR(O,l), ¢ ~ g and ¢(x) ~ 00 as x ~ o?
ii. if g E IR(O,oo), g ~ 0, and g(x) ~ 00 as x ~ 00 then for some convex
function ¢ in IR(O,oo), ¢ ~ g and ¢(x) ~ 00 as x ~ oo?

3.14. If a, b, c, yare nonnegative functions in C ([0,00), 1R) and for all tin
[0,00), yet) ~ J~ [a(s)y(s) + b(s)] ds + c(t) then

3.15. If q E L1 ([0,00), A) and y" + y = -qy, yeO) = 0, y'(O) = 1 then for


some finite M, IIYlloo ~ M.
3.16.
if
If IE C 2 ([0, 1],1R) and A (1-1(0)) = ° then: a) III" exists a.e.; b)

hex) ~ {1/1"(x) if 1/1"(x) exists


° otherwise
then hE Loo([O, 1], A); c) if °~ g E C 2 ([0, 1], 1R) then

11 g(x)h(x) dx ~ 11 I/(x)Ig"(x) dx.

3.17. True or false: for I in (0,1)(0,1),


i. if I is continuous and {an}nEN is a Cauchy sequence then {f (an)}nEN
is a Cauchy sequence?
ii. if I maps every Cauchy sequence into a Cauchy sequence then I is
continuous?

3.18. If I,g E IR(O,oo), limx->og(x) exists, and for all a,b in (0,00),
I/(b) - l(a)1 ~ Ig(b) - g(a)1 then limx->o I(x) exists.
34 3. Real- and Complex-valued Functions: Problems

3.19. For some monotonely decreasing gin JR.[O,411"j and all real r,

A({X: xE[O,41l"], sinx>r})=A({x: xE[O,41l"], g(x»r}).

3.20. a) For some sequence {/dkEN of monotonely increasing functions


in JR.[O,lj, limk-->oo Ik(X) == 1 and for some x, limk-->oo I~(x) = 00. b) If
{ikhEN C JR.[O,lj and each ik is monotonely increasing then

3.21. Is there in JR.IR an I such that Cont(f) = AIR? = JR. \ AIR?


3.22. True or false: if {fn}nEN C C2(JR.,JR.), II/::ILJO ~ M" < 00, and
In ~ 0 then for some finite M' and all n in N, 1I/~lIoo ~ M'?
3.23. If I E JR.IR and I is continuous a.e. then I is Lebesgue measurable.
3.24. There is in JR.IR no function I for which: a) f' exists on an open set
U containing {O}, b) 1(0) = 0, and c) f'(x) = X(-oo,Oj 0 I(x) on U.

3.25. If I E C (JR., JR.) and limhlo I(x + hl- I(x) is never negative then I
is monotonely increasing.

· C(TllI
3 . 26 • For some I In TllI) -I' I(x + h)h - I(x) 2: 0 a.e. and yet I'IS
~,~, ImhtO

monotonely increasing on no nonempty open interval.


3.27. For I E C ((0,1), JR.), there is a metric d for (0,1) so that I is
d-uniformly continuous while the topology induced by d is the standard
topology of (0,1).
3.28. If IE JR.lR n and for all x in some nonempty open ball B(O,r)O in
JR. n , I(x) ~ I(xl, ... ,xn ) = Lkl+"'+kn20akl, ... ,knX~1 ... x~n then either
1= 0 in B(O, r)O or An (1-1(0) n B(O, r)o) = O.
3.29. a) In C (JR. n , JR.), the set V of all I such that for some Mj, kj, and
all x in JR. n , I/(x)1 ~ Mj (1 + IIxll)kJ is a function lattice. b) If P : V f--+ JR.
is a nonnegative linear functional and J.L is a Borel measure such that for all
I in Co (JR. n , JR.),

P(f) = r
ilR n
I(x) dJ.L(x) , (3.1)

then (3.1) obtains for all I in V.


Problems 3.30--3.40 35

3.30. For E ~f {x x E JRn, Ilxll = I} (the surface of the unit sphere in


JRn), if I,~ E C (E x JR, JR) and 12 + (~) 2 > 0 then for each to in JR,
there is an open set U (to) containing to and such that for all x in E, there
is in U (to) at most one t such that I(x, t) = O.
3.31. If {I, g} c C (JR n , JR), for some m, I(tx) =- t mI(x), g(tx) =- tmg(x),
I ;:::: 0, g(x) > 0 whenever x =1= 0, and I(x) = 0 then for some constants
C, D and all x, C I(x) + Dg(x) ;:::: Ilxllm.
3.32. If f E Cl (Bl (JR n ) , JRn) then for some positive 8, if

sup Ildf(x) - idll <8


x

then f is injective on Bl (JRn ).

3.33. For I in C (JRn , JR) if dl (xo) = 0 and (d 2 I (xo)) -1 exists then for
some N (xo) and all y in N (xo) \ {xo}, d/(y) =1= O.
3.34. If I E C ([0,1], q n BV ([0, 1], q and f > 0, there is a positive a
such that for a partition P, {IFI < a} ~ {var[O,lj,P(f) > Var[O,lj(f) - fl.
Is the conclusion valid if I is not continuous?
3.35. If I E BV ([0,1], q then f' E £1 ([0,1], .\).
3.36. For I in JR[O,lj and a in [0,1], V : [0,1] 3 x f--t var[O,x]! is left-
continuous resp. right-continuous resp. continuous at a iff I is of bounded
variation on [0, a] resp. an interval [0, b) containing [0, a] resp. an interval
[0, b) containing [0, a] and I is left-continuous resp. right-continuous resp.
continuous at a.
3.37. If I is of bounded variation on an interval and enjoys the interme-
diate value property there then I is continuous. [For a nowhere continuous
function enjoying the intermediate property on every interval (cf. [GeO]).]
3.38. If 9 E BV ([-1, 1], q, 9 is continuous at ±a, and for every even
continuous I, J~l I(x)g(x) dx = 0, then g(a) + g( -a) = O.
3.39. Some I is in BV ([0,1]' q n AC ([0, a), q for all a in (0,1) and yet
limx---+l I(x) exists and is not 1(1).
3.40. For what values of a is

if x =0
ifO<x:Sl

of bounded variation? absolutely continuous?


36 3. Real- and Complex-valued Functions: Problems

3.41. Some I in C ([0,1], q \ AC ([0,1], q is in AC ([0, a), q for all a in


(0,1).
3.42. In AC([O, 1], JR) there is a strictly monotonely increasing function I
°
such that f'(x) = on a set of positive (Lebesgue) measure.

3.43. For r in (0, ~) and S the corresponding Cantor-like set dis-


cussed in 2.57ii - iii, ¢s is the analog of the Cantor function, what is:
a) f[o,l] ¢s(x) dx; b) the length of the graph !}s of y = ¢s(x)? For 0: in
(0,1), what are the solutions to a) and b) for the Cantor-like set Ca and
its corresponding function ¢a?
3.44. For R symbolizing any of <, >, = and P a polynomial of degree
n, if ER,j(p) ~f {x : p(j)(x)RO} then ER(p) ~f n;=o
ER,j(p) is 0 or an
interval.
3.45. An I in JR[O,l] is Riemann integrable iff >. (Discont(f)) °
= and I is
bounded.
3.46. For some uniformly bounded sequence {In}nEN of functions Rie-
mann integrable on [0,1], limn-->oo In is not Riemann integrable on [0,1].
3.47. There is a Riemann integrable function I and a continuous function
9 such that for no Riemann integrable function h, h ~ log.
3.48. If I' exists everywhere on JR then I is strictly increasing iff f' ~ °
everywhere and D ~ {x : f' (x) = O} is totally disconnected.
3.49. If I E JRIR then the set S of sites of strict local maxima of I is empty,
finite, or countable.
3.50. For I in C (JR, JR), n in N, and

if II~nlloo
constant.
:s: M < 00, n E N, and for all x, limn-->oo ~n(x) = ° then I is

3.51. If {I, g} c C ([0, 1][0, 1]) and log = go I then for some a in [0,1]'
I(a) = g(a). The conclusion is false if {f,g} c C ([0,1][0,1]) is replaced by
{f,g} c C ((0, 00), (0,00)).
3.52. If I E C ([0, 1], JR) and for some c in (0,1),

lim I(c + h) - I(c) ~f L


hEQI\ {O},h-->O h

exists, then f'(c) exists. For some I not in C([O, 1],JR) the conclusion is
false.
Problems 3.53-3.60 37

3.53. If </J, 9 C C (JR, JR) and for all compactly supported h in Coo (JR, JR),
In~.f (x) h( x) dx = - g( x) h' (x) dx then 9 is differentiable and g' = I·
IR
3.54. If a > 0, f E C (JRn , JR n ), and, for all x and y in JRn ,

Ilf(x) - f(y) II ~ allx - yll (3.2)

then f is an auteomorphism.
3.55. For 1 in C (B (0, 1) ,JR) if

I(x) = 2:r f I(y) dy


lIy-xll=r

for all x in B (0, I t and all r in (0,1- IIxll), then 1 is constant iff
11(0)1 = sup II(x)l·
xEB(O,1)

3.56. If 1 E C (JR, JR), 1 is bounded, {t n } nEN C JR, and K is a compact


subset of JR, then {I[tnl} nEN contains a subsequence converging uniformly
onK.
3.57. If 1 E C([O,l],JR) and {XE [0,1]}:::} {11(x)1 ~I; I(t)dt} then
I(x) == 0.
3.58. If f' (exists and) is real analytic then so is I.
3.59. If 1 E Coo (JR, JR) and I(n) (x) ~ 0, n E N, x E JR then 1 is real
analytic.

i
3.60. a) If an 10 then Isin 112:~=m an sin nxl ~ am and if, furthermore,
for some constant K, nan ~ K then for all x in [-71',71'], hence for all x in
[0,271'], IBN(X)I ~f 12::=1ansinnxl ~ K(1+7r). b) If2::::'=1I an- an+11
converges and limn-->oo an = °
then 2::::'=1 an sin n() converges on [0,271']
and uniformly on every closed subinterval of (0,271'). c) If an 1 then °
°
2::::'=1 an sin n() converges uniformly on [0,271'] iff nan -+ as n -+ 00.

3.2. Complex-valued Functions


Conventions

If U is an open subset of JR, Ck (U, C) is the set of functions having con-


tinuous kth derivatives on U; Coo (U, C) ~f nkEN C k (U, C). For 1 in
C k (U, C), 1I111(k) ~f 2:7=0 IIIU) 1100 (~ 00).
38 3. Real- and Complex-valued Functions: Problems

3.61. For I in C ([O, 1], C), n in N, and j in {O, 1, ... , n - I},

x.(t) ~ {:. (~) + (t ~~) t(x. m) :t"Ot,; j: 1


a) For some continuous function x, Xn ~ Xj b)

X{t) = lot lox{s))ds, 0:::; t:::; 1.


3.62. If n in N, and N n in C ([O, 1], C) is the set of I such that for some x
in [0,1] and all h in IR \ {O}, I{x I + h~ - I{x) I : :; n then N n is closed and
nowhere dense in the II lloo-induced topology of C ([O, 1], C).
3.63. If In E C 1 ([0,1], C), 1I/~1100 :::; 1, n E N, and for all g in C ([0,1], C),
limn-too I; °
In (x)g{x) dx = then limn-too Il/nlloo = 0.
3.64. (Wirtinger) If IE C 1 ([0,7I']'C) and I{O) = 1{7I')
K independent of I, Io'll" I/{x)i2 dx:::; K Io'll" 1f'{x)12 dx.
= ° then for some

3.65. The inclusion map T : C 1 ([O, 1], C) "--+ C ([O, 1], C) is compact.
3.66. If X is a subspace of C 1 ([O, 1], C) and also a closed subspace
of C ([O, 1], C) then: a) X is closed in C 1 ([O, 1], C); b) there are positive
constants k,K such that for all I in X, kll/ll(1) :::; 1111100 :::; KII/II(1); c) X
is finite-dimensional.
3.67. If I E C1 ('][', C) and IT
I{z) dr{z) = °then 11/112 :::; 11/'112 and
equality obtains iff for some constants a, b, I{z) = az + bz.
3.68. If
1 1

II II' : C 1 ([O, 1], C) 3 I 1-+ (1oI/{XWdX) 2

1
(10
1

II II" : C 1 ([O, 1], C) 311-+ (1/{x)1 2+ 1/'{xW) dX) 2

are two norms and E1 resp. E2 is the completion of C 1 ([O, 1], C) with respect
to II II' resp. II II" then the derivative operator D has a continuous extension
iJ : E2 1-+ E1 and iJ- 1 {O) = {I : I is constant}.
3.69. If {zn} ;;=1 c '][' then:

a) {tn~O, ;tn=I}=?{I;tnznl:::;I};
b) {I; I I}
tnzn = =? {11 (1 - tn) = o} .
Problems 3.70-3.75 39

3.70. If a < b then {x 1-+ einx } nEZ is a linearly independent set on [a, b).
3.71. (Hadamard) For the matrix A ~f (aij)~j:l with real entries,

Idet(A) I ,; g~ t, a1; .

The following Problems conclude with 3.75, Brouwer's fixed-point


theorem for JR2

3.72. If J E C(D(O, 1), 1l') then for some ¢ in C(D(O, 1), JR), J(z) = ei</>(z).

3.73. If 9 E C(1l', JR) then there two antipodal points p ~f e iO and


ii~ e (6+lI") such that g(p) = 9 (ii).
i

3.74. If hE C(D(O, 1), 1l') then for some e iO , h (e iO ) =I- e iO .

3.75. (Brouwer) If J E C(D(O, 1), D(O, 1)) then for some p in D(O,I),
J(p) = p.
4
Measure and Topology

4.1. Borel Measures


Conventions

For (X, 5,8, fL), a Borel set E is inner resp. outer regular iff

fL(E) = sup {fL(K) : K compact, K c E}


resp.
fL(E) = inf {fL(U) : U open, U:;, E} ;

E is regular iff E is both inner and outer regular; fL is inner (outer) regular
iff every Borel set is inner (outer) regular. (Similar conventions for regular
apply to arbitrary nonnegative set functions.)
For (X, 5, fL) there are defined two set functions:

fL* : 2X 3 E I-t fL*(E) ~ inf {fL(B) : BE 5, E c B}


fL* : 2X 3 E I-t fL*(E) ~f sup {fL(B) : BE 5, BeE},

the outer and inner measures defined by fl. An outer measure is a map
fL* : 2X 3 E I-t fL*(E) E [0,00] that is countably subadditive:

An E in 5 is an atom iff fL(E) > 0 and for all A in 5, fL(AnE) = fL(E) or


fL(AnE) = 0; (X,5,fL) is nonatomic iff there are no atoms in 5. A discrete
measure fL is one for which there is a map f : X 3 x I-t f (x) E [0,(0)
such that for any subset E of X, fL(E) = LXEE f(x); when f(x) == 1, the
discrete measure fL is counting measure and is denoted (.
When fL is complex and E E 5 then the absolute value of fL is

A measurable partition of an E in 5 is a finite or count ably infinite sequence


{Enh:::;n<N:-:;oo contained in 5 and such that E = U;;=lEn . Thus ifLi(E)

40
B. R. Gelbaum, Problems in Real and Complex Analysis
© Springer-Verlag New York, Inc. 1992
Problems 4.1-4.5 41

is the supremum, taken over the set of all measurable partitions of E, of


2::=1 1J.t (En) I·
When J.t is complex, J.t is inner (outer) regular iff 1J.t1 is inner (outer)
regular.
For (X, 5, J.t), M denotes the set of 5-measurable JR-valued functions.
For x in JR, {x} is the fractional part of x: 0 ::::; {x} < 1 and for a
unique n ~f [x] in Z, x = n + {x}: [x] is the greatest integer in x.
A function 't/J in JRIR is concave iff -'t/J is convex.
4.1. a) (Jensen) For a convex function ¢ in JR(u,v), a set X, a vector
space V contained in JRx , and a nonnegative linear functional L : V f-t JR,
if 1 E V, L(I) = 1, and f(X) C (u,v) then ¢(L(f)) ::::; L(¢ 0 I). [In
particular, if (X,5,J.t), J.t(X) = 1, f E L 1 (X,J.t), and f(X) C (u,v) then
¢Uxf(x)dJ.t(x))::::; 1x¢of(x)dJ.t(x).]
b) A function ¢ in JR(u,v) is convex iff for every simple (Lebesgue mea-
surable) function f in JR(O,I), ¢ (1[O,IJ f(x) dX) ::::; 1[o,IJ ¢ 0 f(x) dx (*).

4.2. For a net n : A '3 >. f-t n).. E JRIR , the map 't/J : JR '3 x f-t lim)..EAn)..(x)
is concave while ¢ : JR '3 x f-t lim)..EAn)..(x) is convex.

4.3. For (X,5,J.t) if M '3 fn ~ O,n E Nand limn-+oofn E L 1 (X,J.t), the


inequalities

lim n -+ oo l fn(x) dJ.t(x) ::::; llimn-+<Xlfn(x) dJ.t(x)

limn -+ oo l fn(x) dJ.t(x) ~ llimn-+<Xlfn(x) dJ.t(x)


(the latter is Fatou's lemma) flow from Jensen's inequality.

For a metric space (X,d), p,E positive, and A a subset of X, let ~(A)
be

inf {f:
k=1
(diam (Uk))P : Uk E T,
kEN
UUk ::) A, diam (Uk) < E, kEN}.

The p-dimensional Hausdorff measure of A is PP(A) ~f sup<>o ~(A).


4.4. Show: a) PP(A) = lim<-+o ~(A); b) PP is an outer measure on 2x; c)
if, : [0,1] '3 t f-t ,(t) E X is a simple rectifiable curve then the length £(r)
of, is pI (r ([0, 1])) ~ pI (r*).

4.5. If PP(A) < 00 and q > p then pq(A) = 0.


42 4. Measure and Topology: Problems

4.6. If pEN, and A c IRP , there is an A-free constant Cp, such that, A;
denoting p-dimensional Lebesgue outer measure,

4.7. If A and B are subsets of X and

inf{d(a,b) : a E A, bE B} ~f 8(A,B) > 0

(A and B are a positive distance apart) then PP(A U B) = PP(A) + PP(B).


4.8. For every closed subset F of X and any subset S of X,

PP(S) = PP(S n F) + PP(S \ F),

i.e., every closed set F is Caratheodory measurable with respect to the


outer measure pp.
4.9. There is a constant Kp such that A; = Kppp.
4.10. If (X, d) is a metric space, (X, aR(K(X), Jb) is finite, and each x in
X is the center of a a-compact open ball then (X, aR(K(X), Jb) is regular.
4.11. For (X, aR (O(X)) , Jb), if X is a separable, complete, metric space
and Jb is finite then Jb is regular.
4.12. If Jb is a Borel measure on IR n and Jb (B (0, r)) < 00 for every
positive r then Jb is regular.
4.13. If (X, d) is a compact metric space, (X, Sa, Jb) is finite, for each x
in X, Jb({x}) = 0, and € > 0 then there is a positive 8(€) such that for each
Borel set E, {diam(E) < 8(€)} =} {Jb(E) < €}.
4.14. If Jb is a regular totally finite Borel measure on a compact Hausdorff
space X there is a minimal closed subset F such that Jb(X \ F) = 0, i.e., if
Fl is closed and Jb (X \ F1 ) = 0 then Fl rt F. If fEe (X, C) then f(x) ~ 0
iff f- 1 (0) :J F.
4.15. If X is a compact Hausdorff space, v is a finitely additive nonnega-
tive set function defined on S,e(X), and for each E in S,e(X),

veE) = inf {v(U) : U open, U:J E}


= sup {v(K) : K compact, K C E}
(v is a regular nonnegative set function) then v is count ably additive.
4.16. For Jb a finite Borel measure on a compact Hausdorff space X, there
is a diset r and a net v : r :7 I t--+ v, so that each v, is a discrete measure
Problems 4.17-4.25 43

and for each f in C(X,q, the net IP: r:3 "( 1-+ Ixf(x)dv-y(x) converges
to Ix f(x) df.l(x).
4.17. The difference of two regular complex measures is a regular complex
measure.
4.18. If f.l is a finite nonatomic Borel measure on a compact metric space
(X,d) then for the topology of X, there is a countable base {Un}nEN such
that f.l (aUn) = 0, n E N.
4.19. If {(X, uR (O(X)) ,f.ln)} nEN is a sequence of totally finite measure
situations for a compact metric space (X, d),

lim [ f(x)df.ln(x) == [ f(x)df.l1(X)


Jx
n-->OC! Jx
on C (X, q, U is open, and f.l1 (aU) = 0 then limn-->OC! f.ln(U) = f.l1(U),
4.20. In the context of 4.19, (conversely) if, for every open set U such
that f.l1 (aU) = 0, limn-->OC! f.ln(U) = f.l1 (U), then for every f in C(X, q,
limn-->OC! Ix f(x) df.ln(x) = Ix f(x) df.l1(X).
4.21. In the context of 4.20, the assumption that each f.ln is totally finite
is superfluous.
4.22. If {Ij}jEJ is a set of intervals in IR and if )..(Ij) > 0 for all j then
E ~f UjEJ I j E 5,B (1R).
- - def """""-""---::""::""7
4.23. If x E IIIR then: a) .6.(x) = {{nx} : n EN} = [0,1]; b)

{(a, b) C [0, I]} :=;. CI~~ # ({{x}, ... , ~kX}} n [a,b]) = b - a},

i.e., {{kx}}kEN is equidistributed.


[ Note 4.1: The result b) is a consequence of some classical
work of Hermann Weyl [Wey].]

4.24. If (X, 5, f.l) is nonatomic then f.l (5) is an interval [0, M], 0 ~ M ~ 00.

For the remaining Problems in this Section, X is a locally compact


space, L = Coo (X, C), I is a Daniell functional, and L u , Luz, M, L1, A, etc.
have the meanings given them in Section 1.2.
If K is compact, U is open, and K C U, Urysohn's lemma implies that
for some f in L, K -< f -< u.

4.25. If S eLand {J, 9 E S} :=;. {f 1\ 9 E S}, i.e., if S is I\-closed or


directed downward, and I : L 1-+ IR is a nonnegative linear functional then
{/d f : f E S} = O} :=;. {I'd 1(1) : f E S} = O}.
44 4. Measure and Topology: Problems

4.26. If S C L u , and {f, 9 E S} ::::} {f V 9 E S} i.e., if S is V-closed or


directed upward, then

F ~f V{I : IE S} E L u , and A ~f V{I(f) : IE S} = I(F).

See 1.27, of which 4.26 is, in the current context, a generalization.


4.27. If U E 0, then

p,*(U) = sup {p,*(K) K3 K c U}


= sup {p,*(V) V E 0, U:::> V E K} .

4.28. If E in 2x then p,*(E) = inf {p,*(U) : U E 0, E c U}.


4.29. Show: a) A is a a-algebra and ° U F c A; b) for E in A,

p, (E) = inf {p, (U) : E CUE °};


c) if E E °or E E A and p, (E) < 00 then

p, (E) = sup {p, (K) : K (X) 3 K C E} .

4.30. If, in the context of 4.29, either X is a-compact or (X, A, p,) is


a-finite then for all E in A, p, (E) = sup {p, (K) : K (X) 3 K C E}.
4.31.
i. A subset E of IR is a null set (..\) iff there is a sequence {In}nEN of
intervals such that L~l..\ (In) < 00 and each point of E belongs to
infinitely many In.
ii. For some null set E in [0,1]' if I is Riemann integrable on [0,1] then
Cont(f) n E # 0.
iii. For a given null set E there is a monotonely increasing function I such
that f' exists at no point of E.

4.32. If {In} nEN is a sequence of monotonely increasing functions in ]R[O,l]


and In m~s I then on Cont(f), limn ..... oo In(x) = I(x).

4.2. Haar Measure


Conventions

When G is a locally compact group, e is the identity of G, (G, 5, p,) is the


measure situation such that 5 = aR (K(G)) and p, is left-invariant (Haar)
measure: {{E E S} 1\ {x E Gn::::} {{xE E S} 1\ {p,(xE) = p,(E)}}. The set
Problems 4.33-4.44 45

of Haar measurable sets or simply Haar sets is 5 and 5 is the completion of


S.
Because Haar measure is unique up to a multiplicative constant, when
G is discrete, e.g., when G = Z, since 0 < /-L ({x}) < 00, it is assumed that
/-L ({x}) ~f 1 for (every) x in G; when G is compact, e.g., when G = '][', since
G E 5 and 0 < /-L( G) < 00, it is assumed that /-L( G) ~f 1.

4.33. For (G,S,/-L), if I:=:; p:=:; 00, IE L1 (G,/-L), and 9 E V'(G,/-L) then
1* 9 is well-defined, 1* 9 E V' (G, /-L), and III * gllp :=:; 11/11111gllp·
4.34. If E E 5 and /-L(E) > 0 then EE- 1 contains some N(e).
4.35. For some (G,S,/-L) and a null set E, EE- 1 contains some N(e).
4.36. If G c '][' then #(G) E N or G is dense in ']['.

4.37. If x E JR. then G ~ {exp (27l"inx)} nEZ is a subgroup of'][' and G is


finite resp. dense in '][' if x E Q resp. if x E [JR.

4.38. If a E [JR and ~ def = e''2 1m then G =


def { }
~n nEZ and H =
def { 2 }
~ n nEZ
are dense subgroups of '][' and G : H = 2.
4.39. In the context of 4.38 let R be a complete set of ']['-coset represen-
tatives for the subgroup G and let 8 be H R ~f {hr : h E H, r E R}.
i. For ('][',5,7), 8 ~ 5 and 7*(8) = 7* ('][' \ 8) = O.
, -M 1 -
ii. For w: JR.:3 x ~ e'x E '][',8 = w- (8), and E in 8)" >..*(En 8) = 0,
>"*(E n S) = >"(E).
iii. If P is a subset of JR., p. =f. 0, E E S,e(JR.), and for all pin P, p+E =E
then >"(E) = 0 or >..(E) = 00.

4.40. Every connected subset of JR. is some kind of interval and hence is
Borel measurable. Is every connected subset of JR.2 Lebesgue measurable?

4.41. If H ~f {hn}nEN is a countable, infinite, dense, proper, subgroup of


a locally compact group G and R is a complete set of coset representatives
for H in G then R ~ S.
4.42. If H is a subgroup of a locally compact group G, G is a) compact
or b) connected, and the quotient space G / H is countable and infinite then
H~S.
4.43. If (X, 5, /-L) is totally finite, /-L(X) > 0, X is a group, /-L is translation-
invariant, and X contains subgroup H such that # (G/H) = #(N) then
5 =f. 2X.
4.44. If m, n in Z+ and m + n > 0 then the group G ~f JR.'m X ']['n contains
a subgroup that is not in SA",XTn '
46 4. Measure and Topology: Problems

4.45. If G is a compact group or a locally compact connected group for


which there is a continuous open epimorphism ¢ : G ~ '][' then G contains
a subgroup that is not in S.
4.46. a) If G is a locally compact group then every set E in 5 is the union
of a a-compact set and a null set. b) Some locally compact group G is not
a-compact [whence G rJ. aR (K(G))J.
4.47. If a topological group G is connected then G is generated by each
N(e).
4.48. Let G be a locally compact group containing a closed proper sub-
group H. a) If G is connected and H E 5 then J-L(H) = O. b) If G is
a-compact and connected then J-L(H) = O. c) If H is normal, H E 5, and
J-L( H) > 0 then G / H is discrete in its quotient topology.
4.49. For locally compact groups G and H, Haar measure situations
(G, 5, J-L) and (H, T, II), and a continuous open epimorphism ¢ : G ~ H,
show: a) T c ¢(5); b) if G is a-compact then T = ¢(5); c) T = ¢(5) can
obtain for some G that is not a-compact; d)

{{E E 5} 1\ {J-L(E) = O}} #- {II (¢(E)) = O};


e) if E is a subset of a null set in G, ¢(E) need not be in T.
4.50. For a a-compact locally compact group G, a closed normal subgroup
H, and K ~f G/H, if K contains a subgroup E that is not a Haar set then
G contains a subgroup that is not Haar set.
4.51. For the locally compact group G, (G, 5, J-L), E in 5, and x in G, show
II: 5 :3 E f-t J-L(Ex) is also a Haar measure on G. Hence J-L(Ex) = f).(x)J-L(E).
Show f)., the modular function for G, is in C(G, (0, 00)).

When f).(G) = {l} the group G is unimodular.

4.52. If G is compact then G is unimodular.


4.53. Let H be a closed normal subgroup of the locally compact group G.
View G / H ~f K as the set {xH : x E G} of cosets of H. Let the Haar
measures on G, H, and K be J-L, p, and T and let 8 be the modular function
for H. a) For f in Coo (G, C), F : G :3 x f-t fH f(xy) dp(y) is invariant
on each coset of H and, as a function on K, FE Coo (K, C). b) For some
constant k, and all f in Coo (G, C),
Problems 4.54-4.57 47

c) ~IH = O.
4.54. If G is a locally compact group then U ~f ~ -1 (1) is a closed
normal unimodular subgroup of G and U contains every normal unimodular
subgroup of G.
4.55. If G is a compact infinite group then #( G) ~ c.

A topological semigroup 8 is a Hausdorff space and a semigroup for


which the map 8 2 ::3 {x, y} I--t xy E 8 is continuous. If

{ {xy = xz} V {yx = zx} } ::::} {y = z}


then 8 is a semigroup with a cancellation law.

4.56. If 8 is a semigroup with a cancellation law then 8 is a group


if: a) for some (8,5,1-"), 8 E 5, 1-"(8) = M < 00, and both 5 and I-" are
left- and right-invariant; b) (): 8 2 ::3 {x,y} I--t {x,xy} preserves (product)
measurability [GeKj.
4.57. A compact topological semigroup 8 with a cancellation law is a
compact topological group [GKOj.
5
Measure Theory

5.1. Measure and Integration


Conventions

When (X, 5, JL) is signed JL is the difference of two positive measures JL±:
JL = JL+ - JL- such that for every E in 5, at least one of JL±(E) is in R
There are in 5 two sets p± such that for E in 5, JL±(E) = JL (P± n E) and
IJLI ~f JL+ + JL-.
For a set X and a a-ring 5, Me is the vector space of complex measures
JL defined on 5 and II II : Me :3 JL ~ IJLI(X) is a norm with respect to which
Me is a Banach space over C.
For measure situations (X, 5, JLi), i = 1,2, JLI is absolutely continuous
with respect to JL2 (JLI « JL2) iff {JL2(E) = O} =? {JLI(E) = O}; JLI and JL2
are mutually singular (JLI -.l JL2) iff there are in 5 disjoint sets AI, A2 such
that for all E in 5, JLi(E) = JLi (E n A), i = 1,2. The Lebesgue-Radon-
Nikodym theorem [HeS, Rudj gives general conditions under there is, for
JLI, a Lebesgue decomposition with respect to JL2 into a sum JLla + JLls such
.
t hat JLla « JL2, JLls -.l JL2, JLls -.l JLla, and £or some mtegra e
bl h def dJLla
= -d-
JL2
(the Radon-Nikodym derivative), JLla(E) == IE
h(x) dJL2(X).
When X is a topological space, associated with each measure situation
(X, 5,B, JL) is supp(JL), the support of the measure JL:

supp(JL) ~f X \ U{U : U E O(X), JL(U) = O}.

Thus supp(JL) is the least closed set F such that if U is open and F n U =f. 0
then JL(F n U) > o.
The symmetric difference of two sets is Al:,B ~f (A \ B) U (B \ A). For
(X, 5, JL), if A, B E 5 then A ~ B iff JL(AL.B) = O.
For ((X")'' 5"),, JL")')} ,,),EP a product measure situation

(X,5,JL) ~f X ,,),Er
(X,,),,5")',JL")')

with product measure JL is defined as follows:

48
B. R. Gelbaum, Problems in Real and Complex Analysis
© Springer-Verlag New York, Inc. 1992
Conventions 49

ii. 5 is the u-ring generated by the set of all finite rectangles, i.e., sets of
the form E"(l x ... X E"(n x XTF-"(l, ... ,"(nX"(, E"(i E 5"(i;
iii. when r is an infinite set, it is assumed that p,"( (X"() == 1;
iv.

By abuse of language and notation,

while an element of X is a vector x ~f {x,,(}"(Er. When r' c r, there is the


projection

When x' E X', x" E Pr\p(X) ~f X", and E c X, the x'-section of E is

Furthermore, for some unique x in X,

For this x, and I in jRx, lx' (x") ~f I(x), Ix" (x') ~f I(x). When p,', p,"
denote the product measures for X', X", the Fubini-Tonelli theorems give
conditions for the validity of

r I(x) dp,(x) = lx'r (rlx" lx' (x") dp,II(X")) dp,'(x').


lx
A set F contained in L1 (X, p,) is uniformly integrable iff for each
positive E and some positive 8(E), if p,(E) < 8(E) then for all I in F,
IE I/(x)1 dp,(x) < E.
When t E [0,1] and 1 < kEN, t has one or two k-ary representations:

2: E~) k-
00

t ~ n, E~) E {O, 1, ... ,k - I}, i = 1 or i = 1,2.


n=1

When there are two, the one for which L~=1 E~) = 00 is the k-ary repre-
sentation of t and then E~) is the nth k-ary marker for t. More generally,
50 5. Measure Theory: Problems

the number €~) is an nth k-ary marker for t. When k = 2, 3, €~) is an nth
binary, ternary marker for t; when k = 10, €~) is an nth decimal marker.

5.1. A finite measure, e.g., a complex measure or its absolute value, is


bounded.
5.2. If {an}nEN C eN and, for every permutation 7r of fir, L~=l a7r(n)
converges then: a) the map

(5.1)

is a complex measure and IJLI(E) = LnEE lanl; b) if L~=l an is condition-


ally convergent then (5.1) defines a finitely additive measure on the ring of
finite subsets of fir but this measure is not extensible to a count ably additive
measure on 2N.
5.3. a) If R is a ring of subsets of a set X, and for (X, aR(R), JLi), i = 1,2,

{E E R} "* {O ::; JLl(E) = JL2(E) < oo}


then JLl = JL2 on aR(R).
b) If "< oo"is replaced by"::; 00," the conclusion in a) need not obtain.
5.4. If (X, 5, JL) is a complex measure situation then v : 5 3 E ~ IJL(E)I
can fail to serve as a measure.
5.5. If (X, 5, JL) is complex, E E 5, and

v(E) ~f sup { IJL(A)I : A E 5, AcE}

then v(E) ::; IJLI(E) ::; 4v(E).


5.6. (Borel-Cantelli) For (X, 5, JL), if {En}nEN C 5 and L~=l JL (En) < 00
then JL (limn->ooEn) = 0 (cf. 5.155).
5.7. For (X,S,JL) totally finite, if i E Xx, {E E S} "* {J-l(E) E S},
and {JL(N) = O} "* {JL (J-l(N)) = O} then for some h in L1(X,JL),

{g E LOO(X, JL)} "* {Ix 9 0 i(x) dJL(x) = Ix g(x)h(x) dJL(X)} .

5.8. For (X, 5, JL), if {in} nEN is a sequence of nonnegative measurable


functions, in ~. i, and limn->oo Ix in(x) dJL(x) = 0 then i ~ o.
Problems 5.9-5.16 51

5.9. a) If

{fn}~=o c L1 ([0,1],.x), fn m~as fo, fn ~ 0, n E N,

and lim [ fn(x) dx = [ fo(x) dx


n-->oo J[O,l] J[O,l]

then {E E SA} ::::} {limn-->oo IE fn(x) dx = IE fo(x) dx}. b) For (X, S, JL), if

{fn}~=o C L1 ([0,1],.x), fn ~' fo

fn ~ 0, n E N, lim [ fn(x) dx = [ fo(x) dx


n-->oo J[O,l] J[O,l]

then {E E SA} ::::} {limn-->oo IE fn(x) dx = IE fo(x) dx}.


5.10. For (X,S,JL) totally finite and {fn}nEN in M, fn m~as 0 iff

1, [( Ifn (x) I ) d ( )
n~~Jx 1+lfn(x)1 JLX =0.

5.11. If {cPn}nEN is orthonormal in L2 (X,JL),

9 E L2 (X,JL) , IcPn(x)1 ::; Ig(x)1 a.e., n E N,

and E~=l ancPn(x) converges a.e. then limn-->oo an = O. (This result gener-
alizes the theorem of Cantor-Lebesgue.)
5.12. If {cPn}nEN is orthonormal in L2 (X,JL), and

E ~ {x : :3 lim cPn(X)}
n-->oo

then a) cP(x) ~f limn-->oo XE' cPn(x) ~ 0; b) if X = [0,1]' S = SA, and JL = .x,


then limn-->ooVar[O,l] (cPn) = 00.
5.13. If (X, 5, JL) is totally finite, {fn} nEN C Loo (X, JL), Ifni ::; 1, and
limn-->oo Ix fn(x) dJL(x) = 0 is there a subsequence {fnkhEN such that
limk-->oo fnk (x) ~ O?
5.14. For some (X,S,JL), and some sequence {fn}nEN contained in
L1 (X, JL): a) fn ~' 1; b) Ix (fn(x) - (fn(X))2) dJL(x) -+ 00 as n -+ 00.

5.15. If fELl (X, JL), bn i 00, and En ~f {x : If(x)1 ~ bn }, n E N,


then limn-->oo bnJL (En) = O.
5.16. For (X,S,JL), E,p in (0,00), if f E M then

JL ({ x : If(x)1 ~ E}) ::; Ix If(x;lP dJL(x),


52 5. Measure Theory: Problems

5.17. If (X, 5, Ji.i) are totally finite, i = 1,2,3, then for the Ji.3-Lebesgue
decompositions Ji.3 = Ji.ja + Ji.js, j = 1,2, Ji.1s ..l Ji.2a·
5.18. If (X, 5, Ji.i) are totally finite, i = 1,2, then: a) for some E in
5, if Ji.iE(A) denotes Ji.i (E n A), i = 1,2, for A in 5, then Ji.iE «: Ji.jE and
Ji.iX\E ..l Ji.j X\E, i =f. j; b) if, for all F in 5, Ji.iF «: Ji.jF and Ji.iX\F ..l Ji.j x\F,
i =f. j, then for all F in 5 and E as in a), Ji.1(Ef:::"F) + Ji.2(Ef:::"F) = O.
5.19. If (X, 5, Ji.i), i = 1,2, are complex measure situations then Ji.1 ..l Ji.2
iff for all al,a2 in ee, la111Ji.11 + la211Ji.21 = lalJi.1 +a2Ji.21·
5.20. If (X, 5, Ji.) is totally finite and F~f {f'Y} 'YEr is a set of S-measurable
functions such that {f'Yl' 1'Y2 E 11 => {f'Yl V 1'Y2 E 11 then: a) there is a 5-
measurable 9 such that for all ,,(, I'Y :s; g; b) if h is S-measurable and, for
all ,,(, I'Y :s; h a.e., then 9 :s; h a.e. (g is a minimal measurable cover for :F).
5.21. If (X, 5, Ji.) is totally a-finite and 0 :s; I E L1 (X, Ji.) then

Ix I(x) dJi.(x) = (Ji. x>.) ({ (x, y) : O:S; Y :s; I(x)})

("the integral is the area under the graph").


5.22. If (X, 5, Ji.) is totally finite, Ji.(X) = 1, and for I in L2 (X, Ji.),

then

5.23. If I is as in 5.22, Fn : xn :3 (Xl"'" Xn) f-+

(x n , sn ,Ji.n) ,then V.ar (Fn ) -_ -


Var(f)
-.
n
5.24. (Corollary to 5.23.) If 0 :s; X :s; 1 then

5.25. True or false: for (X, 5, Ji.) if I is measurable and positive a.e. then
Problems 5.26-5.30 53

5.26. If (X,S,J-t) is totally finite, I E C XX [O,l], for each x in X, I is


a continuous function of t, and for each t in [0,1], I is in L1 (X, J-t) then
the dominated convergence theorem obtains: if g E L1 (X, J-t) and for all t,
II(x, t)1 ~ g(x) then Ix
limt_O I(x, t) dJ-t(x) = limHo Ix
I(x, t) dJ-t(x).
5.27. If (X, 5, J-t) is totally finite, I in CXX[O,l], for each x in X, I is
a continuous function of t, and for each t in [0,1], I is in L1 (X,J-t), then
°
Egorov's theorem obtains: if E > for some E in 5, J-t(X \ E) < E and on
E, I(x, t) ~ g(x).
5.28. There is in !R[O,l] a sequence {In} nEN of Lebesgue measurable func-
tions converging everywhere on [0,1] and yet if A(E) = 1, the convergence
is not uniform on E.

The map !R 3 x ~ ei27rx E '][' serves to identify '][' with !RIll. ~f P, which
may be identified with [0,1) according to the following rules:
i. Haar measure T on '][' is identified with Lebesgue measure A on [0,1);
ii. multiplication in '][' is identified with addition (modulo 1) in [0,1);
iii. the!R2 -induced topology of'][' is identified with the topology T for which
a base consists of all open subintervals of (0,1) together with the set
°
of all intervals of the form [0, a), < a < 1. [With respect to T, [0,1)
is homeomorphic to '][' and, in particular, is compact.]

5.29. In the context of P, and S of 4.37 - 4.39, let {gn} nEN be an enumer-
ation of G. For t E (0,1), let kt in N be such that 2-(kt +1) ~ t < 2- kt . If
I :P (0,1) (x, t) {Iif x E ~kt + S and x = 2 kt +1 - 1 then for all
X

x, limt_O
3

I(x, t) = °
° f--+
otherwIse
and for some positive p, A* ({ x : I(x, t) > ! })
> p.
[ Note 5.1: Problems 5.26 - 5.29 illustrate a limited degree
of extensibility of classical results in measure theory.]

5.30. a) If Fe L1 (X,J-t) and for each positive E there is a positive K(E)


such that

{J E F} /\ {k > K(E)} =:;. { (


JE? (lfI,k)
II(x)1 dJ-t(x) < E} (5.2)

then F is uniformly integrable.


b) If (X, 5, J-t) is nonatomic and F is uniformly integrable then for each
positive E there is a K(E) such that (5.2) holds.
c) Absent nonatomicity, F can be uniformly integrable while for some
positive E there is no K(E) for which (5.2) holds.
54 5. Measure Theory: Problems

5.31. a) If lEe (JR, JR) and I is Riemann integrable on every bounded


interval, so is III. b) If, to boot, III is improperly Riemann integrable on
JR then I is improperly Riemann integrable, I is Lebesgue integrable, and
lim it I(x) dx. c) There is a function
rI(x) d>.(x) = s~~ g such that Igl is
iR s
Riemann integrable on [0, 1) although g is neither Riemann nor Lebesgue
integrable on [0,1).
5.32. True or false: for I in C ([0,1), JR), if E > 0, there is a finite set of
(Euclidean) rectangles that cover the graph g ~f {(x, I(x)) : x E [O,l)}
of I and are of total area less than E.

When I E L1 (1l', T), the Fourier coefficients of I are

,
In =
def
ir. . l(x)C n dT(t) , nEZ,
and the Fourier series for I is LnEZ intn. Furthermore,

SN(J,X) ~f
N
L intn
n=-N
~ 1
...
DN(y- 1 x)/(y)dT(Y)

N
O"N(J, x) ~f N ~ 1 ~ Sn(J, x) ~f !... FN(y- 1 x)/(Y) dT(Y),

The function DN resp. FN is Dirichlet's resp. Fejer's kernel.


[ Note 5.2: In many treatments of classical analysis the no-
. 'def1 1271" e- inx , einx
tatlOns In = /iC I(x) /iC dx and LIn /iC' etc. are
y 211" 0 Y 211" nEZ Y 211"
used instead of those introduced in the preceding paragraph. The
transition is accomplished via the identification of 1l' with [0,211"):
0: [0,211") 3 x t--> eix ~f O(x) and dx t--> ~dx ~f dT(O(X)).
In this
211"
identification, the topology of [0,211") is, as for P earlier, not the
topology it inherits from JR.

The classical notation is not readily extended to cover the


general discussion of harmonic analysis on locally compact abelian
groups [HeR, Loo, We2], of which Fourier series, Fourier trans-
forms, etc. are historically important parts.]

5.33. For 1l' viewed as [0,211") and T as ~, what are simple closed formulre
211"
for Dirichlet's kernel DN and Fejer's kernel FN?
Problems 5.34-5.36 55

5.34. If A in C (1I', q is the subset consisting of those functions f for


which the Fourier coefficients f~ satisfy: EnEZ linl (1 + In!) :::; 1 then A is
compact in the II lloo-induced topology of C (1I', C).
5.35. If {an}nEZ C C and for some K in [0,00), all n in N, all t in 1I',
and all sequences {cn}nEZ in C, !E:=_NanCn! :::; K!E:=_NCntn! then
there is a complex Borel measure J1, such that 1J1,1 :::; K and for all n in Z,
an = IT tn ,dJ1,(t).
int
Among the varieties of trigonometric series E:'=-oo Cn ~ are Fourier
series, i.e., for some f in L1 (1I', r),

Cn =
1T
rf(t)C n dr(t) = ~ r27r f(x)
y211" 10
e-;;
y211"
dx, nEZ,

and Fourier-Stieltjes series, i.e., for some (1I', S,B, J1,) resp. ([0,211"), S,B, v),

Cn = i C n dJ1,(t) resp. 127r e- inx dV(x), n E Z.


The characterization in 5.36d) of Fourier-Stieltjes series shows there
are trigonometric series that are neither Fourier series nor Fourier-Stieltjes
series.
5.36. a) The series
(5.3)

is a Fourier-Stieltjes series. b)

{J,g E L1 (1I', r)} :::} {(J * g)";. == in9n}. (5.4)

c) If f E L1 (1r, r) and
n N
Ln(t)~f L Ak tk ,n=O,I, ... ,N, AN(t)~f N~ILLn(t)
k=-n n=l
N N
RN(t) ~ L Anintn, aiv(t) = N ~ 1L Rn(t)
n=-N n=O
then aiv(t) = IT
f (S-lt) AN(S) dr(s). d) 2:::'=-00 cnt n is a Fourier-Stieltjes
series iff for some M in [0,00), and all N in Z,

(5.5)
56 5. Measure Theory: Problems

5.37. For some measure situation ([0,1], 5/3, JL), JL is nonatomic and
supp(JL) = {t : t = I:~=1 EnlO- n , En = or 7}. °
5.38. If FE F (JR) then for some (JR, 5/3, JL), supp(JL) = F.
5.39. For each compact perfect subset K of JRn and some nonatomic Borel
measure JL, supp(JL) = K.
5.40. Is there a ([0,1], 5/3, JL) such that JL (5/3) is the Cantor set Co?
5.41. If JL ([0,1]) = 1, and I in L~ ([0, 1], JL) then

{exp (11 I(x) dJL(X)) = 11 exp (I(x)) dJL(x) }

{:} {f(x) is constant a.e. (JL)}.

5.42. For ([0,1], 5/3, JL), if

{f E C ([0, 1], [0, oo)n =} {/(O) ~ 11 I(x) dJL(X)}

then for some c in [0,1]' {f E C ([0,1], ICn =} {J; I(x) dJL(x) = c/(O)}.

5.43. For ([0,1],5/3,JL) signed, if Jo1 sin k 7rxdJL(x) = 0, kEN, then


{E E 5/3 ([0, ~])} =} {JL(E) = -JL(l - En·
5.44. In 5.43 with sin replaced by cos,

{E E 5/3 ([0,1] \ {~}) } =} {JL(E) = O}.

5.45. If {an} nEN C JR, there is a signed ([0,1]' 5/3, JL) such that

11 cosn7rxdJL(x) == an.

5.46. For ([0,1], 5/3, JL), if F is uniformly integrable then each sequence
{In}nEN in F contains a subsequence {Ink hEN such that for some 9 in
C ([0,1]' q, J~ Ink (x) dJL(x) ~ g(y).
5.47. Let P be the set of all ([0,1]' 5/3, JL) for which JL ([0, 1]) = 1. What
are the extreme points of P regarded as a subset of M, the Banach space
of all signed measures defined on 5/3?
5.48. If {an}~=o' {tn}nEN C JR and
Problems 5.49-5.57 57

there is a unique ([0,1], S{3, JL) such that for all n in N, tn = f; enx dJL(x).
5.49. For X ~f { (x, y) : 0::; x ::; y ::; I} and S{3(X), are there measures
JL1, JL2 such that whenever 0 ::; a ::; b::; 1,

JL1 (([0, a] x [0, b]) n X) = ab + a2b2, JL2 (([0, a] x [0, b]) n X) = ab - a3 b3 ?

5.50. If (~, S{3, JL) is complex, fELl (~, JL), and for all h in Coo (~, C),
- fIR f(x)h(x) dx = fIR h'(x) dJL(x) then JL « dJL
A and dA (x) = jX -00 f(t) dt.

5.51. If E E SA and for all x in ~, A (E.6.(x + E)) = 0 then: a) for all x


in ~, A ((~ \ E).6. (x + (~\ E))) = 0; b) A(E) . A (~\ E) = O.

5.52. For (~, S{3, JL) complex and ji,(x) ~f fIR e- itx dJL(t): a) if P(Z) = °
then for all E in S{3, JL (UmEZ(E + 27l"m)) = 0; b) for some JL and some E,
P(Z) = 0 and LmEZ JL(E + 27l"m) #- O.
5.53. For ([0,1], S{3, JL), if JL .1 A, JL ([0,1]) < 00, and fELl ([0,1], A) then:
a) f; f(x-y) dJL(Y) < 00 exists and is finite a.e. (A); b) the hypothesis JL .1 A
is superfluous.
5.54. a) If {(~, S{3, JLn)} nEN is a sequence of complex (nonzero) measure
situations then for some Borel measure 1/ and all n in N, JLn «1/. b) If
{(~, S{3, JLn)} nEN is a sequence of positive measure situations, need there be
°
a measure 1/ such that #- 1/ « JLn, n E N?
5.55. For (~, S{3, JL) totally finite and an E in S{3, there are measures 1/1,1/2
such that JL = 1/1 + 1/2, 1/1 (x + E) == 0 and for some {xn}nEN'

1/2 (~\ U(x + E)) = O.


n
nEN

5.56. Let Q (a1' a2) in ~2 be { (Xl, X2) : IX1 - all + IX2 - a21 ::; ~ }.
For (~2,S{3,JL), if horizontal and vertical lines are null sets (JL) then

is continuous.
5.57. For (~2, S{3, JL) totally finite, if all lines are null sets, E is a bounded
Borel set, and 0 < a < JL(E) then there is a Borel set F such that JL(F) = a
(cf. 4.24).
58 5. Measure Theory: Problems

5.58. For {(l~n, 5,8, /-Lm)} :'=0 and each f in Coo (JRn , q, if

(5.6)

then: a) for U open, limm--+oo/-Lm(U) ~ /-L0(U); b) if E is a Borel set and


/-Lo (BE) = 0 then limm--+oo/-Lm(E) = /-Lo(E) (cf. 4.19).
5.59. 5,8 (JRm ) x 5,8 (JR n ) = 5,8 (JR m +n ), 5,\ (JR m ) x 5,\ (JR n ) ¥5,\ (JR m +n ).

5.60. For some product measure situation X-YEr(X-y, 5-y, /-L-y) and some
subset r'of r, Pr , does not preserve measurability.
5.61. There are measure situations (Xi, 5i , /-Li) for which 51 x 52 is resp.
is not the set of all empty, finite, or countable unions of pairwise disjoint
rectangles E1 x E 2 , Ei E 5i , i = 1,2.

5.62. For (JRn , 5,8, /-L), if sUPP(/-L) ~f K is compact and for some JR-valued
polynomial p and all JR-valued polynomials q, fn~.n p(x) (q(X))2 d/-L(x) ~ 0
then p ~ 0 on K.
5.63. If {fn} nEN C C ([0,1], JR), fn ~. f, and 0 ::; a < 1 then [0,1]
contains a compact set K such that A(K) > a and f is continuous on K.
5.64.
i. The function f is in AC ([a, b], JR) iff: a) f is continuous, b) f E
BV ([a, b], JR), and c) {A(E) = O} '* {A (f(E)) = O}.
ii. If x) is one of a),b),c) there is an f for which x) does not hold and for
which the other two do hold.
iii. If E E 5,\ and f E AC ([a, b], JR) then f(E) E 5,\.

5.65. If f is not constant on [a,b] and if f'(x) == 0 then f tJ- Lip(l) on


[a,b].
5.66. If f E JRIR n Lip(l) then for all E in 5,\ and for some constant M,
A (f(E)) ::; MA(E).
5.67. For some f,g in JRIR , f is continuous and open, g is measurable (A),
and go f is not measurable (A).
5.68. For some E in 5,\ and some ¢ in C (JR, JR), ¢(E) tJ- 5,\.
5.69. For (X, 5, /-L), if each singleton set {x} is in 5 and 2x \ 5 i=- 0 then
there is a diset A and a net n : A 3 oX 1---+ n,\ EM such that n,\ converges in
the product topology for M to an f in JRx \ M.
5.70. If 0::; f E L 1(X,/-L) then for

v: 5 3 E 1---+ v(E) ~f L f(x) d/-L(x) ,


Problems 5.71-5.82 59

the essential suprema of I with respect to JL and v are the same.


5.71. If E E SA and >.(E) °
> then #(E) = c.
5.72. If ([0, 00), S/3,JL) complex and J[O,oo)e-nxdp,(x) = 0, n E N, then
JL (5/3) = {o}.
5.73. For ([0, 00), S/3,p,), if p, ([0, 00)) = 1 then

r
1[0,00)
(1- p,([O,x))) dx = r
1[0,00)
xdp,(x).

5.74. For ([O,oo),S/3,JL), ifO ~ I E M and I is bounded then

{J E L1 ([O,oo),JL)} '* {r
1[0,00)
JL({Y: x ~ I(Y)}) dx < oo}.

5.75. (Banach) For I in C ([0,1], JR), if I ([0, 1]) ~ [m, M],

v: [m, M] :7 Y f-t v(y)


~f{#oo({X: I(x)=y}) if#({x: I(x)=y})<oo
otherwise,

and 0· 00 ~f ° then J[m,M] v(y) dy = vaqa,b]U).

5.76. If [a, b] c U:=l [an, bn] then 2::=1 (bn - an) ?: b - a.


5.77. The plane JR2 is not the countable union of lines.

5.78. If (0,1] n Q ~f {rd kEN and I(x) ~f 2::=1 21 1 11 then


k x - rk 2

I(x) < 00 a.e.


5.79. If Q ~f {rn} nEN then JR \ UnEN (rn - rn + ;2' ;2)
=I- 0. Is there an
enumeration {Sn} nEN of Q such that JR \ UnEN (Sn - ~, Sn + ~) =I- 0?

5.80. True or false: for ([0,1]2,SA2,>'2) if E E SA2 and >.(Ex.) ~~ a.e.


1
then >'({X2 : >'(EX2) = I}) ~"2?

5.81. The set E ~f JR2 \ { (Xl, X2) : Xl - X2 E Q} contains no measurable


rectangle A1 x A2 such that >'2 (A1 x A 2) > 0.
JR2
5.82. If I E JR , Ix! is Borel measurable for all Xl, and IX2 E C (JR, JR)
for all X2 then I is Borel measurable.
60 5. Measure Theory: Problems

5.83. If E is dense in R and for some I in RIR2 IXi is Lebesgue measurable


for all Xl in E, and IX2 EO (R, R) a.e. (>.) then I is Lebesgue measurable.
5.84. If I E R1R2, IXi E M for all Xl. and IX2 E a (R, R) for all X2 then
for every g in RIR n M, h : R '3 X I--t I (g (X2) ,X2) E M.

5.85. Is I: R2 '3 (Xl.X2) I--t ( :lX22) in Ll ([-1, IJ2,>'2)?


Xl +X2
5.86. For

if x21 + X22 > °,

otherwise

how are ItO,lJ (IO,lJ I (Xl. X2) dXl) dX2 and fro,lJ (Iro,lJ I (Xl, X2) dX2) dXl
related?
5.87. For some I in a ((0,1)2, R), f(o,1)2 II (Xl, X2)1 d>'2 (Xl, X2) < 00 and
for some Xl in (0,1), f(o,l) II (Xl, X2)1 dX2 = 00.
5.88. True or false: if (X, 5, f..Li), i = 1,2, are totally finite then

5.89. If ([0, 1], 5,6, f..L) is totally finite and f..L « >. then

. f..L ([0, IJ n (x - a, X + a))


11m '---'.:-:-~-'----'-----'-'-
aiO >.([O,IJn(x-a,x+a))

exists a.e. (>.).


5.90. True or false: for some E in 5.>.([0,1]),

5.91. If {In } nEN is a set of open intervals in Rand {on}l::;n<N is the set of
components of A ~f UnEN I n, then: a) Ll<n<N >. (an) S Ll<n<N >. (In);
b) for Min N, there is a subset {Jnk}l<k<-K of pairwise disjOInt intervals
such that>. (U~l I n) S 2>' (Ul::;k<KJ~k)'
5.92. True or false: there is a signed ([0,1]' 5, p,) such that f..L to, f..L « >.,
and for all a in [0,1], f..L ([0, a]) = O?
5.93. Lebesgue measure An is rotation-invariant.
Problems 5.94-5.104 61

5.94. There is in JR a Lebesgue null set of the second category.


5.95. If 0 < A(E) < 00 and 0 :=; I E M then g : JR 3 x t-+ fE I(x - t) dt is
in £1 (JR, A) iff I E £1 (JR, A).
5.96. If /-L (JR) = 1 then /-L * A = A, i.e., A(E) == fn~ /-L(E + x) dx.
5.97. If T E (JRm)lRn, then T is continuous if T(x + y) == T(x) + T(y) and
T- 1 (0 (JRn )) C S{3 (JRn ).
5.98. For x in [0,1], let an(x) be the nth decimal marker for x. If k -# 0, 1
and
kif an(x) -# 0, n E N
1:[0,1]3xt-+ { 1 ifmin{n: an(x)=O} is even
o otherwise
then I is Lebesgue measurable. What is fo,1] I(x) dx?
5.99. Let E in [0,1] be the set of x such that the nth decimal marker of
x is 2 or 7. True or false: a) E is closed? b) E is open? c) E is countable?
d) E is dense in [0, I]? e) E E S{3? If E E SA, find A(E).

5.100. For x in [0,1] and m in N \ {I}, let 2::'=1 E~~) be the m-ary
representation of x. If 0 :=; k < m and

AN(x,k,m)~f#({n: En(x)=k, l:=;n:=;N})

then

E ~ {x : J~oo AN (~k,m) does not eXist} E SA \ (0 U F).

5.101. For some E in SA (JR), A(E) < 00, and if a < b then

0< A (E n [a, b]) < b - a.

5.102. For some E in SA, if a < b then A (E n [a, b]) . A ([a, b] \ E) > O.
5.103. True or false: if {En}nEN C SA ([0,1]), the En are pairwise dif-
ferent, and for some positive a and all n in N, A (En) 2: a then there is an
infinite subsequence {Enk hEN such that A (nkEN E nk ) > O?
5.104. If {An}nEN C SA (JR), A (UnENAn) < 00, and

G k ~f {x : x E An for exactly k different values of n}


62 5. Measure Theory: Problems

5.105. If I is integrable (>.) over every interval and for each interval J of
length 1 or V2, JJ I(x) dx = 0 then I(x) ~ O.
5.106. If I E LR ('][',7), 11/1100 = 1, and for some Xo, and some N,
100N(f) (xo)1 = 1 then I(x) is constant a.e. The conclusion need not hold if
I is (>valued.
5.107. There is a bijection I : [0,1] f-+ [0,1] such that: a) E is Lebesgue
measurable iff I(E) is Lebesgue measurable, b) for a Lebesgue measurable
E, >.(E) = >. (f(E)), and c) I (el, ¥)) = [~, ¥].
5.108. For some Lebesgue measurable function I, finite everywhere, if
a < b then Ira.b] I(x) dx = 00.
5.109. If I E JRIR and I is SA-measurable, there is a sequence {fn}nEN
of Q-valued SA-measurable functions converging uniformly and monotonely
to I.
5.110. If lEe (JR, JR), 9 is SA-measurable, and for every null set (>.) N,
l-l(N) is SA-measurable then go I is SA-measurable (cf. 5.67).
5.111. If >'*(A) > 0 and 0 < () < 1, there is an interval J such that
>'* (A n J) > (). >'*(J).
5.112. For (X, S, /-L), if I E M for some 9 in M, g(x) ~ I(x) and
sup", Ig(x)1 = 11/1100'
5.113. If I E JRIO.I] n M then for a unique ao:

1
a) >.({x I(x) ~ ao}) ~ '2;
b) >.({x I(x) ~ a}) < ~ if ao < a < 00.

5.114. If IE Loo ([0, 1], >.) and

then I(x) ~ 0 (cf. 2.79).


5.115. If I,g E L oo ((0,00), >.) and J(o.oo)(I/(x)1 + Ig(x)l)/xdx < 00 then
J(o.oo) (1/(xy)g (l/y)1 /y) dy < 00 a.e.
5.116. If {fn}nEN C L oo (JR, >'), In m~s I, and for some 9 in Ll (JR, >'),
Iln(x)1 ::; Ig(x)1 a.e. then I E Ll (JR, >.) and

lim
n->oo
r r
JIR In (x) dx = JIR I(x) dx.
Problems 5.117-5.128 63

5.117. If >.(E) °
> and {x, y E E} ::::} {~(X + y) E E} then E contains a
nonempty open set.
5.118. # (SA) = 2'.
5.119. If E c JRn and for each a in JR n , there is a positive Ta such that
B (a,Ta) nEE SA, then E E SA.
5.120. If >.(E) = °and #(E) > ~o then E contains a subset F in SA \ Sf3.
5.121. If /-L* is an outer measure on 2[0,1], Sf3 c SI' [the set of (Caratheo-
dory) /-L*-measurable subsets of [0,1]], and /-L «: >. then SA C SI'"
5.122. True or false: if {fn}:=o C JR[O,l], each In is monotonely increasing,
and In m~a.s 10 then lim n-+ oo In(x) = lo(x) at each point of continuity x of
lo?
5.123. If °: :; I E JR[O,l] and I is SA-measurable then: a)

Af ~ {g : 9 E L1 ([0, 1], >'), Ig(x)1 :::; I(x) a.e. }

is a II Ill-closed subset of L1 ([0,1], >.); b) if Af is II Ill-compact then I is


integrable.
5.124. For some E in SA2' E + E t/: SA2.
5.125. For some E in SA, E + E t/: SA.
5.126. If, for {([O, 1], SA, /-Ln)} :=0'

/-Lo ([0, 1]) = 1 = n-+oo


lim /-Ln ([0, 1])

and for each E in SA, /-Ln (E) :::; /-Lo (E) , n EN, then there is a sequence
{/-LnkhEN such that for each 9 in L1 ([0,1]' /-Lo), irO,l] g(x) d/-Lnk (x) exists
and lim ( g(x) d/-Lnk (x) = ( g(x) d/-Lo(x).
k-+oo i[O,l] i[O,l]

5.127. For ([0,1]' SA, >'), if E is in SA and


1.
def (n
In(x) = n io XE(X + t) dt, n E N,

then: a) °: :; In :::; 1; b) In E AC (JR, JR); c) In ~. XE; d) In II-Jl XE.


5.128. If I E AC ([a, b], JR), I is strictly monotonely increasing, and
I ([a, b]) = [c, d] then for every Borel set E in [a, b],

( !,(x) dx = >'(E).
if-leE)
64 5. Measure Theory: Problems

5.129. For some I in BV ([0, 1], C) n C ([0,1], C), and some null set A,
I(A) E SA([O, 1]) and A (f(A)) > 0.
5.130. a) If I E BV ([0, 1], C) then f' is finite a.e. (A) and

f' E L1 ([0, 1], A) .

b) If IE AC ([0, 1], JR) then I is the difference of two absolutely continuous


monotonely increasing functions.
5.131. True or false: if I E JRIR then I is continuous a.e. iff I is equal a.e.
to a continuous function?
5.132. If I and g are nonnegative Lebesgue measurable functions on JR,
Ay ~ {x : I(x) ~ y}, and h(y) ~f fA y g(x) dx then

l I(x)g(x) dx = 1 00
h(y) dy.

5.133. An I is in Lip(l) on [0,1] iff: a) for some g in

L OO ([0, 1], A), I(x) - 1(0) = l x


g(t) dt;

or b) for some sequence {fn}nEN in Lip(1),

lim In(O) = 1(0), nlim


n~oo
var[O 1] (f - In) = 0,
-+(X)'

and the Lipschitz constants K (fn), n E N, form a bounded set.

When I E C IR and for all x, I(x - 0) exists, I(x) = I(x - 0), and
limx->_oo I(x) = 0, then J-tf : [a, b) 1--* I(b) - I(a) is the interval function
associated to I. When, to boot, I is a function of bounded variation, J-tf is
uniquely extensible to a (complex) measure, again denoted J-tf, on SA.

5.134. True or false: if IE C([O, 1], C), there is a complex ([O,I],S,6,J-t)


such that J-t([a,b)) = J-tf ([a,b))?
5.135. If I is monotonely increasing on JR and g = P then J-tg « J-tf.
What is dJ-tg?
dJ-tf
5.136. If I E L1 (JR,A) then for some sequence {In}nEN in Coo (JR,C),
g(x) ~f 2::::'=1 In(x) exists and I(x) ~ g(x).
Problems 5.137-5.146 65

5.137. An f is in L2 ([0, IJ, A) iff fELl ([0, IJ, A) and there is a mono-
tonely increasing function 9 such that if °: :;
a :::; b :::; 1 then

It f(x) <ixl' " (9(b) - 9(a)) . Ib - al·


5.138. If 9 E L2 ([0, 1], A) \ {o} and G(x) ~f foX g(t) dt then IIGI12 < Ilgll2.
5.139. There is a systematic enumeration {{Jnkh<k<2n-i} of the
- - nEN
set of open intervals deleted in the construction of the Cantor set. For
f(x) ~ E:'1 (E!:~1 nXJnk) ' what is 1[0,1] f(x) dx?
5.140. If (JR2 , 5,8 (JR2 ) ,/-l) is totally finite then for the net

v : 5,8 3 A I-t v(A) ~f /-l (A x JR) ,

there is a map 5,8(JR) 3 B I-t fB ELI ([0, 1], v) such that:

{B 1 n B2 = 0} '* {fBiuB2 = fBi + fB2};


{B E 5,8 (JR)} '* {/-l (A x B) = L fB(X) dV(X)} .

5.141. True or false: if a) {Cn}nEN C JR, b) A(A) > 0, and c) for all t in
A, limn -+ oc eicnt exists then lim n -+ oc Cn exists?
5.142.
(1: If(x)1 dx r: ;
If f E Loc (JR, A), 1 :::; p
c(b - a)P-l I:
< 00, -00

If(x)IP dx then f
< a < b<
== 0.
00, °< C < 1, and

5.143. If fEe ([0,1], JR), f(O) = 0, and 1'(0) exists then


f(x)
9 : [O,IJ 3 x I-t x 3/ 2

is in £1([0,1], A).
5.144. If fELl ([0,1], A), 9 E L OC (JR, A), and 9 is periodic with period one
[g(t + 1) = g(t)J then limn -+ oc IrO,I] f(t)g(nt) dt = IrO,I] f(t) dt . 1[0,1] g(t) dt.
5.145. Each finite subset of Ll (X,/-l) is uniformly integrable.
5.146. For (X, 5, /-l), if {fn} nEN C Ll (X, /-l), fn 2:: 0, fn ~. 0, and
Ix max {!I(x), ... , fn(x)} d/-l(x) :::; M < 00 then Ix fn(x) d/-l(x) --+ 0 as
n --+ 00.
66 5. Measure Theory: Problems

5.147. a) lim [R sin x dx = ~; b) lim [R Isin x I dx = 00.


R-HX) Jo x 2 R--+oo Jo x

5.2. Probability Theory


Conventions

A probability measure situation (0, S, P) is one for which 0 E S, P(O) = 1,


and subsets of null sets are in S [(0, S, P) is complete]. In these circum-
stances a subset S of 2n is independent iff for each finite subset {E l , ... , En}
of S, P (n~=l Ek) = TI~=l P (Ek). A subset :F of M is independent iff for
every finite subset {II, ... , In} of:F and every finite subset {Bl' ... ' Bn} of
Borel subsets of~, {/ll (B l ) , ... ,1;;1 (Bn)} is an independent subset of
2n. When S resp. :F is independent, the elements of S resp. :F are called,
by abuse of language, independent.
Traditionally, elements of S are called events and the elements of
M are called random variables. Furthermore, if I is integrable
then E(f) ~f In I(w) dP(w) is the expected value of I; if P is
integrable, then Var(f) ~f 0'2(f) ~ In (f(x) - E(f))2 dP(w) is
the variance of f.
Two particularly elementary and yet important instances of a prob-
ability measure situation (0, S, P) are: a) for some discrete measure P,
(N, 2N, p); b) ([0,1], S.x, A). Every (0, S, P) contains at most count ably
many atoms. When S is metrized according to

d : S x S 3 (A, B) ~ d(A, B) ~f P(A,0.B)

and S is d-separable, then (0, S, P) is, by abuse of language, measure-


isomorphic for some P to

(N,2 N ,P), ([0,1],S.x,A), or (N,2 N ,P)U([0,1],S.x,A).


Unless the contrary is indicated in the current Section, the underlying
probability measure situation is to be taken as a general (0,5, P).

5.148. When some of or all the members of S are replaced by their


complements, the resulting set S is independent iff S is independent.
5.149. a) The set {E.xhEA is independent iff the set {XE>. hEA is in-
dependent. b) If S ~f {{lj k} ~~ 1 } ~ is independent and if 9 j is a Borel
3=1

measurable function on ~Kj, 1 ::; j ::; J, then {hj ~f gj (Jjl, ... , /iKJ}~3=1
is independent.
Problems 5.150-5.156 67

°
5.150. a) If P(A) = or P(A) = 1 and BE S then {A, B} is independent.
b) If I(w) is constant a.e. and gEM then {/,g} is independent.
5.151. If {/,g} is independent and {f,g} C L1 (O,P) then Ig E Ll (O,P)
and

In I(w)g(w) dP(w) = In I(w) dP(w) ·In g(w) dP(w). (5.7)

5.152. If P in (fiI, 2N , p) is such that

P() { 2- n ! if n = 2,3, ...


n = 1_ "cxo
L."n=2
2- n ! if n =1
then: a) if neither A nor B is 0 nor fiI, then {A, B} is not independent; b)
if neither I(w) nor g(w) is constant a.e. then {I, g} is not independent.
5.153. For ([0,1], SA' A) if 1 E M, 1 is strictly monotone on some
nonempty subinterval (a, b) of [0,1], and {f,g} is independent then g(w) is
constant a.e.

For S C L2(X, JL), span(S) is the set of all (finite) linear combinations
of elements in S while

S.l ~f {g : 9 E L2(X, JL), {f E S} :::} {Ix I(x)g(x) dJL(x) o} }.


=

For M a subspace of L2(X, JL), dim(M) is the cardinality of a maximal


linearly independent subset of M. A subset S of M spans M when the
II 112-closure of span(S) is M (cf. Chapter 6).

5.154. If S ~f {I"f} "fEr c L2 (0, P) and S is independent then: a)

{dim (L2 (O,P)) ~ 3}:::} {dim (S.l) ~ I}.


b) {# (r) ~ ~o} :::} {dim (S.l) ~ ~o}.

5.155. If S ~f {An}nEN is independent then P (limn-+cxoAn) is or 1


according as I:~=1 P (An) converges or diverges. [The preceding situation
°
illustrates the zero-one law [Ko].]
5.156. For ([0,1]' SA' A): a) the set

is independent; b) the set W ~ {Wn } ~=o of all finite products of pairwise


different members of R (cf. Solution 5.154) spans L2 ([0, 1], A).
68 5. Measure Theory: Problems

[Note 5.3: The functions Tn resp. Wn are the Rademacher resp.


Walsh functions [Fin, KacS, Wall.]

5.3. Ergodic Theory


Conventions

The context is a totally finite (X, 5, J.L) and an autojection T : X ~ X such


that

{E E 5} '* {{T(E) E 5} 1\ { T- 1 (E) E 5} 1\ {J.L(E) = J.L (T(E))}}.

For f in Ll (X, J.L) and n in N,


n-l
Sn: X 3 x H L f (T k ( ))
x , -F = -1·
def
Imn
Sn
-+ oo - ,
n
and _F = 1·Imn
def
-+ oo
Sn
-
n
k=O

are the principal objects of study.

5.157. If E E 5 then J.L (T- 1 (E)) = J.L(E).


5.158. If
A ~f {x : supsn(x) > o} and E E 5
nEN
then
r
JAnE
f(x) dJ.L(x) ~ o.

5.159. If

=
Aa def {
X:
Sn(X)
sup - - >a~ 0 } and E E 5
nEN n

then
r
JAa nE
f(x)dJ.L(x)~a.J.L(AanE).

5.160. a) F(x) = F(T(x)) and F(x) = F(T(x)); b) F(x) ~ F(x); c)


{ Sn } is uniformly integrable.
n nEN
5.161. For
F(x) ~f {FO (x) if F(x) = F(x)
otherwise
if E E 5 then
Problems 5.162-5.163 69

[ Note 5.4: The conclusions in 5.160b) and 5.161 embody the


pointwise ergodic theorem of G.D. Birkhoff.]
In L2 (X, /L) the autojection T engenders a unitary automorphism U
in [L2 (X,/L)]: U: L2 (X,/L) 3 f e~o [U(f): X 3 x 1-+ f(T(x))].
5.162. In the context just described, if f E L2 (X, /L) then for some L(f)
in L2 (X, /L), L~:~ Uk (f) IIJl2 L(f).
n
[Note 5.5: The assertion in 5.162 is the mean ergodic theorem
of von Neumann. It is valid in the context of any abstract Hilbert
space fl and a unitary (automorphism) U : fl 1-+ fl.]

5.163. For Zo in '[', f in C ('[', C), and the autojection

def
T = Tzo : '[' 3 z 1-+ Zo . Z

of '[', L d~f limn -+ oo Sn (zo) exists. There is a ('[',5/3, /Lzo) such that for each
n
6
Topological Vector Spaces

6.1. The Spaces LP (X, J.L), 1:::; P:::; 00


Conventions

The topological vector space V (over a topological field lK) is locally convex
iff each element of some neighborhood base at 0, is convex. A subset B
of V is bounded iff for every neighborhood U of 0 and some positive t,
B c tu. When some open subset of V is bounded, V is locally bounded.
A subset S of V is circled iff lK=C and for every complex number z such
that Izl :::; 1 and every x in S, zx E S. For a circled bounded neighborhood
of U of 0, the associated Minkowski functional Mu is:

Mu : V '3 x f-+ inf {a : a;::: 0, x E aU} .

A map p : V '3 x f-+ p(x) E [0,00) is a quasinorm iff for some k in [1,00): a)
p(x + y) :::; k (p(x) + p(Y))j b) p(tx) = Itlx. When k = 1, P is a seminorm.
A seminorm p is a norm iff {p(x) = O} <=? {x = O}. A subset R of V is
radial or absorbing iff for every finite subset F of V and some real number
r(F), AR:J F if IAI ;::: r(F).
The dual space of V is [V, q and is denoted V*. Elements of a dual
space are tagged with a superscript *.
The spaces V and V* are paired and for v in V and v* in V*, the
value of v*(v) is denoted (v, v*).
To each T in [V, W] there corresponds in [W*, V*] a unique adjoint T*
satisfying (and defined by): (Tx, y*) == (x, T*y*).
The weak topology for V is a (V, V*)j its basic neighborhoods are

U(xjxi, ... ,X~j€) ~f {y : l(y-x,xk)1 < €, 1:::; k:::; n},


determined by an x in V, an n in N, xi, ... , x~ in V*, and a positive €.
The weak* topology for V* is a (V*, V)j its basic neighborhoods are

determined, mutatis mutandis. For a net n : A '3 A f-+ n(A) E V resp. n :


A'3 A f-+ n(A) E V*, n(A) ~ x resp. n(A) ~ x* indicates the convergence,
in the weak resp. weak* topology of V resp. V*, of the net to x resp. x*.

70
B. R. Gelbaum, Problems in Real and Complex Analysis
© Springer-Verlag New York, Inc. 1992
Conventions 71

For a subset A resp. A * of the topological vector space V resp. V*

Ai. ~f {x* : (a,x*) = 0, a E A} (c V*)


resp.
Ai. ~f {x : (x,a*) = 0, a* E A*} (c V).

When (V, II II) is a normed vector space, e.g., a Banach space, V is


uniformly convex iff for each € in (0,2J and some positive 6(€),

The set B ~f {xn}nEN in a topological vector space V is a basis


for V iff for each x in V and for a unique sequence {an(x)}nEN in EN,
limN ...... "" L;:=1 an(x)Xn = x in the topology of V. Associated with a basis
are the maps

N
SN : V 3 x t-t L an(x)xn, N E N, P n : V 3 x t-t an(x)xn, n E N.
n=1
When V is a Banach space, according as the topology is norm-induced,
weak, or (when V is some W*) weak*, B is a norm-basis (a Schauder
basis), a weak basis, or a weak* basis.
A E-Hamel basis for V is a E-maximal linearly independent subset
H ~f {xAhEA of V. Thus there are unique functions

aA : V 3 x t-t aA(x) E E, .A E A,

such that x = LAEA aA(x)x A. (For each x, only finitely many a A are
nonzero.)
When 1 < p < 00, p' PI' i.e., ~ + I, = 1. When P = 1,
=
def
P- P P
p' ~f 00; when p ~ 00, p' ~f 1. For (X,S,/L), two measurable functions
I and 9 in eX are equivalent iff I(x) ~ g(x) (11,). When 1 ~ p < 00,
LP (X, /L) is the set of equivalence classes of measurable functions I such that
Ix II(x}IP d/L(x) ~f IIIII~ < 00. When p = 00, the condition 11111"" < 00 is
imposed.
When £P (X,/L) is regarded as an lR-module, L~ (X,/L) is a closed lR-
submodule of LP (X, /L).
The notation fP (X) is used for LP (X, /L) when /L is counting measure
(. Similarly, the notation Co (X) is used for Co (X, q when the topology
of X is discrete.
72 6. Topological Vector Spaces: Problems

When X in (X, 5, J.L) is a topological space, it is assumed to be a Haus-


dorff space, 5 is 5!3, for each open set U, J.L(U) > 0, and for each compact
set K, J.L(K) < 00: the situation is symbolized by (Xtop , 5!3, J.L).
For a set X, a subset N of X, and an f in eX,

oscN(f) ~f sup If(a) - f(b)l.


a,bEN

For a topological space X, a set of nonnegative continuous functions


{<p,} ,Er in JRx is a partition of unity subordinate to an open cover {U,} -yEr
of X iff a) each <p-y is not identically zero, b) <p-y (x) = 0 off U-Y' c) for each x in
X, only finitely many <p-y(x) are different from zero, and d) L-YEr <p-y(x) == l.
For a locally compact group G and f,g in L1 (G,J.L), their convolution
f * g is G 3 x ~ Ie f (y) g (y-1X) dJ.L(Y) = Ie f(y)g[x 1 (y-1) dJ.L(Y)·
The set G ~f {a : a E [G, ']['] } of continuous homomorphisms of G
into '][' is the dual group or group of characters a : G 3 x ~ (a, x) E '][' of
~ ~ ~ def --
G. For f in L1 (G,J.L), I: G 3 a ~ I(a) = Iel(x)(a,x)dJ.L(x) is, when G
is abelian, the Gelfand-Fourier transform of f.
For vector spaces V, Wand h in [V, W], ker(h) = h-1(O).

6.1. If I E L1 (X, J.L), and IE I(x) dJ.L(x) ~ a whenever J.L(E) < 00, then
Ix I(x) dJ.L(x) ~ a. If the hypothesis I E L1 (X, J.L) is dropped, does the
conclusion follow?
6.2. For some (X, 5, J.L) and for some sequence {In} nEN in L1 (X, J.L),
In ~ 0 and Ix fn(x) dJ.L(x) i 00.
6.3. If {fn}nEN C L1 (X,J.L) , 0 ~ fn ~ 1, In ~. 1, and for some E of
finite measure, In(x) == 1 off E then limn-too Ix (1- fn(x)) dJ.L(x) = O.
6.4. For (X,5,J.Li), i = 1,2, such that X E 5, J.Ll(X) < 00, andp E [1,00),
and for some f in LP (X, J.Lt),

iff for some a in JR, and all measurable partitions {En} nEN of X,

6.5. If (X, 5, J.L) is totally u-finite then for p in (1,00), I in £P (X, J.L), and
E t ~f {x : If(x)1 ~ t}, II/II~ = pIoootP - 1J.L(Et} dt.
Problems 6.6-6.15 73

6.6. For (X,S,JL), if: a) 1:=:; p < 00, b) 9 is measurable, c) R;.,(g,O) is


a-finite, and d) gh E Ll (X, JL) for all h in £P (JR., A), then 9 E £P' (X, JL).
Without c) the conclusion is invalid.
6.7. For (JR., SA' A) and p in [1, 00), if 9 is measurable and for all h in
£P (JR., A), I~T g(x)h(x) dx has a finite limit as T - t 00 then 9 E £P' (JR., A).

6.8. For pin (1,00), (X, 5, JL), and (Y, T, v), if I in JR.xxY is such that a)
I is 5 x T measurable, b) for almost every x in X, Ix E £P (Y, v), c) for
almost every y in Y, Iy ELI (X, JL) then

[ Note 6.1: The result above may be viewed as a "continuous"


version of Minkowski's inequality, e.g., when Y ~f {1, 2, ... ,n}
and v = ( (counting measure).]

6.9. If {xn}nEN C £1 (N), then Xn IlJil x iffxn ~ x.


6.10. For some I in Coo ([0,00), q n Ll ([0,00), A), :E:'=II(n) = 00.
6.11. For {(X, 5, JLn)} :'=0' if JLo(X) = 1 = limn-+oo JLn(X) and JLn :=:; JLo
then for a subsequence {JLnk hEN' a measure JLoo, and all 9 in Ll (X, JLo),
limk-+oo Ix
g(x) dJLnk(X) = Ix
g(x) dJLoo(x).
6.12. If I is a nonnegative Lebesgue measurable function on ([0,1]
and En ~f {x: n-1:=:;I(x)<n}, n E N, then IE Ll([0,1],A) iff
:E:'=1 nA (En) < 00.
6.13. If
if x # °
if x = °
then f' exists everywhere but f' (j. Ll (JR., A).
6.14. True or false: if I E Ll ([0, n], 00), n E N, and

F(x) ~.!
x 10
r I(t) dt 2: I(x)
a.e. on (0,00) then I is a monotonely decreasing function?
6.15. If I E Ll([O, 1], A) and I; I(x) dx # ° then for some 9 in M,
I; II(x)I'lg(x)1 dx < 00 and I; II(x)I' Ig(x)12 dx = 00.
74 6. Topological Vector Spaces: Problems

6.16. If fELl ([0, l],oA), f(x) ~ 0 a.e., and 101 (f(x)t dx is n-free then
for some E in SA, f(x) ~ XE(X). How is the conclusion altered if the
assumption: f(x) ;:::: 0 a.e. is dropped?
6.17. If fELl (JR, oA) and for every open set U such that oA(U) = 1,
Iu f(x) dx = 0 then f(x) ~ o.
6.18. True or false: if f E L1 (JR, oA) and In~.lxln If(x)1 dx :s: 1, n E N, then
f(x)=Oa.e.onM~f{x: Ixl;::::l}?
6.19. If fELl (JR, oA) and for every open set U, Iu f(x) dx = Iu f(x) dx
then f(x) = 0 a.e.
6.20. If f in Li ([a, b], oA) and

{a < c < d < b} ::::} {lim -h1


Ihl!O
1
c
d
(f(x + h) - f(x)) dx = o}
then for some constant k, f(x) ~ k.
6.21. If f E L1 (JR, oA) and Ilf[t] - fill :s: Itl 2 then f(x) ~ O.
6.22. If q E L1 (JR, oA) then for some constant Cq ,

{f E Coo (JR, en
: : } {IL Iq(x)l· If(xW dxl :s: Cq L If(x)12 + If'(x)1 2 dX} .

6.23. If fELl (JR, oA) then

if x #0
otherwise

is also in L1 (JR, oA) and IIR f(x) dx = IIR T(f)(x) dx.


6.24. If f, 9 E L1 ([0,1]' oA) then f(x) ~ g(x) iff for all h in Coo ([0, 1], JR),
101 f(x)h(x) dx = 101g(x)h(x) dx.
:s: M
it
6.25. If f E L1 ([0,00), A), gEM, and for t in [1,00), Itg(t)1 < 00
then limt-+oo ~t 1
f(8)g(8) d8 = o.
6.26. If fELl ([0,1], oA) and f is continuous at 0 then for each n in N,
fn: [0,1] :') X f-+ f (xn) is in L1 ([0,1], oA).

6.27. If f in L1 (JR, oA) and fn ~f f[n] . X[O,l] then {L~=l fn } :=1 is a


II Ill-Cauchy sequence.
Problems 6.28-6.36 75

6.28. For (X, S, J.t) totally finite,

6.29. If In 2: °and limn---+oo Ix In (x) dJ.t(x) = °then


lim [
n---+oojx
(1- e-fn(X») dJ.t(x) = 0.

6.30. If In E L1 (JR, >..) and In ~. I then In lI.Jl I iff a) for each positive
€ and some A. in S)., >.. (A.) < 00 and sUPnEN IR\A. I/n(x)1 dx < € and b)

lim>'(B)---+O SUPnEN IB I/n(x)1 dx = 0.


6.31. If a E (-1,0), 8 N ~f ~:=o (~)xn, and I(x) ~f (1- x)<> then in
°
L1([0, 1], >..), limN---+oo 118n - 1111 = (cf. 2.72).
6.32. If: a) I> 0; b) I is monotonely increasing on JR; c)

lim I(x) = 0, i.e., x = o(f(x)), {gn}nEN C M([O, 1])

11
X---+CXJ X

gn ~. g, I 0 (Igni) EM, I (Ign(x)l) dx ::; M < 00, n E N;

then {gn}nEN C L1([0, 1], >..) and limn ---+ oo I; Ign(x) - g(x)1 dx = 0.
6.33. If p E (1,00) and {fn}::'=o C LP (JR, >..) then a) and b) following are
equivalent: a) Il/nllp ::; K < 00 and limn---+ oo I; In(t) dt == I; lo(t) dt; b)
In ~ 10·
6.34. Is A ~f {I : IE L1 ([0,1), >..), I/(x)1 2: 1 a.e.} II Ill-closed? Is A
a (L1 ([0,1]' >..) ,Loo ([0, 1], >..) )-closed?

6.35. If I E L1 ([0, 1], >..) and 8 ~f {x : 0::; x ::; 1, I(x) E Z} then


8 E S>. and limn ---+ oo Io1Icos7r/(x)ln dx = >"(8).
6.36. a) If 9 E L1 C'lI', T) then

is a compact linear endomorphism. b) For In : 'lI' 3 x f-+ xn + ~,


xn
n E N,
limn ---+oo liT (fn) - 01100 = 0.
76 6. Topological Vector Spaces: Problems

6.37. If fELl (JR,'x) and for some n in N, f(x) ~ 0 off [-n,n] then
1 E coo (JR, C).
6.38. If fELl (JR,'x) and supp (1) is compact then for some 9 not
identically zero and in Ll (JR,'x) n C (JR, C), f * g(x) ~ O.
6.39. If fELl (JR,'x) and I * I = I then I(x) ~ O.
6.40. If 'x(E) > 0, fELl (JR, ,x), f ~ 0, and IE I(x) dx = 1 then
lIE f(x)e- ix dxl < 1.

6.41. If 0 #I E L1 (JR,'x) and f ~ 0, then for all nonzero t, If(t) I< 11/111'
6.42. If ('ll', S,a, v) is such that v ('ll') = 1 then IIT x dv( x) I :::; 1 and equality
obtains iff for some a in 'll', v(a) = 1.
6.43. For a locally compact group G and the Haar measure situation
(G, S,a, 1-"), if 1 < p:::; 00, IE L1(G, 1-"), and 9 E V(G, 1-") then

1* 9 E V(G,I-") and III * gllp :::; 1I/11t IIgllp'

For a locally compact group G with identity e, the set U( e) of neigh-


borhoods of e is a diset with respect to the partial order:

{U(e) 3 U -< U' E U(e)} {::> {U c U'}.

6.44. If the net n : U 3 U I--t nu E L1 (G,I-") satisfies the following


conditions: a) nu ~ 0 and nu(x) = nu (X-I); b) for all U, IInulll = 1; c)
nu(x) = 0 off U; then for pin [1,00) and I in V (G, 1-"), nu * f IIJr f. [In
particular, n is an approximate identity for L1 (G, 1-"), regarded as a Banach
algebra with respect to convolution as multiplication.]
6.45. If G is a locally compact abelian group and n is an approximate
identity for L1 (G, 1-") then ~ 1. n
6.46. a) For t in (0,00) what is Ct; if Ct I~oo exp ( (f) 2) dx = I? b) For
gt : JR 3 x I--t Ct exp ((f)2) and I in LP(JR, ,x), limt->o IIgt * I-lip = O.
6.47. If fELl (JR, ,x), I(x) = 0 off [-n, n], and supp (1) is compact then
I~O.

6.48. If 9 E Ll (JR,'x) then Tg : L2 (JR,'x) 3 I I--t In~. g(x - y)/(y) dy ~ 9 * I


is not compact unless 9 ~ O.
6.49. For 1-": 2[1 (0) 3E I--t LXEEx, what is (L1 ([1,00),1-"))*?
Problems 6.50-6.58 77

6.50. For some nonempty open interval J,

{{p E J} /\ {J E L1 (JR., A)}} => {


JR
rJ(x - y) Isinlyl2(lfY) I dy E LP (JR., A)} .

6.51. If p E [1,00] and every nonempty open set in X of (Xtop , 5!3, JL) has
positive measure then each equivalence class corresponding to an element of
LP (X, JL) contains at most one continuous function. For some (Xtop , 5!3, JL)
of the kind described, some equivalence class contains no continuous func-
tion.
6.52. If 1 ::; p < 00, the map

J: [0, If 3 (x, y) t-+ {o(x y - 1)-1 if xy #- 1


ifxy=1

is in V ([0,1]2, A2) iff p = 1.

6.53. If (G,5,JL) is the Haar measure situation for a locally compact


group, p E [1,00], 9 E V (G, JL), and n : A 3 A t-+ n(A) EGis a net
converging to the e in G then limn~e Ilg + g[nlil p = 211gllp·

6.54. If J E V (JR., A) and 1 ::; p < 00 then for some sequence {an} nEN in
°
(0,00), an - t and if Ibnl < an,n E N, J[bnl~' J.
6.55. If °: ;
J, g, J E LP (JR. n , An), 9 E Lpl (JR. n , An), and for all positive t,
E t ~f {x : g(x) > t} then

1 (1.
00
J(x) dX) dt = Ln J(x)g(x) dx.

6.56. If 1 < p < 00, {an} nEN' {bn }nEN C C, and for all n in N,

then for some unique J in Vi ([0,1], A), an == f; t nJ(t) dt.


6.57. a) If 1 < p < 00 and for (X, 5, JL), JL #- 0, what is £p, the set
of extreme points of the unit ball B(O, 1) of V (X,JL)? b) What is the
situation when p = 1 and JL is nonatomic?
6.58. If 1 < p < 00 and A and B are closed subspaces of LP (X, JL) then
A = B iff AJ.. = BJ...
78 6. Topological Vector Spaces: Problems

6.59. a) For a uniformly convex normed vector space (Y, 1111), p in (1,00)
and E positive, there is a positive 6p (E) such that if Ilxll, lIyll ~ 1, and
Ilx - yll 2: E then

(6.1)

b) If 1 < p < 00, then I.J' (X, J.L) is uniformly convex.


6.60. If 1 ~ p ~ 00 and (X,5,J.L) is totally finite then for {fn}:=o in
I.J'(X, J.L),

{{fn ~. fo} 1\ tl!..~ Ilfnllp = Ilfollp}} => {fn IIJr fO}'

6.61. For (X,5,J.L), if {En}nEN C 5 and L1 (X,J.L) 3 XEn IIJll f then for
some E in 5, f = XE.
6.62. If (X, 5, J.L) is totally finite and 0 ¥- f E LOO (X, J.L) then

Ilfll = l' Ilfll~+t


00 n~~ Ilfll~ .

6.63. For (X, 5, J.L), LOO (X, J.L) = L1 (X, J.L) iff dim (L1 (X, J.L)) < 00.
6.64. If (X,5,J.L) is a-finite and f E L 1(X,J.L) then

is in (Loo(X,J.L))*.
6.65. If k E Loo (JR, oX) and In~ e-(x-y)2 k(y) dy °
== then k(x) ~ 0.
6.66. If 1 ~ p ~ 00, 8 E [LP ('][', r)]' and 8 (I[t]) == 8(f)[t] (8 and
translation commute), then: a) for all f and 9 in Loo ('][', r),

8(f * g) = f * 8(g) = 8(f) * g;

b) for some sequence {an}nEN in C and all f in LP ('][',r), (§(J))n = anh.


6.67. (Loo ([0, 1], oX))* \ L1 ([0, 1], oX) ¥- 0.
6.68. If f E LOO ([0, 1], oX) and for each x in [0,1], and some gx in
Loo ([0, 1], C), f(t) ~ gx(t) and limt--+x gx(t) ~ Vx exists then for some
9 in C ([0,1], C), f(t) ~ g(t).
Problems 6.69-6.71 79

6.2. Hilbert Space SJ


Conventions

A vector space SJ ~f {x} over C is a Hilbert space when there is defined an


inner product: SJ2 3 {x, y} ~ (x, y) such that

(ax + by, z) = a(x, z) + bey, z)


(x,y) = (y,x), (x,x) ~ 0,
{(x,x) (~f Ilx11 2) = o} ¢:} {x = O}

[(".) is conjugate bilinear and positive definite]. It is assumed that SJ is a


Banach space in the II II-induced metric. The paradigm for SJ is L2 (X, /-t):

(f,g)
def
= r -
ixf(x)g(x)d/-t(x);

Ilfll ~f IIf112;

Because SJ* and SJ are isomorphic, the adjoint T* of a T in [SJ] is itself


in [SJ] and satisfies (Tx,y) = (x, T*y). When U E [SJ]a and, to boot,
(Ux, Uy) = (x, y), U is unitary and U* = U- 1 •

The gradient map is V' : C (IRn , q 3 f ~ (:!1"'" :!n) and the

Laplace map is ~ : C 2 (IRn , q 3 f ~ L:::~=1 uX


~2 {. For h in IR \ {O},
k

f(x + h) - f(x)
~ X ~f { h
hf( ) f (xe i27r{ - f(x)
if f E ClI'

6.69. The span of the image of, : [-~,~] t ~ {n ~ t} nEN is


3

II 112-dense in £2 (N).
1

6.70. For FN : £2 (N) 3 {an}nEN ~ (~n~N lanl2r, FN ~ 0 on each


II 112-compact set.
6.71. If f,g E C2 (1R2,C) n L2 (1R2, A), ~f,~g E L2 (1R 2,A), and each
component of V' f, V'9 is in L2 (1R2, A) then

r f(x)~g(x) dx = iR2r g(x)/).f(x) dx.


iR2
80 6. Topological Vector Spaces: Problems

6.72. For some curve 'Y : [0,1] '3 t 1-+ 'Y(t) E S),

6.73. True or false: if 1[0,00) 1/~(x)12 dx ~ M2 < 00 and Ixln(x)1 ~ 1 then


the sequence {In} nEN contains: a) a pointwise convergent subsequence?
b) a subsequence converging uniformly on [O,oo)? c) a subsequence II 112-
convergent in L2 ([0, 00), 'x)?

6.74. ForE~f{(x,y): 0~lxl~y~I},if/EL2(E,'x)then

A ~f limy=o jY I/(x, y)1 dx


-y
= o.

6.75. If {In} nEN is II 112-bounded in L2 ([0, 1],'x) and In m~as ° then


limn-+oo II/nl11 = 0.
6.76. For I, 9 in L2 (JR, oX), if limh-+O liD-hi - gl12 = 0 then for some
constant c, I(x)';' fox g(t) dt + c.
6.77. For T ~f L2 ([0, 1], oX) '3 11-+ {x 1-+ fox I(t) dt} and P ~f T + T*:
a) IITII ~ 2-~; b) p 2 = P and P (L2 ([0, 1], oX)) = C; c) T is compact.
6.78. If SeC ([0, 1], C) and S is a closed subspace of L2 ([0, 1], oX) then:
a) S is II lloo-closed in C ([0, 1], C); b) for all I in S and some positive M,
11/112 ~ 11/1100 ~ M11/112; c) for all y in [0,1], some ky in L2 ([0, 1], oX), and
all I in S, I(y) = I;ky(x)/(x) dx.
6.79. If I E L2 (1l', r) and for some positive C and a,

then E:=-oo linl < 00.

6.80. The set A ~f {I : I E coo ([0, 1]), fo1 I~) dx = °} is II 112-


dense in L2 ([0, 1], oX).

6.81. If IE L2 ([0,1], oX) then I(t) == tiff [1 t n I(t) dt = ~2' n E z+.


io n+
6.82. If {In} nEN C L2 (X, J.L) and Il/n - In+Iil2 ~ 2- n then for some I in
) f n a.e.
L 2 (X ,J.L, ~
I and f n II~112 I .
Problems 6.83-6.92 81

6.83. If {fn}nEN is orthonormal in L2 (X,J-L) and IIfnlloo :::; M < 00 then


L::'=l Ifn~x)1 converges a.e. D

6.84. For an orthonormal sequence {xn}nEN in j), Xn ~ O.


6.85. Let M be the II 112-closure of

S~f{f: fEC([-I,I),Q, f(x)=f(-x)}.

a) What is a simple formula for the orthogonal projection of L2 ([-1, 1], >.)
onto M? b) There are polynomials Pn, n EN, constituting an orthonormal
basis for M.

6.86. For an infinite orthonormal system {xn} nEN in SJ and

E =
def {
Xm + mXn :
}
n > m, m, n EN:

a) 0 is in the weak closure EW of Ej b) if FeE and F is II II-bounded


then 0 ~ FWj c) no subsequence of E converges weakly to O.
6.87. a) If {xn}nEN is a complete orthonormal set in SJ, e:::; 2-!, °: :;
and llYn - xnll :::; en, n E N, then {Yn}nEN is a norm-basis for SJ. b) If
x = L::'=l anYn then L::'=l lanl2 is finite.

6.88. If <I> ~f {<p-y} -yEr and W ~f {1/10} OEA are complete orthonormal sets
in SJ then # (r) = # (~).
6.89. If K E L2 ([0, 1j2, >'2) then for

T: L2 ([0, 1), >.) 3 f f--t { X f--t 10 1


K(x, y)f(y) dY } :

a) T E [L2 ([0, 1], >')] j b) for each positive a and some Ta in [L2 ([0, 1], >')],
im (Ta) is finite-dimensional and liT - Ta II < a.
6.90. If dim(SJ) = 00 and 5 is the <T-ring generated by the II II-open
°
sets of SJ, there is no (SJ, 5, J-L) such that < J-L (B(O, 1)) < 00 and J-L is
translation-invariant.
6.91. If dim(SJ) = 00, P E (0,00), and pP denotes Hausdorff measure, then
{oo if E # 0
PP(E) =
° otherwise.
6.92. The lillI-induced topology for L2([0, 1], >.) ~f SJ is not stronger than
<T(SJ, SJ*).
82 6. Topological Vector Spaces: Problems

6.93. If M is a closed subspace of 5) ~ £2 (N),

D ~f { {an}nEN {an}nEN' {nan}nEN E 5)},


A ~f { {an}nEN {an}nEN E 5), {nan}nEN E M.L },

and M n D = {O} then An Mis 11112-dense in 5).


6.94. The set

S ~ {f : f(x) = ~ an sin2mrx, X E [0, 1], ~ Inanl :-:; 1 }


is II 112-compact in L2 ([0, 1], A).
6.95. If Sin L2 ([0, IJ, A) is the set of f such that for some 9 in L2 ([0, IJ, A)
and some constant c, f(x) = I;
g(t) dt + c then T : S 3 f f---+ 9 is a well-
defined map for which the graph g ~ { {f, T f} : f E S} is closed in the
II 112-induced product topology of 5)2.
6.96. If (X, 5, JL) is totally finite, M is a subspace of L2 (X, JL), and for all
fin M, If(x)1 :-:; Cllfl12 then dim(M) :-:; C2JL(X) « 00).

6.97. If f E Loo (X, JL) then: a) the operator norm of the map

is Ilflloo iff X contains no atoms having infinite measure, by abuse of lan-


guage, no infinite atoms; b) if X contains no infinite atoms and Ilkll oo = 00
then k . L2 (X, JL) rt L2 (X, JL).
6.98. What are useful necessary and sufficient conditions that Tf in 6.97
be surjective?
6.99. In 6.97 when X = [O,IJ and JL = A: a) if f(t) == t, Tf has no
eigenvalues; b) if S E [5)J and for all f in Loo ([0, 1], A), STf = TfS then for
some 9 in L oo ([0, IJ, A), S = T g •

6.100. For a ~f {an}nEN in £00 (N) and

Ta : £2 (N) 3 x def
= { xn} nEN f---+
{
anxn } nEN def
= Tax:

a) Ta E [L2 (N)] , IITal1 = Iiall oo ; b) A is an eigenvalue of Ta iff for some no,


A = ano; c) if an pO then Ta (B(O, 1)) is not norm-compact.
willi
6.101. If Ilxnll -+ Ilxoll as n -+ 00 and Xn -+ Xo then Xn -+ Xo.
Problems 6.102-6.109 83

6.102. If {xn' Yn} nEN is a subset of the unit ball in fj then

lim (xn' Yn) =


{ n-+oc> I} => { lim
n-+oo
Ilxn - Ynll = o} .
6.103. If B : fj2 3 {x,y} I-t B(x,y) E C is a conjugate bilinear form on
fj2 and IB(x,y)l::; Ilxllllyll then for some T in [fj], B(x,y) = (x,Ty) and
IITII ::; 1.
6.104. The topology a (fj, fj*) is a Hausdorff topology.
6.105. For the topology a (fj, fj*), fj is of the second category iff dim (fj)
is finite.
6.106. If M is a closed subspace of fj and x E fj, then

inf{lly-xll : YEM}=sup{l(z,x)1 : ZEMl.n8B(O,I)}.

6.107. If f E L2 (1l', T) and in #- 0, nEil, then

6.108. For L2 ([0,1],,,\) ~f fj, the domain of the differentiation operator


D is, by definition, the set of functions f such that f E fj, D f exists a.e.,
and D f E fj. The domain S of D is dense and the graph of D is closed.

6.109. For f: JR 3 x I-t f(x) E fj, if F(x) ~f (f(x),y) is, for each y, differ-
entiable with respect to x then for all Xo in JR, limx--+xo Ilf(x) - f (xo)11 = o.

6.3. Abstract Topological Vector Spaces


Conventions

In a topological vector space V over a topological field lK (usually C, occa-


sionally JR), the maps lK x V 3 {ax} I-t ax, V x V 3 {x,y} I-t x+y are
assumed to be continuous.
For a set {VA} AEA of topological vector spaces, the direct sum ffi AEA VA
is the set XAEA VA endowed with the weakest topology for which each map

is continuous. Vector addition and multiplication by scalars are performed


componentwise.
When W is a closed subspace of V, the quotient topology for V jW
is the strongest topology for which the map V 3 x I-t xjW is continuous.
84 6. Topological Vector Spaces: Problems

When V is a normed space or a Banach space, so are Wand V /W, and the
quotient topology for V /W is induced by the quotient norm

II IIQ : V /W 3 x/W ~ inf {Ilyll : y - x E W} .


When V is a Banach space, so is V*, Fx : V* 3 x* ~ (x, x*) is in
(V*)* ~f V**, and e : V 3 x ~ Fx is the isometry mapping V into V**j
V is reflexive iff e is surjective.
For a map f : V ~ W between normed vector spaces and an x in V,
when for some df(x) in [V, WJ,

lim Ilf(x + h) - f(x) - df(x)(h)11 =0


1I~f1~o Ilhll

then df(x) is the differential of fat x.

6.110. For the Banach space X let K(X) be the set of all compact
elements of [X]. If T K = KT for all K in K (fJ), then for some constant c,
T = c· id.
6.111. For Banach spaces V, Wand {Tn}nEN in [V, WJ, if, for all {v, w*}
in V x W*, sUPnEN I(Tnv,w*)1 < 00 then sUPnEN IITnl1 < 00.
6.112. For a Banach space V if T E VV resp. S E (V*)

and for all
{v, v*} in V x V*, (T(v), v*) = (v, S (v*)) then T E [V] resp. S E [V*]
and S = T*.
6.113. Any two C-Hamel bases for a vector space V are of the same
cardinality.
6.114. If V is an infinite-dimensional Banach space and H is a C-Hamel
basis for V then #(H) ~ c.

6.115. If H ~ {xAhEA is an lR-Hamel basis for an infinite-dimensional


Banach space V then at least one of the (linear) maps aA is discontinuous.
6.116. If T is a not necessarily continuous homomorphism of the Banach
space V into the Banach space Wand ker(T) ~f T- 1 (O) is closed then
true or false: a) T is continuous? b) T is open?
6.117. Let W be a closed subspace of the Banach space V. If x E V \ W
and a > 0 then for some z* in V*, Ilz* II :::; 1 and

(x, z*) > inf { liz - wll : w E W} - a ~f d(x, W) - a.

6.118. If V is a Banach space, there is no involution

#: V 3X~x# E V
Problems 6.119-6.131 85

such that: a) (x#)# = x (# has period two); b) (ax + bx)# = ax# + by#
(# is conjugate linear); c) for all {x,x*} in V x V*, (x#,x*) = (x,x*).
6.119. If V is a Banach space and M c E* then (M1-)1- is the weak*
closure of span(M).
6.120. If M is a closed subspace of a Banach space V then (M1-) 1- = M.
6.121. For some Banach space V, V* contains a II II-closed subspace M
such that M ¥
(M1-)1-.

6.122. a) If K is a convex subset of a Banach space and K is II II-closed


then K is weakly closed. b) If K is a convex subset of a Banach space then
the II II-closure and weak closure of K are the same.
6.123. If W is a finite-dimensional subspace of a normed vector space V
then W is closed and [V, Wl e contains an idempotent P.
6.124. If 1 is a linear but not necessarily continuous map of a Banach
space V into C then either ker(f) ~f 1-1(0) is closed or ker(f) is a dense
proper subset of V (cf. 6.116).
6.125. If a) V and W are Banach spaces, b) M is a subspace of W,
c) dim (W/M) < 00, and d) for some T in [V, WJ, T(V) = M then M is
closed. The conclusion can fail if d) is dropped.
6.126. If V is a Banach space then:

a) {V* is separable} =? {V is separable} ;


b) {V is separable} =fr {V* is separable} .

6.127. If V is a Banach space and V* is separable then the a-algebra


generated in 2v ' by the II II-open sets of V* is the same as the a-algebra
generated by the weak* open sets of V* .
6.128. If V is a separable infinite-dimensional Banach space then V
contains a dense linearly independent subset.
6.129. Let V be a Banach space containing a sequence {vn}nEN such that
for all v* in V*, 2:::~=ll(vn, v*)1 < 00. True or false: if {an}nEN is in Co (N)
then {2:::;;'=1 an Vn } NEN is a II II-convergent?

6.130. If {xn}nEN is a dense subset of B(O, 1) in the Banach space V then:


a) T : £1 (N) 3 {bn}nEN ....... 2:::~=1 bnxn is in W
(N), vt;
b) £1 (N) /ker(T)
endowed with its quotient norm is isometrically isomorphic to V.
6.131. If V is a Banach space, T E [VJ, IITII < 1, and y E V then the
equation x = Tx + y has a solution.
86 6. Topological Vector Spaces: Problems

6.132. What is the adjoint B* of

Although II(B*tll == 1, for each x, (B*t x II.!' O.


6.133. If {Mn}nEN is a sequence of closed subspaces of a Banach space V
and M ~ UnEN Mn is a closed subspace of V then for some no, M = Mno'
6.134. If V is a normed vector space over C then V is a Banach space iff

(6.2)

6.135. For a II II-compact subset A of a Banach space V and K the


II II-closure of the convex hull Conv(A) of A: a) K is II II-compact; b) for
x* in V*, I(x, x*) I IK achieves its maximum value on K; c) for each x in
K there is defined on S(A) a Borel measure f.lx, such that IIf.lxll = 1 and for
all x* in V*, (x, x*) = fA (y, x*) df.lx(Y).
6.136. For Banach spaces V, W, Z and a bilinear map B : V x W f--t Z, B
is (jointly) continuous: a) iff for some constant C, IIB(v, w)1I :::; Cllvllllwll;
b) iff B is continuous separately in each argument.
6.137. If V and Ware Banach spaces, f E W V , and df == 0 then f is a
constant.
6.138. For a Banach space V, a linear map T : V f--t V is continuous iff

(6.3)

6.139. If V is a normed vector space, x* E V*, and M = ker (x*), then


d(x,M) 'lIx*11 == I (x,x*) I·
6.140. If B ~f {xn}nEN is a Schauder basis for a Banach space V then:
a) the maps x~ : V 3 x f--t an(x) are in V*; b) {xn,x~}nEN is a maximal
biorthogonal set in V x V*; c) BN E [V), BF. = BN; d) Pn E [V], P~ = Pn ,
n E N; e) there are constants Band P such that IIBNII :::; B, N E N,
IlFnll :::; P,n E N.
6.141. If {xn,x~}nEN is a biorthogonal set then for N in N, the maps
BN : V 3 x f--t 2:;;=1 (X,X~)xn are in [V]. Furthermore, X ~f {xn}nEN is
a II II-basis iff L ~f span ({xn}nEN) is dense in V and for some M,

(6.4)
Problems 6.142-6.148 87

6.142. The set X ~f {xn}nEN is a basis for a Banach space V iff: a) no


Xn is 0; b) L ~f span (X) is dense in V; c) for some finite K

6.143. A a (V, V*)-basis X ~f {xn} nEN for a Banach space V is a II II-basis


for V.
6.144. If X ~f {xn}nEN is a basis for a Banach space V then for some
sequence {En} nEN in (0,00),

{llYn - xnll ~ En, n E N} ::::} {{Yn}nEN is a II II-basis for V}.

6.145. For some infinite-dimensional normed vector space V, there is a


countable lR-Hamel basis.
6.146. Let {xn}nEN be a Schauder basis for a Banach space V. For
t ~f L:'=l En (t)2- n (the binary representation of tin [0,1]) and

C(X) ~ {t : t E [0,1], ~En(t)(X'x~)xn II II-converges }:


a) C(x) is Lebesgue measurable; b) C
~f nXEV C(x) is Lebesgue measur-
°
able and dense in [0,1]; c) A(C) = or A(C) = 1; d) A(C) = or C = [0,1]. °
6.141. Let k be a measurable function in lRlRn and for a in (0, n), some c,
and all x, assume Ik(x) I ~ cllxll a - n , n = 1,2. For the map

K: Coo (lRn,lR) 3 1 f---* k * I,


b positive, and Y fixed: a) if 1 - a/n < l/q then

[ Ik(x - yW dAn (x) ~f Mq < 00;


JB(O,b)

b) if 1- a/n < l/q < 1 then K has a unique extension to L1 (B(O, b), An),
and the extension is in [L1 (B(O, b), An), Lq (B(O, b), An)].
6.148. For a positive let Ea be

{ I: 1 E ee, 1(0) = 0, IIU)II ~f sup 1/(8) -


S#t Is -
l(t)1 <
tl a
oo} .
88 6. Topological Vector Spaces: Problems

As a function space, (Ea, II II) is a Banach space.


6.149. For

F ~f {I : IE e[O,I], 1(0) = 0, 11111 ~f sup II(s) - l(t)1


si-t Is - tl
< oo}
and lilli' ~f 1111100 + 11111, there is no constant K satisfying lilli' ::::; KIIIII
for all I in F.

6.150. The set K ~ {I : IE iC[x], deg(f) ::::; k, fol II(x)1 dx ::::; 1 } is a


II II-compact subset of Ll ([0,1], oX).
6.151. Both Ll (JR, oX) n Co (JR, q and (Ll (JR, oX)) ~ are II lloo-dense in
Co (JR,q.
6.152. For m in N, let Pm be {p : p E JR[x], deg(p) ::::; m}. If

is a linear functional such that L (p2) > 0 when p -I- 0 and deg(p) ::::; n,
then for some Pn+1 in Pn +1: a) for all q in Pn , L (Pn+1 . q) = 0; b) Pn+1 has
n+ 1 different real zeros; c) if a) and b) obtain for some Pn+1 then Pn+l is
a constant multiple of Pn+l.

6.153. By abuse of notation, V ~f L! ([0,1], oX) is a topological vector


space with respect to the weakest topology making the map

V 3 I...... 1
o
1 1
II(x)12 dx
def
= 1IIIIi
!

continuous. It follows that V* = {O}.


6.154. For (X,Sj3,p,), (L2 (X,p,)r normed according to
1

II{II, ... , In}11 ~f (~II!kII~) 4

is a Banach space V. What is V*?


t
6.155. If V is a Banach space and T E [V, £1 (N) then: a) for some P in
[V], p2 = P and P(V) = ker(T); b) for the product topology of the direct
sum, F : ker(T) EEl {(id - P) (V)} 3 {x, y} ...... x + y E V is a continuous
open map.
6.156. If V is a normed vector space, {ydr'=1 is a finite subset of V, and
M is a II II-closed subspace of V then span ({ydr'=1 ,M) is closed.
Problems 6.157-6.162 89

6.157. a) If {an}nEN E £1 (N), for (X,S,/L), {/n}nd~ C L 1 (X,/L), and


s
II/nl11 M < 00 then F : [0,1] 3 x f-+ L::'=1 an In (x) is in L1 ([0,1], A).
b) For what pairs ({an}nEN' {fn}nEN) in C N x (L1 ([0, l]A))N is
00

11F111 = L
n=l
lanl'll/nl11 (6.6)

valid?
c) For what {an}nEN does (6.6) hold for each sequence {fn}nEN con-
tained in £1 ([0,1]' A)?
d) For what sequences {fn}nEN does (6.6) hold for each {an}nEN in
W (N))?
6.158. If V and W are separable infinite-dimensional Banach spaces, there
is an isomorphism T : V f-+ W. [Absent further hypotheses, T need not be
continuous. If V and W are Hilbert spaces, some such T is continuous.]
6.159. The set

V ~f {I : IE C ([0, 1], 1R), fa! I(x) dx - hI(x) dx = 1 }


1

is a II lloo-closed convex subset of C ([0,1], 1R) and no element in V is of


minimal norm.
6.160. If E is a locally convex topological vector space and U is a convex
circled neighborhood of 0 then the Minkowski functional Mu is a seminorm.
6.161. If p is a seminorm for a topological vector space E then the
following statements are equivalent: a) p is continuous at 0; b) p is uni-
formly continuous; c) U ~ {x : p(x) < I} is a convex, open, and circled
neighborhood of 0; d) p is the Minkowski functional of U.
6.162. (Kolmogorov) A topological vector space E has a norm-induced
topology iff E contains a bounded and convex neighborhood of O.

6.4. Banach Algebras


Conventions

A Banach algebra is a Banach space (A, II II) and a C-algebra in which mul-
tiplication A 2 3 {x, y} f-+ xy E A is separately continuous. In consequence,
it may be assumed that IIxYl1 ::; Ilxllllyll.
An ideal f in A is a proper subalgebra that is A-invariant: Afuf A c f.
A regular or modular ideal f is one such that the quotient algebra AI f
contains an identity e (thus when ul f = e, u is identity modulo f). Ideals in
the category of Banach algebras are closed unless the contrary is indicated.
90 6. Topological Vector Spaces: Problems

When A is commutative, M is the set of regular maximal ideals of A and


the radical 'R(A) of A is the intersection of all its regular maximal ideals:
'R(A) = nMEM M.
A generalized nilpotent in a Banach algebra A is an x for which
1
limn-+ oo Ilxnlln = O. When A is commutative, 'R(A) is the set of gener-
alized nilpotents.
By definition, [A, q consists of the continuous nonzero algebra e-
homomorphisms.
When G is a locally compact group, the Banach space L1 (G,/L) is a
Banach algebra A( G) with respect to convolution as multiplication. When
X is a locally compact Hausdorff space, the Banach space Co (X, C) is a
Banach algebra A(X) with respect to pointwise multiplication.
A derivation D : A 3 x f-+ Dx is an element of [AJ that satisfies the
product rule for derivatives: D(xy) = D(x)y + xD(y).
The disc {z : Iz - al ::; r} in e is denoted D(a, r). For a region f! in
e, H(f!) is the set of functions f in en and holomorphic in f!.
6.163. If n is a bounded approximate identity for a Banach algebra A
then for each k in N, n k is also an approximate identity.
6.164. For the norm II 1100, the set A(U) of functions holomorphic in
U ~ {z : z E e, Izl < I} and continuous on {z : z E e, Izl ::; I} is a
Banach algebra with respect to pointwise multiplication of functions. If
f E A(U) and 9 ~f fhr then: a) an
= 0 if n < OJ b) for some sequence
{Pn}nEN of polynomials, Pn ~ f on D(O, 1).
6.165. For A (D(O, 1)°) as in 6.164, if h E [A, q then for some Zh in
D(O, 1), h(f) = f (Zh)'
6.166. Let AH be the same set as A (U) in 6.165 but as a Banach
algebra with respect to convolution as multiplication:

f * g(z) ~f r
i[o,zj
f(z - w)g(w) dz,

and with norm 111100' Each f in A(*) is a generalized nilpotent.


6.167. If X is a compact Hausdorff space then for each h in [A(X),q
and some Xh in X, h(f) = f (Xh).
6.168. a) If G is abelian and a E G then

h: A(G) 3 f f-+ fa f(x)(a, x) d/L(x) E e ~f f(a)


is in [A(G), q.
b) If hE [A(G), q for some a in G, h(f) = f(a).
Problems 6.169-6.172 91

6.169. The algebra A(G) has an identity iff G is discrete.


6.170. If I is a regular ideal in A then the closure I of I is also an ideal.
6.171. For G the locally compact group JR, what is an example of an ideal
I dense (whence not closed) in A(G) (= L1 (JR,>.))?
6.172. Let A be a commutative Banach algebra with identity. If D is a
continuous derivation on A then D(A) c R(A).
COMPLEX ANALYSIS:

PROBLEMS
7
Elementary Theory

7.1. Geometry in C
Conventions

For each subset S of C, the set of complex conjugates of elements of S


is denoted S. On the other hand, the topological closure of S, i.e., the
intersection of all closed sets containing S, is denoted SC.
The symbol 0, with or without subscripts, denotes a region, Le., a
nonempty connected open subset of C. Concordantly, H(O) denotes the
set of functions mapping 0 into C and holomorphic in 0; in particular,
H (C) ~f £, the set of entire functions; when OC is compact,

A(O) ~f H(O) n C (OC, C) .


For a in C and r in [0,00), D(a,r) ~ {z : zEC,lz-al ~r} and
Ca(r) ~ aD(a, r); in particular, U ~f D(O, 1)° and 'f = Co(I).
The symbol z denotes both a complex nwnber and the geometric object
that z represents as an element of the Gau13iari plane. If ZI, Z2 E C are two
points, then {ZI, Z2} ~f {z : Z = ZI + t (Z2 - ZI) , t E 1R} is the straight
line they determine and [ZI, Z2] ~ {tZl + (1- t)Z2 : 0 ~ t ~ I} is their
convex hull.
The lexicographic partial order ~ of C is defined according to

{ZI ~ Z2} {:} {{lR (ZI) ~ lR (Z2)} 1\ {$S (ZI) ~ $S (Z2)}} .


A topological space X is simply connected iff every closed curve 'Y such
that 'Y* C X is homotopic to a constant in X. Although a simply connected
space X need not be connected, e.g., when X = U U (2 + U), unless the
contrary is stated, only simply connected regions in C are considered.
When ! ~f u + iv E CC and 'Y(t) = pet) + iq(t), 0 ~ t ~ 1, the symbol
J-y ! (z) dz means that the Riemann-Stieltjes integrals
11 (u (P(t), q(t)) + iv (P(t), q(t))) dp(t) ~f A
11 (u (P(t), q(t)) + iv (P(t), q(t))) dq(t) ~B
exist and J-y fez) dz ~f J; !('Y(t)) d'Y(t) ~ A + iB.
95
B. R. Gelbaum, Problems in Real and Complex Analysis
© Springer-Verlag New York, Inc. 1992
96 7. Elementary Theory: Problems

The extended complex plane Coo, intuitively the construct CL:J{ oo}, is
the set E ~f {(e, T], () : T], ( E JR,e, e
+ T]2 + (2 = I}. Furthermore the
set E \ {(O, 0, I)} may be identified with C according to the stereographic
projection: e :E \ {(O, 0, I)} 3(e, T], () 1---+ (1 ~ (' 1: () E C while
(0,0,1) is identified with 00. The center of the stereographic projection is
(0,0,1) and e maps (e, T], () in E \ {(O, 0, In
into the intersection of the
line through (0,0,1) and (e, T], () and the plane

{(a, b, 0) : a, bE JR} ~f C.

If Zi, 1 ::; i ::; 4, are four elements of C, the number

is their cross ratio or anharmonic ratio. For Z2, Z3, Z4 fixed and pairwise
different, X (z, Z2, Z3, Z4) ~f M(z) is a function on Coo \ {Z3} and may be
extended to Coo by defining M (Z3) to be 00. There are constants a, b, e, d
az + b def
such that M(z) = - - d = Tabed(Z) and
ez+

def az + b
More generally, when ad-be = ~ =f:. 0, the map Tabed : C 3 z 1---+ --d
ez+
is a Mobius transformation. By definition,

Tabed ( -~) = 00 and Tabed(OO) = ~.


Correspondingly, there is Tabed ~f e-1Tabede : E \ {(O,o, 1---+ E, which In
may be extended by continuity to a self-map of E. When ambiguity is
unlikely, the subscript abed is dropped. Note that if 0: =f:. 0 then

T(aa)(ab)(ae)(ad)(Z) = Tabed(Z)

whence if 0: = ;X then (o:a)(o:d) - (o:b)(o:e) = 1, and as the need arises


the value of ~ may be taken to be l.
Each Mobius tranformation T is invertible. The inverse of Tabed is
-dz+b .
Tab!d : C3 z 1---+
ez - a
= T( -d)be( -a). Thus each T IS one-one:

{T(z) = T (z'n {:} {z = z'}.


Conventions 97

The set of all Mobius transformations is denoted Mo.


2
When r > 0, z '# a, and k ~f 1 r 1 ' the point zP ~f a + k(z - a) is
z-a 2
the reflection of z in Ca(r) and z is the reflection of zP in Ca(r): z = (zP)P.
If 0 E [0,211'), the image ,* of, : IR 3 t I-t a + te i8 is a straight line
La(O) in C. If z fj. La(O) the reflection of z in La(O) is zP ~f a + ei8 (z - a)
and z is the reflection of zP in La(O).
In Figure 7.1 there is a depiction of the geometry of the situation for
the circle Ca(r).

zP

Iw - zl2 =Iz - al . IzP - zl = Iz - al . IzP - al - Iz - al 2


= Iz - al . IzP - al - (r2 - Iw - z12)
Iz - al . IzP - al = r2.

Figure 7.1.

For the line Lo(O), the reflection zP of z is z and for La(O) regarded as
a mirror, zP is the mirror image of z.
Thus the superscript P is used as a generic notation for a map z I-t zP
performed with respect to some circle or line.
For f E H (D(a, R)O) and 0 < r < R, M(r; f)) ~f maxlzl=r If(z)1 and
mer; f) ~ minlzl=r If(z)l·
98 7. Elementary Theory: Problems

7.1. If K is a circle lying on ~ then e (K \ {(O, 0, I)}) is a circle or a


straight line.
7.2. a) What are the algebraic descriptions of the boundary and interior
of the triangle !:J.(pqr) having as vertices the noncollinear points p, q, r? b)
If p, q, E D(O, 1)°, p, q, 1 are noncollinear, and s lies in the interior of the

trIangle !:J.(pql) then R(s) = 1 _
def 11 - lsi::;
sl K(p, q) < 00.

7.3. If T I ,T2 E Mo and neither is the identity id then: a) TI(z) = Z


has at most two solutions in C (each such Z is a fixed point of T I ); b) T I ,
the extension by continuity of TI to all ~, has at least one fixed point; c)
if ZI is the only point fixed by TI and Z2 and Z3 are two points fixed by
T2 then TIT2 -=1= T 2T I ; d) if TI and T2 have the same fixed point(s), they
commute: TIT2 = T 2T I ; e) when aid l - blCI = 1 and CI -=1= 0, ITI(z)1 = Izi
iff ICI Z + dil = 1 (the equation of the isometric circle of T 1 ). What are the
center and radius of the isometric circle of TI?
7.4. If {ZI' Z2, Z3} resp. {WI, W2, W3} are two sets of three points in Coo
then for precisely one T in Mo, T (Zi) = Wi, 1 ::; i ::; 3.
7.5. If T E Mo then for any four p, q, r, s,
X(T(p), T(q), T(r), T(s)) = X(p, q, r, s).

7.6. For the family F of all lines and circles in C and for T E Mo: a) if
K is in Fthen T(K) E F; b) if zP is the reflection of Z in the circle Ca(r)
and T E Mo then T (zP) is the reflection of T(z) in T (Ca(r)).
7.7. Which Tin Mo map U onto U?
2
7.S. If z -=1= a then the reflection zP of z in Ca(r) is a + _ r _.
z-a
7.9. The elements of (z, z') in C;, are a pair of mutual reflections in C
resp. L iff X (z', ZI, Z2, Z3) = X (z, ZI, Z2, Z3) and the validity of the equation
is independent of the choice of the triple ZI, Z2, Z3.

7.10. Three points p, q, r are collinear iff p - q is real.


q-r
7.11. a) If 8 E Mo, and 8- 1 (0,00,1) = (Z2' Z3, Z4) then
X (ZI, Z2, Z3, Z4) = 8 (zd .
b) Four points p, q, r, s are co circular or collinear iff X(p, q, r, s) is real.
7.12. With respect to composition of transformations: a) Mo is a group
generated by the subset
1
To : C \ {O} 3 z 1--+ - ,Tab : C 3 z 1--+ az + b, a, b E C;
z
Problems 7.13-7.22 99

b) as a group, Mo is isomorphic to 8L(2, q, the multiplicative group of all


2 x 2 matrices M with entries from C and for which det(M) = 1; c) each
T in Mo is the composition of an even number of reflections.
7.13. If a, b, c, dE JR and ad - bc > 0 then Tabcd leaves {z : ':;S(z) > O}
invariant.
7.14. If Ca(r) and Cb(S) are two nonintersecting circles then for some T
in Mo, T (Ca(r)) and T (Cb(S)) are concentric.
7.15. For what values of () in [0, 21r) does limrioo ereiO exist?

7.2. Polynomials

7.16. If Pn(z) ~f L~:~(k + l)zk and 0 < r < 1 then for some no, if
n > no then Pn has no zeros in D(O,r).
7.17. If a E JR, b > 0, and for n in N, In(z) = z2n + az2n - 1 + b then In
has exactly n zeros with positive imaginary parts.
7.18. If I E H(U) and for each z in U and some n z in N, I(n z ) = 0 then
I is a polynomial.
7.19. If ao > al > ... > an > 0 then L~=o akzk ~f I(z) =I- 0 in D(O, 1).
7.20. For a polynomial p, an a in C, and E(a) ~ {z : p(z) = a}, the
number of components of E(a) does not exceed deg(p).

7.21. Ifp,qEClz] anddeg(p) =nthen: a) n~ {z # (p-l(Z)) =n}


is open; b) q(z) ~f Lp(w)=z q(w) is holomorphic in n.
7.3. Power Series
Conventions

The radius of convergence of the power series L~=o cn(z - a)n is Re.
For I in CC, Z (f) is the set of zeros of I, 8 (f) is the set of singu-
larities, P(f) is the set of poles, and E8(f) is the set of isolated essential
singularities of I. The statements Z(f) = Z(g) or P(f) = P(g) are to
be interpreted as meaning not only that the sets are equal but that the
corresponding multiplicities or orders are the same as well.
When w = re i9 , r > 0, and 0 ~ () < 21r then W Z ~f ez(ln r+i9) .

7.22. If L~=o ICn - Cn+ll < 00, limn -+ oo Cn = 0, and 0 < {j < 2 then
L~=o cnz n converges absolutely and uniformly in the compact set

K ~f {z : Izl ~ 1, Iz - 11 2: {j} .
100 7. Elementary Theory: Problems

7.23. If {fm}mEN C £, Jm(Z) = 2::=0 CmnZ n , {IJm(z)l}mEN is bounded


·
on every compact set, and 1lmm--+oo = Cn t h en ,",00
Cmn def L.Jn=O Cnz n converges
everywhere and represents an entire function J such that Jm ~ J on every
compact set.
7 . 24 . L.Jn=O CmnZ n ,m = 1, 2,1'f ,",00
ror ,",00
D
L.Jn=O Cln C2n Zn def L.Jn=O CnZ n then
= ,",00
Rc ~ RCl Rc 2 • What is an example for which Rc > Rc 1 Rc 2 ?

7.25. a) What are the Ck such that for z near 2,

b) What is Rc?
7.26. If J(z) = 2::=0 cnz n in D(O, R)O and for some k and some r in
(O,R), ICkl = M(r~f) then J(z) = CkZ k.
r
7.27. If !R (c n ) ~ 0, n E N, and Rc = 1 then 1 is a singularity of
J(z) ~f 2::=1 cnz n .
7.28. If J(z) = 2::=0 anz n , an ~ 0, Izl < 1, f' (1) exists, and in D(1, t)O,
J(z) = 2::=0 bn(z - 1)n then Ra ~ 1 + t.
7.29. If J(z) = 2::=ocnz n , Rc = 1, and S(f) n'lf' = P(f) then sUPn Icnl
is finite.
7.30. If, for k in Nand z in U, Pk(z) ~f (1 - z)k+l 2::=1 nkzn then
Pk E N[z] and deg (Pk ) = k.
7.31. If K ~f {J : J(z) = 2::=0 anz n , lanl ::; n + 1} then K contains a
sequence {In}nEN converging uniformly on every compact subset of U.
d f zk n zn+k
7.32. If L(z,n) ~ - 2:~=1 k +L -k-' n E N then: a) there is
n + 1- k=l
n2
Z2 L (z 2n)
a constant K such that IL(z,n)IID(O,I)::; K; b) 2::=1 n2 ' repre-
sents a function J in A(U) although the power series for J does not converge
absolutely at any point of 'If'.
7.33. If (X, S, J.-t) is a complex measure situation, 0 is a region in C, and
9 is a measurable function such that g(X) C C \ 0 then

is in H(O).
Problem 7.34 101

7.34. (Inverse function theorem) If 1 E H(U), 1(0) = 0 =I- /,(0), r E (0,1),


I(z) =I- 0 for z in A(O,O: r), and

g(w) ~f ~
21l"l
1 z/'(z) dz
Izl=r I(z) - w
(7.1)

then for some s in (0,00), 9 E H(D(O,s)O) and for all win D(O,s)O,

log(w)=w.
8
Functions Holomorphic in a Disc

8.1. General Results


Conventions

For I in H(U) and t in [0,271"),

f*(t) ~ {~mril I (re it ) when the limit exists


otherwise
is the radial limit function. The (Hardy class)

{ I: IE H(U), zEU
sup I/(z)1 < oo}

is Hoo.

8.1. In H(U) there is no I, g, or h such that:

{I (~)} nEN = {1,0,~, ~,o,~,~, ~,o, ... };


{g(~)LEN ={1,-~,~,-~, .. -};
{h(~)}nEN ={~,~,~,~,~,~, .. -}.
8.2. If IE H(U) and a) II (z2) I ~ I/(z)1 or b) !R(f) = (~(f))2 then I is
a constant.
8.3. nEN C H(U), and for all r in (0,1),
i:
If IE C(U, q, {In}

nl.!..~ lIn (rei/}) - I (rei/}) I = °


dO

then I E H(U).
8.4. If IE H(D(O,r)O) and n E N then for some 8 in (0,00) and some 9
in H (D(O, 8)°),6

{/(z) = (g(z))n} V {/(z) = z(g(z))n} V··· V {J(z) = zn-l(g(z))n}.

102

B. R. Gelbaum, Problems in Real and Complex Analysis


© Springer-Verlag New York, Inc. 1992
Problems 8.5-8.14 103

8.5. If I(z) = I::=oCnz n , limn-+ oo Icnl > 0, Rc = R < 00, and the only
singularity of 1 in C is a pole a of order 1 then limn-+ oo ~ = a.
Cn+l
8.6. If 1 E H(U) and J'(U) c D(O, M) then for some F in C(D(O, 1), c),
Flu= f.
8.7. ForA~f{ei8: 0::;a::;O<b::;211"},if/EH(U)nC(UUA,C)
and 1 (A) = {O} then I(z) == O.
8.8. If I, 9 E H(U), 1 is injective, 1(0) = g(O) = 0, and g(U) c I(U) then
for r in [0,1), g(D(O, r)) c I(D(O, r)).
1
8.9. If z E U then 1 _ z = II (1 + z2n) .
00

n=O
8.10. If, for z in U,

I(z) ~ II (1 + z2n) and g(z) ~ II (1 - z2(2n-l»)


nEN nEN

then I,g E H(U) and I(z)g(z) == 1 in U.


8.11. For no 1 in A(U), l(z)ly= ~.
z
8.12. If 1 E A(U) and I/(z)lly== K then 1 is a rational function.
8.13. If 1 E A(U)
and 0 ~ 1(1f) then for some r in (0,1), the number of
1
zeros N of 1 in U is 211"
rTf J' (re i8 ) .
10 1 (re i8 ) re,8 dO.
8.14. For I(z) ~ I::=ocnz n E A(U): a) I::=olcnI2 < 00; b) if z =
i8
re , 0 ::; 0 < 211", r < 1 then:

~
I(z) =
10[21<!R (e~t + z) 1 (e it ) d;
211" e,t - z

= 1- 21<
o
1 1- r2 .
e't dt·
211"1-2rcos(O-t)+r 2/ ( ) ,
[21<
~f 10 Pr(O - t)1 (e it ) dt;

c) for Pr , Poisson's kernel, there obtain:


1
- o·,
00 .
i. P.r (t) = -211" '"
~
rlnletnt >
n=-CX)

[271:
ii. 10 Pr(t) dt == 1;
iii. Pr(t) ~ 0 if 0 < 0, t E [0,11" - 0], and r i 1.
104 8. Functions Holomorphic in a Disc: Problems

[Note 8.1: Because Pr(t) is a periodic function with period 271",


i-iii above may be re-interpreted for Pr(t + 71") ~f ITr(t). The
interval of integration [0,271"] is replaced by [-71",71"] and iii becomes:
iii'. if 0 < 10 < 71" and t rt (-71"+ 10,71" - 10) then ITr(t) ~ 0 as r -+ 1.
In sum, {ITr}rE[O,l)' is, like the set {FN} NEN of Fejer's kernels, an
approximate identity in the sense of 6.44.]

8.15. If f E H (D(a, R)O) and 0 < r < R then

8.16. If f E H(U) and f(U) c U then 11'(0)1 ~ 1.

8.17. If f ~f U + iv E H(U) then f(k) (0) = k!~


7I"r
r11" u (reiO ) e-ikO dO.
10
2

~(J(z)) ~ u(r, 0) ~ M < 00

1211" 2(M - a)

m D(O,R)O then: a) a
def
= ~(co) = -1 u(r, 0) dO; b) Icnl ~ Rn .
271" 0 c

8.19. For f(z) ~f L::'=o cnz n in H (D(O, R)O) and injective on D(O, R)O,
when r in (O,R), L(r) resp. A(r) denotes the length of f(Co(r)) resp. the
area of f (D(O, r)). Then:

= 71" L
00

a) L(r) ~ 271"r 11'(0)1; b) A(r) n Icn l2 r2n; c) A(r) ~ 7I"r 2 I1'(0)1 2 .


n=l

8.20. If f E H(U) then 12(r) ~f J~1I" If (re iO ) 12 dO is a monotonely


increasing function on [0,1) (cf. 15.9).
8.21. If f E H(U), f(U) c D(O, 1), and f(z) = zn (L:~=n Ckzk-n) then
for z in U, If(z)1 ~ Izln.
8.22. If f, f- 1 E H(U), f(O) = 0, and n E N then for exactly n functions
gk, 1 ~ k ~ n, in H(U), f (zn) = (gk(Z)t, 1 ~ k ~ n.
8.23. If f E H(U), Xn ! 0, and f (xn) E R., n = 2,3, ... , then for each v in
z+, 1(")(0) E R

8.24. For S ~f Un} nEN C H(U) and r in [0,1), if J~1I" lin (re iO ) dO ~ 1 I
then S contains an L1 (U, A2)-Cauchy sequence.
Problems 8.25-8.31 105

8.25. For no function I in H(U), II (zn)1 -+ 00 whenever IZnl -+ 1.

8.2. Applications of Mobius Transformations

8.26. If I(z) = 1 + L::'=l cnz n , Rc = 1, and ~(f(z» > 0 for z in U then:


. 1 - Izl 1 + Izl .
a) III U, 1 + Izl ::; I/(z)1 ::; 1 -Izl; b) hmn->co Cn = O.

8.27. If I E H(U), Z(f) n U = {Zd~=l' and I(U) c U then: a) when


u, I/(z)1 ::; ITk=l
K I1- Zk I def
z - Zk K .
K < 00 and z E Z = ITk=ll<I>zk(z)l; b) lf K = 00
and limk->co IZkl = 1 then

I/(z)l::; II l<I>zk(Z)1 (8.1)


kEN

and either L:~l (1 -Izki) < 00 or 1=0.


8.28. (Blaschke) If {zn}nEN C U and L::'=l (l-lzni) < 00 then
co
B(z) ~f II sgn(zn) (-<I>zn(z» E H co . (8.2)
n=l

8.29. If I E H(U), I(U) c U, and for two points a, bin U, I(a) = a and
I(b) = b, then I(z) = z: U contains at most one I-fixed point.
8.30. a) If I E H(U) and I(U) C U then

1f'(z)1 1
1 - I/(z)1 2 ::; 1 - Iz1 2 •

b) If g(z) ~f L::'=o cnz n E H(U) and g(U) C D(O, R) then

8.31. If I E H(U), I(U) = U, and I is injective then I is a Mobius


transformation (cf. 7.7).
9
Functions Holomorphic in a Region

9.1. General Regions


Conventions

The sets {z : <;s(z) ~O} are denoted II±.

9.1. a) If f E H(n) and a E 0. for two points b, c in 0., f'(a) =


= f(b~ :(c).
b) For "( a Jordan curve and 0. a region containing "(* together with
the bounded component 0. 1 of C \ "(*, if f E H(n) and on "(*, If(z)1 == M,
a constant, then f is a constant or for some a in 0.1, f(a) = O.
9.2. For a region 0., if f, 9 E H(n) and

{f(a) = f(b)} =} {g(a) = g(b)}


then for some h in H(f(n)), 9 = h 0 f.
9.3. For some region 0. and some f in H(n), no sequence {Pn}nEN of
polynomials converges to f uniformly on compact subsets of n.
9.4. If 0. is simply connected, f, 9 E H(n), and k E C (f' (0.) x g' (0.), C)
then for some h in H(n), h' = k (f',g').

9.2. Regions 0. Containing D(O, 1)


Conventions

For the items in this Section it is to be understood that the region 0.


contains D(O, 1).

9.5. If f E H(n) and f (1l') c IR then f is a constant.


9.6. If f E H(n) and f has n simple zeros in U then 'iR(f) has at least 2n
zeros on 1l'.

9.1. If f, 9 E H(n) n C (n C, q,f (nC) U 9 (nC) c UC, 9 (80.) = 1l', and L


9
is bounded on 0. \ g-l(O) then If(z)1 :S Ig(z)1 on n.

106

B. R. Gelbaum, Problems in Real and Complex Analysis


© Springer-Verlag New York, Inc. 1992
Problems 9.8-9.19 107

9.8. If I in H{n), In{z) ~f I (zn) , n E N, and I{O) = 0 then: a)


limN-+(X) FN ~f limN-+(X) 2:;:'=1 In ~f F exists in D{O, 1); b) F' = 2::=1 I~.
9.9. If I E H{n) and I{D{O, 1)) c U then U contains precisely one a such
that I{a) = a.
9.10. If I E H(n) and I(U) = U then I(T) = T.

9.3. Other Special Regions


Conventions

When 0 ~ r < R < 00, the symbol A( a, r R) denotes the annulus


{z : r ~ Iz - al ~ R}.

9.11. If n = {z : ~(z) > O}, I E H{n), III ~ R, limdo I{r) = a, and


bEn then limr!o I(rb) = a.

9.12. If I E H (A(O, 0: R)O) ~f H{n) and ~(f(z)) ~ M on n then zero


is a removable singularity of I.
9.13. If I E H (A (0, rl : r2t) and I(z) ;f:. 0 then for some r in (rl, r2),
I{z) =F 0 on Co{r).
9.14. If I E H (A(O, r : R)O) ~f H(n), {Pn} nEN C iC[z], and Pn ~ I on
every compact subset of n then for some F in H (D(O, R)O), F(z)ln= I{z).
9.15. If

I E H (A{O, 1 : 2t) ~f H(n), I E C {n {I}, <C) , C \

lim II (rei/}) I ~ 1, 0 ~ B < 21T, and lim II (rei/}) I ~ 1, 0 ~ B < 21T,


rj2 r!l

then I(n) c Uc•

9.16. If {Lkh<k<n is a set of straight lines in ee, E ~ ee \ U~=l L k , and


IE H (E) n c«e,
C) then there is an entire function F such that FI~= /E.
9.17. For n ~f A(O, r : 1)°, if I E H(n) then for some M, n contains a
sequence {zn}nEN such that limn-+(X) IZnl = 1 and II (zn)1 ~ M (cf 8.25).
9.18. If n is convex, IE H(n), I is not a constant, and

~ (f'(z)) . ~ (f'{z)) =F 0
in n then I is injective on n.

9.19. If n ~f ee \ {O}, 9 E H(n), 9 is injective, and g(n) c n, then for


a
some a, g{z) = az or g(z) = -.
z
108 9. Functions Holomorphic in a Region: Problems

9.20. If {fn}nEN C H(O), fn ~ f on each compact subset of 0, and each


fn is never zero in 0 then f == 0 or f is never zero in O. There are examples
for each situation.
9.21. Assume {fn} nEN C H(O), for each a in 0, fn ~ f on compact
subsets of some neighborhood V(a), and f ¢. O. If b E 0 and f(b) = 0
then for each neighborhood V(b) contained in 0, and some n(V(b)) in N,
{n> n(V(b))} '*
{Z Un) n V(b) -I- 0}.

9.22. If 0 ~f {z : ~(z) > O}, f E H(O), and f(z + 1) == 2f(z) then


there is an entire function F such that F (z ) = f (z ) . In
9.23. Assume for 0 ~f {z : !S(z) > O}, f in H(O) n C (OC, C), z in
oc \ {O}, and M, r positive, that If(z)1 ~ Mlzl-r. True or false: a) for z in
0, f(z) = -21 .1
00
(f(t)) dt? b) if f is not a constant then
7l'Z -00 t - z

9.24. For f, 9 in H (A(O, r : 00)°) ~f H(O), if r < R ~ I~I then: a)

fCo(R) f(z - w)g(w) dw ~f f * g(z) exists and its value is independent of Rj


b) f * 9 E H(O)j c) if f, 9 are rational so is f * g.
9.25. If f E H(O), a is a zero of order k of f, and 9 has a pole of order
m at zero then h ~ 9 0 f has a pole of order km at a.
9.26. True or false: if f E H(D(a,r) \ {a}) then for some unique A in
C, f 'Y (f(Z) - ~)
z- a
dz = 0 for every closed rectifiable curve-image ,* in
A(a,O:r)?
9.27. If a E U, f E H(U \ {a}) n C (D(O, 1) \ {a}, C), and f(1f) C llHhen
for some A in C, some B in JR, and some entire function F,

f(z) =
F ( AZ2
(z
+ Bz +
+ a)(1 + az)
A) .
9.28. If {fn}nEN C H(O), for each z in 0, {lfnl}nEN is bounded in some
N(z), {adkEN C 0, limk-+oo ak = a E 0, and limn -+oo fn (ak) = 0, kEN,
then fn ~ 0 on every compact subset of O.

9.29. True or false: for E ~f U \ (-1,1) and f in H(E): a) if If(z)1 is


bounded in E then for some F in H(U), F(z)I1;= f(z)? b) if for some F
in C(U,C), F(z)in= f(z) then F E H(U)?
Problems 9.30-9.33 109

9.30. For some simply connected region 0 and some J in H(O), J (0) is
not simply connected.

9.31. For 0 ~f {z : I~(z) I < I} and some J in H(O),

lim
!R(z)->oo
J OR(z)) = 0

while
lim J(z)
!R(z)->oo

does not exist.

In one form, the maximum modulus theorem states that if V is a


bounded open set (not necessarily a region) and J E H(V) then IJ(z)1
has no local maximum in V. Hence if J E A(V) and for each a in oV,
limz->aIJ(z)1 ~ M then maxzEVc IJ(z)1 ~ M.
For a subset S of C, oooG denotes the boundary of S in Coo.

9.32. If J E H(O) and for each a in 000 0 ~f F, limz->aIJ(z)1 ~ M then


SUPzEO IJ(z)1 ~ M.
9.33. The open mapping theorem implies the maximum modulus theorem.
10
Entire Functions

10.1. Elementary Theory


Conventions

Unless the contrary is indicated, every function introduced in this Chapter


is entire. An entire function that is not a polynomial is transcendental.
When I in H(n) is injective, I is conformal [and 1- 1 E H (f(n))].

10.1. If limlzl-+oo I~~)I = 0 then I is a constant.

10.2. If 9 E (0,00)[0,00), limx -+ oo Ig(x)1 = 0, and

I/(z)1 s Izl'lg(lzl)l in C \ {O}


then I == O.
10.3. Ifrn E Nand limr -+ oo M(~ I) = 0 then IE C[z] and deg(f) S rn-l
r
(cf. 7.26).
10.4. If, for some k in Z+, some g, and all large Izl, I/(z)1 s Izkg(z)1 then
for some rational h, 1= hg.
10.5. If Ct, A> 0 and I/(z)1 s Alzl<> then I E C[z].
10.6. For a sequence {zn} nEN' what are necessary and sufficient conditions
that for each sequence w ~f {wn}nEN' and some I (depending on w),
I (zn) = W n , n E N?
10.7. If 1(lR.) c lR. and l(ilR.) C ilR. then = - I(z).
I( -z)
1O.S. For some g, h, g(lR.) U h(lR.) c lR. and 1= 9 + ih.
10.2. General Theory

10.9. True or false: a) for some transcendental I, I(c) = C? b) for some


I, I(c) = {z : ?R(z) > O}? c) for some injective I, I(c) ¥C.
10.10. An I is transcendental iff for some sequence {zn}nEN' Zn ---+ 00
and sUPnEN II (zn)1 < 00.

110

B. R. Gelbaum, Problems in Real and Complex Analysis


© Springer-Verlag New York, Inc. 1992
Problems 10.11-10.21 111

10.11. If f is transcendental then for some sequence {zn}nEN' IZnl i 00


and for each p in C[z], limn-+ oo f (zn) p (zn) = 0.
10.12. If 9 E C[z], FE G(C, C) and F(g) = f then FEe.
10.13. If p, q E C[z] and e!(z) + p(z) = eg(z) + q(z) then p = q.
10.14. If f is not a constant then (j3 + j2) (C) = C.
10.15. If a E (0,00) and Ea ~f {z : If(z)1 < a} then: a)

E~ = {z : If(z)l::; a};

b) every bounded component G of Ea meets Z(f).


10.16. If fez) = fez + 1) then for some sequence {cn}nEZ'

L
00

fez) = cne211"inz,
n=-(X)

the series converging to fez) uniformly on compact sets.


10.11. If fez) = fez + 1) = f (z + v'2) then f is a constant.
10.18. If f is not a constant then for some curve 'Y : [0,1] 3 b--+ 'Y(t) E Coo,
'Y(1) = (0,0,1) and If b(t)) I i 00 as t i 1.
10.19. If mEN and fez) ~f /ro,z] exp (w 2m ) dw then f is not injective
and fCC) = C.
10.20. If e!(z) + eg(z) == 1 then f and 9 are constants.
10.21. If Z(f) = 0 then either fez) = ce Z or fez) +e Z has infinitely many
zeros.
10.22. If a,b are two points in C and both f-1(a) and f-1(b) are finite
nonempty sets then f E C[z].
10.23. If f is injective then f E C[z] and deg(f) = 1.
°
10.24. If fCC) = C and f' -# then f is injective.
[ Note 10.1: When fez) = eZ then f is not injective although
f' -# 0.]
10.25. If #(Z(f)) is finite then for some p in C[z] and some g, f = pg.
10.26. fiue or false: a) if 1~(f)1 is bounded then If I is a constant? b) if
f E H(U) and 1~(f)1 is bounded then If I is bounded?
10.21. Let L1 and L2 be lines in C and assume that f- 1 (L 1) C L 2. If
Ai, Bi are the two components (half-spaces) of C\Li , i = 1,2 then: a) for an
112 10. Entire Functions: Problems

appropriate choice of labels, 1 (A 2) = Al and 1 (B2) = B l ; b) 1 (L 2) = L l ;


c) for some constants a, b, I(z) = az + b.
10.28. Is there a nonconstant 1 such that for all z, ~(f(z)) =1= (SS(f(z)))2?
10.29. If Z(f) n Z(g) = 0 then for any h and some a, (3, al + (3g = h.

10.3. Order of Growth


Conventions

When I IS . and not a constant, p (I)


· entire =
def -1'
Im r- HXl
InlnM(r; I) is t h e
Inr
order (of growth) of I. When 0 < p(f) < 00,

is the type of its order. Thus 0 :::; p(f), r(f) :::; 00.

When 0< lanl i 00 and L::'=l la:lr < 00,

v(a) ~f inf {r : r E JR, ~ Ia1 I


n
r
< 00 }
resp.

o(a) ~f sup { m : mE Z+, ~ Ia1Jm = 00 }

is the exponent of convergence resp. the exponent of divergence.


The function

l-Z lip=O
E(z,p)~f { (l-z)exp [L:1=1 z:] ifpEN
,pEN

is useful in the discussion of WeierstraB factorization theorem, viz.:


If IZnl i 00, 1 E £, and Z(f) \ {O} = {zn}nEN and IZnl i 00 then
for some gin £, some sequence {Pn}nEN contained in N, and some
A in Z+, I(z) = zAeg(z) n:'=1 E (z: ,Pn) .

10.30. a) If I(z) ~f L::'=o cnz n and 0 < r < Rc then for some vf(r)
(the central index) in Z+, IC Vf (r)rVf(r) I ~f /-Lf(r) ~ lenlrn,n E Z+, and
li k > vf(r) then ICvf(r)lrVf(r) > Icklrk. b) vf(r) is a right-continuous
Problems 10.31-10.39 113

monotonely increasing Z+-valued step-function. c) If J is transcendental


then vf(r) i 00 as r i 00. d) If a> 0 and g(z) ~f J(az) then p(g) = p(J).
10.31. For the situation in 10.30c): a)

p(J) = inf {>. : IJ(z)1 < exp (lzIA) } ~f w(J);

b) if 0 < p(J) < 00 and

K(J) ~f {k : k > 0 and for large r, M(r; 1) < exp (kr P )} =1= 0

then r(J) = inf K(J) ~ v(J). c) p(J) = limr -+ oo In 17 J.tf(r) ~f ((J).


nr
10.32. a) If p, r > 0 then eTZP is of order p and of type r of that order.
b) ee z is of order 00. c) If Iql < 1 then J(z) ~f L::=o qn 2 zn is of order zero.
d) If a> 0, Fa(z) ~ f: (!.':.. ) -~
n=l ae
zn is of order a and of type one of that
order.

10.33. What is vf(r) for: a) J(z) ~ L::=o zn? b) J(z) ~ L::=o (:!~a
when a > O?
10.34. a) p(J) = a ~f inf {a : a ~ 0,limr-+ooIJ(z)lllz1=r::; e 1z1a }. b) If
J(z) ~ L::=o cnz n then, by abuse of notation when Cn = 0,

-.- nlnn
e;
() def
p J = hmn-+oo -In Icnl =
1 - e.ill
c) If p(J) < 00 then r(J) = ep(J) limn-+oon lenl n •
zn
10.35. When J(z) = L::=l (n!)a what are p(J) and r(J)?
10.36. If kEN and g(z) = zkJ(z) then: a) p(g) = p(J); b) p(J) = p(J');
c) r (J') = ep(f)r(J).
10.37. How are a) p(J + g), b) p(Jg), and c) p(J 0 g) related to p(J) and
p(g)?
10.38. If J(z) ~f L::=o cnz n is entire then, by abuse of notation when
- n
en = 0, limn-+ oo -In Icnl = O.

10.39. If In(z) ~ J (zn) , n E N, what is P (In) resp. r (In) in terms of


p(J) resp. r(J)?
114 10. Entire Functions: Problems

10.40. If {an}nEN C (0,00) then for some f, limn --+ M(n; f) = 00.
DO
an
10.41. If a> 0, fez) = 2::'=0 cnz n , and If(z)1 ::; e 1zl " then lenl ::; n-;;;.

10.42. If Z(f) = {an}nEN ~f a and to > 0 then v(a)::; p(f).

10.43. For fez) ~f n:'=2 (1 + n(l: n)2 ), p(f) = 1and r(f) = O.


10.44. If p < s ::; p + 1 then for some A(s), IIE(z,p)1 < exp (A(s)lzIS).

10.45. For 'Y ~f limn --+ DO [ ( 2:~=1 ~) - In n] (Euler's constant) and

10.46. In the context and notation of the WeierstraB factorization theo-


rem: a) if Izl ::; 1 then IE(z,p) - 11 ::; IzIP+1; b) if the exponent of diver-
gence 8 (a) of a ~f {an}nEN is positive then, A denoting the order of the
zero of f at 0, fez) = zAeg(z) n:'=l E (:n ,8(a)).
10.47. (Hadamard) If the order of fin 10.42 is p then: a) if 8(a) = 0 the
exponential factors in the E (:n '0) can be omitted; b) in any event, the
function 9 may be chosen in qzl and so that deg(p) ::; [pl.

10.48. For fez) ~f sinz: a) p(f) = 1 and r(f) = ~; b)


e

sin7rz=7rz II E(~,I) (=7rZ IT (1- ~:)) ~ Jcz).


nE(Z\{O}) n=l

10.49. True or false: if 9 E C(JR, JR) then for some f,


If (lR(z)) I > Ig (lR(z))I?
11
Analytic Continuation

11.1. Analytic Continuation of Series


Conventions

When fh, O2 are two regions such that 0 3 ~f 0 1 n O2 =I 0 and Ii E


H (Oi) ,i = 1,2, while It (z) = h(z) in 0 3 then the function element
(12, O2 ) resp. (fl, 0 1 ) is an immediate analytic continuation of the function
element (h,O I ) resp. (12, O2 ). When Oi, 1 :::; i :::; n, is a finite sequence of
regions such that

and each (fH1, OiH) is an immediate analytic continuation of (Ii, Oi) then
(fn, On) is an analytic continuation of (h, 0 1 ).
[ Note 11.1: If 1 - k > 1 and z E Ok n 0 1 then Ik(Z) and Il(Z)
can be different.]
When "( : [0,1] :3 t ~ "((t) E U:=10i is a curve and, in the context
above, "((0) E 01, "( (ti) E (Oi n OiH) , 1 :::; i :::; n - 1, "((1) E On then In is
an analytic continuation of It from 0 1 to On along ,,(, by abuse of language,
along "(*, by further abuse of language, an analytic continuation from h
(to In).
If 0 is a simply connected region, I E H(O), and 0 tj. 1(0) then
k ~f ~ E H(O). If a, Z E 0 then for any rectifiable curve "( such that

"(* C 0, "((0) = a, "((1) = z,

I,), k (W) dw ~f K (z) is independent of the choice of "( and therefore defines a
Kin H(O). It follows that if eK = G then
(e K )'
~ = 7f' whence 1= l(a)e K .
For some unique () in [0,271'), I(a) = I/(a)le iO and thus I = eK+lnlf(a)l+iO.
Any of the functions K +In If(a) I+i(2m7l'+()),m E IE, are branches oflnl
in O. In particular, if 0 is a simply connected region not containing {O},
there are infinitely many branches of In z in O.
When 0 is a region, a E a~, r > 0, and I in H(O) admits an immediate
analytic continuation 9 from 0 to D(a, r)O then a is a regular point of a~.

115

B. R. Gelbaum, Problems in Real and Complex Analysis


© Springer-Verlag New York, Inc. 1992
116 11. Analytic Continuation: Problems

Otherwise a is a singular point of 80,. If each point of 80, is a singular point


then 80, is a natural boundary for I.

11.1. a) For I(z) ~f 2::'=1 nzn and N(a) an open neighborhood of a in


C \ {I}, there is a unique function element (fa, N (a )) that is an analytic
continuation of I from U. b) When a =f. 1 what are the cn(a) and Rc(a) in
I[-a] (z) = 2::'=0 cn(a)(z - a)n?

11.2. For I(z) ~f 2::'=1 ::, if 0, n U =f. 0, 0" U u 0, and UnO. are
simply connected, and {I} rt 0" there is an analytic continuation of I from
U along any curve in U U n.

11.3. The natural boundary for I(z) ~f 2::'=1 zn! E H(U) is 11.'.
11.4. a) (Ostrowski) Assume N 3 P1 < P2 < ···,0 < >. E JR, {qn}nEN E N,
and qn > (1 + ~) Pn, n EN. If

00

I(z) ~f LcnZ n , Rc = 1, n = O,Pk < n < qk, lk E N,


C
n=l

and 1 is a regular point of I then the sequence {SPk hEN of partial sums of
the series converges in some nonempty neighborhood of 1.
b) (Hadamard) In the notations and conditions of a), if

then 11.' is a natural boundary for I.


[ Note 11.2: In a) although the series representing I diverges
for each z outside D(O,I), a sequence of partial sums of the se-
ries does converge in some nonempty neighborhood of 1: there is
overconvergence. By contrast, any sequence of partial sums of the
series in 11.3 diverges at each point of 11.'.

In a), the sizes of the gaps [the sequences of successive zero


coefficients] increase rapidly while the sizes of the nongaps [the
sequences of successive nonzero coefficients] may increase as well;
in b), the length of each nongap is 1.]
c) For a given series 2::'=0 cnz n for which Rc = 1, N contains infinitely
many pairwise disjoint sequences {{ m(p, q)} qEZ+ } such that 11.' is a nat-
pEN
ural boundary for each subseries 2::0 Cm(p,q)Zm(p,q), P E No
Problems 11.5-11.11 117

d) Let J(z) be represented by L:=l Cnz n, for which Rc = 1. For each


tin [0,1], let {En(t)}nE'll' be the sequence of binary markers for t. For all
but at most count ably many of the functions,

L En (t)CnZ n,t E [0,1]'


00

Jt(z) ~f
n=l
'JI' is the natural boundary.

11.5. a) If a E J[ then J(z) ~f L:-1


- 2n (z - 1e 2lI'ona
' ) is in both

H(U) ~f H (0 1 ) and H (C \ D(O, 1» ~f H (0 2 ),

b) The natural boundary for JIni ' i = 1, 2, is 'JI'.


11.6. a) The domain of convergence 0 of the series L:=l n(z ~ n) is
C \ N. b) For some F, S(F) nC = P(F) (F is meromorphic in C) and
F(z)ln= L:=l 1. (
(z ). c) When r E lR+ \ N, -2 F(z) dz = [r].
n z- n 71'Z Jizi=r

11.7. a) The domain of convergence 0 of the series L:=o Z2 ~ n 2 is


C \ (z \ {O}). b) The series in a) defines a meromorphic function J. c)
When r E lR+ \N, -2
1.
71'Z
1izi=r
-
[r]
J(z)dz - -2'

11.2. General Theory

11.8. If J E H(O), a E 0, and for some M,

then for some F in £, Fln= J.

11.9. Assume 0 :J 'JI', J E H(O), Cn = -1


def 1211' J (e ot. ) e- .
mt dt, nEZ, and
271' 0

r ~f limn->_ooicnli-,R- 1 = limn->oolcnli-. For some F in H(A(O,r: R)O),


Fln=l.
11.10. If 0 :J 'JI' and I E H(O) for some F in H(O U U), Fln= J iff for
each n in Z+, ITI(z)zn dx = 0.
11.11. Let 0 be a region containing the compact set K and let 0 \ K be
a region 0 1 . If {fn}nEN C H(O) and In ~ I on every compact subset of
0 1 then for some F in H(O), Fln , = f.
118 11. Analytic Continuation: Problems

11.12. If lEe (D(O, 2), C)nH (D(O, 2)° \ [0,1)) then I is in H (D(O, 2)°).

11.13. If 0 :::> Q ~f {z : O:S ~(z), ~(z) :S I}, I E H(O), I (aQ) c lR


then for some F in e, Fln= I.

11.14. For 0 = def{ z: -"27r < ~(z) < "2'~(z)


7r > 0 }.
,If

I E H (0) n C (OC ,C) and I (aO) c lR


then for some Fin e, Fln= I and F(7r - z) = F(z + 27r) = F(z).
11.15. Assume: a) IE H(U), sUPzEU I/(z)1 < 00, and for some positive
r, there is an analytic continuation Ir of I from U along any curve in
U U D(l, r)Oj b) for some positive 8, 11 - zl + I/(z)1 ~ 8 on U. For some
g, h in Hoo, (1 - z)g(z) + I(z)h(z) == 1 on U.

11.16. Assume I E H ( D (1, ~) 0) and that I may be continued analyt-


ically along every half-line L ~f (1,00) not passing through zero. For any
branch f3 of In z in 0 ~f C \ ( -00,0] and some G in e, the resulting analytic
continuation F of I along any curve in 0 satisfies: F(z) = G(f3(z)) in O.

11.17. If IE H(U \ {O}) ~f H(O) then for some F in

each D(a, r)O contained in 0, and some branch f3(z) of In z, F(f3(z)) = I(z)
in D(a,r)o.

The Problems 11.18 - 11.20 lead to some interesting properties


of the Gamma function r first mentioned in 10.45. Furthermore, they
simplify a derivation of the connection between r and the Zeta function

2::>-zlnn
00

((z) ~f
n=1
and they lead to a method for the analytic continuation of ( from its domain
0 1 ~ {z : W(Z) > 1} along curves in O 2 ~f C \ {I}.

11.18. a) r is meromorphic and per) = Z \ Nj b) for z in 0 ~ C \ P(r),


Problems 11.19-11.26 119

c) for z in n, r satisfies the functional equation

F(z + 1) = zF(z); (11.1)

d) if 9 is meromorphic, P(g) = P(r), and in n, g(z+l) = g(z) then h ~ gr


. r(z + n + 1) .
also satisfies (11.1); e) for each n III N, r(z) = n-1 ; f) for z III n,
IIk=o(z + k)
. n!n Z 7r
r(z) = hmn-+oo ( 1) ( ); g) r(z)r(1 - z) = -.-.
z z+ ... z+n SIll7rZ

11.19. Show: a) if ?R(z) > 1 then

r(z) = 1 00
e- t t z - 1 dt; (11.2)

b) if fi,i = 1,2, in (O,oo)(a,b) are log convex then It +gl is also log convex;
c) for x in (0,00), lnr(x) is a convex function.

11.20. Show: a) for n1 ~f {z : ?R(z) > I}, ( EH (nt) and for z in n1 ,


1 roo t z - 1
(z) = r(z) Jo eZ _ 1 dt. (11.3)

b) there is an analytic continuation of ( along any curve in n2 ~f C \ {I}.


11.21. If f E H(U) and for z in ~U, f(2z) = 2f(z)f'(z) then for some F
in E, Flu= f·
. 1 2f(z)
11.22. If f E H(U) and for z III "2 U, f(2z) = 1 _ (I(z))2' there is a
meromorphic function F such that Flu= f.

11.23. a) For some power series ~~=o cn(z - l)n ~f f(z), ef(z) = z in
D (1, 1) o. b) How is f continued analytically along any curve in C \ {O}?

11.24. a) For some power series ~~=o cn(z - l)n ~f g(z), g(z)2 = z in
D (1, 1) o. b) How is 9 continued analytically along any curve in C \ {O}?
11.25. (Poincare) If f E H(U), a E C, and N(a) is an open neighbor-
hood of a, there are at most count ably many pairwise different analytic
continuations (g, N(a» of (I, U).
11.26. Let n be a region. For each a in n let there be at least one
function element (fa, D (a, Tat) and assume that each function element is
an analytic continuation of every other one. Assume further that for each
C in ooon, lim z -+ c Ifa(z)1 ::s: M < 00. If a E n, sUPzED(a,ra)O Ifa(z)1 ::s: M.
120 11. Analytic Continuation: Problems

[ Note 11.3: Informally one describes a collection of function


elements conforming to the conditions in 11.26 as a regular func-
tion F: F is not single-valued (not a function) but for each value
w of F at a point a, there is a function element (fa, D (a, rat)
such that w = fa(a).

Thus 11.26 may be paraphrased as the maximum modulus


theorem for regular functions.]

11.27. For Pk,l::; k::; n, in (0,00), Udl<k<n a set of functions holo-


morphic in a region fl, assume that the hypotheses of 11.26 obtain for
the regular functions fk(Z)Pk, 1 ::; k ::; n. If ¢(z) ~ L:~=ll!k(z)IPk then
SUPzErl ¢(z) ::; Mn.
12
Singularities

12.1. General Theory


Conventions

Each function introduced in this Chapter is assumed to be differentiable


at all points except those specified as singularities. A singularity need not
be i""lated, e.g., the ,ingularity of . (1) at "'''".
SIn -
z
When J is defined in a region 0 and S(f) nO = P(f) nO, J is
meromorphic in O. The set of functions meromorphic in 0 is denoted
M(O).
When J E M(O) and a E P(f) no, the Laurent expansion for J near
a is, for some N in N, J(z) = E:'=_NCn(z - a)n. The sum

L
-1
cn(z - a)n ~ Pa(f)
n=-N

is the principal part or singular part of J near a. The residue at a is


Res(f, a) ~f C-1.

12.1. If a is a pole or an isolated essential singularity of J then a is not a


pole of e f .
12.2. If J E H(C \ {O}), zero is a simple pole of J, and J(1f) c lR then for
some a in C \ {O} and some b in lR, J(z) = az + a~ + b.
z
12.3. If {Ianl} nEN C (0,1), limn _ oo an = 0, J E H (U \ ({an} nEN U {O})),
and each an is a pole of J then C = (J (U \ {an} nEN) ~f E. r
12.4. If a is an isolated singularity of J and near a and for some m in N,
~(f(z)) ::; -mIn Iz - al then a is removable.

12.5. Is there in H (C \ {O}) an J such that for each positive r,

= -2
Jr(f ) def 1.
7n
1Co(r)
J(z)dz = 0

121

B. R. Gelbaum, Problems in Real and Complex Analysis


© Springer-Verlag New York, Inc. 1992
122 12. Singularities: Problems

and zero is: a) a pole of order not less than two? b) an essential singularity?
c) a pole of order one?
12.6. If I E H (U \ {o}) and
removable singularity.
fu I/(x + iy)1 2 dA2(X, y) < 00 then ° is a

12.7. a) For 0: in C and b in C \ {a},

b) If 0:=1= 0, k E z, (h(z) E [0,211"), and Ib(Z) ~f I/b(Z)1 eilh(z) then for z in


D(b, Ibl)°, exp Cn I/b(Z) I + i ~Ob(Z) + 2k7r)) = z. c)
S(f) = {0 if 0: E Z+
{o} otherwise·

d) Iff 0: E Z, there is a (Laurent) series L:=-oo cnz n ~ g(z) valid in


A(O,O: (0) and such that g(z) = Ib(Z) in A(O,O : (0) n D(b, Ibl)°. e) If
0: ¢. Z, S(f) (= {O}) is neither a pole nor an isolated essential singularity
of lb.

12.2. Meromorphic Functions

12.8. If D(O, 1) c n, I E M(n), and 1(11') c 11' then I is a rational


function.
12.9. If IE M (Coo) then I is a rational function.

12.10. If {an, bn}nEN C C and L~llan - bnl < 00 then IlnEN z - abn
z- n
converges in n ~ C \ ({bn}nENt and represents a function in M(n).

12.11. For n
~f C \ (Z \ H), if I E H(n) n M(C) and for z + 1 in A,
I(z + 1) = zl(z) then: a) every pole of I is simple; b) P(f) = Z \ H; c)
Res(f, -n) = (_1)n 1(1).
n!
12.12. If A c U and S ~f A- n U is empty or finite then: a) U \ A is a
region; b) if hE H(U\A), g E M(U), and Ih(z)1 :::; Ig(z)1 on U\A then for
some k in M(U), k(z)lu\A= h(z).

12.13. If P(f) n U ~f {Pn}nEN' limn->ooPn ~f Po E U, b E C, and


I E H (U \ {Pn} nEZ+) ~f H(n) then U contains a sequence {zn} nEN such
that limn->oo Zn= Po and limnEN I (zn) = b.
Problems 12.14-12.22 123

12.3. Mittag-Leffler, Runge, and WeierstraB Theorems


If 0 is a nonempty region, D(a,r) c 0, and J E H(O) then for some
sequence {Pn}nEN in C[z], Pn ~ J on D(a,r). The previous sentence is a
special case of Runge's theorem to which the following Problems lead and
from which Mittag-Leffler's and Weierstrafl's representations flow.
12.14. If F is a closed subset of C then C \ F consists of at most count ably
many components.

12.15. If K is a compact subset of C and P ~f {Pn}l<n<N<oo meets each


component of C \ K in one point then R, the set of rationa,l functions F
such that P(f) c S, is dense in C(K, C).
12.16. For some region 0 in Coo, Coo \ 0 consists of uncountably many
components.
12.17. If 0 E O(C) for some sequence {Kn}nEN in K(C): a)

b) each compact subset K of 0 is a subset of some Kn; c) each component


of Coo \ Kn contains a component of Coo \ O.
12.18. (Runge) If 0 is an open subset of Coo, and S meets each component
of Coo \ 0 in exactly one point, then for each J in H(O) and some sequence
{Rn} nEN of rational functions, P (Rn) C Sand Rn (z) ~ J(z) on every
compact subset of O.
12.19. (Mittag-Leffler) For an open subset 0 of Coo, if S cO, S· nO = 0,
and for each a in S, there is a rational function ra(z) ~ 2:N~~)
n
( Cn(a~
z-a n
then for some J in M(O), P(f) = S and Pa(f) = ra'
12.20. (Weierstrafl) If 0 is an open subset of C, S c 0, S· n 0 = 0 then
for some J in H(O), Z(f) = S.
[ Note 12.1: The open sets 0 in 12.19 - 12.20 need not be
regions.]

2
1
12.21. For z .
ill C \
Z, J(z) =
clef 7r
-.-2- = '"'"
L ( )2'
SIn 7rZ nEZ Z - n

12.22. For z in C \ Z, 7r cot 7rZ = ~ +L


Z ncJ.Z
(_1_ + .!.).
Z - n n
13
Harmonic Functions

13.1. Basic Properties


Conventions

When 0 is an open subset of C, h(x + iy) in C 2 (0, JR.) is harmonic iff


t1h ~f hxx + hyy = 0 in O. The operator t1 is the Laplacian and the set of
all functions harmonic in 0 is denoted L(O). When 0 is a region and h is
?R(f) for some I ~f U + iv in H(O) then v is an harmonic conjugate of h.
When 0 is open, an h in C 2 (0, JR.) enjoys the mean value property iff
{{a E O} 1\ {D(a, r) c O}} ~ {h(a) = ;7[" 127r h (a + re it ) dt}. The set
of all functions enjoying the mean value property (in 0) is MV P(O).

13.1. If 0 is open, 9 E H(O), and h E L(g(O)) then hog E L(O).


13.2. If 0 is open and h E L(O) then for each a in 0, some open N(a),
and some la in H(N(a)), h = ?R (fa) on N(a). ("Each harmonic function
h is locally the real part of a holomorphic function.")
13.3. If 0 is a simply connected region and h E L(O) then L(O) contains
an harmonic conjugate of h.
13.4. If 0 is a region and h E L(O), then any two harmonic conjugates of
h differ by a constant.
13.5. If 0 is a simply connected region, h E L(O), and 0 tJ- h(O) then
L(O) contains a pair {p, q} such that h = p2 _ q2.
13.6. If R > 0, 3 ~f oD(a, R), and 0 ::; r < R then for the complex
measure situation (3,5/3(3), /1-),

13.7. a) L(O) c MVP(O). b) If 0 is a region and I E MVP(O) then


the following principles of the maximum and the minimum obtain:
i. (Maximum Principle) If for some a in 0 and all z in 0, I(a) ~ I(z)
then I is a constant in 0;

124

B. R. Gelbaum, Problems in Real and Complex Analysis


© Springer-Verlag New York, Inc. 1992
Problems 13.8-13.17 125

ii. (Minimum Principle) If for some a in n and all z in n, f(a) ::; fez)
then f is constant in n.
c) If n is a region then h E MVP(n) iff hE L(n).

i
13.8. If a E U then for some measure situation (T, 5/3(T), /La) and for each
h in L(U) n C(D(O, 1), C), h(a) = h(z) d/La(z).

13.9. If n is a region and h, h2 E L(n) then h or Ii is in H(n).


13.10. If f ~ U + iv E H(n) and °~ fen) then In If(z)1 ~f h(z) E L(n).

13.11. If n is a region, f E H(n), and If I E L(n) then f is a constant.

13.2. Developments

13.12. (Schwarz) If h E L (D(O, R)O) n C(D(O, R), C) then

fez) ~ 1 w + z hew) dw E H (D(O, R)O)


Iwl=R w- Z W

and )R(f) = h.

13.13. Assume D(O, 1) c n, {fn ~f Un + iVn} C H(n), {Un}nEN


nEN
converges uniformly in D(O, 1), and {vn(O,O)}nEN converges. The sequence
{fn(z)}nEN converges uniformly on each compact subset of D(O, 1)°.
13.14. a) Assume h is harmonic in D(O, R)O, h(D(O, R)) c D(O, M),
and v, an harmonic conjugate of h, is such that v(O) = 0. If Izl :::; r < R
2M R+r . .
then Iv(z)1 ::; - I n - R . b) For some regIOn n and some h III L(n),
7r -r
SUPzEf! Ih(zl < (Xl while no harmonic conjugate of h is bounded in n.
13.15. (Harnack) If hE L (D(a, R)O)nC(D(a, r), JR) and h(D(a, R)) C JR+
. R - Izi R + Izi
then for z III D(a,R)O, h(a)· R+ Izi ::; h(z)::; h(a)· R-izi'

13.16. Jor27r In 11 - eitl dt = 0.


13.17. (Jensen) If R > 0, n ::l D(O, R), f E H(n), fen) =I- 0, and
Z(f) n D(O, R) = {Znh::;n::;N (listed according to multiplicities) then

In If(O)1 +~
N
In IIZnl
R I = 27r
1
Jo r 27r
In If (Re it ) I dt.
126 13. Harmonic Functions: Problems

13.18. (Poisson-Jensen) In the context of 13.17, if Z E D(O, R), then

13.19. a) If f E H oo and

In+t~f{lont ift:::::1
if t < 1
then

ior If (rei8 ) I dO < 00.


27r
sup In+
rE[O,l)
b) If f E H(U), Z(f) n U = {an}nEN (each zero of f is listed according to
its multiplicity), and sUPrE[O,l) J~7r In+ If (re i8 ) I dO < 00 then
L (1 -
00

lanl} < 00 or f == O.
n=l

13.20. (Harnack) For a region n


and a sequence {hn}nEN contained in
L(n): a) if h n ~. h, h E C(n, iC), and for every D(a, r) contained in n, there
is a function fa,r integrable on 8D(a, r) and dominating Ihn(z)llaD(a,r) then
hE L(n); b) if hn ~ h on every compact subset of n then h E L(n); c) if
h n i h then either for some h, h n ~ h on every compact subset of n (in
which case h E L(n)) or h == 00.
14
Families of Functions

14.1. Sequences of Functions


Conventions

A set F of functions in H (0) is normal iff every sequence in F contains a


subsequence converging uniformly on compact sets of 0 to a function [not
necessarily in H(O)]. When the ranges of the elements of F are regarded
in Coo and convergence is regarded as taking place in Coo, where 00 is a
legitimate limit, there is a corresponding notion of spherical normality.
For a set S of maps f : X 1-+ Y between topological spaces, a base
for the compact-open topology for S is the set of all sets of the form
{f : f(K) C O,K E K(X),O E O(Y)}.
When f E H(U), r E [0,1), and 1 ~ p ~ 00, then fr(t) = f
def "t)
( re~ and

Mp(r; f) ~f (2~ 127r Ifr(tW dt) f;


{ if 1~ p < 00
M(r; f) if p = 00.
The set of f such that, by abuse of notation,

Ilfllp ~ sup Mp(r; f) < 00


O~r<l

is the Hardy class HP. [The Hardy class Hoo is the natural counterpart of
Loo (X, Jl).]
For a region 0 and a positive 8"

06 ~f {z : z E 0, inf Iz -
wEan
wi ~ 8}.

14.1. If {fn}nEN c H(O) is normal then for some sequence {mn}nEN'


mn i 00 and {f;:'n} nEN is normal.
14.2. For some {fn}nEN in H(U), each fn maps U conformally onto U
and f n ~ 1 on compact subsets of U.

14.3. For which regions 0 is Fn ~f UnEN {f f(z)ln= zn} normal?

127
B. R. Gelbaum, Problems in Real and Complex Analysis
© Springer-Verlag New York, Inc. 1992
128 14. Families of Functions: Problems

14.4. For which regions n is Fn ~f {sinnz!n}nEN normal?


14.5. If 2 ::; q < 00 and limn->oo fu I/n(zW d>"2 = 0 then In ~ 0 on
compact subsets of U.

14.2. General Families

14.6. For any n, H(n) is a Banach space with respect to the norm II 1100
and the unit ball B(O, 1) is a normal family.
14.7. If 1 ::; p < 00 then: a) II lip is a norm for the vector space HPj b)
with respect to II lip, HP is a Banach spacej c) the unit ball B(O, 1) of HP
is a normal family.

14.8. If I(z) ~f L::'=o Cnz n E H2 then for some 1* in L2 ([0, 27r], >..): a)
111* - r
Irl12 -+ 0 as r Ij b) Ir ~. 1* as r l. r
14.9. a) If I E H2 and 1* is the function defined in 14.8 then

-f* = { Cn 1·f '77+


n E ILl
n 0 otherwise.

b) If 9 E L2 ([0, 27r], >..) and, for n in Z \ Z+, 9n = 0 then for some gin H2,
g* ~ g.

14.10. If L::'=o bnz n converges in U and bn 2: 0, n E Z+ then

is normal.

F~f {I : I E H(U), I(U) c U,/(O) = I (ak) = 0, 1 ::; k ::; n},

what is SUP!EF II'(O)I?

14.12. The family F ~f {I : I = L:~o ck(f)zk,J E H(U) } is normal


iff for Kk ~ sup!EFlck(f)I, limk->ooKt ~f K::; l.
14.13. Assume 0 E n, I E C(n x D(O, 1), q, for each w in n,

81(w,z)
I(w, z) E H(n), 8z E C(n x D(O, 1), q,

and for some n in N, 1(0, z) == zn. For each w in n and for some points
gk,w, 1 ::; k ::; n, not necessarily pairwise different, in U, I (W,gk,w) = O.
Problems 14.14-14.19 129

14.14. The family

F~f {I : IE H(U),j(U) C U,I(O) = 1'(0) = 1"(0) = O}

contains one and only one F such that F (~) = SUPfEFi' (~) ~f 1 0:.

14.15. Let S ~f {zn}nEN be a sequence of pairwise different points in


C and assume {qn}nEN C N. For some h in 'I, the set of entire functions
having at Zn a zero of order at least qn, 'I = h . E. (Thus 'I is a principal
ideal in the algebra E.)

14.16. If F ~f {I : IE H(U),j(U) C U} then for some 10 in F,


orR (fo) (0) 1_ 1orR(f) (0) 1
1 ox - SUPfEF ox .
14.17. The family F ~f {I : IE A(U), 1'][' If(z)1 dr :::; 1} is a normal
subset of H(U).

14.18. For a region 0, if F ~f {I : I E H(O), In


II(z) IdA2 :::; 1 } then:
a) for each a in 0 for some M (a) in ~, and for each I in F,

II(a)1 :::; M(a) 10 II(z)1 dA2;


b) Fis normal; c) Fis compact in the compact-open topology for H(O).
14.19. Assume 0 J D(O, 1), I E C(O x [0,1]' C), 0 ¢. I ('j[' x [0,1]), for
each t in [0,1]' I(z, t) E H(O), and Ol~:, t) E C(O x D(O, 1), C). a)

J(t) ~f 2~i i (Ol~:, t) /I(Z, t)) dz

is t-free; b) If 9 E C([O, 1], C) then Fg(z) ~f 1[0,1] I(z, t)g(t) dt E H(U).

14.3. Defective Functions


Conventions

For a region 0 and a subset S of C, a function I in M(O) is S-defective in


o iff 1(0) c (C \ S). A family F of functions in M(O) is S-defective in 0
iff UfEFI(O) c (C \ S). The symbol Fs is used to denote such a family.
The items below show in what (minimal) circumstances a defective
family is necessarily normal. The development is due to Landau [La] who
created a strikingly direct proof of Bloch's and Schottky's theorems. From
130 14. Families of Functions: Problems

these Landau derived the (great) Picard theorem, which may be phrased
as follows:
If r > 0, a is an isolated essential singularity of I, and 1 is 8-
defective in D(a, r)O \ {a} then #(8) ::::; 1.

14.20. If 0 is simply connected, 1 E H(O), and 1 is {a, l}-defective, there


is in H(O): a) a p that is Z-defective; b) a q such that p = q2; c) an r such
that p - 1 = r2; d) an s such that q - r = e8 •

14.21. For8~f{z: z=±ln(Vm+vm-1)+n;i,mEN,nEZ},


and in the context of 14.20d), s is 8-defective.
14.22. For 8 in 14.21, C \ 8 contains no translate of U.

14.23. For 1 E H(O) and 0:) D(O, R): a) 1(0) = 0; b) 11'(0)1 ~ a > 0; c)
II'(z)1 ::::; M in D(O,R); d) 1 is {-y}-defective in D(O,R)O then bl ~ ~i:.
14.24. (Bloch) If 0:) D(O, 1), 1 E H(O), and 11'(01 ~ 1 then for some a,
I(U) :) D (a, 116 ).
14.25. (Schottky) Assume 0 :) D(O, 1) and 1 E H(O). For r in [0,1) and
some positive cI>(f(O),r), if 1 is {O,l}-defective then I/(z)1 < cI>(f(0) , e) in
D(O, r).
14.26. If a, b are two numbers and 0 is a region in C then Fa,b is spherically
normal.
14.27. If the region 8 is a simply connected proper subset of C then for
any region 0, F~f {I : 1 E H(O), 1(0) c 8} is spherically normal.

14.28. For a region 0 and 9 ~f {I : 1 E H(O), 1(0) c II+}, the set


F ~f {e f : 1 E g} is spherically normal.

14.4. Bergman's Kernel Functions


Problems 14.29 - 14.34 below lead to Bergman's kernel functions, which
have practical applications in conformal mapping [Berl, Ber2, Hi]. The
overriding hypothesis is that # (aO) ~ 2.

14.29. For S) ~f L2 (0, '\2) n H(O): a) S) is II 112-complete; b) if K is a


compact subset of 0,5, and for 1 in S), II/IIK ~f SUPzEK If(z)1 then for some
constant MK, II/I1K ::::; MKll/lb c) the unit ball of S) is normal.
Problems 14.30-14.34 131

14.30. If {¢n} nEN is a complete orthonormal system in 5) and 1 E 5) then


~:=l (J, ¢n) ¢n(Z) ~ I(z) in 06.
14.31. If {¢n}nEN is a complete orthonormal system in 5) and tEO what

iSf-t~finf{11/1I2: IEspan({¢nh~n~N)~fMN,/(t)=l}?
14.32. For Bergman's kernel function K(z,w) ~ ~:=1 ¢n(z)¢n(w): a)
IK(z,w)12::; K(z,z)K(w,w); b) {w E O} '*
{K(w,w) > O}; c)

{J E 5)} '* {f(z) = In K(z, w)/(w) d>'2(W)}.

14.33. If wE 0, 1 E 5), and I(w) = 1 then 11/112 ~ II :~',~ t


14.34. If 0 is simply connected, 1(., w) is a conformal map of 0 onto
D (0, V 1 )0 ~f
7rK (w,w)
D(O R)O and 81(z,w)
" 8z
I
Z=W
1
= then

I(z,w) = K(w,w)
1 lW
z
K(s,w)ds.

(The integral is path-independent!)


15
Convexity Theorems

15.1. Thorin's Theorem


In such disparate contexts as the Hahn-Banach theorem, seminorms, Kol-
mogorov's theorem (cf. 6.162), von Neumann's theory of almost periodic
functions on groups (cf. 2.93), his theory of games [NM] and linear pro-
gramming, Jensen's inequality, Hadamard's three-lines/three-circles theo-
rems, the Hausdorff-Young theorem, probability theory, etc., the rOle played
by convexity is central. The following discussion attempts to illustrate that
rOle in complex analysis.

Conventions

When {Vkh<k<n is a set of vector spaces, Fk E IR[O,oo), 1 :::; k :::; n, and


G E 1R1R, ¢ hI iRV1X"'XVn is (G; Fb ···, Fn))-convex iff whenever 0 :::; tk,
I:~=l tk = 1, and Go¢ is defined then Go¢ (Vb"" Vn) :::; I:~=l tkFk (Vk).
[Note 15.1: In the context of Jensen's inequality as discussed in
Chapter 4, a function ¢ convex in the ordinary sense is (id; id, id)-
convex.
When

then Holder's inequality states that In f is (G; Fb F 2 )-convex:

1n Ii Xl (X)X2 (X) dj.£(x) I: :; ~ In II xlli p+ ~ In IIx211pl .


If hE MVP(D(a,r)O) then for n in N and s in [O,r),

rh (a + st) dr(t) ~ L ~h (a + se~).


n-l
= (15.1)
J
T k=O

132
B. R. Gelbaum, Problems in Real and Complex Analysis
© Springer-Verlag New York, Inc. 1992
Problems 15.1-15.5 133

Thus when "=" and "~" are read ":::;," (15.1) may be viewed as a
convexity property of h. By abuse of language, subharmonic func-
tions, i.e., upper semicontinuous JR-valued functions I for which

may be viewed as a class of convex functions.]


When X is a set, B, Dc X, and I E JRx then the maximum principle
in D relative to B obtains for I iff sUPxED I(x) :::; SUPxEB I(x).
For example, the maximum modulus theorem asserts, i.a., that if

D(a,r) c n and I E H(n)

then the maximum principle in D ~f D(a, r)O relative to B ~f oD(a, r)


obtains for III. For more details cf. Chapter 9.
Note that if DeB, the maximum principle in D relative to B obtains
for all I in JRx .

15.1. True or false: a) if a function I in JRIR is monotone or convex then for


every finite interval [a, b], the maximum principle in [a, b] relative to {a, b}
obtains for I? b) if, for every finite interval [a, b], the maximum principle
in [a, b] relative to {a, b} obtains for I in JRIR , then I is convex?
15.2. For a vector space V, a ¢ in JR v is convex iff for each A in JR, each
pair {x, y} in V 2 , and each t in [0, 1], the maximum principle in the interval
[x,y] ~ {z : z=tx+(I-t)y,tE [0, I]} relative to {x,y} obtains for
¢(tx + (1 - t)y) - At.

15.3. If each element of {¢AhEA is convex then ¢ ~ SUPAEA ¢A is convex.


15.4. If 9 in JRIR is a monotonely increasing function continuous on the
right and the maximum principle in D relative to B obtains for I then the
maximum principle in D relative to B obtains as well for 9 0 I.
15.5. (Thorin) Assume X is a vector space, C is a convex subset of X, Y
is a set, and I E JR xxY . If, for every line segment

[xo, Xl] ~f {xo + t (Xl - xo) : 0:::; t :::; 1 }

contained in C and every A in JR, the maximum principle in

[xo, Xl] x Y relative to (xo x Y) U (Xl X Y)

obtains for FXO,Xl,A : [0,1] x Y 3 (t, y) f--t f (txI + (1 - t)xo, y) - At then


M(x) ~f SUPyEY I(x, y) is a convex function of x.
134 15. Convexity Theorems: Problems

15.6. If Z is a vector space, K c IZ, Xl, C is a convex subset of X, and


n
z E LEK L -1 (C) ~f r then, in the context of 15.5,

M(z;K) ~f sup{F(L(z),y) : L E K,y E Y}

is convex on r.
I Note 15.2: If r = 0 the conclusion above is automatic. In any
event, since each L is linear and C is convex, r is convex.
The result asserts in particular that if M(z; K) is finite at
both endpoints of a line segment J lying in r then M(z; K) is
bounded above on J.J

15.2. Applications

15.7. (Hadamard's Three-Lines Theorem) If

o ~f (a,b) x JR, IE H(O) n C (oc,q, and 1(0) c D(O,K)

then In (M(x; f) ~f sup {1/(x + iy)1 : -00 < y < oo}) is convex.
15.8. (Hadamard's Three-Circles Theorem) If

IE H (A(a,r: Rt) n C(A(a,r: R),q


then for t in Ir, RJ, In M(t; f) is a convex function of In t.
I Note 15.3: The result 3.12 combined with 15.7 implies that
M(x; f) is itself a convex function of x; by the same token M(r; f)
is a convex function of In r. The greater strength of 15.7 is em-
phasized by 3.10: {f is convex} =fo. {In I is convex }.J

15.9. If IE H(U), r E (0,1), p E (0,00), and

Ip(r) ~f ~ r
27r io
27r
II (re it ) IP dt
then: a) on (0, 1), I P (r) is a monotonely increasing function of r; b) In I p ( r )
is a convex function of In r.
15.10. (M. Riesz) For a, f3 positive,
m n
B : em x en 3 (x, y) f--+ L L:>jkXjYk ~ B(x, y),
j=1k=1
Problem 15.10 (cont.) 135

and
Pj > 0, O'k > 0, 1 :-:; j :-:; m, 1:-:; k :-:; n,

P =
def (
P1,···, Pm ) , (T =
def (
0'1,···, an ) ,
let S be the set of all (x, y, p, (T) such that
m n
I>j IXjl~ :-:; 1, and LO'k IYkl! :-:; l.
j=l k=l

Then the logarithm of M(a, (3) ~f sup(x,y,P,lT)ES B(x, y) is a convex func-


tion on the quadrant Q ~f {(a, (3) : a, (3 > O}.
[Note 15.4: The M. Riesz convexity theorem has many conse-
quences. In particular it gives rise to the famous theorems of F.
Hausdorff, G.C. and W.H. Young, and F. Riesz. These theorems
apply in the context of a locally compact abelian group G and for
any pin (1,2] [We2, Zy). They relate the norm of a function f in
LP (G, C) to the norm of its Fourier transform 1, viz., for appro-
priately normalized Haar measures on G resp. {} and p' ~f -p-:
p-l

(15.2)

It is important to note that the condition p E (1,2] is essential.


When p > 2, there are examples for which (15.2) fails.]
REAL ANALYSIS:

SOLUTIONS
1
Set Algebra and Function Lattices

1.1. Set Algebra


Conventions

References to items in SOLUTIONS are tagged with the prefix s to


distinguish them from items in PROBLEMS.

1.1. If {An}nEN C M then BN ~f U:=l An E M and BN C B N+1 whence

lim BN
N.-oo
(= Un=l
An) E M.

1.2. As a monotone class, oR (R) :::> M (R). Hence it suffices to show M (R)
is a O"-ring.
For C in M and 8(C) ~f {D : C \ D, D \ C, CuD EM}:
a) {C E 8(D)} ~ {D E 8(C)};
b) 8( C) is a monotone class;
c) {D E R} '*
{R c 8(D)};
d) {D E R} '*
{M (R) c 8(D)};
e) {A E M (R)} '*
{M (R) c 8(A)};
f) M (R) is a ring (cf. 1.1). 0

1.3. For F in F(X), if N n ~f UyEF {x d(x, y) < ~ }, then for n in N,


M :::> O(X) 3 N n :::> Nn+l and F = nnEN N n E M whence F(X) C M. 0
1.4. From 1.3 it follows that M :::> F (JR) whence if a :.::; b, [a, b] E M.
Because M is monotone, [a,b) (= limn .- oo [a,b-~]) E M. Since a finite
union of closed intervals is closed, hence in M, any finite union of half-open
intervals [a, b) is in M, i.e., M contains the ring R consisting of all finite
unions of half-open intervals [a, b). From 1.2 it follows that M :::> O"R (R). If
a < b then (a, b) = lim n .- oo [a + ~,b) E O"R (R) c M, whence M :::> O"R (0).

139

B. R. Gelbaum, Problems in Real and Complex Analysis


© Springer-Verlag New York, Inc. 1992
140 1. Set Algebra and Function Lattices: Solutions

Because R E oR (O)naR (F), it follows that aR (F) = aR (0). The argument


thus far shows that the a-ring generated by the set of all bounded closed
intervals la, b] is aR (0) whence aR (0) = aR (F) ::) aR (K) ::) aR (0). 0
1.5. If S is neither empty nor a finite set of sets, and if S is a sequence
of pairwise disjoint members of S then the set E of all countable unions of
members of S is an uncountable subset of S.
First proof. By induction, define a sequence {En}nEN as follows:
i. E 1 , an arbitrary member of Sj
ii. if Sn ~f {El' ... , En} is a finite set of pairwise disjoint sets in S, there
is some set E in S \ Sn and among Ek \ E, E \ Ek, Ek 1 ~ k ~ n, there
are at least n + 1 pairwise disjoint sets, each in Sj for one of them, say
En+! not in Sn, Sn+l ~f {El, ... ,En+!}.

The sequence S ~f {En}nEN is an infinite subset of S and consists of


pairwise disjoint sets.
Second proof. Call a set E in S indivisible iff

{S 3 FeE} '* {{F = 0} V {F = E}}.


Pairwise different elements of the set I of indivisible elements of S are
disjoint. If I is infinite then E is uncountable. If I is finite or empty
let U be the union of the elements of I. Direct calculation shows that
5 ~f {A \ U : A E S } is a a-ring and 5 \ 0 is free of indivisible elements.
If 5 = {0} then # (S) < ~o. If 5 =I- {0} choose a nonempty Fl in S.
Hence for some F2 in 5,0 =I- F2 ~Fl and, by induction, for n in N, for some
Fn+1 in 5, 0 =I- Fn+! ~Fn. The set

S =
clef {
En =
clef
Fn \
Fn+1 : n E N}

is infinite and its members are pairwise disjoint.


Third proof. If 2X ::) S ~ {En}nEN and S is infinite then for each x
in X, Dx ~ (nxEEn En) \ (Ux~En En) E S. Furthermore, if Dx n Dy =I- 0,
then Dx = D y. Hence the pairwise different elements of {Dx : x EX} are
pairwise disjoint and constitute a finite or infinite subset S of S. If x E En
then Dx C En. i.e., En = UXEEn Dx. Hence S is infinite. 0
1.6. a) Let Eo be E. If 0 < (3 < 0, let Et3 be the set of all countable unions
of differences of sets drawn from Ua<t3 Ea.
If A,B E S, for some a, A,B E Ea whence A \ B E Ea + 1 C S. If
{An}nEN C S, for some a in [0,0), {An}nEN C Ea whence UnEN An E Ea +1 .
It follows that LJa<n Ea ~f S is a a-ring and aR (E) C S. Transfinite
Solutions 1. 7-1.8 141

induction implies that for all 0: in [0,0), Eo c aR (E), and there emerges
the required conclusion: aR (E) = S.
°
Because # (E) 2:: 2, # (Eo) < (# (E)/~o. If ~ (3 < 0 and, for all 0: in
[0,(3), # (Eo) ~ (#(E))NO then

# ( U EO) ~ No (# (E))NO = (# (E))NO


0'5,0<13

# (Et3) ~ (# (E))N~ = (# (E))NO .

Finally, # ([0, 0)) ~ c and (# (E))NO 2:: c whence

# (aR(E)) ~ C. (#(E))No ~ (#(E)No)2 = #(E)No.


b) For

aR (E) = 5/3 (JR n ) whence, by virtue of a), # (5/3 (JR n )) ~ N~o = c. For all x
in JR n , {x} E 5/3 (JR n ) whence # (513 (JRn )) = c.
c) Since M C [O,oo]Si3(JR) and # (513 (JR)) = c [cf. b)], it follows that

#(M) ~ CC = (2 No)' = 2Noc = 2 c .


On the other hand, if

B d~f { {A, B} : A, BE 513 (JR), A n B = 0, #(A) = #(B) = c}


°
then # (B) = c, and if {A, B} E Band < s, t < 00, there is a nonatomic
measure p, living on Au B and such that p,(A) = s,p,(B) = t. Hence

o
1.7. For

o ~f {D : DEaR (Eo) , Eo c E, # (Eo) ~ No },

aR (E) :J 0 :J E. If {An} nEN C 0 then for each n in N, there is a countable


set Eon such that An E aR (Eon); U nEN An and Al \A2 are in aR (UnEN Eon)
whence 0 is a a-ring and so aR (E) :J 0 :J aR (E). 0
1.8. For each sequence {p, q, r, ... } in N, consider sequences
142 1. Set Algebra and Function Lattices: Solutions

such that

Bp = U A pq , Apq =
qEN
n
rEN
B pqr , ... (s1.1)

resp.

Ap = UBpq , Bpq =
qEN
n
rEN
A pqr , .... (s1.2)

If B is the set of all countable unions of sets Bp then B c A and, symmet-


rically, A c B: A = B, i.e., A is closed with respect to the formation of
countable unions and intersections of its members.
If A ~f UPEN Ap then

pEN qEN
If iii obtains as well and A E A then X \ A E A whence A is a a-algebra
and A :::> aA (E). If the last inclusion is proper and A E (A \ aA (E)) then
A = UPEN A p, some Ap rJ- aA (E), some Bpq rJ- aA (E) , .... Consequently
(s1.1) and (s1.2) are denied. 0
1.9. Items a) and b) are translations of the symbols used to define £ resp.
e..
c) If x E £ then X£(x) = 1. Because x is in infinitely many En,
sup XEn (x) = 1, limn-tooXEn (x) = 1.
nEN

If x rJ- £, = O. Since x is in at most finitely many En, there is an no


X£(x)
such that sUPn>no XEn (x) = 0, limn-tooXE (x) = O.
If x E e. then X&(x) = 1. Because x is in all but finitely many En,
n

there is an no such that infn>no XEn(x) = 1,limn -tooXEn (x) = 1. If x rJ- e.


then X&(x) = O. Since x is not in all but finitely many En, for infinitely
many n,
x rJ- En, i.e., XEn (x) = 0 for infinitely many n, whence for each no,
infn>no XEn (x) = 0, limn-tooXEn (x) = O. 0
1.10. a) The formula XEnF = XE . XF shows F is intersection-closed.
Because 1 E U it follows that for any E in F, XX\E (= 1 - XE) E U
whence F is closed with respect to the formation of complements, hence
also with respect to the formation of set differences. If {En} nEN C 2X ,
E = UnEN En,
def
and S N = UN
def
n=l En then

L
00

XE = XTN E U whence E E :F.


N=l
Solutions 1.11-1.14 143

b) If K is a compact subset of X and N n ~f {x d(x, K) < ~} then


K = nnEN N n · The function

In : X 3 x 1--+
d (x, X \ N n )
d (x, X \ N n ) + d (X, K) =
{I if x E K
0 if x ~ N n

is in Coo (X, C) while for all x in X, 0 ~ In(x) ~ 1. Hence In E U, n E N.


Because XK = limn __ oo In it follows that XK E U and thus KEF. Since F
is a a-ring by virtue of a), it follows that aR (K(X)) c :F. 0
1.11. There is a bijection F : NN 3 v ~ F(v) ~f (a,(3) E (NN)2. If
9 E A2 (E) then g: NN 3 a 1--+ g(a) E (A(E))N, g(a) ~f {g(a)m}mEN' and
for each pair {m, a}, some I mOl. : NN 3 (3 1--+ I mOl. ((3) E EN is in E so that
g(a)m = U~ENNnkEN/mOl.((3)k' For each pair {a,(3} let {fmOl.((3)k}:,k=1
be enumerated as {s(a,j3)p}PEN ~f {s(F(V))P}PEN ~f {r(v)p}PEN' Then

u n m,kEN2
ImOl.((3)k = U n
pEN
s(a,(3)p

= U n
(0I.,~)E«N)N)2 (0I.,~)E«N)N)2

r(v)p E A(E). o
vENN pEN

1.12. a) For v~f {nl,n2, ... }, let I(V)k be M n1 , kEN. It follows that

b) If I is the constant map I : NN 3 v 1--+ (M1 , M 2 , •. . ), i.e., for all v, k,


I(V)k = Mk, then Mf = UVENN nkEN Mk = nkEN Mk. 0
1.13. For each v in NN, if g(V)k ~ n:=l I(v)m (in E) then 9 is regular,
Mg = nkEN g(V)k = n mEN I(v)m = M f · 0
1.14. a) If x is in the left member of (1.1), then for some m and some
v ~f {nl,n2, ... }, x E nkENMnl,n2, ... ,ni,m,ni+l, ... ,nHk' However, for the
_ def { }
sequence V = nl,n2, ... ,ni,m,ni+I,.'"

n I (i/)k = kENn Mn1 ,n2, ... ,ni,m,nHl,... ,nk·


00

k=i+2
144 1. Set Algebra and Function Lattices: Solutions

Since I is regular, n~i+2 I (vh = nkEN I (vh, whence x is in the right


member.
If x is in the right member of (1.1) then for some v ~f {nI, n2, ... },
x = nkEN I(V)k. Because I is regular, x E n;:O=i+ 2 /(v)k which, if m =
ni+I, is nkEN Mnl ,n2, ... ,n;,m,n;+2, ... ,n;+k·
b) Because I is regular,

UM n1 ,n2, ... ,nk = Mnl


kEN
U UM n"n2, ... ,nk = U Mnl = M1 UM 2 U ... = UMn.
vENN kEN vENN nEN

c) If x is not in the right member of (1.2) and x E M then

Hence if x E M n1 ,n2, ... ,nk then for some m in N, x E M n1 ,n2, ... ,nk,m. By
hypothesis, x E M (~f M n1 ,n2, ... ,nk when k = 0) whence for an m ~f mI,
x E M m1 . Thus, for an m ~f m2, x E Mml ,m2' etc. It follows that

x E n
kEN
M m"m2, ... ,mk'x E U n M m1 ,m2, ... ,mk = MI,
vENN kEN

i.e., x is not in the left member of (1.2). D


1.15. Since every open or half-open subinterval of [0,1] is a countable
union of closed intervals, 1.12 implies that all relatively open subsets of
[0, 1] are in A (E). Furthermore, A (E) is a monotone class. In 1.4, IR may
be replaced by [0,1] without affecting the conclusion. D
1.16. Because all elements of E are compact, h (E) consists of compact
sets. Since 1.13 is applicable, each E in A (E) is, for some regular g,

Each set g(V)k ~f Kk is compact and Kk J Kk+1.


The relations IoU = U0/, I- i on = n 0/-1, and f 0 n enol ob-
tain for any map f: X ~ Y.
Moreover, if y E n kEN h (Kk) then for k in N, and some Xk in Kk,
y = h(Xk). Because {Xd kEN C KI, for some point x in nkENKk and some
subsequence {Xk",}mEN' limm-->ooxk", = x, y = h(Xk",) -+ hex) as m -+ 00,
i.e., y E h (nkENKk).
Solution 1.17 145

Thus, for the sequence {KkhEN of compact sets and the continuous
map h, not only does h 0 n c n oh hold but h (nkEN Kk) = nkEN h (Kk)
is true as well, i.e.,

h(E) = Uh
vENN
(nkEN
Kk) = U n
vENN kEN
h (Kk) E A (E) . o

1.17. When E = S,X, 1.13 implies an f in Fmay be assumed to be regular.


Any subset E of [0,1] is contained in a Borel set A such that >'(A) =
>'*(E). Because {M1 J M2 J"'} ::::} {nkENMk = n':=l U:'NMk}, for
any sequence {ml, ... , mil there is a Borel set Amb ... ,m. such that

Amb ... ,m; J U n


vENN kEN
Mmb ... ,m;,nb ... ,nk

-unM
- nl,···,nk ~fM
vENN kEN
- f

Since Amb ... ,m; n Mmb ... ,m; serves as well as Aml, ... ,m; in (s1.3), it may be
assumed that Mml,. .. ,m; J Aml, ... ,m;' Since M \ (M \ Mf) = Mj, to show
that Mf E S,X it suffices to show that >'* (M \ Mf) = O. However, from
1.14c) it follows that

A \ Mf = A\ (U n
vENN kEN
Anb ... ,nk)

C U
vENN k=O
U (Anl, ... ,nk \ U
m=l
Anl, ... ,nk,m) . (s1.4)

The set of all finite subsets {nl,"" nk} of I'll is countable. Each
{nl, ... ,nk} determines Anlo ... ,nk \ U:=l Anb ... ,nk,m' Hence there are at
most count ably many different summands in (s1.4). They may be enumer-
ated, say as {Bp}l:S;P<P:S;oo' and A \ M f C Ul:S;p<P:S;oo Bp.
However, from 1.14c),
146 1. Set Algebra and Function Lattices: Solutions

whence

Am1, ... ,mi \ U Am1, ... ,mi,q C Am1, ... ,mi \ U


qEN vENN kEN
n Mm1, ... ,mi,nl,... ,nk·
The inner measure of the right member above is zero whence the left
member above is a null (Borel) set, i.e., each Bp is a null set. Hence
).* (A \ M j ) = °
and M j E SA' 0
[ Note s1.1: An examination of the proof shows that the fol-
lowing generalization of Szpilrajn [Sz] is valid.
Let B be a a-algebra contained in 2x. Assume further that
when E C X then for some A in B, a) A :J E and b) if E c B E B
and F C A \ B it follows that FEB. Then for B in Band
f : NN f-+ BN, Bj E B.]

1.18. Let M be a set such that #(M) = 0 and let E be {M}. Then
# (A(M)) = o. 0
1.19. If E ~f F ([0, 1]) then Sf3 ([0, 1]) c A (E) (1.15) whence

h (Sf3 ([0, 1])) c h (A (E)) c A (F (JR)) c A (SA (JR))

(cf. 1.16). Because JR is the countable union of intervals, the argument for
1.16 may be extended to show that A(SA (JR)) = SA (JR). 0

1.2. Function Lattices

1.20. a) It suffices to assume that a ::; b ::; c. Then

a V b = b, a V c= c, b V c = c
(a V b) /\ (a V c) /\ (b V c) = b /\ c /\ c = b /\ c = c.

b) See a).
c) If f(x) == -1, g(x) == x, and h(x) == 1 then
-I ifx<-1
mid (f,g, h) (x) = { x if -1::; x < 1
1 if 1 ::; x,

whence mid(f,g,h) tJ. {f,g,h}.


g(x) + f(x) if f(x) ::; g(x)
d) f(x) V g(x) + f(x) /\g(x) = { f(x) + g(x) if f(x) > g(x)" o
1.21. Because V is a lattice, if f, g, h E V then f /\ g, f /\ h, 9 /\ h E V. It
follows that mid(f, g, h) = (f /\ g) V (f /\ h) V (g /\ h) E V. 0
Solutions 1.22-1.25 147

1.22. For (1.3)-(1.8), honest toil succeeds when applied to the sorting out
of cases as they are governed by the definition of mid. D
1.23. a) Since V is a vector space, - f E V. Direct calculation shows
If I = f V 0 - f 1\ O.
b) Because V is a lattice (1.6) implies M is a lattice. If 11, hEM, if
11 + h is meaningful, and V 3 g ~ h E V then for n in N,

4>in ~ mid (Ii, -n(lgl + Ihl), n(lgl + Ihl)) E V, i = 1,2,


4>n ~f mid (4)ln + 4>2n,g, h) E V,
g ~ 4>n ~ h,4>n i mid (11 + h,g, h).

The hypotheses imply the validity of the Lebesgue monotone convergence


theorem, hence the validity of the dominated convergence theorem for
J. Because g ~ 4>n ~ h, it follows from (1.7) and the hypotheses that
mid (11 + h,g, h) = limn_oo 4>n EM.
c) If V 3 g ~ h EV and M 3 fn i f then mn ~f mid (In,g, h) E V
and -00 < J(g) ~ J (m n ) ~ J(h) < 00. The conclusion follows from (1.7).
If M 3 fn ! f the same proof, mutatis mutandis, applies.
d) See c) and the definitions of lim, lim, and lim.
e) The result follows from the formula: mid (I, -If I, If I) = f.
f) The implication::::} is valid because V is a vector space. On the other
hand, if V 3 P ~ q E V then (1.4) implies

p +q . ( q - p q - p) .
-2- + mId f, --2-' -2- = mId(l,p, q).

Because V is a function lattice, the implication ~ is valid. D


1.24. For C d~f UnEN An, from the equations

XAUB = XA V XB, XA \B = XAUB - XB, Xc = V XA n ,


nEN

and the properties 1.23b) - d), it follows that A is a a-ring. When 1 EM,
the formula XX\A = 1 - XA implies A is a a-algebra. D
1.25. a) Since each Eo (I, a) E f- 1 (St3(lR)), it suffices to note that when
a1 ~ a2,

f- 1 ([a1' (2)) = E< (I, (2) \ E< (I, (1) = E? (I, ad \ E? (I, (2),
f- 1 ((ab a2]) = E~ (I, (2) \ E~ (I, (1) = E> (I, ad \ E> (I, (2),

and that St3(lR) is e.g., aR ({ [a, b) : a ~ b, a, bE Ql}).


148 1. Set Algebra and Function Lattices: Solutions

b) For (lR, S>., A), there is a set S in 21R \ S>. (cf. 4.39). For all a, if
I(x) ~f XXs(x) - XXIR\s(x) then

o
1.26. If IE M then, since 1 E M, it follows from 1.23 that for n in N,

n(a-f)EM, 1I\n(a-f)EM, and (ll\n(a-f))VOEM.

Furthermore, XEd/,a) = limn ..... oo (11\ n(a - f)) V 0 whence again, 1.23
implies Ed/, a) E A.
When IE lR x and (k, n) E Z x N, let Ink be the characteristic function
of E:s, (I, k2- n ) \ Es, (I, (k - 1)2-n). Then I = limn..... oo E~=-n2n k2:1 Ink
and iffor every a, Es,(I, a) E A, then I is in M. 0
1.27. Because In:::; lim m..... oo gm,

Moreover, I (h m ) ~ I (In - gm) whence

Hence, lim m..... oo I (gm) ~ lim n ..... oo I (In). The argument is symmetrical in
the pair {{fn}nEN' {gm}mEN}· 0
1.28. Multiplication by nonnegative numbers preserves order among func-
tions. Addition is a continuous operation. 0
1.29. For each n and some sequence {gmn}mEN in L, gmn im In. Hence
if k n ~f gIn V ... V gnn then

· k n ~f
L 3 kn :::; In, gmn :::; k n :::; In if m :::; n, gmn:::; 11m - k,
n ..... oo

In = lim gmn :::; k :::; I, k


m~~
E Lu, I = lim
n~~
in:::; k:::; f.

Hence I = k E Lu and similarly the inequalities

imply I (In) i 1(1). o


1.30. For some {fn}nENinL,In i I. Ifh~f /Iandp=E':=2(1n-In-l)
then h E L, p E Lu, p ~ 0, and I = p + h. 0
Solutions 1.31-1.32 149

1.31. Trivialities aside, if I ::; h E L u , 9 ::; k E Lu then

I(f + g) ::; I(h + k) = I(h) + I(k),


I(f + g) ::; I(f) + I(g).
a) The definitions imply the conclusion. In particular, I is positive
homogeneous: {a 2: O} :::::> {I(al) = aI(f)}.
b) 0 = I(f + (-I))::; I(f) +1(-1).
c) Since I ::; I, if I E Lu then I(f) ::; I(f). If I ::; 9 E Lu then
I(f) ::; I(g), whence I(f) ::; II).
d) It may be assumed that I (fn) < 00, n E N. If t > 0 then for
some gn in L u , gn 2: In and I (gn) < I (fn) + 2:' n E N. From 1.29 it

follows that 9 ~f L:~1 gn E Lu and I(g) = L:~=1 I (gn) ::; L:~=1 I (fn) +t.
Furthermore, 9 2: I and hence I(f) ::; I(g) ::; L:~=1 I (fn) + t.

[Note 81.2: From {I 2: L:;:'=1 In} :::::> {II) 2: I (L:;:'=1 In)}'


and the subadditivity of I it does not follow that I can be dis-
tributed validly over the sum. J
e) Again it may be assumed that I (fn) < 00, n E N. From a) it follows
that limn-too I (fn) ::; I(f). If t > 0 for each n then for some h n in L u ,
- t
hn 2: In and I (hn ) < I (fn) + 2n . Then

· (h 1
11m V ... V hn ) def
= 1·1m gn = 9 E L u,
def
n~~ n~oo

and I (gn) i I(g), (cf. 1.29).


Since gl = hI, 1.29 and mathematical induction show

I (gn) + I (hn 1\ gn-l) = I (hn + gn-1) = I (h n ) + I (gn-t)


- t - t - - t
::; I (fn) + 2n + I (fn-t) + t - 2n - 1 = I (fn) + I (fn-l) + t - 2n .

Because In-I::; h n 1\ gn-1 it follows that I (fn-d ::; I (h n 1\ gn-t) and so

- t-
I (hn V gn-1) = I (gn) ::; I (fn) + t - 2n + I (fn-t) - I (h n 1\ gn-1)
- t
::; I (fn) +t - 2n .

Because I (fn) ::; I (gn) it follows that I(f) ::; I(g) ::; I (fn) + t. 0

1.32. a) The first inclusion follows from the fact that IlL = I and the
second inclusion is a matter of definition.
150 1. Set Algebra and Function Lattices: Solutions

b) Since 1 is additive on L, 1 is additive on Lu and on L1. If I E L1


and E > 0 then for some 9 in Lu, 1(g) < 1(f) +~. Similarly, for some
-h in Lu, -I ::; -h and 1( -h) < 1(-1) +~. Hence h ::; I ::; 9 and
1(g + (-h)) = 1(g) + 1( -h) ::; 1(f) + 1( - I) + E = 1(0) + E = E.
Conversely, if there are a 9 and a -h as described then

0::; 1(f) - (-1(-1)) ::; 1(g) + 1(-h) = 1(g + (-h)) < E.

c) If 0 stands for any of +, V, /\, but the same one throughout each
string of relations, and if hi ::; Ii ::; gi, i = 1,2, then

When 11, hE L1 and gi and hi, i = 1,2, are chosen as in b), it follows that
1 (g1 0 g2) + 1 (- (hI 0 h 2)) < E whence 11 0 h E L1; similarly, if c E lR. then
cl1 E L1.
d) The definitions imply the statements.
e) If I E L1 then J (fn) ::; J(f) < 00. For the converse, it may
be assumed that J (fn) < 00, n E N. If 9 ~f 1-11 then 9 ~ 0 and
9 = :E:'=1 (fn+l - In). The subadditivity of I and d) in 1.31 imply

00

" J (fn+l - In) = n-+CX)


l(g) ::; 'L...J lim J (fn) - J (ft)
n=1
1(f) = 1(11 + g) ::; J (11) + l(g)::; lim J (fn).
n--->oo

Since I(f) ~ I (fn), it follows that I(f) ~ limn--->oo J (fn) which, in light of
the previous inequality, shows I E Ll and J(f) = limn--->oo J (fn).
f) The countable additivity of J.l is a consequence of e).
g) If I E L1 (= V!) then, since L1 is a function lattice, it follows that
I E M and III = I V 0 - I /\ 0 ELI. Conversely, if I E M and III E Ll
then -III ::; I ::; III and I = mid(f, -III, II)I E L1.
h) If E E M and I ~f XE E Ll, the result follows by definition. In
general, when I E L\ for some sequence {In}nEN in L\ In i I and then
e) yields the conclusion. 0
1.33. If 1= 9 - P as described, then 1.32a) shows I E L1. If I E L1 then
_ - 1
1(f) E lR. and for n in N, and some In in Lu, In ~ I and 1 (fn) < 1(f) +-.
n
Thus {gn ~ 11 /\ ... /\ In} is a sequence such that for some 9 in LuI,
nEN
Moreover, 1(g) = J(f), 0 ::; 9 - I = PEL, J(p) = 0,
- ~ 1
gn ! g. and
I=g-~ 0
Solutions 1.34-1.36 151

1.34. a) Any gin L1 is in M whence if 0 S f E M then f 1\ gEM and


If 1\ gl SIgl E L1. It follows from 1.23e that f 1\ 9 E L1.
Conversely, if 0 S f and for all 9 in L1, f 1\ 9 E L1, assume 0 S k and
k E L1. Then mid(f, -k, k) = f 1\ k E L1 and 1.23f implies f E M.
b) If 0 S f E M assume h E L. Then hEM n L1 whence a) implies
f 1\ hE L1.
Conversely, assume 0 S f and for all h in L, f I\h E L1. The lattice and
monotone convergence properties of L1 imply that if k E Lu and I(k) < 00
then f 1\ k E L1 and that if q E Lul then f 1\ q E L1. If 9 E £1 then (1.33)
9 = q - r, q E Lui, 0 S r, J(r) = O. Because 9 S q,

f 1\ q(x) - f 1\ g(x)
q(x) - g(x) = rex) 2:: 0 if q(x),g(x) S f(x)
= { f(x) - g(x) 2:: 0 if g(x) S f(x) S q(x) . (s1.5)
o if f(x) S q(x),g(x)

Note that in (s1.5), 0 S f(x) - g(x) S q(x) - g(x) = rex). It follows that
OS f 1\ q - f 1\ 9 = r V 0 and since, J(r) = 0, J(r V 0) = O. Hence, owing
to 1.33, f 1\ 9 E L1 and so a) implies f E M. D
1.35. If f E L1 then for some gin Lul,

J (If - gl) (= Ix If(x) - g(x)1 d/-L(X)) = 0

(cf. 1.33). For some h in L u , II(h) - J(g)1 < ~ and for some f in L,
II(h) - I(f)1 < ~. Note J/ Lu = I and J/ L = I. D
1.36. The definition of /-L* and the properties of] imply that /-L* is an outer
measure, whence the set of Caratheodory measurable sets is a a-algebra.
If D is Daniell measurable and /-L*(A) = 00, the subadditivity of]
implies /-L* (A) = /-L* (A n E) + /1-* (A \ E).
If /1-* (A) < 00 and E > 0 then for some 9 in L u , 9 2:: XA and

] (XA) + E 2:: I(g) 2:: I(g 1\ 1) = I (g 1\ XD + 9 1\ XX\D)


2:: I (g 1\ XD) + I (g 1\ XX\D)
2:: ] (XAnD) +] (XA\D) .

Hence Daniell measurability implies Caratheodory measurability.


When C is Caratheodory measurable, its Daniell measurability is es-
tablished by showing that Xc E M, i.e., that for all 9 in L, Xc 1\ 9 E L1.
Since the range of Xc is the set {O, I}, it suffices to show:

{{g E L} 1\ {O S 9 S I}} =} {xc 1\ 9 E L1} .


152 1. Set Algebra and Function Lattices: Solutions

Case 1: 1 (xc) < 00. If f > 0 then for some gin L u , 9 ~ Xc and 1(g) <
l(g) + f. Because 1 E M, L1 :3 1/\ 9 ~ Xc. Each of a) - e) below follows
from its predecessors or from the statements made thus far: a) 1/\ gEM;
def -
b) B = E? (1/\ g, 1) E A; c) 9 ~ XB EM; d) 1 (XB) ::::; l(g) < 00, whence
XB E L1; e) XB ~ xc.
Because XB E L1, B is Daniell measurable and hence Caratheodory
measurable. The following equations and inequalities are validated, i.a.,
because both B and C are Caratheodory measurable, XB E L1, I is super-
additive, and for all sets A, 1(XA) = J1.*(A) (by definition):

1 (XB) = 1 (XB /\ xc) + 1 (XB\ c) = 1 (Xc) + 1 (XB - Xc)


1 (XB - Xc) = 1 (XB) - 1 (xc) = I (XB) + I (-Xc) ::::; I (XB - Xc) .
Thus XB - Xc E L1, Xc = XB - (XB - Xc) E L1, i.e., C E A.
Case 2: 1 (Xc) = 00. Let A be E>(g, 0) which, owing to 1.24, is in A.
Case 2a: XA E L1. Then A is Daniell measurable, hence Caratheodory
measurable, and Xc /\ 9 = Xc /\ (g /\ XA) = (xc /\ XA) /\ 9 = (XcnA) /\ g.
Thus en A is Caratheodory measurable and, since 0 ::::; XcnA ::::; XA E L1,
1 (XcnA) < 00. The conclusion in Case 1 shows that XcnA /\g E L1, whence
Xc /\g E £1.
Case 2b: XA EM \ L1. For f > 0 let E>(g, f) be A,. Then XA< EM
and 1 (XA e
) ::::; 1 (~€ . g) = ~€ l(g) < 00 whence XA e E L1. The conclusion

in Case 2a implies that en A E A. Setting f at 2:., n E N, yields the


n
result. o
[ Note 81.3: Because the set of Caratheodory measurable sets
is complete (subsets of null sets are Caratheodory measurable), it
follows that A is complete if 1 E M.l
2
Topology, Limits, and Continuity

2.1. Topology

2.1. a) If A contains the range of the net n converging to a then for each
for each N(a), n is eventually in N(a) whence N(a) n A -j. 0 and so a E A.
If A is the diset N(a) then the range of the net n : A 3 N f-+ n(N) EN n A
is contained in A and n converges to a.
The proofs for b) - f) are similar exegeses of the definitions. 0
2.2. For N(x) and AN ~f {A : A E A, n(A) ENE N(x)}, since n is
frequently in each N of N(x), r ~f UNEN(x) AN is a subdiset of A. Then
m = n Ir
def
converges to x. 0
2.3. If F converges to x, and n is a net corresponding to F, then every
N(x) contains an element of F and hence n is eventually in N(x).
Conversely, if every net corresponding to F converges to x, and if F
does not converge to x then some N (x) fails to contain any element of :F.
Hence some net n corresponding to F is such that for each member F of F,
n(F) E F \ N(x). Thus n fails to converge to x, a contradiction. 0
2.4. The net n converges to x iff n is eventually in each N(x), i.e., iff for
some AO, if A r- AO then B).. eN, i.e., iff F converges to x. 0
2.5. a) limx=af(x) = -limx=a (- f(x)). If f is usc and x E Edf, ex) then
limy=xf(Y) = f(x) < ex, whence for some open N(x), SUPYEN(x) f(y) < ex,
i.e., N(x) C Edf, ex).
Conversely, if each Edf, ex) is open and f(x) -j.limy=xf(Y) ~f ex then
f(x) ~f f3 < ex and so for some N(x) and all Y in N(x), f(y) < ex; f3. It
ex+f3 -.- ex+f3
follows that SUPYEN(x) f(y) < - 2 - < ex and so hmy=xf(y) < - 2 - < ex,
a contradiction.
For lsc, the dualities among lsc/usc, sup/inf, and +/- apply. 0
2.6. For some N(a) and some L in [0,00), sup xEN
-(-)
a
f(x) = M + L.
The set A ~f {N : N = N(a) C N(a) }, partially ordered according to:
N' r- N iff N' eN, is a diset. For N in A and some EN in [0,00),
sUPxEN f(x) = M + EN and the net E : A 3 N f-+ EN E [0,00) converges to

153
B. R. Gelbaum, Problems in Real and Complex Analysis
© Springer-Verlag New York, Inc. 1992
154 2. Topology, Limits, and Continuity: Solutions

O. Furthermore, for some XN in N, f (XN) 2 M + ~EN. Finally, the net


n: A 3 N ~ XN converges to a and f 0 n converges to L.
For limx=af(x), the duality lim/lim applies. 0
2.7. Because # ({O, 1}N) = c, and since each element of S is representable
by a sequence in that element, it follows that #( s) S; c. on the other hand,
if s E s and a represents s, the sequence a ' arising by replacing each term
in an infinite subsequence of terms in a by its opposite counterpart (0 by
1, 1 by 0) is not equivalent to a: #(s) 2 c. 0
2.8. If x adheres to F then every n( x) meets every element of F whence
the filter :F ~f {n(x) n f : f E F} is finer than F and converges to x.
If F converges to x then each N (x) contains some element E of F. if
F E F then F nEE F whence N (x) contains every F n E and thus, for
each F in F, N(x) n F i= 0, i.e., x adheres to F. 0
2.9. Assume X is compact and U ~f {F} is an ultrafilter in 2x. each F
is closed and if nFEU 1 = 0 then for some finite subset {FI, ... ,Fn} of U,
n~=l Fk c n~=l Fk = 0, which is impossible for elements of a filter. Hence
some x is in nFEUF. For the filter N(x),

{N(x) n F : N(x) E N(x), fEU}


generates a filter V that converges to x. Furthermore, V)- U and, since U
is an ultrafilter, V = U.
Conversely, if every ultrafilter converges, let £ ~f {E} be a set of closed
sets such that every finite subset of £ has a nonempty intersection. then
£ generates a filter F contained in an ultrafilter U that converges, say to
x. For N(x) and E in £, there is in U a U that is contained in N(x) and
N(x) :J Un E i= 0. since E is closed, x E E: x E nEEeE. In other
words, if a set £ of closed sets enjoys the finite intersection property then
the intersection of all the sets of £ is nonempty. Hence X is compact.
If X is compact and n : A ~ X is a net, the corresponding filter
F~ {B).. ~f {n(J-L) : J-L)- A} } is a subset of an ultrafilter U converging to
some x in X. Hence x is in the closure of each element U of Uj in particular
x is in the closure of each B)... Thus for each N(x) and each A in A, there is
a U contained in N(x), and in the nonempty set B).. n U, there is an n (A').
Note that>.' )- A. The set A' of all such>.' is cofinal in A and the cofinal
net nl)..'
converges to x.
Conversely, if U is an ultrafilter in 2x , and n is a net corresponding to
U, there is a cofinal net converging to some x in X. Hence U converges to
x whence every ultrafilter converges and x is compact. 0
[ Note 82.1: When X is not compact a point x may lie in the
closure of the range n (A) of a net n : A ~ X and yet no cofinal
net nlA'
converges to x [Ar, GeO, Rin].)
Solutions 2.10-2.14 155

2.10. The map I : [0,1) 3 x f-+ 1 ~ x sin (1 ~ X) E R. is continuous and

1([O,I))=R D
2.11. Define the equivalence relation rv on lR according to x Y iff
x - Y E IQ. Then rv decomposes [0,1] into equivalence classes and, via
the Axiom of Choice, there is a set E consisting of one element from each
equivalence class. Thus E - E \ {O} C 1I1R and (E - Et = 0. D

2.12. ThefunctionD: [0,1)2 3 {x,y} f-+ \_x_ - -Y-\ is anew metric.


I-x l-y
Furthermore,

Ix-YI (s2.1)
D(x, y) = 1(1 _ x)(1 _ y)1 ~ Ix - YI = d(x, y)

and D(x, y) = d(x, y) iff x = y. Therefore, Xn .:!:" x iff Xn E. x, i.e., the d-


topology and the V-topology are the same. A d-Cauchy sequence {xn}nEN
fails to d-converge iff limn---+ooxn = 1. For such a sequence {xn}nEN'
{V (Xm,xn)}m,nEN is unbounded and thus {xn}nEN is not a D-Cauchy se-
quence. On the other hand, if {Yn} nEN is a V-Cauchy sequence, for some
positive 10, d (Yn, 1) ~ E. The inequality (s2.1) implies {Yn} nEN is ad-Cauchy
sequence that fails to d-converge to 1 and thus d-converges to a Y in [0,1).
Finally, V (Yn,Y) ::; c 2 d(Yn,Y), i.e., Yn E. y. D
2.13. Because [0,1] is compact and connected, each of Ax B, A, and B
is compact and connected. If al and a2 resp. b1 and b2 are two elements of
A resp. B then, because 1-1 is a homeomorphism, each of the four sets

is compact and connected, i.e., a closed interval. Since la,nh j = 1-1 (ai, bj )
and lal n la2 = hl n h2 = 0, the four intervals constitute an impossible
configuration of subintervals of R D
2.14. a) If an a in Co has two different ternary represenur2.tex tations in
°
which each marker is or 2 let the representations differ for the first time at
the nth marker. Then in one representation the nth marker is 2 and in the
other it is 0. Thus a - a has a ternary representation in which the first n-l
markers are zeros and the nth marker is not zero, i.e., la - al = ~ 3- n . °
b) The map ¢>: VN 3 e ~f {En}nEN f-+ ¢>(e) ~ 2:::=12En3-n E Co is a
homeomorphism.
c) If x, Y E Co and Ix - YI < 3- n then I/n(x) - In(Y)1 < 2- n .
d) Ifx,y E Co and Ix-YI < 3- n then I¢>o(x) - ¢>o(Y) I < 2- n . Further-
°
more, if [0,1] 3 t ~f 2:::=1 En2- n , En = or 1 then ¢>o (2:::=1 2En3-n) = t.
156 2. Topology, Limits, and Continuity: Solutions

e) If some ternary representation of a t in [0,1] is given, successive


ternary markers for elements x and y of Co can be determined by mathe-
matical induction so that x + y = t. A similar argument applies for a t in
[-1,1].
f) 4>0 (~) = 4>0 (~). 0
2.15. a) For e~f {~m}mEM E VM, a minimal base B ~f {Nm}mEM for
X, and each m in M, there is a dyad Ai ~ {Nm if i =0 of
m X \Nm ifi =1
closed subsets of X. If A~ ~f nmEM A~ =1= 0 then, since X is a Hausdorff
space, A~ is a single point x~.
If A~ = 0, the compactness of X implies that there is in M finite
subset f..L such that nmEtL A~ = 0. Then {e' : ~:n = ~m' m E f..L} is an
e
open subset of V M . Hence the set 8 of those such that A~ =1= 0 is closed
in V M .
Furthermore, if x E X, let M' be {m : x E N m }. If

e = {O1
m
ifm E M'
otherwise

and e' ~f {~:n}mEM then x~, = x. Hence the map F : 8 3 e I--t x~ E X is


surjective. If N m is a neighborhood of x~ then V ~f {e : ~m = O} n 8 is
an open subset of 8 and F(V) C N m . Since B is a base for the topology of
X, F is continuous.
b) If x E BnCo let hex) be x. If x E Co \B and L:'=l ~: is the ternary
representation of x, define a sequence {Zn}nEN by induction as follows:
When a E {0,2}, let a' denote a + 2 mod 4. If B contains a b
for which the first ternary marker bl is Xl, let Zl be Xl; otherwise
all bl =1= Xl, in which case let Zl be x~. Thus B contains a b such
that the first ternary marker bl is Zl. Having defined Zl, ... , Zn so
that B contains a b for which the first, ... , nth ternary markers
bl , ... , bn are Zl, ... , Zn, let Zn+l be Xn+1 if B contains a b such
that the ternary markers bl , ... , bn , bn + l for b are bl , ... , bn , Xn+l;
otherwise, all bl, ... , bn , bn + l are bl , ... , bn , X~+l' in which case let
Zn+1 be X~+l. Thus B contains a Z such that the ternary markers
for bare Zl, Z2, ....
For Z as just defined let h(x) be z. If x, Y E Co then Ix - yl < 3- N
iff the ternary markers x n , Yn, 1 :s: n :s: N, are equal, in which case the
(corresponding) ternary markers for hex) and hey) are the same. Hence h
is a continuous surjection of Co on B.
c) If W = No then VM is homeomorphic to Co while 8 is a closed
subset B of Co. Thus F 0 h (Co) = X. 0
Solutions 2.16-2.17 157

2.16. a) Since {0, X} c Tand Tis closed with respect to the formation
of arbitrary unions and finite intersections, (X,1) is a topological space.
If U ~f {U>.hEA is an open cover of X then Xo is in some U>.. Hence
X \ u>. is finite and thus can be covered by finitely many elements of U:
they, together with U>., constitute a finite sub cover of U.
If x =f:. y and neither is Xo then each is open; if x = Xo then X \ {y} is
a neighborhood of Xo and y is open.
b) Because Xo ~ {x}, {x} E T.
c) 8 ~f {{x} : x =f:. Xo }U(X \ {xo}) is a base for Tand # (8) = #(X).
Because each x other than Xo is open, the weight of X is at least #(X).
o
2.17. Each nonempty 0"( contains a basic neighborhood

n
N (a, E) = X{xmJ x VM\O"
i=1

specified by: a) a finite subset a ~f {m1, ... , m n } of M; b) some finite set


E ~f {Xml' .•. , xm n } of binary markers.
It suffices to show that any set N of pairwise disjoint basic neighbor-
hoods is empty, finite, or countable.
i. If two basic neighborhoods N(a, E) and N (a', E') are disjoint then
a n a' =f:. 0 and for some m in a n a', the coordinates Xm and x~ are each
other's opposite: Xm + x~ = 1. The index m is a hit-index for the pair
{N(a, E), N (a', En.
ii. If, for k in N, 1;k ~f {N (a, E) : N(a, E) EN, #(a) = k} then
N= UkEN1;k.
iii. If 1;1 =f:.0 then i and ii imply # (1;1) = 1 or 2.
iv. If each of 1;1, ... ,1;k-1 is empty or finite and # (1;k) 2: 2, fix
N ~ N(a, E) in 1;k. Owing to i and the existence of another N' in 1;k,
some hit-index m1 is in a. Let T1 in 1;k be the set of all N' such that
X~l = 1- x m1 . Replace each N' in T1 by a basic neighborhood N' for which
the restriction at index m1 is removed. Since the N' are pairwise disjoint
and X~l = 1 - X m1 , any pair of the N' has a hit-index different from m1.
Each N' is determined by e~ctly k -1- indices and, owing to the conclusion
in the last sentence, the set T1 ofthe N' from Tl consists of pairwise disjoint
basic neighborhoods. By inductive assumption # (Tl) (= # (T1 )) is finite.
If 1;k \ T1 =f:. 0, then some hit-index m2 is in a \ m1 and 1;k \ T1 contains
a subset T2 consisting of an N' such that X~2 = 1- x m2 • The corresponding
set T2 consists of pairwise disjoint basic neighborhoods, each determined
by precisely k - 1 indices. By inductive assumption, # (T2) [= # (T2)] is
finite.
158 2. Topology, Limits, and Continuity: Solutions

For some K in N, K ::; #(a) and in finitely many repetitions of the


procedure just described, there are generated

Each Ti is a finite set of basic neighborhoods, and the union of the 1i is


Ek : # (E k ) is finite. Owing to ii, .N is empty, finite, or countable. D
2.18. Let B).. ~f {U)..,n} nEN be a countable base for X)... Since each set
in 0 contains a basic neighborhood constructed with elements from finitely
many bases, it suffices to show that a set .N~f {N( a, E)} of pairwise disjoint
basic neighborhoods is empty, finite, or countable.
A basic neighborhood N(a, E) is determined by a finite set a of indices
from A, and a finite set E of sets U)..,n)o.' A E a, n).. E N, from B)...
The finiteness of each index set a permits the exhaustion process de-
scribed in 2.17 to be carried out, mutatis mutandis. The following should
be noted.
i. Disjointness arises at a hit-index because of inequality of the base ele-
ments used at a hit-index.
ii. The set T1 is here a set Tn consisting of all N' having at the hit-
index Al a fixed base element different from the base element for N
at the hit-index AI; T12 consists of all N' having at the hit~index Al
a fixed base element different from the base element for N and from
the base element used in the construction of Tn; ... . Once the first
hit-index Al has produced its countable string Tn, T12"'" the process
is repeated for a second hit-index A2.2....-etc. The refinement Tij permits
the conclusion that the elements of Tij are pairwise disjoint.
iii. Each Ek may be infinite but cannot be uncountable. D
[Note 82.2: If basic neighborhoods are determined by not more
than count ably many indices, the argument remains valid. If each
space X).. has weight Wand if neighborhoods are determined by
not more than count ably many indices, the cardinality of a set of
pairwise disjoint open sets in X cannot exceed W. At the heart
of the reasoning is the fact that if n is an infinite cardinal then
Non = n.]
2.19. If I : VM ~ X is continuous then for each of the uncountably many
points x other than xo, the set {x} is open and 1-1 ({x}) is an open subset
of V M . The sets {x} are pairwise disjoint, whence the sets 1-1 ({x}) are
pairwise disjoint and constitute an uncountable set, in contradiction of the
conclusion in 2.17. D
2.20. Because V ~f U)..EA (a)..,b)..) is an open subset of JR, which is
separable, V is the countable union of open intervals, each contained in some
Solutions 2.21-2.23 159

(a.x,b>.): for some countable subset {(a.xn,hJ}nEN' V = UnEN (a>'n,b>'n)'


Let Ao be {A : b.x rt V}. If (a.x/,b.x/l C (a>.,b.xl then (a).',b).'l may be cast
out of the union. If A, A' E Ao and A I- ,XI then (a>., b>.l n (a).', b.x/l = 0 since
otherwise, b.x or b.x' is in V. Because there are at most count ably many
pairwise disjoint intervals (a>., b>.l in JR, Ao is countable. It follows that

F= U (a>.,b.xl U U(a.xn,hnl· o
.xEAo nEN

2.21. Because limm,n-too d( m, n) = 0, N is a d-Cauchy sequence but JR


contains no x such that limn-+ oo d (n, x) = o. 0
2.22. Ifu rt s then for some sequence {xn}nEN in S, Xn i u. Furthermore,
Xn Xn . { }
- - E S, whence - - < u and thus If p, n C N then 0 < Xn < uPxn+P'
Xn+l Xn+l
Since u < 1 it follows that Xn == 0, a contradiction. 0
l+_t_ ift>O
2.23. The map f: JR \ Q 3 t f-+ { 2 2t + 2 illustrated in
1 t 'f 0
2+2-2t 1 t<
Figure 82.1 maps JR \ Q homeomorphically onto (0,1) \ Q.

y-axis

(0, 1)

\
\

\
\

---L-----''---------L----....L---~--x-axis
(0,0)

Figure s2.1. The homeomorphism f : JR \ Q ~ (0,1) \ Q.


o
160 2. Topology, Limits, and Continuity: Solutions

2.24. If Po E U then for some positive ro, B (Po, ror c U. Because


there are no isolated points in X, there are in B (Po, ro) two different points
POO,POl and a positive rl such that

By induction, for each dyadic rational number 'Lf=l €k2-k, there can be
defined a point Pfl, ... ,fK and for each K a positive rK so that rK 10,

B (Pfl, ... ,fK_l,o,rK) n B (Pf l, ... ,fK_l,1,rK) = 0, and


B (Pfl, ... ,fK-l,O, rK) U B (Pfl> ... ,fK-1>1, rK) C B (Pfl, ... ,fK-l' rK -1) .

The cardinality of the closure of the set of all points P... is c. o


2.25. If no point of K is isolated, each point of K is a limit point and
since K is compact, K is perfect. Because X is a complete metric space,
#(K) ~ c (cf. 2.24), a contradiction. 0
2.26. If R. n =I- UnEN F;:' then R. n \ UnEN F;:' contains a nonempty compact
ball Band B = UnEN B n (Fn \ F::J. Since each summand is closed and its
interior is empty, each summand is nowhere dense. The complete metric
space B is the countable union of nowhere dense sets, a contradiction.
o
2.27. a) Since R. n is separable, it has a countable base {Nn}nEN. Every
N(a) contains an Nna containing a. If every AnN(a) is at most countable,
then A c UnEN A n N na , which is countable.
_
b) If s E S then s = nnENB s, ~ ( 1) 0
and B
(1 ) n
s, ~
0
S
def
= Sn =I- 0.

For t in Sn and some m in N, B (t, ~) c B (s,~) whence s S. E

c) #(S) ~ # (Sn) > ~o.


d) If n = 1 and AnN(a) is uncountable then lim {x : x E An N(a)}
serves for band lim {y : yEA n N(a)} serves for c. 0
2.28. Let P be the self-dense kernel of X. If x E p. then {x} U Pis
self-dense whence x E P, i.e., P is closed and hence P is perfect.
If x E C ~f X \ P then x is isolated: C consists of isolated points.
Because D is dense, xED and since D is countable, #(C) ::; ~o· 0
2.29. Because A(F ([0, 1])) c A (S,\ ([0, 1])) and A (S,\ ([0, 1])) = s,\ ([0, 1])
(cf. 1.16), it suffices to prove that every Lebesgue measurable subset of
[O,IJ is empty, finite, countable, or of cardinality c.
The Cantor-Bendixson theorem implies that every closed subset of [0, IJ
is empty, finite, countable, or of cardinality c. If E E S,\ ([0, 1]) it follows
that >.(E) = sup {>'(F) : FE F ([0,1]), FeE}. Hence #(E) ::; ~o or
#(E) = c. 0
Solutions 2.30-2.32 161

2.30. a) Let Y be X x X metrized according to the formula

Since (X, d) is compact, so is (Y, D). The map I serves to define a map
F : Y 3 Y ~f (Xl, X2) 1-+ (f (Xl) ,I (X2)) ~f F(y) E Y. If, for some y in Y,
some positive E, and all n, D (Fn(y), y) ~ E then, since I is expansive, for
each k in N, D (Fn+k(y), Fk(y)) ~ E. In other words, for any two p, q in
N, D(FP(y),Fq(y)) ~ E, i.e., {Fm(Y)}mEN contains no Cauchy sequence, a
contradiction of the compactness of Y.
Hence if E > 0 then for each y in Y, and some n ~f n(y) in N,
D(Fn(y),y) < E. Ify = (a, b) then:

d(a, b) ::; d (r(a), r(b)) ::; d (r(a), a) + d(a, b) + d (b, r(b));


d(r(a),a) +d(b,r(b)) = D(Fn(y),y) < E;
d(a, b) ::; d (f(a) , I(b)) ::; d (r(a), r(b)) < d(a, b) + Eo

b) Because I is expansive, I is injective. Since I is an isometry, I


is continuous and, because X is compact, 1- 1 is continuous on I(X). If
I(X) '¥X then Z ~ X \ I(X) is open and nonempty whence it contains
some closed ball B(a,r) of positive radius. But then I (B(a,r)) C I(X)
and it follows that the sequence {r (B(a, r))}nEN consists of pairwise dis-
joint closed balls, each of radius r. Hence {r (a)} nEN contains no Cauchy
sequence, again in contradiction of the compactness of X.
c) For each X in X, 1- 1 (x) =I- 0. The Axiom of Choice implies there is
a map 9 : X 1-+ X such that 1 0 g(x) = X and, since I is contractive, 9 is
expansive. From a) and b) it follows that 9 is an isometric auteomorphism.
Since log = id, it follows that

o
2.31. For T: 1I' 3 e 27rilJ 1-+ e 27ri (2IJj(1+IJ)) E 1I' and all x, limn->CX) Tn(x) = l.
If d is any T-invariant metric compatible with the topology of X then
d (Tn(x), Tn(y)) == d(x, y), whence d(x, y) == 0, i.e., d (T(x), T(y)) = d(x, y)
iff X = y. 0
2.32. It suffices to show that every sequence {Tn} nEN in E contains a
convergent subsequence. The proof follows the lines of the proof of the
Arzela-Ascoli theorem. Because X is a compact metric space, X contains a
dense sequence S ~f {xn}nEN. Because each Tn maps X into itself, there is
a convergent subsequence Tnrn (Xl)' a convergent subsubsequence Tnrn. (X2),
1

etc. The diagonal sequence {Tnp T n rn2 ' ... } ~f {G I, G 2 , •.• } is such that
for each m, {G p (xm)}PEN converges.
162 2. Topology, Limits, and Continuity: Solutions

Owing to the equicontinuity of {Gp}PEN' if E > 0 there is a positive


TJ such that {d(x,y) < TJ} ~ {\f(p){d(Gp(x), Gp(Y)) < En.
Furthermore
{N m ~f {x : d (x, xm) < TJ} } mEN is an open cover of X and thus contains
a finite subcover {Nmh::;m::;M' Furthermore, there is a k(E) such that if
p, q > k(E) and 1 ~ m ~ M then d (Gp (Xm) , Gq (xm)) < E. It follows that
if if x E X then for some m in [1, MJ, x E N m and if p, q > k(E) then

d (Gp(x), Gq(x)) ~ d (Gp(x), Gp (xm)) + d (Gp (Xm) , Gq (xm))


+ d (Gq (Xm) , Gq(x)) < 3E. o

2.33. If I is continuous then

F: X :7 x ~ (x,/(x)) E X x Y

is also continuous. Because X is compact, F(X), which is g(J), is compact


and thus closed.
Conversely, if g(J) is closed and I is not continuous, there is a se-
quence {xn} ~=o such that limn->oo Xn = Xo
and for some positive E and all
n, {j (J (Xn) , I (xo)) ~ E. Owing to the compactness of Y, there is a sub-
sequence {xnkhEN such that limn->oo I (x nk ) ~f y exists and is not I (xo).
Since g(J) is closed, (xo, y) is in g(J). On the other hand, for some subse-
quence {x mz hEN' liml->oo xmz = Xo and liml->oo I (x mz ) ~f z exists, whence
(xo, z) E g(J). In sum, the two different points (xo, y) and (xo, z) belong
to g(J), a contradiction. 0
2.34. Let B ~f {Un}nEN be a base for X. Then every open set in X
is a countable union of members of B, whence # (0) ~ c. Furthermore,
() : 0 :7 U ~ X \ U E F is a bijection. o
2.35. a) The maps In : [0,1] :7 x ~ :: are in Ai whereas In ~ 0 and
n
01'- Ai: Ai is not closed.
b} If {fn}nEN C As and In ~ I then for y in [0,1] and n in N, there is
an Xn such that In (Xn) = y. If Xnk ---. Xo as k ---. 00 then I (x nk ) ---. I (Xo)
as k ---. 00. Because Iy - I (xnk)1 = link (X nk ) - I (xnk)1 ~ Il/nk - 11100, it
follows that I (xo) = y and so As is closed.
c) As depicted in Figure 82.2,
Solution 2.35 (cont.) 163

y·axis

(0, 1)

1
(0, 2:)

tL-_ _ _-----.l_ _ _ _.....L_ _ _ _.l....-_~ x-axis


(0,0) (I, 0)

Figure 82.2. Graph of the equation y = fn(x).

when n = 3,4, ... , the maps

3(n- 2)x
2n
fn; [0, 1J 3 x f---> X - ~
6 --+- 1 'f -<x
1 < -2
n 2 1
3 - 3

fn(X-~)+~ if~<x::;1
are in A b • On the other hand,

3x
2
1

and f rJ- Ab whence Ab is not closed.


164 2. Topology, Limits, and Continuity: Solutions

d) Because A is convex, it is connected.


e) The maps In : [0,1] 3 X f---+ x n , n E N, are in A, whereas no
subsequence of {In}nEN converges uniformly: A is not compact. D
2.36. If B is not connected there are in X two closed sets F1 and F2 such
that (F1 n B) L!J (F2 n B) = B and Fi n B =F 0, i = 1,2.
If Xi E Fi n B, i = 1,2, and I (Xl) = I (X2) ~f Y then YEA, Xl and
X2 are in the connected set C ~f 1- 1(y), C c B, C = (C n F1) L!J (C n F2),
and Xi E C n Fi , i = 1,2, contradicting the connectedness of C. Hence
I (F1 n B) L!JI (F2 n B) = A.
Since A is connected and neither of I (F1 n B), I (F2 n B) is empty, it
suffices to show that each is closed in A.
Thus let n : A -+ I (F1 n B) be a net converging to some y in A. For
each n(A) and for some x), in F1 n B, I (x),) = n(A), whence for the net
m : A 3 A f---+ X)" I(m) converges to y. Owing to the compactness of X,
for the net m, there is a cofinal diset A' such that miN
~f m' converges to
some X and thus I(m) converges to I(x). Because I (m') c I(m) and I(m)
converges to y it follows that I(x) = y whence X E B. Since F1 is closed,
X E F!, i.e., X E (F1 n B), y = I(x) E I (F1 n B). The same argument
shows I (F2 n B) is closed in A.
If X = Y = {O, 1} and the topology of X is 2x while that of Y is
{0, Y}, then for I : X 3 X f---+ X E Y the hypothesis above is fulfilled but Y
is connected whereas 1- 1 (y) (= X) is not. D
2.37. If E > 0, lR contains open sets {Un} nEN such that diam (Un) < E
and UnEN Un = lR. Since X is compact, for some N, X = U:=l 1- 1 (Un).
Each open set 1- 1 (Un) is a union of basic neighborhoods whence for finitely
many basic neighborhoods, say Vi, ... , Vp , X = U:=l Vp. Let Zl be V1 , and
Zp be (V1 u .. · u Vp) \ (V1 U .. · U Vp-1) , p = 2, ... , P. Then X = U:=l Z p-
If Zp =F 0, fix an xp in Zp- Each x in X is in precisely one Zp. Thus
9 : X 3 x f---+ I (xp) if x E Zp is well-defined.
Each basic neighborhood Vp is determined by finitely many indices
li,p, 1 s:: i s:: N p. Hence if x, y E X and x'Y;,p = Y'Y;,p then x and y are in
the same Zp and g(x) = g(y) = I (xp): 9 is finitely determined. If x E Zp
then II(x) - g(x)1 = II(x) - I (xp) I < E.
For n in N, there is a finitely determined gn with a corresponding finite
index setr nand SUPxEX II(x) - gn(x) I < ~. If x and y share coordinates
n
indexed by the countable set r ~f U nEN r n then

II(x) - l(y)1 s:: II(x) - gn(x)1 + Ign(x) - gn(y)1 + Ign(y) - l(y)1


1 1 2
::;: - +0+ - = -, n E N,
n n n
whence I is countably determined. D
Solutions 2.38-2.42 165

2.38. The function I is bounded above. Otherwise, for each n in N, there


is an Xn such that I (xn) > n whence the set {/- 1 ([n, 00)) : n EN} of
closed sets has the finite intersection property. Hence there is a y in their
intersection: for all n in N, I(y) ~ n, a contradiction. It follows that
def
SUPxEIR I (x) = M < 00.

The sequence {Fn ~f { X : I(x) ~M - ~}} consists of closed sets


and Fn ::l Fn+1. n E N. Hence there is an Xo in nnEN Fn and I (xo) = M.
D
2.39. The compact sets Li ~f K \ Ui , i = 1,2, are disjoint, whence
there are disjoint open sets Vi 'such that L1 C VI C U2 and L2 C V2 CUI'
Furthermore, Ki ~f K\ Vi are compact, Ki CUi, i = 1,2, and K 1 l:JK2 = K.
D
2.40. For each finite subset {'Y1,'" ,'Yn} ofr, each finite subset {tt, ... , tn}
of [0,1] n Q, and each m in N,

is a neighborhood in 2. The set of all such U ( ... ) is countable. Any


neighborhood in 2 is, for some {Sl"",Sn} contained in [0,1] and some
positive E, of the form

E. 1 E
If Iti - sil < 4,1:::; z :::; n, and m < "8 then

2.41. As the product of compact spaces, X is compact. If X is metrizable,


X is separable and hence (cf. 2.34) #(X) :::; c. However #(X) = 2' (> c).
As a function space, X may be endowed with the metric

d: X x X 3 {f,g} 1-+ sup I/(x) - g(x)1 ~f d(f,g) E [0,1].


xE[O,l]

(The reader is urged to use the language of cylinders to describe the topol-
ogy induced by d. The previous paragraph implies that any metric induces
a topology in which X is not compact.) D

2.42. If Y ~f N in the discrete topology then for alln in N, Y and yn


are homeomorphic (and countable). On the other hand, # (yN) = C > No.
D
166 2. Topology, Limits, and Continuity: Solutions

2.43. If {Fn}nEN is a sequence of closed sets and [0,1] = UnENFn , there


is an open set U2 such that F2 c U2 and U2 n Fl = 0. Some component
C2 (necessarily a closed interval) of U2 meets F 2 • On the other hand, if
C 2 c U2 then for some open interval (a, b), C2 C (a, b) C U2 , which implies
that C 2 is not a component of U2 • Thus U2 \ C 2 =f. 0 and so C 2 \ F2 =f. 0.
It follows that C 2 c U~=3 (C2 n Fn), whence for some n in N \ {1,2},
C2 n F n =f. 0. The argument just given and applied to the closed interval C 2
and then continued by induction, leads to a sequence {Ctn }:=2 of closed
intervals such that C tn :::) C tn+ b C tn =f. 0, C m n Fm - 1 = 0, m = 2,3, ....
Thus n:=2 Ctn =f. 0 while

a contradiction. o
2.44. Because F =f. ]Rn, the Cantor-Bendixson theorem implies of is
finite or countable. Thus for some M in N U No, there are in ]Rn points
X tn , 1 ~ m < M, such that B = {xtnh<tn<M' Assume that y E ]Rn \ F,
z E Fa, and N (z) C Fa. Because n 2': 2, the cardinality of the set L of line
segments joining points of N(z) to y is c and hence infinitely many of the
segments in L fail to meet the countable set of. Any such line segment
f is a connected set meeting both F and ]Rn \ F whence it meets of, a
contradiction. Thus Fa = 0 and hence F = of, i.e., F is countable or
finite. 0
2.45. Since, is rectifiable, if E > 0 for some N in N:

i. if m = 0, ... , N -1 and E[~, m; 1] the distance of ')'(t) from the


t

line segment , (~) , ( m; 1) connecting , (~) and , ( m; 1)


does not exceed E;
ii. the sum L of the lengths of the segments

,(~),(m;l), m=O, ... ,N-l,


does not exceed f (r).
It follows that for some constant Kn, ,*
is contained in a finite union of
n-dimensional boxes the total volume of which does not exceed KnEn-1f(r).
o
2.46. If -00 ~ a < 00 then (a, b) = UnEN ( a, b - ~] whence Tis at least
as strong as e: T:::) e. Because (a, bJ E T\ e, Tis (strictly) stronger than e.
Solutions 2.47-2.48 167

If two basic intervals intersect, their union is again a basic interval.


Hence if U E T then U is a union of a countable set of pairwise disjoint
basic intervals. It follows that if Tis separable there is a base consisting of
a sequence {(am' bm]} mEN of basic intervals. If b fj. {bn } nEN and a < b then
for some sequence {ndkEN' (a, b] = UkEN (a nk , bnk ]. Since b fj. {bn}nEN' for
some ko, anko < b < bnko ' whereas sUPkEN bnk ::; b, a contradiction: Tis not
separable. D
[ Note s2.3: The discussion above illustrates the distinction
between the notion of a space containing a countable dense set
and that of a separable space. Although JR is not separable with
respect to the topology T, Q is nevertheless a countable subset of
JR and is dense with respect to the topology T. If X is separable
and S consists of one point from each element of a countable base
of neighborhoods, then S is a copuntable dense set: each separable
space contains a countable dense set. For a metric space (X, d)
the notions coincide: X is separable iff X contains a countable
dense subset (cf. Solution 2.110).]

2.47. If U is an open set containing Q and x E JR, then for any N(x),
N(x) n U =f. 0 and N(x) n U is an open set not meeting A ~f JR \ U: A is
nowhere dense. Hence if Q is a Gli then lIJR is a set of the first category, a
contradiction. D
2.48. a) Because for all U in U, L,. c U, it follows that x E V",(U). Since
U is a filter base, V( x) is a filter base.
n
b) If 6. = UEU U and x and yare two points in X then {x,y} fj. L,.
and thus for some U in U, (x,y) fj. U and for some W, (y,x) fj. W. Some
Z in U is contained in W n U. The diagram in Figure 82.3 shows why
V",(Z) n Vy(Z) = 0.
Conversely, if T is Hausdorff topology and x and y are two points in
X there are in U a U and a W such that V",(U) n Vy(W) = 0. If Z in U is
contained in Un W then {(x, y)} U {(y, x)} rt z, cf. Figure 82.3.
c) If n : A ~ X is a net then (cf. 2.9) there is a cofinal net nlAI
converging to some x in X. Hence if U E U then for some A~ in A',
n (A') E V",(U) if A' »- A~. Choose in U a W such that WoW c U. If n
is a Cauchy net, then for some AO in A, (n(A),n(J.t)) E W if A,J.t »- AO. It
follows that n(A) E V",(U) if A »- AO, A~. D
168 2. Topology, Limits, and Continuity: Solutions

w
... '" ""

x y x

Figure s2.3.

2.49. If U, V E U then some W in U is contained in un V. It follows that


Ow >-- Ou, Ov· 0
2.50. If 0 is a cover of X then each x in X is in some U(x) belonging to O.
The set 0' ~f {U(X)}xEX is an open cover of X and # (0') :::; #(X). 0
2.51. If #(D) > 1, Z is a connected subset of D, and #(Z) > 1 then, since
D is closed, it may be assumed that Z is closed. If z E Z then z 1. UNEN VN
and for some N in N, zEN. Choose x and y in Z and an N(x) in N so
that y 1. N(x). Thus Z = (Z \ N(x)) l:.J (Z n N(x)) and neither summand
is empty. Because N(x) is open, Z \ N(x) is closed.
Furthermore, because VN ::) aN, N(x) \ VN(x) = N(x) \ VN(x) and thus
Z n N(x) = Z n (N(X) \ VN(x)) , i.e., Z n N(x) is closed and so Z is not
connected, a contradiction. 0
2.52. If 0 < a < b then for some mo in N,

{m > mol =} {(m + l)a < mb}

(otherwise a 2: b). Hence Um>mo (ma, mb) = « mo + 1) a, 00). Since G


is unbounded and open, it contains a nonempty interval (c, d) contained in
«mo + 1) a, 00), i.e., for some ml in (mo + 1, oo)nN, (mIa, mlb)n(c, d) i- 0,
i.e., (a, b) contains an Xl such that ml Xl E (c, d) (c G). Thus if

Am ~f {x : mx E G} and Dn ~f U Am
m~n
Solutions 2.53-2.56 169

then each Dn is dense in [0,00). Because G is open, each Am is open and


thus each Dn is a dense open subset of [0,00). However, D = nnEN Dn and
the Baire category theorem implies that D is a G6, dense in [0,00). 0

2.2. Limits

2.53. The inequalities


2N + 1 N <Xl <Xl

L an 2: 2a2 + L 2N - 1a2N , L (a2 n +l + ... a2 n + S L


1) 2na2n
n=l n=l n=l n=l
imply the assertion. o
2.54. For n, K in N,
K K
L an+k > L A n+k - A n+k - 1 = 1-~. (s2.2)
k=l A n+k - k=l An+K An+K
For large K, the last members in (s2.2) are near one, whence the leftmost
members of (s2.2) do not constitute a null sequence.
°
On the other hand, if < d, x < 1, the mean value theorem implies
1 - x d > d(l - x) whence if 2> r = 1 + d > 1

(s2.3)

The first member of (s2.3) converges. o


def (_l)n
2.55. It may be assumed that b1 > 1. If En = Ln b then
k=l k
<Xl

L( -l)nEn bn = 00 (2.54),
n=l
<Xl

L( -l)nbn converges,
n=l

whence L::"=l (-l)nb n (1 + En) ~f L::"=l (-l)na n diverges. o


2.56. For large n, I~: I < 1 and
170 2. Topology, Limits, and Continuity: Solutions

Furthermore, tin ~f 2::'1 (~:) 2k is small and positive and

For some N in N, if n > N then

2.57. i. Because 2::'=1 an converges absolutely, there is an autojection


IT : N 1-+ N such that alT'(n) ! 0, 2::'=1 alT'(n) = 1, and 8 for the latter series
is the same as that for the former. Thus it may be assumed that an ! O.
If, for some N, aN > 2::'N+1 ak then 8 fails to contain the interval
(2::'N+l an,aN)'
On the other hand, if an ::; 2::'n+l ak, n E N, and 0 <t < 1 then
for some nl in N, 8 1 ~f anl ::; t < a nl -l ::; 2::'nl ak. By induction, there
can be defined in N a strictly increasing sequence {np}l~p<P~oo such that
8 p- 1 ::; 8 p ~f anl + ... + a np ::; t ::; 8 p- 1 + a np -l ::; 8 p- 1 + 2::'n p ak.
Either P < 00 and 8 p = t or P = 00 and 8 p i 2:;':1 a np = t.
If t = 0 then t = 2::'10. ak and if t = 1 then t = 2::'11 . ak.
ia. The argument in i remains valid owing to the added hypothesis
lim n -+ oo an = O.
ii. For the map

() .. C0 3 x ~f
- ~~
2En(x)
3n 1-+
()()
X
~
-
~ ()
~ En X an,
n=1 n=1

if'T/ > 0, there is an no such that 2::'=no+l an < 'T/. If Ix - x'i < 3-(n o+1)
then IB(x) - B (x')1 < 2::'=no+1 an < 'T/ whence B is continuous.
ror a gIven
10' •
= ~oo
a def L.."n=1 Enan , 1'f X = ~oo 2En then ()()
L.."n=1 ~ .
x = a, 1.e., () IS
.
a continuous surjection of the Cantor set Co onto 8.
If x > x', there is an no such that

if n ::; no
ifn=no+1
()(x) - () (x') ~ ano +1 > 0
Solutions 2.58-2.59 171

whence () is a strictly monotonely increasing function. Since Co is compact,


() : Co ~ 8 is a homeomorphism. (Thus 8 is a perfect nowhere dense subset
of [0,1]).
.. If
zza.
def h
1 > a = 1 - r > t en arn ° -1 k ar n '
~ Lm=k ar = 1 _ r 1ff r ~ 2
,,",,00 1

( whence ar n - 1 > ,,",,00 k arn. 1)


un=k ar = - - 1ff r < - .
1-r 2
°
iii. If r ~ ~ t~n 8 = [0,1] and ..\(8) = 1. If ~ r < ~, [0,1] \ 8 is
the union of pairwise disjoint open intervals, the first of length 1 - 2r, each
of the next two of length (1 - 2r)r, ... , each of the next 2n - 1 of length
(1- 2r)rn-1, ... , and ..\(8) = 1- L::':'=1(1- 2r)(2r)n-1 = 0.
iv. By induction, there can be defined au ~f a1, a12 ~ the first ai
= the fi rst ai such that a12 > ai and au > a12 + ai, etc.
1ess than au, a13 def
The process is repeated on the sequence {an}nEN \ {a1Q}qEN' to produce a
sequence {a2Q}QEN' ....
v. The condition implies that for every subsequence {a pQ } QEN as de-
scribed, ap1 > L::;:2 apQ . 0
2.58. The series converges resp. diverges according as limn-+ooqn > 1 resp.
< 1. However, if P is the set of primes in N then
limn-+ooqn

" 21 converges although hmn-+ooqn


'~ . = 0,
nEP n

L ..!:.n diverges although limn-+ooqn = 1. o


nEP

2.59. If In : llt 3 X ~ L:~:~(1 - x)n-1 then xln(x) = 1- (1 - x)n and


00 00

M ~f L (1- (1- Tkt) = LTkin (2- k ).


k=l k=l
Direct integration and graphical considerations imply

Inn
Because n ~ 2, 1 ~ In 2 and so

n 1 (l
In(n + 1) < L - = 10 In(x) dx ~ M,
p=l P 0
172 2. Topology, Limits, and Continuity: Solutions

1
"2 + "2 M
1
::;
11 f n (x) dx = L -
1
n
< 1 + In n,
o p=l P

In (n + 1) ::; M ::; 1 + 21n n, In n ::; M::; (2 + 1: 2) In n. 0

2.60. If Pn(x) ~f L~l ankxk, n E N, and the M + 1 points at which


convergence takes place are a1, ... , aM+b the determinant of the Vander-
monde matrix

V ~f (~~1 a~ )
1 aM aM+1
is TI1:S;k<I:S;M+1 (al - ak) which is not zero because the am are pairwise
different. Thus, if limn -+ oo Pn (am) ~ Am then

and f(x) = limn-+coPn(x) = L~l akxk.


On the other hand, the Weierstrafi approximation theorem implies that
if M = 00 then f need not be a polynomial. 0
2.61. a) If x E Q, then for large all large m, mIx E 2N whence 8 m = 1
and the right member is one. If x ¢. Q then for each m, Icos (m!x7r) I < 1
whence 8 m = 0 and the right member is zero.
b) If x = 0 then arctan (nx) = 0; if x > 0, limn -+ co arctan (nx) = if i;
x < 0, limn -+ co arctan (nx) = - i.
c) If En ~f L~=n+1 ~! then (n + l)!En - 1 ~f on 10. Thus

n (1n+l
+ On) sin (27r (1 + On ) )
n+l
nsin(27ren!) = nsin(27rE n n!) = --------:::---'-..,,--'----'-''- ---- 27r
1 + On
n+l
as n ---- 00.
If e E Q then for some n, en! E N and if m > n then m sin (27rem!) = 0,
a contradiction.
Solutions 2.62-2.64 173

d) If x E Q, for all large m, sin 2 (m!7rx) = OJ if x ~ Q, sin 2 (m!7rx) > 0.


D
2.62. For x in [0,00),

As x i 00 the integrands in I and I I decrease monotonely to zero. The


dominated convergence theorem applies. D
2.63. For a in (0,1), each

Ekn(a) ~f {x : x E [2k, 2(k + 1)7r], Isin (x + r~7r) I > a } , k, n E N,


is the union of two disjoint intervals of total length £( a) that is independent
of both k and n and decreases monotonely to zero as aiL Thus

1 2(k+1)1r I - (. (
2k1r
e x sm x+ -rn7r))nl dx
n

~ lkn(a) + l2k1r,2(k+1)1r]\Ekn(a)
~ 2e- 2k1r (1 - e-£(a)) + 2an (2 + e- 21r ) e- 2k1r ,
11 00
e- X (Sin (x + r~7r) ) n dxl

(1 - e-£(a)) L e- 2k1r + 2an (2 + e- 21r ) L e- 2k1r


00 00

~2
k=O k=O

~ 2 ((1 _
1 - e- 27r
e-£(a)) + a n (2 + e- 21r )) ~f 2
1 - e- 21r
(I + II).
If a in (0,1) is near 1, I is small and for such an a, II is small if n is
large. D
2.64. If f E (0,1) then

11 - e(ex)21 ~ 2fxe(ex)2 and 11 _ e(ex)21 e- x3 ~ 2fxe- x2 (x-e 2 ).

The integrand converges to zero on [0,00), is bounded on [0,2]' and is


bounded on [2,00) by an integrable function. The dominated convergence
theorem applies. D
174 2. Topology, Limits, and Continuity: Solutions

2.65. The dominated convergence theorem implies the limit is zero. D


2.66. If 0 < E < 1 - fi < 1, the change of variable x -+ 1 - y yields

1 E
1-6 I(x) dx = [6 (1 - y) In(l - y) dy
J1-E-y

= f n2(~n_ I)}
{-y + n=2 1
6

1-£
_ ~ fin - (1 - E)n
- 1 - fi - E +~ n 2(n _ 1) < 00.
n=2

If € ~,fi = ~,p,
p
=
q
q EN, the dominated convergence theorem permits a
passage to the limit as p, q -+ 00. D
2.67. Because ~:=1 Itmnl :S M, the infinite product
converges for all Z in C and represents an entire function 1m.
n:=l (1 + tmnz)
If x E (-1, 1) then

00 (_l)n+1xn def X2
In(l+x)=2: =x--(a(x)),
n=l n 2

1+x = exp (x _ ~2 a(x») .

Hence there are functions amn(z) such that if Izi < ~ then for some constant
K, lamn(z)1 :S K and 1 + tmnz = exp (tmnz - t~;z2 amn(Z»). Thus if
1
Izl :S 2 then
00

2: It~namn(Z)1 < 00,


n=l
00

Sm(Z) ~f 2: t~namn(z) exists,


n=l
00

ISm(z)1 :S K2:t~n (-+ 0 as m -+ 00),


n=l

=
Im(z) def II (1 + tmnZ) = exp (~sm(z»)
~ tmnz - - 2 - -+ eZ as m -+ 00.
nEN n=l
Since 1m and exp are entire functions, limm->oo Im(z) = e for all z in C.
Z

D
Solutions 2.68-2.71 175

2.68. If Sn i S < 00, the convergence-preserving properties of Toeplitz


matrices imply that limm--+ DO am = s.
If Sn i 00 then for each positive M there is and N M such that Sn > M
if n > NM. Thus

NM DO

am = ~ tmnsn + ~ tmnsn
n=l n=NM+1

As m ~ 00 and NM is fixed, 8 1 ~ 0 and 8 2 ~ M, whence am ~ 00. 0


[Note 82.4: Absent the monotonicity of {sn} nEN' the conclusion
lim n --+ DO an = S can fail to obtain. If, e.g., Sn = (_1)n+1n, n E N,
1
then a2n = -2' a2n-1 > 0, and limn --+ DO an does not exist.]

2.69.
N
For Ln=l --;;
an def
= SN, { LN_1
nN
Sn def}
= aN .
IS the result of ap-
NEN
plying the special Toeplitz matrix T1 ~f {t mn }:,n=l for which

if1~n~m

otherwise

1 N
more, N ~an =
n=l
summation yields

A N =-N_1
1 -1 - 1) +sN~-s·1+s=0
(N~sn . (N~
)

as N ~ 00. o
2.70. For all real x, Isinxl ~ Ixl whence IU n+11 ~ Iunl (~ 1), n E N. Thus
limn --+ DO lunl ~f A exists. Let {Pm}mEN resp. {qm}mEN be the sequence of
positive resp. negative terms in {un} nEN' Then Pm ! P 2:: 0 and qm i q ~ O.
Owing to the continuity of sin, sinp = P and sinq = q, whence P = q = o.
o
2.71. If x > 1 then fn+1(X) > fn(x) and if a as described exists and is
finite then 1 < a = x a , X = ai whence 1 < x ~ sUPa>l aI/a = e 1/e.
176 2. Topology, Limits, and Continuity: Solutions

If 0 < x ::; 1 then 1 ;::: 12(n+l){X) > hn{x) > 0 whence limn->oo In (X)
exists and is finite. D
2.72. The ratio test implies that for any real a, the series converges
absolutely and uniformly in every closed subinterval of (-1, 1); if a = 0 or
a E N, the same conclusion obtains on [-1,1] itself.
I I
If a E (0,00) \ N and an ~f (~) then for n in ([a] + 1,00),
nan - (n + 1)an+1 = aan > 0
whence for some "( in [0,00), nan 1 ,,(, and

The WeierstraB M-test implies the uniform and absolute convergence of


E~=o (~)xn on [-1,1].
For la{x) ~f E~=o (~)xn, a in 1R, and x in (-1,1), the following
equations may be derived by valid term-by-term differentiation and multi-
plication by 1 + x of the convergent power series representing 10.-1 (x) and
I~(x): I~ = ala-I, (1+x)/a_l(X) = la(x), (1+x)/~(x) = afa{x). Hence

the derivative of F{x) ~f (fa(x~ a is zero, i.e., F is a constant. Further-


1+x
more, F(O) = 1. D
[ Note 82.5: If LEe ([-1,1]' iC) and L is piecewise linear then
L is a linear combination of translates of p : t f---* t and q : t f---* Itl.
1
Because It I = (1 + (t 2 - 1) p.,
q is uniformly approximable by
polynomials. Since every 9 in C ([-1,1]' iC) is uniformly approx-
imable on [-1,1] by functions like L, there emerges the WeierstraB
approximation theorem:
Every 9 in C ([-1,1]' iC) is uniformly approximable on
[-1,1] by polynomials.
The WeierstraB approximation theorem is a vital ingredient in the
proof of the Stone-WeierstraB theorem.]

2.3. Continuity

2.73. = UnEZ A n [n, n + 1], it follows that A is the union of


Since A
compact summands Kn: A = U nEN Kn. Since I is continuous, each f (Kn)
is compact, whence closed, and I(A) = UnEN f (Kn), an Fu· D
2.74. Because V is a union of open intervals (am, bm ), mEN, and each
(am, bm ) is a union of closed intervals [am + ~,bm - ~] ,n E N, it follows
that V is an Fu and thus I(v) is also an Fu (cf. 2.73). D
Solutions 2.75-2.77 177

2.75. The function 9 : 'lI' :3 z f-+ J(z) - J( -z) is such that g(z) = -g( -z).
Hence 9 ('lI') is a connected set containing meeting both [0,00) and (-00,0],
whence 0 E 9 ('lI'). D
2.76. If x '" y iff J(x) = J(y) then '" is an equivalence relation and
if 3 ~f [0, III "', there is a map () : [0,1] :3 x f-+ xl ",~f e
E 3. For
T~f {()-I(U) : U open in [0,1] }, (3,1) is a topological space,

are well-defined and continuous, and the diagram below

Figure 82.4.

is commutative. Furthermore, J is injective and, owing to the definition


of T, 1-1 is continuous, i.e., 1 is a homeomorphism. For 901-1 ~f G,
if U is open in [r, s] then 9- 1 (U) is open in 3 and leU) is open in [p, q]:
G ~f 901-1 is a continuous map of [p, q] onto [r, s].
Stripped of its disguise, the discussion above shows that for y in [p, q],
the equation G(y) = g(x) defines G(y) uniquely and that G is continuous
on [p, q]. The Weierstra13 approximation theorem implies that there is a
sequence {Pn} nEN of polynomials such that Pn ~ G on [p, q], i.e., Pn 0 J ~ 9
on [0,1]. D
2.77. i. Let G be the set of all 9 as described. From 2.76 it follows that
there is a sequence {Pn} nEN polynomials such that Pn 0 J ~ g. Since L is
linear and continuous, 0 = L (Pn 0 f) --+ L(g) as n --+ 00.
ii. Let H be the set of all h as described. For each n, J{n} is a con-
tinuous, odd, and piecewise linear function. The zeros of J{n} are ±xn and
limn -+ oo ±xn = ±1. The graph of y = J{n}(x) is a polygon consisting of
three segments: ((-1, -1)( -Xn, 0)), [-xn' x n ], and ((xn' 0), (1, 1)). Fig-
ure 82.5 is offered to provide a graphical presentation of the form of J{n}.
Note that limn -+ oo Xn = 1.
178 2. Topology, Limits, and Continuity: Solutions

Y = f {n)(x)

Figure 82.5.

The linear span S of {I{n}} nEN is a set of continuous, odd, and piece-

wise linear functions, each vanishing on [- ~, ~ ]. The graph of each h in H


is uniformly approximable by polygonal graphs corresponding to functions
in S: S is II lloo-dense in H. Because L is continuous, for each such h in H,
L(h) = 0. D
2.78. Since the R-span RN of AN is a separating algebra of functions in
C ([0,1], R), the Stone-WeierstraB theorem implies that RN is II lleo-dense
in C ([0,1], R). Consideration of the real and imaginary parts of functions
in C ([0,1], C) shows that SN is II lloo-dense in C ([0,1], C). D
2.79. Fejer's theorem for Fourier series implies the WeierstraB approxi-
mation theorem and thus implies both conclusions. D

2.80. In A ~f {I : 1 E C([O, 1], C), 1(0) = O} there is a II lloo-dense


sequence {gn}nEN' For n in N, there are polynomials Pl,P2,'" , so that a)
deg(P1) ~f db P1(0) = 0, and IlgI-P111oo < 2- 1, and b) after Pb ... ,Pn
have been chosen so that deg (Pk) ~f d k , 1 :S k :S n, and
Solutions 2.81-2.84 179

there is a polynomial Pn+ 1 such that

Thus L::~=l Pn (x dn - 1 +1) is a power series L::~=l anxn. If I E A then there


is a subsequence {gnkhEN such that gnk(X) ~ I(x) whence there are mk
such that L::;:'::l anx n ~ I(x) as k -+ 00. D
2.81. If I(x) -1+ 0 as x -+ 00 there is a sequence {xn}nEN and a positive
8 such that Xn i 00 and II (xn)1 ~ 28. Because I is continuous, each Xn
belongs to an interval (an, bn ) where I/(x)1 > 8. Hence G ~f U nEN (an, bn )
is an open set unbounded above. Hence for G, D as in 2.52 is dense in
[0,00) and for some h in (a, b) n D and infinitely many n in N, nh E G.
Thus infinitely often, I/(nh)1 > 8, whereas I(nh) -+ 0 as n -+ 00. D
2.82. a) For n in N, let En be 1;;1(0). Then each En is closed while
the hypothesis implies UnEN En = [0,1]. The Baire category theorem im-
plies that some Eno is not nowhere dense and hence contains a nonempty
subinterval (a,b). For x in [a,b], 0 = Ino(x) = foX Ino-l(t)dt. Further-
more, I~ = In-l whence on [a,b], Ino-l(X) = Ino-2(X) = ... = lo(x) = 0
and so (a, b) C [a, b] cEo.
b) If 1/0(0)1 = 8 > 0 then for some positive b, I/o(x)1 ~~ if x E [O,b].

Thus for each n in N, I/n(b)1 ~ ~bn > 0, whereas for some nb, Inb(b) = 0,
a contradiction. Thus 10(0) = O.
If 0 < b:::; 1, lo(x) "I- 0 in [0, b]' and lo(x) does not change sign in [0, b]
then for each n in N, In(x) =I=- 0 on (0, b], a contradiction when n = nb. It
follows that if 0 < b :::; 1 then at some bl in (0, b), 10 changes sign, at some
b2 in (0, bl ), 10 changes sign, ... , whence 10 changes sign infinitely often in
(O,b] and thus lo(x) = 0 infinitely often in (0, b). D
2.83. For each n in N, let En be {x : I(n)(x) = O}. If x E (0,1) then
x E En", whence U~=oEn = (0,1). Because each En is closed, the Baire
category theorem implies that some EN contains a nonempty open interval
(a, b), i.e., 1(N)(x) == 0 on (a, b). By successive integration it follows that
there are constants Co,Cl, ... ,CN-l such that on (a, b), I(N-l)(X) = Co,
I(N-2) (x) = cox + Cl, ... , and
N-l + N-2 + +
I( X ) = (N Co
_ 1)!x
Cl
(N _ 2)!x ... CN-l· D

[Note 82.6: Compare the argument above with that for 2.82a).]
2.84. If 0 :::; x < y :::; 1 and g(x) = g(y) then for m as described,
x = g{m} (x) = g{m}(y) = y, a contradiction. Thus 9 is strictly monotone
180 2. Topology, Limits, and Continuity: Solutions

and since g(O) = l-g(l) = 0, 9 is strictly monotonely increasing. If g(x) ~x


for some x then g{m} (x) ~ g{m-l} ~ ... ~ g(x) ~ x, a contradiction. 0

2.85. a) If f(x) = {~
if 0 < x, then f is left-continuous
ifx<O o
and unbounded on any open interval cOntaining O.
b) If f(x) is unbounded in [0,1] then for some sequence {xkhEN in
[0,1]' limk->ooxk ~f Xo exists and If(Xk)1 > k. Thus there emerges the
contradiction: limx=xof(x) = 00 t- f (xo). 0
2.86. If IIflloo ~f 8 ~ 1 then for some a in (0,1]' If(a)1 = 1 and if
0:::; x < a then If(x)1 < 1. If 0 < € < 1 and if Sis equicontinuous, there is
a positive 8 such that if Ix - al < 8 then for all n in N,

€ > Ir(a) - r(x)1 ~ Ir(a)I-lr(x)1 = l-lr(x)l·

However, for large n and x in (O,a), Ifn(x)1 < 1- € whence there emerges
the contradiction 1 - € > Ir(x)1 > 1 - €.
Conversely, if IIflloo ~ € < 1 then for x,y in [0,1],

2.87. If, for n in Nand f in C ([0, 1], C), Ln(f) =


def t0
Xn f(x) dx
1 then
fo xndx
Ln E C ([0,1], C)* and IILnll :::; 1. If p is a polynomial and p t- 0, then
limn->oo Ln(P) = p(I). If fEe ([0,1], C) then

The Weierstrafi approximation theorem implies limn->oo Ln(f) = f(I).


o
2.88. If SUP!EA II!"II ~f K < 00 then for f in A and all x in [0,1],

II'(x)1 :::; 11'(0)1 +K 1 x


1 dt :::; 11'(0)1 + K,

f'(O) = f(l) - f(O) -1 (1 1 X


!,,(t) dt) dx,
K 3K
If'(O)I:::; 2+ 2' M < 2+ 2 < 00.
On the other hand, if K = 00 then for each n in N, and for some fn and
some Xn in [0,1), If:: (xn)1 > n. Iffor some Xo and some n, If:: (xo)1 ~ n-l
then, since Ilf::'ll oo :::; 1, for any x, If::(x)1 :::; If:: (xo)1 + f:o If::'(t) I dt :::; n,
Solution 2.88 (cont.) 181

I::
a contradiction. Hence for all n in N and all x in [0,1], I (x) I > n - 1
and, since I:: is continuous, it must be of one sign, say I::(x) > n - 1,
throughout [0, 1].
1"(0) 1
If In(x) = In (0) + 1~(O)x+ ~x2 +R2(X) then IR2(X)1 ~ "6 in [0,1].
The hypotheses imply

Hence if 1::(0) ~f an, and 2/~(0) ~f bn the discussion turns on behavior


of the quadratic polynomial an x 2+bn x on the interval [0, 1]. Since an ~ n-l
the graph of the equation y = an x 2 +bn x has one ofthe four forms indicated
in Figure 82.6.

Case I Case II
O<
-
-~ <1
2a -
O<
-
-~ <
2an -
1
n

y-axis y-axis

x-axis

y-axis

Case III Case IV

y-axis

x-axis x-axis

Figure 82.6.

In Cases I and II,


b2
-5 < -4 n , -2.J5an < bn < 0, an - 2.J5an < an + bn ~ Ian + bnl ~ 5.
an
Upon division by ..;a;., there emerges the inequality n - 1 ~ an ~ 20, a
contradiction if n > 21.
182 2. Topology, Limits, and Continuity: Solutions

In Case III,

a contradiction if n > 7.
In Case IV, bn > 0 whence

a contradiction if n > 7. Hence K = 00 is an impossibility. A similar


argument applies if it is assumed that f~ (x) ~ 1). -en -0
2.89. Note that for a) and c), Co (X,·.·) = C (X,,· .).
a) The WeierstraB approximation theorem implies that P is true.
b) For f : [0, (0) 3 x t-+ e- x2 E C, lim x_ oo f(x) = 0 whereas for
any nonconstant polynomial p, lim x_ oo Ip(x) I = 00 and, for a constant
polynomial p, sUPo~x<oo If(x) - p(x)1 ~ ~: P is false.
c) The function f : X 3 Z t-+ Z is continuous on X but not holomorphic
in xo, whereas any uniform limit of a sequence of polynomials is holomor-
phic in xo: P is false. 0
2.90. The Stone-WeierstraB theorem implies that the compactness of F is
sufficient while Solution 2.89b) shows that the compactness of F is also
necessary. 0
2.91. There is a positive 8 such that if Ix - yl < 8 then If(x) - f(y)1 < 1.
If x a, b) then Ix- a; b I ~ b; a= [b ~ a]
E ( ~. If N 3 no > b ~ then a
. . a + b def
for any x III (a, b) and for some m III N n [1, no], between -2- = Xl and

x ~f x m , there are X2,'" ,Xm -1 for which IXk+1 - xkl < 8, 1 ~ k ~ m - 1.


Hence k(x) - f (a; b) I ~ 2:::;;'=1 If (Xk) - f (xk-dl ~ no· 0

2.92. For x in JR, let F(x) resp. G(x) be f (e 21t"iX) resp. 9 (e 21t"ix). Then

Iv f(z)g (zn) dr(z) = 11 F(x)G(nx) dx


1 (H1 (k)
~ ;;: L ik F;;: G(x) dx
n-1

k=O

~ (;;:1~ F;;:
(k)) io(1 G(x) dx.
n-1

The last expression converges to 101 F(x) dx 101 G(x) dx. o


Solutions 2.93-2.97 183

2.93. The Arzela-Ascoli theorem implies that the closure Conv(A) of the

i
convex hull of A is an equicontinuous subset of B (0,11/1100)'
If F(x) ~f I(xy) dr(y) then, since r (1I') = 1 and r is a nonnegative

in 1I', F(zx) i.e., F is constant and C(f) = i


measure on 1I', F E Conv(A). Because r is translation-invariant, for all z
= F(x), I(y) dr(y).
If G is constant and in Conv( A) then G is uniformly approximable by
linear combinations of translates of f. Again the fact that r (1I') = 1 and
integration show that for any positive D, IG - C(f)1 < D. 0
2.94. Because UnEN {x : sUP!EF I/(x)1 ::; n} ~f UnEN Un = lR, at least
one Un, say UM, contains a nonempty open set V. If x E V then for some
{x n } nEN contained in UM, limn-;oo Xn = x whence

However, for I in F,II (xn)1 ::; M. D


2.95. For L in C (X, q*, the Riesz representation theorem implies that for
some complex Borel measure J-L and all I in C(X, q, L(f) = Ix I(x) dJ-L(x).
The bounded convergence theorem applies. D
2.96. Since AIR, consisting of the elements of A that are formed with
the use of lR-valued functions Ii, gi, is a separating sub algebra of C (X, lR),
it follows from the Stone-WeierstraB theorem that AIR is II lloo-dense in
C (X, lR). Moreover, A = AIR + iAIR · D
2.97. a) If x E Cont(f) then for each n in N, there is an Nn(x) such that
d (f(y), I(x)) < .!.n if y E Nn(x). For each n in N,

Cont(f) C On ~f U Nn(x)
xECont(f)

whence Cont(f) C nnEN On ~f O.


On the other hand, if y ¢. Cont(f), there is a positive f such that in
each neighborhood N(y) there is an x such that d(f(x),/(y)) ~ f. Choose
N in N so that ~ < ~. If yEO then y E ON and for some z in Cont(f),
y E NN(Z). There is a neighborhood N(y) contained in NN(Z) and ~r N(y)
there is an x such that d(f(x),/(y)) ~ f. Hence,

1 1
f::; d(f(x),/(y))::; d(f(x),/(z)) + d(f(z),/(y)) < N + N < f,

a contradiction. Thus {y ¢. Cont(f)} ::::} {y ¢. O}, Cont(f) = 0, a Go.


184 2. Topology, Limits, and Continuity: Solutions

b) The sets

Fkm ~f [D. {x , I/m(x) - 1.(x)1 ~ ~ }] ,


Fk =
def U Fkm , Gkm = Fkm , and
def 0

mEN

G k ~f
- U Gkm, G~f- nGk
mEN kEN

are such that:


I> each Fkm is closed because the fn are continuous;
I> each Fk is X because the f n converge everywhere;
I> if Fkm = 0 then Fkm is nowhere dense because Fkm is closed;
I> the set Rkm ~f Fkm \ Fkm is closed and its interior is empty, whence
Rkm is nowhere dense and Rk ~f UmEN Rkm is of the first category;
I> each Gk, as a union of open sets is open, and G, as the intersection of
a countable set of open sets, is aGo.
If G = 0 then G C Cont(f). If x E G then for each k in N and some
mk in N, x E Fkmk , whence /fmk(X) - fn(x)1 ~ ~, mk ~ n. Because
Fkmk is open (and nonempty), it contains an N(x) and for every z in N(x),
1 1
Ifmk (z) - fn(z)1 ~ k' mk ~ n, whence Ifmk (z) - f(z)1 ~ k and so

If(z) - f(x)1
~ If(z) - fmk(z)1 + Ifmk(z) - fmk(x)1 + Ifmk(x) - f(x)l. (s2.4)

1
The first and third terms in the right member of (s2.4) do not exceed k.
Owing to the continuity of fmk' N(x) contains a neighborhood W(x) such
that if z E W (x) then the second term is less than ~. Hence at each point
x of G, the limit function f is continuous: Cont(f) ::) G.
c) When X is a complete metric space then, since Rk is nowhere dense
and G k = X \ Rk, G k is dense. The set G k , as a union of open sets, is open
and so Gk is a dense open set. Baire's Theorem implies that G is dense.
It follows that Cont(f) is both dense and a Go· 0
2.98. a) If f is an extreme point in B(O, 1) then Ilflloo = 1.
If If(x)1 == 1 then sgn(f)f = If I = 1. If g, hE B(O, 1) \ {f}, g =1= h,
t E (0,1), and If I = tg+ (1- t)h then for some a, g(a) =1= h(a) and thus near
a, h(x) =1= g(x). Because every point on '][' is an extreme point of D(O, 1),
f(a) =1= tg(a) + (1 - t)h(a). Hence f is an extreme point of B(O, 1).
Solution 2.99 185

If 1 is an extreme point of B(O, 1) and I/(x)1 ¢ 1 then for some a,b,c


in (0,1), a < I/(b)1 < c and for some positive 10, if Ix - bl < 10 then x E (0,1)
and a < I/(x)1 < c. For some D in B(O, 1), D ~ and °
D(X) = {1° if
if
Ix - bl < ~ .
Ix - bl > 10
Then for small 'fI,

a contradiction.
In sum, 1 is an extreme point iff I/(x)1 == 1.
b) As in a), 11/1100 = 1. If I/(x)1 == 1 then X is compact and the
argument in a) shows that 1 is an extreme point.
If I/(x)1 ¢ 1 then maxxEX I/(x)l- minxEx I/(x)1 ~ 3~ > 0. The sets

E M def
= {
x I/(x)1 ~ max
xEX
I/(x)l- ~} ,

Em ~f
- { X min I/(x)1 + ~} ,
I/(x)1 ::; xEX

are disjoint, nonempty, compact, and contained in disjoint compact neigh-


borhoods NM resp. N m . Since X is connected, X \ (NM U N m ) =f. 0.
In Co(X,q there is an 10 such that E(X) ~ 0, EINMl:JN", = 0, 10 =f. 0, and
11101100 < ~. The functions 9± ~ 1 ± 10 are in B(O, 1), neither is I, and
1=~9++~9-.
In sum, if X is a connected, locally compact and not compact Hausdorff
space then there is no extreme point in B (0, 1). D
[Note 82.7: For C ([0, 1], q, the cardinality of the set of extreme
points of B(O, 1) is c. However, in C ([0, 1], JR), the set of extreme
points of B(O, 1) consists of the two constants ±l. By contrast,
if X = N, then Co (X,JR) = Co and each vector en ~f {Dmn}mEN
is an extreme point of Bd

2.99. a) If X = Co (the Cantor set) then, since Co is totally disconnected,


for each x in Co, ICo1 (x) contains a component. It follows that

c~ # (C) ~ # (Co) = c.
b) Each component of A is either a point or a nondegenerate closed
interval. There can be no more than count ably many components of the
186 2. Topology, Limits, and Continuity: Solutions

°
form [an, bn ], S; an < bn S; 1,1 S; n < N S; 00, since components are
pairwise disjoint and each of the form described meets Q. Thus

is closed and nowhere dense and a) implies #(B) = c. The Cantor-


Bendixson theorem implies that B is the union of a set P homeomorphic to
Co and a countable set D: B = Pl:.JD. Furthermore, the self-dense kernel
of D is empty. If U ~f Ul:5n<N (an, bn ) then P, D, and U are pairwise
disjoint and A = Pl:.JDl:.JU. If 1 S; n < N then fe o ([an, bnD is a single point
and thus fe o(B) = feo (A).
c) Because every compact metric space X is the continuous image of
Co (cf. 2.15), there is a continuous surjection hx : Co ~ X. Hence if
gx ~ hx 0 feo IB then gx is a continuous surjection of Bon X. 0
2.100. If fo °
= and for n in N,
. 1
nx IfO<x<-
- - n
fn : [0,1] :3 x f--+ if .!.n < x S; 2n ,
. 2
If-<x<1
° n -

then IlfnlLx> S; 1 and lim n-+ oo fn(x) == 0. If L E C ([0, 1], C)* then for some
complex measure J.L on [0,1],

{f E C ([0,1], Cn =? {L(f) =
JrrO,lj f(x) dJ.L(X)} .
The bounded convergence theorem implies limn -+ oo L (fn) ° whereas

fn(~)==1. o
2.101. For Ln : C ([0,1], C) :3 f ( 1)def
f 1 -;;; and L = 2:n=1
f--+
00 (-1)nLn
2n '
if IIfiloo S; 1 then IL(f)1 s; 2::'=1 2- = 1, i.e., IILII s; 1. For each k in N
n
and some fk in C ([0, 1], C), IIAlioo = 1 and

A (1 _ .!.) = { (_1)n ~f n = 1,2, ... , k


n ° If n = k + 1, k + 2, ...
.

Then L (fk) = 1 - 2- k whence IILII = 1.


On the other hand, if IIfiloo S; 1 and IL(f)1 = 1 then for some () in
[0,27l"], L(f) = eiO , L (e-iOf) = 1. For all n in N,

e- io(-1)nLn (f) E D(0,1).


Solutions 2.102-2.104 187

Assume that for some no, e-''0 (-I)noLno ( f) = a =f.1.


def
Then

b ~f Ln#no Ln(f) E D(O 1) 1 = L (e-iOI) = _1 a + (1 - ~) b


1" 2no 2no '
1--
2n o

a contradiction since 1 is an extreme point of D(O, I), It follows that


e- iO ( _1)n Ln(f) == 1 and so limx--+1/(x) does not exist, a contradiction.
Hence there is in C ([0,1], C) no I such that 11/1100 ~ 1 and L(f) = 1. 0
2.102. a) Let Q be the set of all polynomials with rational complex co-
efficients. Then A ::> Q and the WeierstraB approximation theorem implies
A is II lloo-dense in C ([0,1], C).
If {rk hEN is an enumeration of Q then for {k, n} eN,

is closed in C ([0,1]' C) and A C U{k,n}CN Ekn ~f B. If IE Band kEN,


then I (rk) E Q whence I E A: A = Band B is an Fu .
b) The set of all polynomials with exactly one irrational coefficient
is also II lloo-dense in C ([0, 1], C) and no such function maps Q into Q:
AO = 0. 0
2.103. As a closed subspace of a Banach space, A is itself a Banach space.
The distributive character of multiplication implies Mg is linear. For x in
[0,1] and T", : A 3 I I-t g(x)/(x) E ee, it follows that IT", (f) I ~ Ig(x)1 .
11/1100 whence T", E C([O,I],C)*. Because Mg(A) c A, sUP"'E[O,l] IT", (f) I =
Ilg/lloo. The uniform boundedness principle applies to the set {T",} "'E[O,l]
of linear functionals: SUP"'E[O,l] IIT",II ~f M < 00. Hence

Ilg/lloo = sup Ig(x)/(x)1 ~ Mll/iloo.


"'E[O,l]
o

°
2.104. a) For 9 in C ([0,1], C) and I in A, g·1 (!) = whence A is an ideal.
If A is a principal ideal then for some k in C ([0, 1], C), k· C ([0, 1], C) = A.
°
Hence k (1/2) 1 (1/2) = k (1/2) . 1 = and so k (1/2) = 0. If Ikl ~f r then
both rand Vr are in A whence for some gin C ([0, 1], C), kg = Vr. Since
Vr 2: 0, it follows that Vr = kg = Ikgl = Ikl . Igl = rlgl whence Igl = r-~.
However, since r G) = 0, Ig(x)1 is unbounded and hence 9 r;. C ([0,1]' C), a
contradiction: A is not a principal ideal.
b) If go(x) ~f x - ~ then go . C ([0, 1], C) contains the set of all poly-

nomials vanishing at ~. From the Stone-Weierstral3 theorem it follows that


go· C ([0, 1],C) = A. 0
188 2. Topology, Limits, and Continuity: Solutions

2.105. a) Because BK C AK and AK is closed, it follows that BK C A K .


There is a sequence {Vn }nEN of open sets such that Vn ~ Vn+l, n E N, and
nnEN Vn = K. For n in N and some In in A K , K -< In -< Vn.

For I in A K , min N, and some open set Um containing K, I/(x)1 < ~


1 m
if x E Um . If E > 0 and - < mo E N then for some no greater than mo,
E
Vn C 1m . I E BK and 111m' 1- 11100 < E.
Um if n > no. If m > no then
Hence BK = A K .
b) As in 2.104 it follows that if K =f. 0 then AK is an ideal. The
argument showing that A in 2.104 is not a principal ideal may be repeated,
mutatis mutandis, to prove that AK is not a principal ideal. If K = 0 then
AK = C ([0,1], C), which by definition is not an ideal. 0
2.106. a) If a E X and N is a neighborhood of a then for x in N and I
in S, I(x) ::; hex), whence

inf I(x) ::; inf h(x)


xEN xEN
limx=al(x) = sup (inf I(X)) ::; sup (inf heX)) = limx=ah(x).
N3a xEN N3a xEN
Because I is continuous, limx=al(x)= lim x. . . . a I(x) = I(a) and since IE S,
I(a) ::; h(a), whence h(a) ~ VfEB I(a) ::; limx=ah(x). By definition,
h(a) ;::: limx=ah(x). A dual argument leads to the dual conclusion for A,
etc.
b) If X is locally compact, his lsc, h;::: 0, and a < h(a), there is an
N(a) such that a < inf (h(N(a)). Urysohn's lemma implies that for some
I in Coo (X, JR), I(a) = a, 0 ::; I ::; a and I = 0 off N(a). Hence I < h
and h = V {I : I E Coo (X, JR)}. The lsc/usc duality applies for upper
semicontinuity. 0
2.107. a) Because limx=a (I(x) + g(x)) ;::: limx=al(x) + limx=ag(x), if I
and 9 are lsc then

I(a) + g(a) ;::: limx=a (I(x) + g(x)) ;::: I(a) + g(a).


Hence, via Isc/usc duality, the sum of two lsc resp. usc functions is lsc resp.
usc. If I is lsc resp. usc then - I is usc resp. lsc. If I is lsc and 9 is usc then
1+ 9 may be lsc (e.g., if 9 is continuous) or usc (e.g., if I is continuous)
or neither lsc nor usc (e.g., if I = X[O,l] and 9 = X(2,3))' All the previous
remarks and the lsc/usc duality resolve the question.
b) For similar reasons, the product of two nonnegative lsc resp. usc
functions is lsc resp. usc. If I is lsc and 9 is usc then Ig may be lsc (e.g., if
9 is continuous) or usc (e.g., if I is continuous) or neither lsc nor usc (e.g.,
if I = X[0,2] and 9 = X(1,3))' For the last I and 9 described, I and -g
are usc whereas I· (-g) is neither lsc nor usc. Hence the product of two
Solutions 2.108-2.110 189

usc functions may fail to be usc. The other implications can be resolved by
Isc/usc duality.
c) The implicit assumption here is that f is defined on the range of g.
If 9 = X[O,lj and f(x) = 1- x then f is continuous and 0 = f(l) = 1- f(O).
For any neighborhood N of 1, g(N) = {O, I},

sup f 0 g(x) = max{J(O), f(l)} = 1 > 0 = f(l) = f 0 g(I).


xEN

Hence limx=d 0 g(x) = 1 > f 0 g(I): fog is not usc even though f is
continuous and 9 is usc. Similar examples for combinations of Isc and usc
show that there are no instances of valid implications. 0
2.108. a) If f is continuous at x then for every neighborhood N of f(x),
there is a neighborhood V of x such that f(V) C N. If F converges to
x then F refines the neighborhood filter at x and thus f (.1) refines the
neighborhood filter at f(x), i.e., f (.1) converges to f(x). The argument for
the converse proceeds similarly.
b) If f is continuous at x and n converges to x then n is eventually in
each neighborhood of x and thus f(n) is eventually in each neighborhood
of f (x). The argument for the converse proceeds similarly. 0
2.109. Let W be a vicinity in W. If x E X and y = f(x) then for
Vy(W) (cf. 2.48) and some U(x) in U, f (Vx(U(x))) C Vy(W). Since
X = UXEX Vx(U(x)) and X is compact, it follows that for some finite
set {Xl, ... , XN}, X = U:=l VXn (U (x n )). If (x, x') E n:=l U (x n ) then
(f(x),f(x')) E W. 0
2.110. i. If G is a group, H is a subgroup, and G / H ~f K is the coset
space then #(G) = #(H)#(K). Hence if Hand K are countable, so is G:
count ability is a QL property in Q.
ii. The group 8 3 of permutations of the set {I, 2, 3} contains the normal
subgroup A3 of even permutations, the cyclic group of order three, and the
quotient group 8 3/A3 is 8 2 , the cyclic group of order two. Both A3 and 8 2
are abelian while 8 3 is not: abelianity is not a QL property in Q.
iii. If, in the context of i, the groups are topological and both H
and K are compact then every open cover {UAhEA of G has a refinement
V ~f {V-y} -yEr such that each V-y is open and V-y is compact.
Let h : G f---> G / H be the quotient map taking elements of G to their
cosets. Then h (V) is an open cover of K. Because K is compact, it is
covered by finitely many h (V-yJ, 1 :S i :S n, and G C U~=l V-Yi . H. Each
summand, as the continuous image under the map G x G 3 {x, y} f---> xy E G
of the compact set V-Yi x H in G X G, is compact. As the finite union of
compact sets, G is compact: compactness is a QL property in .ceQ.
190 2. Topology, Limits, and Continuity: Solutions

iv. For a metric topological group G, a closed subgroup H, and the


coset space K ~f G I H, let {k n } nEN resp. {h m } mEN be a dense subset of K
resp. H. In G, there is a set {9n} nEN such that 9nl H = k n .
If U is open in G then UI H ~ V is open in K and for some kn in
V and some z in U, zlH = k n . Because zlH = 9n1H, it follows that
Z9;;1 E Hand U 9;;1 n H is open in H. Thus there is an h m in U 9;;1 n H,
i.e., U9;;1 3 hm, and h m 9n E U: {h m 9n}m,nEN is dense in G: separability
is a QL property in M'lQ.
v. If, for the topological group G and closed subgroup H, both H
and K ~f G I H are separable, their associated uniformities have countable
bases and so both Hand K are metric and contain countable dense subsets
whence, as the argument in iv shows, G contains a countable dense subset
{9n}nEN·
Furthermore: a) for some sequence {Vn}nEN of open neighborhoods
containing the identity of G,

and {Vn n H}nEN is a base of neighborhoods of the identity in H; b) for


some sequence {Un} nEN of open neighborhoods containing the identity of
G, {UnIH}nEN is a base of neighborhoods of the identity coset of K.
If U is an open neighborhood of the identity in G, there is an open
neighborhood Wand a Vn such that W 2 c U and Vn n HeW n H. For

Pn ~f ( G \ Vn+l . (H \ Vn )) n (Un H ) ,
Qn ~f PI n··· n Pn , n E N,

and some k greater than n,

UklH c (W n Vn+l) IH,


Qk c Pk n Pn C (UkH) n (G \ (Vn+l . (H \ Vn ») ,
c (Wn Vn+dHn [(Wn Vn+l)· (G \ (H\ Vn»)] ,
c (W n Vn+l) . (H n Vn ),
C (W n Vn+1)· (W n H) c W2 C U.

Hence G is metric and contains a countable dense subset {xn}nEN" The


set {B (xn' rn r>O is a countable base for G.
rEIQi
0
[ Note s2.8: The argument in v can be extended to show that
if K contains a dense subset of cardinality e and H contains a
dense subset of cardinality lJ then G contains a dense subset of
cardinality lJ . e.
Solutions 2.111-2.114 191

The metrizability of a topological group for which there is a


countable base at the identity is a theorem of Kakutani [KakJ.
Thus v is a consequence of two propositions: a) if H and G / H
have countable bases at their identities then G has a countable
base at its identity; b) iv.J

2.111. a) If lim x -+ a I(x) exists and V(f) is a typical vicinity, then a is in


a neighborhood U such that if Y E U then I/(Y) - LI < ~. It follows that
In (U') -LI < f if U' c U.
Conversely, if n as described is a Cauchy net then, since lR. is complete,
n converges to some L and L = limx -+ a I(x).
b) By definition, I is continuous at a iff I(a) = lim x -+ a I(x). 0
2.112. The conclusion is a consequence of the definitions. o
2.113. The argument is, mutatis mutandis, the argument for the case of
sequences. 0
2.114. If X is not compact, then some sequence S ~f {xn}nEN in X
contains no Cauchy subsequence. For each n and some positive Tn, the
open balls Un ~f B (Xn' Tnt are pairwise disjoint. It may be assumed that
Tn! O. Because X contains no isolated points, for some Yn in Un, Yn i Xn ,
i.e., d (xn' Yn) ~f 2bn > O. If

then In is continuous and

The series 2::'=l/n(x) converges throughout X since for each x, there


is an n(x) such that In(x) = 0 if n > n(x). For similar reasons, I ~f
2::'=1 In is continuous. However, contrary to the hypothesis, I is not uni-
formly continuous: if f = 0.5, II (xn) - I (Yn)1 = 11 - 01 = 1 > f while
limn -+ DO d (xn, Yn) = O. 0
3
Real- and Complex-valued Functions

3.1. Real-valued Functions

3.1. For each n in D and some index m greater than n, Sm > Sn-l. Let
n' be the least such m. If n < p < n' then

Sn' - Sn-l = Sn' - Sp-l + Sp-l - Sn-l > 0,

whereas Sp-l - Sn-l :s; 0, and so Sn' - Sp-l > 0. In other words, each p
in (n, n') is distinguished, i.e., the numbers n, n + 1, ... ,n' - 1 constitute a
block or a part of a block.
Thus a monotonely increasing enumeration of the elements of D begins
with a block: nl ~f nl, nl + 1, ... , n/ -1. Let n2 be the first distinguished
index after nl/-1 and then continue the enumeration: n2, n2+1, ... , n2' -1,
etc. In this way D is completely enumerated and if n# is the last member
of a block to which n belongs and if n# > n, then Sn# > Sn-l· 0
3.2. If D = 0 thenD is open and no further discussion is required. If
D ::f. 0, xED, limy=xf(y) ~ Lx, x' > x, and f (x') > Lx, for some N(x),
x E N(x), x' f{. N(x) and SUPYEN(x) f(y) < f (x'). Hence N(x) c D, D is
open, and uniquely expressible as follows: D = Un (an, bn ).
Assume x E (an, bn ) and f(x) > Lb n ' Because bn f{. D, if x" > bn
then Lb n ~ f (x") whence Lx ~ f(x) > f (x"). Thus if f (x') > Lx then
x' E (x,b n ]. Let c be the supremum of all such x'. Then x < c :s; bn . If
c = bn then f(c) ~ Lx as claimed. If c < bn there is a c' such that c' > c
and f(c') > Lc k f(c) ~ Lx) and so c' E (x,b n ], c:S; c', a contradiction.
Hence c = bn . 0
[ Note s3.1: Figure s3.1, showing the graph of y = f(x) for
a continuous f, suggests the origin of the name running water
lemma for the result in 3.2. The situations in 3.1 and 3.2 are
analogous. Both discoveries are due to F. Riesz who used them to
give perspicuous proofs of a) the differentiability a.e. of a mono-
tone function and b) the Birkhoff pointwise ergodic theorem, (cf.
5.160, 5.161).]

192

B. R. Gelbaum, Problems in Real and Complex Analysis


© Springer-Verlag New York, Inc. 1992
Solutions 3.3-3.4 193

y-axis

y = [(x)

o an+ l x-axis

Figure s3.1.

3.3. If E ~f {p : p E lR[x) , deg(p) ::; R + 8 - I} then E is isomorphic to


C R+S . For T : E 3 P 1-+ (p(l), ... ,p(R-I)(I),p(2), ... ,p(S-I)(2)) E C R+S ,
if T(p) = 0 and p(x) = E~;-I Ak (x - l)k then

Ao = Al = ... = A R - I = 0.
If 1 $ s ::; 8 then

R+S-I 1
whence Ek=R (k _ s + l)!k!A k = 0, 1 ::; s $ 8, a system of 8 homo-
geneous linear equations in the 8 variables, k!A k , R ::; k ::; R + 8 - 1.
= R + 8 - 1, the matrix of coefficients is, modulo column/row
For N def
permutations,

1 1

M(N, S) 'Jf (
N!
;
(N -1)!

i
(N - 8 + I)!
1
i
1
(N - 8 + I)! (N - 8)! (N - 28+2)!

Mathematical induction and the row operations for reducing M(N, 8) to


a manageable form show there are positive constants K(N, 8) such that
det(M(N, S)) = K(N, S)det(M(N -1, S - 1)). Hence det(M(N, 8)) i= 0,
Ak == 0, i.e., T is surjective. 0
3.4. If
def{O ifx<b n
8n : 1R 3 x 1-+ = 1 if bn ::; x ' n EN,

f : 1R
3 x I-+~f E~=I a n 8n (x), and E > 0, there is an N such that N> n
and E:=N +1 am < Eo There is a positive 8 such that if y < bn ::; x and
194 3. Real- and Complex-valued Functions: Solutions

x - y < 8 then, among bb"" bN , only bn lies in (y, x]. Hence


an + E ;::: f(x) - fey) = L am = an + L am;::: an,
bmE(Y,x] m#-n
bmE(Y,x)

whence f (b n + 0) - f (b n - 0) = an and so ContU) C S.


Furthermore, if xES and E > 0 for some N, L::=N+l an < E. There
is a positive 8 such that

(x - 8, x + 8) n {bn }nEN C {bn }N+1:::;n<oo ,


whence if y E (x-8, x+8) then If(x) - f(y)1 < E. Thus S C ContU)· 0
3.5. Direct calculations reveal the truth of each of the following state-
ments.
i. On each of the intervals (-00, Xl) , (Xb X2) , ... , (Xn-b xn) , (xn' 00), 8
is a linear function.
ii. On JR., 8 is continuous.
iii. If Xk ~ Xl + 15k, 1 ::; k ::; n, then 0 = 81 < 82 < ... < 8n and
=

!
8(0)
L:~=l Xk - nO

(2p - n): + (n - 2p)Xl - L:1=1 15k + L:~=P+1 15k


nO - L:k=l Xk
iv. On (-OO,Xl], 8 is monotonely decreasing, on [Xn' 00) 8 is monotonely
increasing, and on [Xk' Xk+l]
monotonely decreasing if 2k - n < 0
8 is { constant if 2k - n = 0 .
monotonely increasing if 2k - n > 0
Hence if n is odd, 8(0) reaches its minimal value when 0 = X n+l .
2
If n is even, 8( 0) reaches its minimal value throughout the interval
[x1j,x1j+d· o
3.6. The Heine-Borel theorem implies there is a finite set {Xdl<k<K in
JR. and a set {8 k }1:::;k:5K of positive numbers such that: - -

Xl - 81 < p < Xl < X2 - 82 < Xl + 81 < X2 < ...


< XK - 15K < XK < XK-l + 8K, < q < XK + 15K;
U (Xk - 15k, Xk + 8x ) :) [p, q];

{Xk-l - 8k- l < ak < Xk-l < bk < Xk-l + 15 k- I }


=} {f (ak) ::; f (bk)} ,k = 2,3, ... ,K + 1.
Solutions 3.7-3.8 195

Hence if X2 - 02 < Y2 < Xl + 01, ... ,XK - OK < YK < XK-I + OK-I then
f(p) :S f (Y2) :S f (Y3) :S ... :S f (YK-I) :S f(q)· D
3.7. If ¢ is convex,

def b- C def C - a
u < a < c < b < v, a = -b- , and (3
-a
= -b-
-a

then a, (3 > 0, a + (3 = 1, and c = aa + (3b. Hence

¢(b) - ¢(a) = ¢(b) - a¢(a) - (3¢(b) < . :. . .::¢(--:-b)_-.. :. . ;¢(,-,-c)


b- a b - aa - (3b - b- c

and, by a similar calculation, ¢(c) - ¢(a) :S ¢(b~ - ¢(a). The inequalities


c-a -a
u < X :S x' < Y :S y' < V can be applied first when x = a, x' = c, and Y = b
and then when x' = a, Y = c, and y' = b. The argument is reversible.
The sketch in Figure s3.2 shows the geometry of the situation. D

t-axis

o x x' y s-axis

Figure 83.2.

3.8. If ¢ is neither monotonely decreasing nor monotonely decreas-


ing, there are numbers a, b, c such that a < b < c and either a) ¢(b) >
max{¢(a) , ¢(c)} or b) ¢(b) < min{¢(a) < ¢(c)}. The convexity of ¢ rules
out a) and implies for b) that if a" < a' < a then ¢ (a') ~ ¢(a) [whence
¢(a") ~ ¢(a' )] andifb" > b' > bthen¢(b') ~ ¢(b) [whence¢(b") ~ ¢(b')].
Hence ¢ is monotonely decreasing on (-00, a) and ¢ is monotonely
increasing on (b,oo).
196 3. Real- and Complex-valued Functions: Solutions

Ifp~sup{a: ¢! on (-00, a)} , q~finf{b: ¢ion(b,oo)}then


[p, q] is the required interval.
p :::; q and
Examples: a) ¢(x) = e"; b) ¢(x) = x+ Ixl; c) ¢(x) == 0; d) ¢(x) = e-";
e)
(x + a)2 if -00 < x :::; -a <0
¢(x)= { 0 if-a<x:::;a
(x-a)2 if a < X,oo
f) ¢(x) = Ixl - x.
o
3.9. If u < c :::; x < y :::; d < v then

¢(e) - ¢(a) < ¢(y) - ¢(x) < ¢(b) - ¢(d)


e-a - y-x - b-d

and so ¢ E Lip(l).
The criterional inequality of 3.7 shows that the difference quotient

¢(x + h) - ¢(x)
h
is a monotonely increasing function of h. It follows that the right- and
left-hand derivatives D±¢ of ¢ exist everywhere.
In particular, at each point a, D_¢(a) :::; D+¢(a). [Any line through
(a, ¢(a)) and with a slope m satisfying D_¢(a) :::; m :::; D+¢(a) is a sup-
porting line.]
From 3.9 and the differentiability a.e. of a monotone function it follows
that ¢ is differentiable a.e. 0
[ Note 83.2: The fact that (u, v) is open is essential to the
continuity and differentiability properties of ¢. For example, if
(u,v) is replaced by (0,1] and

ifO<x<l
¢(x)={~ if x = 1
then ¢ is convex but is discontinuous when x = 1. Similar obser-
vations apply when (u, v) is replaced by any nonopen interval.]

3.10. The functions 'if; : x f--+ x 2 and ¢ : x f--+ e-" are convex, but
¢ 0 'if; : x f--+ e-,,2 is not convex. 0
3.11. a) If ¢" exists, ¢" > 0, and ¢ is not convex then there is a triple
r-q q-p
p, q, r such that u < p < q < r < v and ¢(q) > --¢(p) + --¢(r). The
r-p r-p
map f : x f--+ ¢(x) - r - x ¢(p) + x - p ¢(r) is continuous and positive at
r-p r-p
q, whence f has a positive maximum on [p, r]. Because f(p) = fer) = 0,
Solution 3.12 197

this maximum occurs at some S in (p, r) and thus 0 < ¢" (s) = f" (s) ::; 0,
a contradiction.
b) If p < rand ¢'(p) > ¢'(r), the convexity of ¢ implies

¢(r) > ¢(p) + ¢'(p)(r - p),

whence

¢(r) + (p - r)¢'(r) > ¢(r) + (p - r)¢'(p)


> ¢(p) + ¢'(p)(r - p) + ¢'(p)(p - r) = ¢(p).

In geometric terms, the inequalities above say that the point (p, ¢(P)) does
not lie above the supporting line through (r, ¢(r)), a contradiction. Hence

{p ::; r} =* {¢' (P) ::; ¢' (r)} ,

i.e., ¢' is monotonely increasing and so ¢" ::::: o.


An alternative proof uses a truncated Taylor series. If x E (s, t) then
for suitable ~ and 1],

(s x)2
¢(s) = ¢(x) + ¢'(x)(s - x) + ¢"(~) ~! '

¢(t) = ¢(x) + ¢'(x)(t - x) + ¢"(1]) (t _,X)2.


2.
For some A in (0,1), x = AS + (1 - A)t and, owing to the nonnegativity of
¢", for some nonnegative P,

A¢(S) + (1 - A)¢(t) = ¢(x) + ¢'(x) (AS - AX + (1- A)t - (1- A)X) +P


::::: ¢(x) + 0 = ¢(AS + (1 - A)t).
Note that if ¢(x) = x then ¢ is convex and yet ¢"(X) == 0: the implication
{¢" exists and ¢ is convex} =* {¢" > O} is invalid. 0
3.12. a) Ifln(¢) is convex, x, y E JR,O ::; a, (3, a + (3 = 1 then
In(¢)(ax + (3y) ::; a In(¢)(x) + (3ln(¢)(y)
¢(ax + (3y) ::; (¢(x))a. (¢(y))~.

However for u, v positive, uav~ ::; au + (3v. (PROOF. Because

(t a )' It=c = {~ca-l


aca -
:;
1 :::::
a
a
if c = 1
if c > 1
ifO<c<l
(at + (3)' == a,
talt=l= at + (3lt=l= 1,
198 3. Real- and Complex-valued Functions: Solutions

it follows that for all positive t, t a ~ at + (3. Hence if t = !!:. then


v

U
va
a
V ~ (a!!:.v + (3) v,
u a v/3 ~ au + (3v.]

By induction it follows that if


K
ak, Uk 2 0, 1 ~ k ~ K, and L ak =1
k=l
then, ambiguous cases excluded,
K K
II U~k ~ Lakuk,
k=l k=l
as required.
b) Graphical considerations show that the map ¢ x 1--+ 1 + Ixl is
convex. However, for 'l/J ~ In¢,

{~~:
if x> 0
'l/J'(x) =
if x < 0
1-x
whence
{x =I O} '* {'l/J"(x) = - (1 ~ x)2 < o} .
Hence 3.11 implies'l/J is not convex. D
3.13. i. True: There is a sequence {xn}nEN such that Xn < Xn2+l and
g(x) 2 n if x ~ x n. If ¢(xn) = n - 1,n E N, and if ¢ is piecewise
.linear , continuous, and nonnegative then ¢ is also convex and monotonely
decreasing, ¢ ~ g, and ¢(x) --t 00 as x --t o.
ii. False: For 9 : [0,00) '3 x 1--+ In(1 + x), if the convex function ¢ is
such that ¢ ~ 9 then (cf. Figure s3.3), since -g is convex, 9 is concave
and

en In (1 + e- n )
As n --t 00,
n
--t 00 and 1 + n
--t 1 whence ¢(x) -f 00 as
x --t 00. D
Solution 3.14 199

y-axis

y = In(1 + xl

x-axis
o
Figure s3.3. The graph of y = In(1 + x)

[ Note s3.3: Associated with the notion of convexity is that of


midpoint convexity, viz.:
A function f in JR(a,b) is midpoint convex iff for all x, y
in (a,b), f (~+~) ~ ~f(X) +~f(Y).
Hence if a, f3 E {t : t = k2- n , k, n EN}, and a + f3 = 1,
then f(ax + f3y) ~ af(x) + f3f(y). If, to boot, f is continuous,
then f is convex. The existence of a discontinuous linear (hence
convex) function ¢ in JRIR (cf. [GeO] and, for generalizations,
6.115) shows that the converse is false.]

3.14. For the maps

k: [0,00) :3 s I-t foB b(r) dr,


g: [0,00) :3 s I-t k(s) + c(s),
T: C([O,oo),JR):3 f I-t (F: [0,00):3 s I-t foB a(r)f(r)dr) ,
200 3. Real- and Complex-valued Functions: Solutions

the following obtain:

d (T~:(S)) _ a(s)T(y)(s) ::; a(s)g(s);

d (e- I: a(r)drT(y)(s)) t

ds ::; ef. a(r)dr a(s)g(s);

T(y)(t) ::; lot a(s)g(s)eI: a(r)dr ds;


lot a(s) [yes) - g(s)eI: a(r) dr] ds::; 0;
from which the result follows.
Owing to the importance of fixed-point theory, the alternative solution
below is of interest.
If Mo is an endomorphism of a Banach space X and if, for all x in X,
:E~=1 M(j(x) converges then for each y in X, the map

My : X 3 x 1--+ Y + Mo(x)

has a unique fixed point, namely P ~f Y + :E~=1 M(j(x). If, to boot, X is


a function space and Mo preserves positivity then

{x::; y + Mo(x)} ~ {x::; P}.

More particularly, if X = C ([0, al, R), 0 ::; t ::; a, and Mo = T, the


results just cited are applicable: for y read g, for x read y, and for P read

o
3.15. The solution involves two steps: a) showing that for a, bin R, the
differential equation problem:

y" + (1 + q)y = O,y(O) = a, y'(O) = b (s3.1)

has a unique solution; b) the solution is bounded.


a) Because q E L1 ([0,00)), the map

T: C ([0, Rl, R) 3 f 1--+ ([0,00) 3 t 1--+ lot sin(t - s)q(s)f(s) dS)


is one for which E:'l IITnlloo converges if 0 ::; t ::; R. Hence (cf. 3.14)
if w E C([O,oo),R), the map Mw : C([O,oo),R) 3 f 1--+ W - T(f) has a
unique fixed point v. If w" is continuous and w(O) = a, w'(O) = b then
Solution 3.16 201

v(O) = a,v'(O) = b and direct calculation shows v" + (1 + q)v = 0: (s3.1)


has a solution.
Since the differential equation is homogeneous, the question of unique-
ness of solution for the problem is resolved by examining the case in which
a = b = O. The original equation is equivalent to the system

(s3.2)

Any solutions Yl, Y2 of the system must satisfy

Because

the second proof in Solution 3.14 applied to (s3.2) shows Yl = 0 (whence


Y2 = 0): the problem has at most one solution.
b) If w" + w = 0, w(O) = a,and w'(O) = b then

Iv(t)1 :::; Ilwll oo + lot Iq(s)llv(s)1 ds;


3.14 applied again yields

whence v is bounded. o
3.16. a) Since I is continuous, E-I(f,O) is the union of a null set Nand
an open subset U of (0, 1). Furthermore,

and U± are uniquely of the form lJnEN (a n ±, bn ±). On U±, III = ±I and
III", as the limit of second order difference quotients for ±I, is measurable.
Because I is in C 2 ([0, 1], lR),

0 if x E [0,1] \ U
h : [0,1] :3 x ,........ { III" ot h erwise

is in LR'([O, 1],,\).
202 3. Real- and Complex-valued Functions: Solutions

b) Since

ior g(x)h(x)dx= ~
o
1 00

n=1
l bn ±

an ±
g(x)h(x)dx,

it suffices to consider a single term of the sum. Then, (a, b) denoting any
one of (a n ±, bn ±), integration by parts twice yields

lb g(x)h(x) dx = g(x)III'(x)l~ -g"(x)ll(x)ll~ + lb g"(x)ll(x)1 dx.

°
Since 9 (b) III' (b) ::; and 9 (a) III' (a) ~ 0, the first summand in the right
member above is nonpositive. Because III (a) = III (b) = 0, the second
summand is zero. Hence

lb g(x)h(x) dx ::; lb g"(x)ll(x)1 dx. o


3.17. i. The map
I: (0,1) 3 x ~ ~ sin (~)
is continuous and

is a Cauchy sequence, whereas its I-image is not.


ii. If I is not continuous then for some Xo in (0,1) and some sequence
{xn}nEN'
lim
n--->oo
Xn = Xo and I (x n ) -1+ I (xo)

as n -+ 00. If
Y2n = xn , Y2n-1 = xo, n E N,
then {Yn} nEN is a Cauchy sequence and its I-image is not, a contradiction.
o
°
3.18. The hypothesis implies that if Xn -+ as n -+ 00 then {f (x n )} nEN
is a Cauchy sequence. Thus the argument in Soluition 3.17ii implies
limx--->o I(x) exists. 0
3.19. The map I : JR. 3 r ~ A {x : sinx > r, x E [0,471']} is monotonely
decreasing, 1([-1,1]) = I (JR.) = [0,471'], and on [-1,1], I is injective. Thus
9 ~f 1- 1 1 [0,41fJ is monotonely decreasing and g(x) > r iff x < I(r). 0
3.20. a) If Ik is the function for which the graph of Y = !hex) is that in
Figure 83.4 then limk--->oo Ik(X) == 1 and I~ (~) = k, kEN.
Solution 3.21 203

y-axis

-l-----l-------<j__--*-----<__--::----- X- axis
o (2,0) (1,0)

Figure s3.4.

b) Because each fk is monotone, each f~ exists a.e.,

E ~ {x : f£{x) exists and is finite, kEN}

is measurable, and )..(E) = 1. The Fundamental Theorem of Calculus for


Lebesgue integration implies fk(l)- fk(O) ~ IE
fHx) dx, kEN, and Fatou's
lemma (cf. 4.3) permits the conclusion:

However, f~(x) ~ 0 if x E E. D
3.21. Since AIR is countable and contains Q, AIR is a dense Fu' If, for
some f, AIR = ContU) it is also a (dense) Go (cf. 2.97) and therefore a set
of the second category, a contradiction.
As an Fu , AIR is the union of count ably many closed sets Fn such that
Fn c Fn+ 1. If Bn ~ (Fn \ Fn- 1) \ (Fn \ Fn-1t and
2-n if x E Bn
f(x) = { 0
if x (j UnEN Bn
204 3. Real- and Complex-valued Functions: Solutions

then Discont(f) = AIR whence Cont(f) = R \ AIR [GeO]. 0


3.22. Because f~(x) - f~(y) = f::(t)dt,J:
the sequence {f~}nEN is
equicontinuous. If M' as described does not exist then, via passage to
subsequences as needed, it may be assumed that an ~f IIfnlloo 10 and that
M~ ~ Ilf~lIoo i 00. Hence for some positive E and for each n in N, there is
an Xn such that {Ix - xnl < E} :::} {lf~(x)1 > ~M~}. For some Cn,

fn(x) = 1 f~(t)
x

Xn
dt + en, n E N,

and since fn (xn) = Cn, it follows that len I :::; an, n E N. If Ix - xnl < E
1 1
then Ifn(x)1 2:: 2M~E - an· Hence Ilfnlloo 2:: 4M~E - an -+ 00 as n -+ 00, a
contradiction. 0
3.23. If U is open in R, f-l(U) is the union of two disjoint sets:

Cu ~ Cont(f) n f-l(U) and Du ~f Discont(f) n f-l(U).

By hypothesis, oX (Du) = O.
If x E Cu , for some positive E(X), {y : If(x) - yl < E(X)} C U and
there is a positive b(x) such that

f({x': Ix'-xl<b(x)})c{y: If(x)-yl<E(X)}CU.

Hence f-l(U) = (UXECu {x' : lx' - xl < b(x)}) UDu, a union of an open
(hence measurable) set and a null set. 0
3.24. If f'(x) = X(-oo,Oj 0 f(x) is valid on an open set U containing {O}
then
1'( ) =
x
{O1 if f(x) > 0
if f(x) :::; 0 '
(s3.3)

and, by hypothesis, f(O) = O.


i. If E > 0 and f(x) = 0 on
[0, E) then f'(x) = 0 on (0, E) while (s3.3)
requires that f' (x) = l.
ii. If f(x) > 0 throughout (0, E) then f'(x) = 0 throughout (0, E) whence
for some positive constant p, f(x) = p throughout (0, E) and hence f
is not continuous at zero.
iii. If f(x) < 0 throughout (O,E) then f'(x) = 1 throughout (O,E) and for
some constant k, f(x) = x+k throughout (0, E). Because f(O) = 0, k =
0; the hypothesis implies f' (x) = 0 throughout (0, E), a contradiction.
Hence if E > 0 then f:
iv. cannot be ze.. o throughout [0, E);
Solution 3.25 205

v. can be neither only positive nor only negative throughout (0, e).

Thus, owing to (s3.3), if e > 0, f' assumes both values zero and one
on (0, e) and no others. As a derivative, f' enjoys the intermediate value
property and a contradiction emerges. 0
3.25. If, for some x,y, x < y and f(x) > fey) then for some positive p
and all e in (O,p), fey) < f(x) - e(y - x).

u ·axis

(y, j(x) - E(y - x»

(y,j(y»

L-..._ _ _.....I.-_ _ _ _....L_ _.....I.-_--L._ _ _ _ _ x-axis


o x x~ y

Figure s3.5. The situation: x < y, f(x) > fey), and Xoo < y.

As indicated in Figure s3.5, by hypothesis, for some Xl in (x, y),


206 3. Real- and Complex-valued Functions: Solutions

If
y > Xoo ~f sup {Xl : X < Xl < y,J (Xl) ~ f(x) - f (Xl - X)}
then f (xoo) ~ f(x) - f (xoo - x) and, by hypothesis, for some z in (xoo, y),

fez) ~ f (x oo ) - f (z - x oo )
~ f(x) - f (x oo - x) - f (z - x oo ) = f(x) - feZ - x),

a contradiction. Thus

Xoo = y and fey) ~ f(x) - fey - x).

Since the last inequality obtains for all f in (O,p), fey) ~ f(x), another
contradiction. 0

3.26. For the Cantor function <Po, let h be <Po.


i. On the closure of each interval J ln , n E N, deleted in the construction
of the Cantor set Co, define the homolog hn of h.
ii. On the closure of each interval J 2n , n E N, deleted in the construction
of the sequence {hn}nEN' define the homolog!2n of h·
iii. By induction, for each k in N, there can be defined a sequence {fkn} nEN
such that: a) the domain of fkn is the closure of one of the intervals
deleted in the construction of {fk-l,n} nEN; b) each /kn on its domain
is the homolog of h.
The sequence {hn}nEN' {!2n}nEN' ... can be rearranged so that there
emerges a single sequence {hn}nEN' each member of which, on its original
domain, is homologous to h on [0,1]. Furthermore, each h n may be ex-
tended to be constantly zero to the left of its domain and constantly one to
the right of its domain. Then h ~f 2::=1 ~: is continuous and monotonely
increasing while 1 - h ~ f is monotonely decreasing. Owing to Fubini's
theorem on derivatives of sequences of functions, h' ~ 2::=1 ~~ and, since
h~ ~ 0, n E N, f' ~ 0. In sum, the function f meets all the requirements:
f is a monotonely decreasing function, hence monotonely increasing on no
interval and yet f' ~ ~ o. ° 0

3.27. For x, y in (0,1), the formula

d: (0,1)23 {x,y} 1-+ max {If(x) - f(y)l, Ix - yl}


Solutions 3.28-3.30 207

defines a metric and furthermore, d(x, y) 2:: Ix - yl. Because I is con-


tinuous, if limn -+ oo IXn - xl = 0 then lim n -+ oo II (xn) - l(x)1 = 0, whence
limn -+ oo d (Xn' x) = 0: d and I I endow (0, 1) with the same topology.
Since {d(x,y) < f} ::::} {1/(x) - l(y)1 < f}, I is uniformly continuous
on ((0,1),d). 0
3.28. If n = 1, I may be regarded as the restriction to (-r, r) of a
function F holomorphic in D(O,r)o. If I ~ 0 then F- 1 (0) is countable and
thus A (1-1(0)) = O.
Assume the result is valid when n = 1,2, ... , N - 1. When n = N,
let A be 1-1(0) n B(O, r)o. Fubini's theorem for product measures implies
that if A"N ~ {(Xl'"'' XN-l) : (XI,"" XN-l, XN) E A} then

For fixed XN in (-r,r), AXN in JR N - 1 is the possibly empty subset on


which I, regarded as a function on JR N - l when Xn is fixed, is zero. By
inductive assumption, AN -1 (AXN) = 0 whence AN (A) = o. 0
[ Note 83.4: The last result implies, when Mat nn is regarded
2
as a subset of JRn , Nondiagnn is the set of nondiagonable n x n
matrices, and Singnn is the set of singular n x n matrices, that

Thus "almost all" n x n matrices are invertible and diagonable.


In the same vein, every algebraic or analytic variety in JRn is
a null set.]

3.29. For 1,9 in V, Mf V Mg resp. kf V kg serves for Mfvg resp. k fvg


and Mf 1\ Mg resp. k f 1\ kg serves for Mf!\g resp. kf!\g. The vector space
character of V flows from the definitions.
The Daniell construction in Chapter 1 shows that there is a measure
v such that P(f) = JlRn I(x) dv(x) whence v = fL· Each nonnegative I in
V is the limit of a monotonely increasing sequence in Co (JR n , JR) and the
monotone convergence theorem implies the validity of (3.1) for f. For an
arbitrary I in V, 1= 1+ + 1-· 0

3.30. For to fixed in JR, the map 9 : ~3x 1--+ (f (x, to))2 + (O/~' to)) 2
is in C (~, JR) and 9 > O. Because ~ is compact, m ~f minxEI; 9 (x, to) > 0
and for some Xto in ~, 9 (Xto' to) = m.
If, arbitrarily close to to, there is a tk and in ~ a corresponding Xtk
such that 9 (Xtk' tk) = 0 then it may be assumed that for some Xoo in ~,
208 3. Real- and Complex-valued Functions: Solutions

limk-+oo IIXtk -xooll = 0 and thus g(xoo,to) = 0, a contradiction. Hence,


for some open set N (to), if tEN (to) then minxEE 9 (x, t) > O.
If, for each k in N, there are in N (to) two points, tkl, tk2 such that
Itkl - tol+l tk2 - tol < ~ and for some Xk in~, 1 (Xk, tki) = 0, i = 1,2, then

Rolle's theorem implies for some tk3 between tkl and tk2, of (~t' tk3) = o.
It may be assumed that for some x in ~, limk-+oo IIXk - xii = O. Hence
01 (x, to) .. _
at = O. SInce mInxEE 9 (x, t) > 0, 1 (x, to) =I- O. Thus for large k,
1 (Xk, tkl) . 1 (Xk, tk2) =I- 0, a contradiction. It follows that for h, t2 close
enough to to and different from each other (one may be to), and any x in
~, (f (x, tl))2 + (f (x, t2))2 > O. 0
3.31. For ~ as in 3.30, let K be ~ n 1-1(0). Then K is compact and
g(x) > 0 on K. Hence for some positive € and some open U containing K,
g(x) ;::: € on U. For some positive 8, I(x) ;::: 8 on the compact set ~ \ U.
Because U may be chosen so that 0 fJ: U and since if x =I- 0,

it follows that

{ x~} * {I(x)
IIxll E \ U IIxll m ;::: 8
}
and
{ x }* {g(x)
IIxll E U Ilxll m ;::: €
}

The required inequality obtains if C = ~ and D = ~. o


3.32. For a ~f (al,.'" an) in Bl (JR n ) let f(a) be (It (a), ... , In (a)). If
b ~f (b l , ... ,bn ) E Bl (JR n ) then for some Cij between aj and bj ,

There is a positive {) such that

{II;~; 1100 < 8,i,j = 1, ... ,n} * {lIf(a) - f(b) II < ~lla - b U}.
Solutions 3.33-3.34 209

Because d(id) = id, it follows that if g =f - id then d(g) = d(f) - id.


There is a positive"., such that

{s~p IId(f) (x) - idll <".,} => {II~~; -8iilloo <8,i,j = 1, ... ,n},
1
whence Ila- bll-Ilf(a) -f(b)11 ~ IIg(a) -g(b)1I < "2l1a- bll. Consequently,
IIf( a) - f(b) II ~ ~ Iia - bll and thus f is injective. D
3.33. For x in lRn , d(f)(x) E [lR n , lR] and, since [lR n , lR] and lRn are
naturally isomorphic as vector spaces, if f E C 2 (lR n , lR) then d 2 (f) is in
[lR n , [lR n ll. [For each x, d 2 (f) (x) may be represented by the Hessian matrix

( ::~~:~ ) :i=l']
By definition, there is a map f; : [-1,1] 3 t 1--+ (0,00) such that
limt_O f;(t) = 0 and for some positive 8,

{Ilull < 8} => {lldf (xo + tu) - df (xo) - d 2 f (xo) (tu) II = f;(t)ltl·llull}·
Since df (xo) = 0, Ildf (xo + tu) - d 2 f (xo) tull = f;(t)ltl·llull·
If df (x o + tu) = 0 for some nonzero t in [-1,1] and some nonzero
u, then, since d 2 f (xO)-l exists, IId 2 f (xo)-lll ~f K > O. If Y ~f d2 f (xo)
then tu = d 2 f (XO)-l (y) ~ Kllyll, whence y i= 0 and lIyll ~ f;(t)KIlYII,
i.e., 1 ~ f;(t)K.
1 1 8
If IIxn - xoll < -, df (x n ) = 0, n E N, and - < -2 then when n > no,
n no
there is a Un such that II Un II =~ and a tn such that Xn = Xo + tn Un and

Itnl < .!..


n
But then 1 ~ f; (t n ) K, a contradiction, since limn_oo f; (tn) = O.
Hence for some open set N (xo), df(y) i= 0 for all y in N (xo) \ {xo}. D
3.34. If W = var[O,l] (f) - ~, there is a partition Po determined by n +2
partition points for which Var[O,l],Po(f) > W -~. There is a positive 8 such
that {Ix - yl < 8} => {If(x) - f(y)1 < 8:}'
If IFI < min{8, IPol} ~f a
and PI is the common refinement of Po and P, then
f; f;
Var[O,l],P(J) + 2n 8n ~ Var[O,l],P! (J) ~ Var[O,I],Po (f) > W - "2
3f;
Var[O,l],P(f) > W - "4 = Var[O,l] (f) - f;.
When f = XU} then Var[O,I](J) = 2. However, for any partition P

determined by points in [0,1] \ {~}, Var[O,l],P(J) = O. D


210 3. Real- and Complex-valued Functions: Solutions

3.35. There are functions Ii, 1 :s; i :s; 4, each monotonely increasing,
nonnegative, and bounded for which I = It - h + i (13 - 14). For each Ii,
fro,l] II(x) dx :s; li(l) - li(O) < 00, whence!, E Ll ([0, 1), .\). 0
3.36. Only the situation for left-continuity is discussed. The other situa-
tions are treated, mutatis mutandis.
If I is of bounded variation and left-continuous at a, if E > 0, and if

o def
= Xo < Xl < ... < Xn < Xn+1 < a
n
L II (Xk+1) - I (xk)1 + I/(a) - I (xn+l)1 > var[O,a](I) - E
k=O

then var[O,X n+l](I) > var[O,a](I) - E - I/(a) - I (xn+dl· As Xn+l i a,


limxn+l ja var[O,X n +l] (I) ~ var[O,a] (I)-E. Because var[O,x] (I) increases mono-
tonely, var[O,a] (I) ~ lim xn +1 ja var[O,X n +l] (I) ~ var[O,a] (I) - E.
Conversely, if var[O,x] (I) is left-continuous at a, then

and as Xn+1 i a the right member above descends to zero. o


3.37. If, for some a, I/(a + 0) - I(a - 0)1 = 6. > 0 then for all small
positive 8, II(a + 8) - I(a - 8)1 > ~ a~ so on [a - 8, a + 8], I assum: no
values between (I(a - 0) /\ I(a + 0)) +3 and (I(a - 0) V I(a + 0)) - 3' a
contradiction. 0
[ Note s3.5: Hence if a function I is of bounded variation and
is a derivative then I is continuous.)

3.38. There is a sequence {/mn}m,nEN of piecewise linear, continuous,


even functions such that

I if Ix - al < ~ or Ix + al < ~
I mn (x) = {
o if x ~
(m 2 2) m(
-a - m' -a + m U a -
2 2)
m' a + m

Imn 1 X[-a-.l.tn' -a+.l.] 1'n


+ X[a-.l. a+.l.]
Tn' 'Tn
as n -+ 00.

Then 0 = limn-->oo J~l Imn(x)g(x) dx = J~:~r + J:~r g(x) dx. Because


g is continuous at a and -a, if E > 0 and m is large, '"

o
Solutions 3.39-3.43 211

3.39. The assertion holds if I(x) = X{l}(x). o


3.40. Direct calculation shows that on [0,1] if a ::; 1 then la is not of
bounded variation and that if a> 1 then la is absolutely continuous. 0
3.41. If
ifO::;x<1
if x =1
then I has the required properties. o
3.42. If 0< a < 1, Co< is a Cantor subset of [0,1], and >. (Co<) = a, then

I(x) ci;1 1 x
(1- XC.,(t)) dt

is absolutely continuous.
If [a, b] c [0, 1] then (a, b) is either a subinterval of some interval deleted
in the construction of Co or some such interval is a subinterval of (a, b). It
follows that
I(b) - I(a) ~ b - a.

Furthermore f' ~ 1 - Xc., whence I' ~ °


on Co. o
3.43. The map () in Solution 2.57ii resp. (}-l ~f ~ is a continuous strictly
monotonely increasing function on Co resp. S. The intervals deleted from
[0,1] in the construction of Co are in bijective order-preserving correspon-
dence with intervals deleted from [0,1] to yield S.
An analog CPs of CPo should be monotonely increasing, continuous, and
constant on each interval deleted in the construction of S. Owing to the
denseness of [0,1] \ S, the values of CPs on the deleted intervals then deter-
mine the values of CPs throughout [0,1]. Thus there is some arbitrariness
about the definition of CPs.
A straightforward analogy with CPo is

CPs : S 3 x = f:
n=l
fn (x)(1 - r)r n - 1 t---+ f: fn2~)
n=l

which is a continuous and monotonely increasing map of S onto [0,1]. Con-


sequently, at the ends of each interval deleted in the construction of S, the
values of CPs are the same and CPs may be extended in one and only one way
to a continuous and monotonely increasing map of [0,1] onto itself. Figure
s3.6 depicts the graph of the second approximant to CPs.
212 3. Real- and Complex-valued Functions: Solutions

y-axis

-O-f"---r-('--I-_-'-2r-)--"-1--2r"'---r-'-(1---2'-r)------ x-axis

Figure s3.6.

a) The complement in the unit square of each approximant to 98 con-


sists of two congruent figures. Hence frO,1] 4>8 (X) dx = ~.
b) The length of 98 may be approximated by the lengths of polygons
that represent the successive approximants to 98. Induction shows that, f3n
denoting the length of each interval in the nth group of deleted intervals, at
the Nth stage of approximation, the length of the polygonal approximant
is 2::=12n-1f3n + VI + (1- 2::=12n-1f3n/ . Hence the length of 98 is 2
(I).
1
c) Symmetry arguments show J[0,1]4>a{X) dx = '2. On the other hand,
the discussion in b) now leads to 1- a + VI + a 2 as the length of r a. The
length of r a is less than 2 if 0 < a < 1. 0
[ Note s3.6: When 0 < r < ~, a more interesting analogy with
4>0 (for which r = ~) follows.
Dual to an ~ {1- r)rn-l, n E N, is bn ~ r{l- r)n-1, n E N.
Furthermore, 2::'=1 bn = 1 and bn ::; 2:~n+1 bk, whence the set
T of all subsums of 2:::1 bn is [0,1].
Solutions 3.44-3.45 213

Thus the formula

L En(x)(l - L En(x)r(l -
00 00

<Ps : S 3 x ~f r)r n- 1 ~ rt- 1


n=1 n=1

defines a continuous monotonely increasing map of S onto [0,1].


Hence the values of <P s at the endpoints of each interval deleted
in the construction of S are the same and <P s may be extended in
one and only one way to a continuous and monotonely increasing
function mapping [0,1] onto itself.
The reader might wish to calculate f[o,IJ <ps(x) dx and the
length of the graph of y = <ps(x).]

3.44. If p == 0, E=(p) = JR, E>(p) = E<Cp) = 0. The definitions imply


that if p(x) ~f aoxn + ... + an and ao =f. 0 then

Thus it suffices to discuss only E> (p) and to assume that ao > o. When
n=l,E>(p)= ( ~,oo. -a1 )
Ifn=Nand,whenn=1, ... ,N-1,for
some a in JR, E>(p) = (a, 00), then for some {3 in JR, E> (p') = ((3,00) and
E>,o(p) is a union of a finite number of pairwise disjoint open intervals.
If E>,o(p) = JR then for some {3 in JR, E> (p) = ({3, 00).
If rk < Sk < rk+1, E>,o(p) = Uf=1 (rk, Sk), and {3 ~ SK-l, then for
some 'Y in (SK-b 00), p'("() = 0 in contradiction of the definition of {3.
Hence for a ~f max{{3,rK}, E>(p) = (a, 00) (and SK = 00). 0
3.45. If J is Riemann integrable, it must be bounded since otherwise the
set of approximating Riemann sums is unbounded in JR.
Since

Discont(f) = U{x : limy=xJ(y) -limy=xJ(y) ~~}~ UE::"


nEN nEN

if >. (Discont(f)) = {j > 0 then for some no, >. (E n1J


~f E > O. For each
partition used to calculate the upper and lower Riemann sums, the sum of
214 3. Real- and Complex-valued Functions: Solutions

the lengths of the partition intervals containing E..L is at least € and thus
"0
the difference between the upper and lower Riemann sums is at least ~,
no
whence I is not Riemann integrable, a contradiction.
Conversely, if I is bounded and >. (Discont(f)) = 0 then for positive
€, E. is compact and >. (E.) = O. Hence E. can be covered by finitely
many pairwise disjoint open intervals of small length-sum. For a partition
consisting of such intervals and others (on each of which I is uniformly
continuous), the difference between the upper and lower Riemann sums is
small if the mesh of the partition is small. 0
3.46. For Qn [O,lJ ~f {rdkEN,ln ~ L;=lXrk,limn-oo/n = XlQln[O,lj,
each In is Riemann integrable while Discont (XlQln[O,lj) = [0, 1J and XlQln[O,lj
is not Riemann integrable (cf. 3.45). 0
3.47. Let <Po be the Cantor function and let I be <Po . X[O,lj\co' Because
I is bounded and >. (Discont(f)) = 0, I is Riemann integrable.
For O! positive, there is a homeomorphism H : Co. ho~eo Co [GeOJ and
the Tietze extension theorem implies that H has a continuous extension
g: [0, 1J I-----t [O,lJ. If h ~ log then >.(Discont(h)) = >. (Co.) > 0 and h is not
Riemann integrable. 0
3.48. If I is strictly increasing, all its difference quotients are nonnegative
and thus f' 2: 0 everywhere. If D contains a connected set containing two
points, then f'(x) = 0 on a nonempty open interval (a, b), whence I is
constant on (a, b), a contradiction.
Conversely if f' 2: 0 everywhere, Rolle's theorem implies I is mono-
tonely increasing. If a < band I(a) = I(b) then I'(x) = 0 on (a,b), an
impossibility if D is totally disconnected. 0
3.49. If m is a strict local maximum value of I, if Am ~f 8 n 1- 1 (m),
and if # (Am) > No then by virtue of 2.27, for some a in Am, and every
N(a), # (N(a) n Am) > No, whence arbitrarily close to a there are points
where the value of I is m, i.e., a is not the site of a strict local maximum:
# (Am) ::; No·
Let M be the set of strict local maxima of f. If #(M) > No then
for some m in M, and all pin (0,00), # ((m,m + p) n M) > No whence
# (1-1 ((m,m + p) n M)) > No. Thus in E ~f 1- 1((m, m + p) n M) there
is a b such that for every N(b), # (N(b) n E) > No. Because # (Am) ::; No
for some c in N(b)nE, I(c) i:- m and so I(c) is in (m, m+p) whence m ~ 8:
#(M) ::; No and #(8) = LmEM # (Am) ::; No· 0
3.50. If a < b, there is a sequence {[an' bn )} nEN such that

an ~ Pn Tq " , bn ~ (Pn + 1) Tq", Pn, qn EN, (a, b) = U [an' bn).


nEN
Solutions 3.51-3.52 215

For some no, a* ~f infn~no an and b* ~f sUPn~no bn are such that

If(a) - f (a*)1 + If(b) - f (b*)1 < b - a


no
If(b) - f(a)1 ::; If(b) - f (b*)1 +L If (bn) - f (an)1 + If (a*) - f(a)1
n=l
no
< b - a + LTqn . M < (M + 1)(b - a).
n=l

Hence f E Lip(1) and so f E AC whence for all x,

The Fundamental Theorem of Calculus for Lebesgue integration implies


that f'(x) ~ O. Hence for each n in N and each kin Z, f(O) = f (k2-n).
Since {k2- n hEZ is dense in lR and f is continuous, f(x) == f(O). 0
nEN
3.51. If no such a exists then, for all x, f(x) - g(x) =f:. O. Since f and 9
are continuous, f - 9 is of one sign, say f - 9 > 0 and on [0,1], for some
positive 8, f(x) > g(x)+8. Thus fof > gof+8 = fog+8 > gog+28. By
induction, it follows that {n E N} => {fin} > gin} + n8}, an impossibility
ifn8> 1.
For f: [0,00) 3t---+ In(1 +x) and g: [0,00) 3 x t---+ e Hx , it follows that
f 0 g(x) = 1 + x = go f(x) and yet, for all nonnegative x, In(1 + x) < e Hx .
o
3.52. Because f is continuous, if f, 181 > 0 th~p for some h6 in Q \ {O},
Ih6 - 81 ::; 82 and If(c + 8) - f (c + h6)1 < f181. Hence

1 fCc + 8~ - f(c) _ LI ::; 1 fCc + 8) - / (c + h6) 1

+ f (c +~] - f(c) . h; - LI.


1

For small 181, the second term in the right member above is small since ~
is near one and the first member is small by virtue of the choice of h6.
. f(c+h)-f(c)
If f = XiQi and c = 0 then hmhEiQi\O,h->O h = 0 although,
since f is nowhere continuous, f'(O) does not exist. 0
216 3. Real- and Complex-valued Functions: Solutions

f:
c [a, bJ then for F(x) ct;bf f(t) dt, integration by parts
f: f:
3.53. If supp(h)
implies fJR f(t)h(t) dt = - F(t)h'(t) dt = - g(t)h'(t) dt whence F = g.
Because F' = f, g' exists and is f. 0

3.54. If limn -+ oo f (xn) = y then limn -+ oo Xn ct;bf x exists and f(x) = y.


Hence f (ll~n) is closed.
The inequality (3.2) implies that f is injective and hence, confined to
a closed ball in jRn, is a homeomorphism. From Brouwer's invariance of
domain theorem it follows that f is open whence f (jRn) is both open and
closed. Since jRn is connected, f is surjective: f is bijective. Finally, (3.2)
also implies f- 1 , which exists because f is bijective, is continuous. 0
3.55. The proof is an application of the average principle: an average lies
between the supremum and infimum of the numbers averaged.
If If(O)1 = SUPxEB(O,I) If(x)1 ct;bf M, Ilxoll = r < 1, and If (xo)1 < M
then

If(O)1 = M = 1-2
1 1
nr 1Ilyll=r
1 1
f(y) dyl ::; -2
nr 1Ilyll=r
If(y)1 dy. (s3.4)

Because f is continuous, If(y)1 < M near Xo whence the value of the last
member in (s3.4) is less than M, a contradiction.
If f is constant then If(O)1 = SUPXEB(O,I) IIf(x)ll· 0
3.56. Because f is uniformly continuous on K, if f > there is a positive
8 such that on K, {Ix - yl < 8} =? {If(x) - f(y)1 < fl. Consequently,
°
{Ix -yl < f} =? {If[tj(x) - f[tj(y)1 = If(x+t) - f(y+t)1 < f},

whence {f[tnj} nEN is uniformly bounded and equicontinuous. The Arzela-


Ascoli theorem implies the result. 0
3.57. For any x in [O,IJ and some ~ in (0, x),

f(x) ::; If(x)1 ::; fox f(t) dt = f(~)x ::; Ilflloox.


It follows by repeated integration that for each n in N and each x in [0,1),
xn
If(x)1 ::; IIflloo,.
n.
0
3.58. For each real a, there are coefficients bn(a), n E N, and a positive r(a)
such that for x in (a-r(a), a+r(a», 1'(x) = E~=1 bn(a)(x-a)n. The series
' '=--1 bn (a) (x n- +a)n+1
"'L.Jn_ 1 converges in (a - r( a), a + r( a» and represents a
function gao
On the other hand, foX l' (t) dt = ga (x) for x near a and, since l' is
continuous, g~ = 1', f = ga + constant, whence f is real analytic. 0
Solutions 3.59-3.60 217

3.59. If t E (0,1], f(n+l)(tx) is a monotonely increasing function of x.


Taylor's formula

f(x) fen) (0) xn +


= f(O) + f'(O)x + ... + __
n!
(1 (1 - tt f(n+l) (tx) dt
0
1
) n 1
-x +
n!
~f f(O) + j'(O)x + ... + f(n) (0) xn + R,,(x)
n!

leads to the conclusion that if 0 S; x < c then

x n+1 J/(l-t)n+1f(n+1)(tc)dt
OS;Rn(x)S; 0 ,
n.
fen) (O)cn )
Xn+1 ( f(c) - f(O) - j'(O)c - ... - n!

whence for all x, limn --+ oo Rn(x) = o. o


3.60. a) Abel summation implies

Ii an sin nx sin ~I
= ~ lam cos (n-~) x
+ i (an+1 - an) cos (n +~) x - aM cos (M +~) xl
S; ~ [am + f: (an - an+1) + aM 1 am·
=

If x E (0,71") and n = -;
def [1] then when m S; n

L
m

18m (x)1 S; nan x S; Kmx S; Knx S; K.


n=1

If m > n then, since


1.1 Ix 111
--Slll-X> - - - > - >-
an+1 2 - an+171" - an+1(n+l)7I" - K7I"'
218 3. Real- and Complex-valued Functions: Solutions

it follows that

18m (x)1 ~ 18 (x)1 + I f


n aksinkxl ~ K + ~n+~ ~ K +K1r.
k=n+! sm"2

Hence if x E (0,1rJ, 18N (x)1 ~ K(l + 1r).


b) When 0 E (0, 21r), Euler's formula e iO = cosO+sinO and the formula
for the sum of a geometric progression imply that there is a constant C 1
such that

It, sin nol ~f IKN(O)I ~ Isin~~/2)1·


Abel summation implies that if N 2 M and 0 <a ~ 0 ~ b < 21r then
for some constant C2 ,
N
18N(O) - 8M(O)1 ~ L lak - ak+11·IK k(O)1
k=M+1
+ laNI·IKN(O)1 + laM+11·IKM+!(O)1
~ C2 (laNI + laM+11)

whence convergence is uniform on [a, b].


c) If 2:;;=1 sin nO converges uniformly on [0,21r] as N -4 00, 0 = 2~'
and f > 0 then for large M and all N in (M,oo),

"2f > 8N(O) - 8M(O) 2 (N - M)aN = N-M


N NaN = (1-
M N) NaN·

Conversely, if an 10 and limn-;co nan = 0 the conclusion in a) implies


that it suffices to prove uniform convergence on [O,~]. Owing to point-
wise convergence everywhere on [0,21rJ, consideration can be focussed on
,",co . 0 def
R d def {}
L.m=M+1 anSm ; = [~]a:ef fM = maxn
2':M nan..
If 0 < 0 ~ 4" then (j = lI(O) EN and for M m N,

RM = {E~~L-+1 + 2:~=v(O)+l an sin nO ~f R'u + R'J.r if M + 1 ~ lI(O) .


2:~=M+1 an sin nO ~f 8M if M + 1 > lI(O)

If M + 1 ~ lI(O), the inequality Isin nOI ~ InOI implies

v(O) 1
/R'u/ ~ 0 L nan ~ lI(O) fM+1 (lI(O) - M) ~ fM+!·
n=M+1
Solution 3.61 219

In sum, R~ ~ 0.
If °< 7r • 0
< - then sm -2-8
- 0 -4
7rO •
> -. Smce

if M + 1 :::; v(O) Abel summation reveals that for some constant C3 ,

L
00

IR~I:::; (an - an+d IKn(O)1 + av(O) IKv(o)-l(O)1


n=v(O)
8 16C2
:::; 2C1 aV (O)-O :::; - - (v(O)
7r 7r
+ l)av(o) :::; C 3 EM.

If M +1 > v(O) the same kind of calculation reveals that for some
constant C4 , ISMI :::; C4 EM. Hence if C ~f max{Cl,C2 ,C3 ,C4 } then

f an sin nol : :; CEm·


In=M+l o

3.2. Complex-valued Functions

3.61. For n in Nand j in [0, n - 1],

Xn (j+1) = (j) +;:1 f ( (j))


----;:;- Xn ;: Xn ;:

and induction shows, since f is defined on [0,1]' that

Direct calculation shows that

whence {xn} nEN is uniformly bounded and equicontinuous. The Arzela-


Ascoli theorem implies that there is a subsequence {Xnk hEN and a contin-
uous function x such that x nk ~ x.
The equicontinuity of {x nk } nEN and the uniform continuity of f imply
that if "1 > 0, there is an no and a positive {j such that if nk > no then
220 3. Real- and Complex-valued Functions: Solutions

.
Hence If nk > max {I}
~,
u
no and -j ::; s
nk
j+l
< t ::; - - then
nk

Xnk(t) - xnk(s) = I (xnk


t - s
(L))
nk
I (Xnk (s)) - 'fJ ::; x nk (t~ =:nk(s) ::; I (Xnk (s)) + 'fJ
x(t) - x(s)
I (x(s)) - 2'fJ ::; t_s ::; I(x(s)) + 2'fJ.
It follows that

I(x(s)) - 2'fJ ::; limt=s X(t~ =:(s) ::; limt=s X(t~ =:(s) ::; I(x(s)) + 2'fJ

whence x'(s) exists and x'(s) = I(x(s)).


Because X(s) ~f J; I(x(t)) dt and x(s) are both solutions of
z' = I(z), z(O) = 0, (s3.5)

the uniqueness theorem for differential equations implies X = x. If the


entire sequence {xn}nEN fails to converge to x, some subsequence converges
uniformly to a different limit, say y. However both x and y are solutions of
(s3.5). 0
3.62. If {AhEN C N n and limk~oo Il/k - 11100 = 0, then for each k and
some Xk .
In
.
[0,1], If h 1= 0, Ilk (Xk + h)h - I
Ik (Xk) ::; n. It may be assumed

that for some x in [O,IJ, x - Xk ~f Ok --+ 0 as k --+ 00. Thus for large k,
1
0k : h I is near one and

II (x + hh - I (x) I::; II(x + h) ~ A(x + h) I


Ik (Xk + Ok + h) - A (Xk) I· 0k + h I
+I
1
Ok + h h
+ IIk (Xk) : I (Xk) I+ II (Xk) : I (x) I
def I+II+III+IV.
For any nonzero h and large k, I, III, and IV are small while II does not
exceed a number near n. Hence I E N n and so N n is closed.
There is a piecewise linear continuous "sawtooth" function G for which
at each point x, G(x + I hh - G(x) I > n if his sufficiently small while IIGlloo
Solutions 3.63-3.64 221

is arbitrarily small [GeO]. For any neighborhood N(I) of an f in N n , there


is a polynomial p such that p + G E N (I) \ N n. Hence the interior N~ of N n
is empty and so, as a closed set with no interior, N n is nowhere dense. 0
[Note 83.7: The set A ~f 0([0, l],q \ (UnENNn ) is of the
second category and consists entirely of (continuous) nowhere dif-
ferentiable functions.]

3.63. Write fn = !R (In) + i<;J< (In) ~ Un + iv n· If Ilfnlloo +0 as n -+ 00,


it may be assumed that for some positive 8, some x oo , and some sequence
{xn}nEN' Un (xn) ~ 8 and limn-+ oo Xn = Xoo. The mean value theorem and
the hypothesis imply

whence for large n and all x in [xoo - ~'Xoo +~], un(x) ~ ~.


If 9 is the piecewise linear function such that

_{1 on [xoo-~,xoo+~]
g(x)-
o
[8
off Xoo - 2'xoo +2
8]
f1 82
then for large n, io un(x )g(x) dx ~ 16' a contradiction. o
3.64. Because f(O) = 0, for some F in 0 1 ([-11",11"], q,
F(x) = {f(X) if x E [0,11"]
- f( -x) if x E [-11",0].
Integration by parts and the Schwarz inequality show that if
00

F(x) = I>nsinnx
n=l

[since F E 0 1 ([-11",11"], q, the right member converges everywhere to F(x)]


then

bn = :n (i:I'(x)cosnxdx + 1'" f'(x) cos nx dX)

Ibn 1< 111'112


- .j1rn

21'" If(x)12 dx = J~ IF(x)12 dx = 11" ~ Ibn 2 ~ (~:2 )1'" II'(xW dx


l

as required. 0
222 3. Real- and Complex-valued Functions: Solutions

3.65. The Fundamental Theorem of Calculus implies that if E is a


bounded subset of C 1 ru.,
C) then E is equicontinuous. The Arzela-
Ascoli theorem implies T(E) is compact. 0
3.66. a) Since II 11(1) ;::: II 1100, X is closed in C 1 ([0, 1], C).
b) Because II II 00 ~ II II (1), the inclusion map
T: c 1 ([0,1], C) '-+ C ([0,1], C)
restricted to X is a continuous bijection of Banach spaces and thus T is
open, whence T- 1 is continuous. Hence for some M, II 11(1) ~ Mil 1100' Let
1 .
k be M and let K be 1.
c) The mean value theorem for derivatives implies that a bounded
subset of C1 ([0,1], C) is equicontinuous. By virtue of 3.65, the inclusion
map T of b) above is compact, whence T(E) is compact. In particular,
T (B (0, It) is compact while, since T is open, T (B (0, It) is open. Since
a Banach space is finite-dimensional iff it is locally compact, it follows that
X is finite-dimensional. 0
3.67. Since I E C1 ('][', C), I is represented by its Fourier series:

I(z) = L anz n .
nEZ\{O}

Furthermore, the Fourier series for f' is 2:nEZ\{O} nan z n - 1, whence by


virtue of Parseval's equation,

1If'11~ = L n2 lan l2 , II/II~ = L la n l2 ~ II!'II~·


nEZ\{O} nEZ\{O}

Equality obtains iff for n other than ±1, an = O. o


3.68. Note that E1 = L2 ([0,1], ),), D (C 1 ([0, 1], C)) eEl, and for all I
in C1([0,1],C), IID(f)II' ~ 11/11". Because C 1 ([0, 1],C) is dense in Ell D
may be extended to a linear continuous map D : E2 ~ L2 ([0, 1], ),).
If D(f) = 0 then for some sequence {In} nEN in C 1 ([0,1]' C),

lim
n---+OCl
Il/n - III" = lim
n-+OCl
liD (fn)II' = O.
The Schwarz inequality and the choice of the sequence {fn}nEN imply

lim {1 I/n(x)1 dx = 0
n-oo}o
lim
m,n--+oo io(1 (1/m(x) _ In(x)12 + 1/:n(x) - 1~(x)12) dx = O.
Solutions 3.69-3.71 223

It may be assumed that limn --+ oo Jn ~ J and lim n --+ oo J~ ~ O. The Funda-
mental Theorem of Calculus implies that for all x in [0,1],

Hence limn --+ oo In(O) ~f C exists and, since x may be chosen off the null set
on which {fn}nEN fails to converge, J(x) ~ c as required. D
3.69. a) The triangle inequality la + bl ~ lal + Ibl for complex numbers
shows that D(O, 1) is convex.
b) If 0 ~ tn ~ tn+1 and L:;;=I ti = 1, the Euclidean structure of ]R2
yields the result if N = 2. If N > 2 and 0 < h < 1 then the formulre

and mathematical induction reduce the argument to the case when N = 2.


D
e
3.70. If L:;;=I ane inx == 0 on [a, b] then J : 3 z 1---+ L:;;=I ane inz is an
entire function vanishing on a compact infinite subset of e. Hence J(z) == 0
and thus J(x) == 0 on [0,271']. The orthogonality relations

1o
e· e -.mx dx =
271"" nx {O
271'
ifm #n
ifm =n

lead to the conclusion that an == o. D


3.71. If A ~f (aij)~j:l and det(A) = 0, the result follows. Hence it
may be assumed that det(A) # 0, whence the rows of A are linearly inde-
pendent vectors al, ... ,~ in en.
The Gram-Schmidt orthonormalization
process applied to {ai} 1<i <n provides an orthonormal set U ~f {Ui} 1<i <n .
Furthermore, there is a rower triangular matrix r such that - -

(s3.6)

If Ui = (Uil,"" Uin), (s3.6) may be written r (aiJ-t1,,)-


'-I = (UiJ'Y>2,3-
'-I' The
diagonal entries of rare 'Yii ~f Ilai - L:~:i (~, Uj)r I 2: Ilaill-i. Because
(Uij)~j:I is a unitary matrix, Idet [(Uij)~j:I] 1= 1, whence
D
224 3. Real- and Complex-valued Functions: Solutions

3.72. For m in Nand z in D(O, 1), the functions

J (~z)
(p _1 )' 1 ~ p~ m,
def
Fp(z) =
J --z
m

are continuous and IFp(z)1 == 1, 1 ~ p ~ m. For large m, and all z, because


the numerator and denominator of the fraction defining Fp are near each
other, Fp(z) f:. -1, 1 ~ p ~ m. Hence Fp (D(O, 1)) is a connected compact
subarc Ap of '][" \ {-I}, i.e., there is a continuous function

'TIp: D(O, 1) ;:, z 1-+ 'TIp(z) E (-11",11")

such that Fp(z) = ei '1/p(z) E Ap. Since

J (E-z)
J(z) = J(O)· II (p ~ 1 ) '
l$p$mJ - - z
m

it follows that if J(O) ~f ei '1/0,-11" < 'TIo ~ 11", and ¢(z) ~f 'TIo + EZ'=o'TIk(Z)
then J(z) = eiq,(z). 0

3.73. If G(O) ~ 9 (ei(l)+1r)) - 9 (e ilJ ) then G(11") = g(l) - g(-l) = -G(O).


Hence either G(O) = 0, p = 1, and p = -lor for some 0 near 11", G(O) and
G(O) are of opposite sign, whence at some 'IjJ in [0,11"), G('IjJ) = O. Thus if
p = ei'lj! and p = e i (1/!+1r) then g(p) = 9 (P). 0
3.74. Because hiT
maps '][" into itself, 3.73 implies that at least one of
some pair of antipodal points is not left fixed. 0
3.75. If no point is left fixed by J then for z in D(O, 1), the closed half-line
determined by z and J(z) and of which J(z) is the endpoint meets '][" in a
point h(z). The map h is continuous and h leaves each point of'][" fixed, in
contradiction of 3.74. 0
4
Measure and Topology

4.1. Borel Measures

4.1. a) Because L(f) E (u, v), if y = m(x - a) + </>(a) is the equation of a


supporting line through (a, </>( a)), for x in X,

m (f(x) - a) + </>(a) :::; </> 0 I(x)


L (m (f(x) - a) + </>(a)) = m(L(f) - L(f)) + </>(L(f)) :::; L(</> 0 I).

b) If (*) obtains for every simple Lebesgue measurable function I,


assume p, q E JR, 0 :::; a, f3, and a + f3 = 1. For I def
= PX(O,a] + qX[a,l) '

[ I(x) dx = ap + f3q

r </>(p) dx + 11 </>(q) dx = a</>(p) + f3</>(q)


1[0,1]

[ </> I(x) dx =
k
0
1~~ a

</> ( [ I(x) dX) = </> (ap + f3q):::; [ </> 0 I(x) dx = a</>(p) + f3</>(q).
1~,~ 1~,~
o
[ Note s4.1: The idea behind the original version of Jensen's
inequality is that approximating Lebesgue sums for I(x) dJ-t(x) Ix
are, because J-t(X) = 1, convex combinations of values of I. The
convexity of </> implies an inequality between </> applied to the
Lebesgue sums for I and the Lebesgue sums for </> 0 I. The con-
tinuity of </> permits a valid passage from the inequalities for the
Lebesgue sums to the same inequalities for Lebesgue integrals.

The impact of b) is that Jensen's inequality when X = [0,1]


implies Jensen's inequality for any X as described in a).]

4.2. By abuse of language, the functional lim resp. lim is superadditive


resp. subadditive and positive homogeneous, i.e., if t ?: 0 then

lim (f + g) ?: lim 1+ limg


lim(f+g):::; lim I + limg
tliml = limtl, tliml = limtl. o
225
B. R. Gelbaum, Problems in Real and Complex Analysis
© Springer-Verlag New York, Inc. 1992
226 4. Measure and Topology: Solutions

4.3. With respect to inclusion as a partial order, the set A of all finite
subsets of N is a diset. If /-L(X) = 1, for ,X E A, /-L('x) ~f max {n : n E ,x},
and for {fn}nEN' n", ~f f/1-("')' Jensen's inequality applies for the convex
functionals lim"'EAn", and -lim"'EAn",.
More generally, if {fm}mEN C L 1(X,/-L) thenB;o' (fm,O) ~f Em is (7-
°
finite: < /-L (An) < 00 and UnENAn = UmEN Em. Then the conclusion in
the previous paragraph applies and, owing to the positive homogeneity of
lim and lim, leads to:

Mutatis mutandis, limk---+oo Ix Jk(x) d/-L(x) :::; Ix limk---+oofk(x) d/-L(x) ,


[which is true as well iflimk---+oofk rJ. L1(X,/-L)]. D
°
4.4. a) If < 101 < 102 then {diam(U) < Ell ::::} {diam(U) < E2} whence,
for fixed A, ~(A) is a monotonely increasing function of E.
b) If 15 > 0, {Umn }m,nEN is a double sequence of open sets, each of di-
ameter less than 10, if UmEN Umn :J An, n E N, and 2::=1 (diam (Umn))P <
15
~ (An) + 2n ' then

: :; L ~ (An) + 15 :::; L
00 00

pP (An) + 15
n=1 n=1

°
c) It may be assumed that < £b) < 00 and, by a change of scale, that
£b) = 1. The parametrization of l' may be chosen so that £ b ([0, tJ)) = t,
i.e., so that the parameter is arc-length.
Solution 4.4 (cont.) 227

In these circumstances, if mEN and 1 ::; k ::; m - 1 then

d(,(~),,(k:1)) < ~.

Hence U;'=l B (, (2~: 1) '2m1_1) 0 J A and so P~"?--l (A) ::; 2~r: l'
As m --+ 00 there emerges the inequality pleA) ::; 1.
For some partition Q ~ {Sj}O::;j9 of [0,1],

J-I
Ld(l'(sj+1),,(Sj)) > 1-8
j=O

and it may be assumed that minj d (, (Sj+l) " (Sj)) ~f ~ > O.


If a < < max: { 4~' ~ }, then for some positive 7], It' - t"l < 7] and
fl

d(,(t'),,(t")) < fl. If IFI < min{7], IQI} then for some kl' k 2 , .. ·,

Hence

1- 8< ~ Cf., db(tw),> (t k »)


+ Ld(,(sj),,(tk,)) +d(,(tkp+l) ,,(Sj))'
j

8
The second sum in the right member above is less than J . 2fl < 2" and
so the first (double) sum exceeds 1 - 3:. Because, is a homeomorphism,
there is a positive f such that

[In other words, if points corresponding on , ([0,1]) to successive partition


points of [0,1] are close to each other, the associated approximants to £(,)
are close to £(,).]
For 8, f as in the preceding paragraph and a positive, let {Um } mEN be
a sequence of open sets such that

U Um, diam (Um) < L


00

A c f, mEN, diam (Um) < p! (A) + a.


mEN m=l
228 4. Measure and Topology: Solutions

Since A is compact, for some M in N, A c U:;;=1 Um. Let to be 0


and for some m not exceeding M, let Vo be a Um containing "/ (to). If
{Vi}o~i~k have been defined so that each is a Um, m ::; M, "/ (ti) E Vi, no
two Vi are the same, and tk = 1, the process stops. Otherwise, tk+1 ~f
sup {t : ,,/(t) E Vk} and Vk+1 is a Um, m ::; M, covering "/ (tk+1). Thus
Vk+1 is different from all the Vi previously defined. Since M is finite, there
emerges a set {Vih<i<K<M such that no two Vi are the same, tK = 1, and
maxi d b (ti+d ,,,/ (~)) <-€ whence 2:~o d b (ti+1) ,,,/ (ti)) > 1- 8. On the
other hand, 2:~o d b (ti+1) , "/ (t i )) ::; 2:~o diam (Vi) < p!(A) + a. Thus
1-8::;p1(A)+a. 0
4.5. It suffices to show p~(A) ::; €q-P~(A). If {Um}mEN is a sequence of
open sets such that UmEN Um :J A, diam (Um ) < €, mEN, it follows that
pHA) ::; 2::=1 (diam (um))q ::; €q-p 2::=1 (diam (Um)t· 0
[ Note 84.2: Besicovitch [Be8, GeO] showed that in ]R3 there
is a homeomorphic image 8 of 8B(O, 1) and A3(8) is large while
the two-dimensional surface area of S is small. However it is im-
possible for p3(8) to be positive while p2(S) is small.]

4.6. If E c ]RP then A;(E) ::; (diam(E)t. Hence if {Um}mEN is a


sequence of open sets such that U mEN Um :J A, diam (Um ) < €, mEN, then
A;(A) ::; 2::=1 Ap (Um) ::; 2::=1 (diam (Um))P whence A;(A) ::; PP(A).
If A;(A) = 00 then for any constant cP' A;(A) 2': cpPP(A). If A;(A) <
8
00 and 8,€ > 0, then for some open U, A c U and A;(A) > Ap(U) - 2".

Furthermore, U is the countable union of pairwise disjoint half-open p-


dimensional cubes Km and so Ap(U) = 2::=1 Ap (Km). For a sequence
{17m}mEN of positive numbers, each Km is contained in an open cube Lm
such that
8
L
00

Ap (Lm) < Ap (Km) + 17m and 17m < 2".


m=1

Consequently Ap(U) > 2::=1 Ap (Lm) - ~.


For an open cube L of side s,

8
2" > cp L
00

Hence A;(A) > Ap(U) - (diam (Lm)t - D ~ cp/Y.'(A) - D. 0


m=1
4.7. Because (cf. 4.4) PP is an outer measure, it is count ably subadditive
and it suffices to show PP(A U B) 2': PP(A) + PP(B).
Solution 4.8 229

If 0 <f < ~ and {Um}mEN is a sequence of open sets such that

UU m :) AUB, diam(Um) < f, mEN,


mEN

then no Um meets both A and B. Hence there are disjoint subsequences

such that UI~k<K Umk :) A and UI~l<L Um; :) B. For a positive 'T/, the
Um can be chosen so that
00

m=1
2: L (diam(Umk)t+ L (diam(Um;)Y -'T/
l~k<K 1~l<L

PP(A U B) 2: ~(A) + ~(B) - 'T/. o


4.8. If PP(8) = 00 then the subadditivity of pP implies

PP (8) = PP (8 n F) + PP (8 \ F) .

If PP(8) < it suffices to show PP(8) 2: pP (8 n F) + pP (8 \ F). If, for


I} .
00,

. N, Um
m m =
def { x: d ( )
x,F >m then Um IS open, Um C Um+l, and
UmEN Um = X \ F. Since ((8 \ F) \ Um) = U%"=m+1 (8 U (Uk \ Uk-I)), it
follows that

L
00

PP ((8 \ F) \ Um)::; PP (8 n (Uk \ Uk-d)· (s4.1)


k=m+1

If k and k' are two integers of the same parity and greater than one,
then 8n (Uk \ Uk-I) and 8n (Uk' \ Uk,-d are a positive distance apart and
thus (cf. 4.7)

k~n,k odd

k~n, k even

L
00

PP (8 n (Um \ Um-I)) ::; 2PP(8) < 00.


m=1
230 4. Measure and Topology: Solutions

It follows from (s4.1) that

lim p1' ((8 \ F) \ Urn)


rn-+oo
= °
p1' (8 n Urn) ::; p1' (8 \ F) ::; p1' (8 n Urn) + p1' ((8 \ F) \ Urn)
lim p1' (8 n Urn) = p1' (8 \ F) .
rn-+oo

Because inf {d(x, y) : x E 8 n F, y E 8 n Urn} > 0, from 4.7 it follows


that pP (8) 2: pP (8 n F) + pP (8 n Urn) and PP(8) 2: PP(8 n F) + PP(8 \ F).
o
4.9. From 4.8 it follows that all Borel sets are pp-measurable and 4.6 im-
plies Ap and pp are mutually absolutely continuous. Since pp is translation-
invariant, the Radon-Nikodym derivative ~~ is a constant Kp. Hence, for
p
any Borel set A, Ap(A) = KpPP(A).
For any set 8,

A;(8) = inf {Ap(U) : U open, U::J 8}


= Kp inf {p1'(U) : U open, U::J 8}
2: Kpp1'(8).

If €, 8 > 0, there is a sequence {Urn} rnEN of open sets such that

U Urn ::J 8, diam (Urn) < €


rnEN

L
00

p1'(8) 2: rJ:(8) 2: (diam (Urn))P - 8


rn=l

2: inf {rJ:(v) : V open, V::J 8} - 8.

Hence for each n in N, some open set Vn contains 8 and

1
p1'(8) 2: rJ: (Vn) - - - 8.
n

As first, € ---+ 0, then 8 ---+ 0, and finally n ---+ 00, the result emerges. 0
4.10. The set R of fl.-regular Borel sets contains 0: R i- 0. Direct
calculations using the formulre of set algebra imply R is a ring.
Solution 4.11 231

E
If E > 0, R 3 An cAn+! c Un+! E O(X), and J.L (Un \ An) < 2n then

00

A ~f U An C U Un ~ U E O(X),
nEN n=2

n=2

N def (\ E
If Un=lAn = BN then BN E R, for large N, J.L A B N) < 2' and BN
. E
contams a compact KN such that J.L (BN \ K N ) < 2. Hence

AER.
If E > 0, R 3 Dn :J Dn+! :J Kn+l E K(X), and J.L (Dn \ Kn) < E,
then D ~ nnEN Dn :J n:=2 Kn ~ K E K(X) and J.L(D \ K) < E.
N
If nn=l Dn def = EN then for large N, EN E R, J.L ( EN\ )
D < 2'
E
and
E
EN is contained in an open set UN such that J.L (UN \ EN) < 2. Hence
J.L (UN \ D) :::; J.L (UN \ EN) + J.L (EN \ D) < E; D E R.
Thus R is monotone, whence R is a a-ring (cf. 1.1, 1.2).
If an open ball B(x, r)O is a-compact it is regular by virtue of the
monotone convergence theorem. If 0 < 8 < r then B(x, 8)° is also regular.
Hence for each n in N and each k in K, there is a regular open ball B (k, rkt
such that 0 < < !n and so for some finite set {kdl<i<I
rk contained
--
in K, K c U:=l B (ki' rk;t ~f Un and Un is regular. If x tJ. K then
inf {d(x, k) : k E K} ~f 8 > 0 and for some n, ! < 8: x tJ. Un, i.e.,
n
K = nnEN Un. Since R is a a-ring, K is regular: aR(K(X)) C R. 0

4.11. Let {xn}nEN be dense in X. For m in N, X = UnENB (Xn' ~) and


·f FmN
1
def = Ul~n~N B (1)
n, x m .
then FmN C Fm,N+1. hmN-+oo FmN = X,
and limN-+oo J.L (FmN) = J.L(X).
E
If E E (0, J.L(X)) and mEN, for some N m , J.L (X \ FmN".) < 2m whence
232 4. Measure and Topology: Solutions

If {Yn}nEN C nmENFmN", ~f K€ then: in some B (Yn"~) there is

an infinite sul>requence {y•• hEN; ... ; in ",me B (y....: ~) there is an

infinite ,"h... sul>requence { y....} ;.... Flrrth",more,


Pq qEN

E =
def {
Ynl,Ynr2, ... ,Yn ... pq' ••• }

is a Cauchy sequence. Because X is complete, E converges and because K€


is closed, E converges to a Y in K€, whence KE is compact.
Because X is a metric space, every closed set is a G{j. Since J.L is finite,
every closed set is outer regular. Furthermore, if F is closed then F n KE is
a compact subset of F and J.L (F \ K€) < f, Le., every closed set is regular.
The set R of regular sets is a a-ring (cf. 4.10) and contains all closed sets,
whence every Borel set is regular, Le., J.L is regular. 0
4.12. The result in 4.11 is applicable to J.L restricted to any B (0, r), i.e.,
J.L so restricted is regular.
For a Borel set E and a positive r, if Er ~f E n B(O, r) then

E = U Em = U (Em \ U Ek) .
mEN mEN k<m
If f > 0 then for each m, there is an open set Um and a compact set Km
such that

Um:J U Ek :J Km
k<m

J.L (Um) - 2~ < J.L (Em \ k<rn


U Ek) < J.L (Km) + 2~ .
If J.L(E) < 00 then Em \ Uk<m Ek is both inner and outer regular
because f..£ restricted to B(O, m) is regular.
def M def - . - .
Hence U = UrnEN Um :J E,E:J Um=l Km = KM, U 1S open, KM 1S

compact, and J.L(U) - f < f..£(E) < J.L (Km) + 2f, whence E is regular.
If J.L(E) = 00 then E is outer regular and

J.L(E) = mlim J.L(Em):::::: m--t


--t (X)
lim00 J.L (Urn Kk) +f,
k=l

whence E is also inner regular. o


Solutions 4.13-4.16 233

4.13. As a compact metric space, X is separable and complete. According


to 4.12, J.L is regular. Hence for each x there is a positive 8(x) and a finite set
{Xd1::;k::;K such that J.L (B(x, 8(x) )0) < i and X C Uf=l B (Xk, 8 (Xk) r. If

8 = ~ m~n {8 (Xk)}, if diam(E) < 8, and if En B (Xk, ~8 (Xk)) ° '" 0 then


E C B (Xk, ~8 (Xk)) ° whence J.L(E) < E. 0

4.14. For U ~f {U : U open J.L(U) = O} partially ordered by inclusion,


let {U y } 1'Er be a maximal linearly ordered subset of U and let F be the
closed set X \ U1'Er U y•
Because J.L is regular and totally finite, if 10 > 0, there is an open set V
containing F and J.L(V) < J.L(F) +10. Then {V, U1'}1'Er is an open cover of X,
whence for some finite set bd1<k<K'_ _ X = V U Uf=l U1'k· Since {U1'} l' Er
is linearly ordered, for some k o, X = V U U1'ko and

J.L(X) ~ J.L(V) + J.L (U1'kJ = J.L(V) + 0 < J.L(F) + E.

Since F c X, J.L(X) = J.L(F) and J.L (U1'Er U1') = O. If F1 is closed and


J.L (X \ F1) = 0 then X \ F1 ~f U* E U.
1.
If F1 C F then U* U1'Er U1' which denies the maximality of {U1'} 1'Er·
Hence F1 ct F.
Because f- 1 (0) is closed, f(x) == 0 iff f- 1 (0) :) F. 0
4.15. If E is a Borel set and if 10 > 0, then for some compact set K,
contained in E, v(E) < v (K,) + E. If, furthermore, E = UnENEn and
each En is a Borel set, for each n some open set Un contains En and
v (En) > V (Un) - 2:. For all N, l:;:'=lV (En) = V (U;:'=l En) ~ v(E)
whence l:~=lv (En) ~ v(E). For some N 1, U;:'~l Un :) K, and so

Nl
v(E) ~ V (K,) + 10 ~ L v (Un) + 10
n=l
00 00

o
n=l n=l

4.16. A finite Borel partition 71" of X is a finite sequence {En} ;:'=1 of Borel
sets such that X = U;{=lEn. When U ~f {X n };:'=l is finite, write u rv 71"
iff Xn E En,1 ~ n ~ N. If {Ul,7I"1} -< {U2,7I"2}, iff U1 -< U2 in the natural
partial order of sets and 71"2 refines 71"1, i.e., 71"1 -< 71"2 in the natural partial
234 4. Measure and Topology: Solutions

ord er 0 f part 1't'Ions then { {


ordered by -<.
<7, }
7r = =,
<7 rv 7r } def r def { }"YEr IS
. partl'ally

For two elements {<71' 7rd and {<72' 7r2} of r, there is a 7r3, a refinement
of both 7rl and 7r2, such that <71 U <72 ~f <73 rv 7r3 and {<73, 7r3} »- {<71' 7rl}
and {<73, 7r3} »- {<72' 7r2}. Thus r is a diset.
For each, = {{Xn}~=l' {En}~=l} and each Borel set E let IL"Y(E) be
LXnE E IL (En).
If IL(X) = 0, the conclusion is automatic. If IL(X) > 0 then for I in
C(X,q and E positive, there is a Borel partition {En}~=l of I(X) and
maxn {diam (En)} < 2IL~X)' If

then {<7,7r} ~f, E r and if {<7l,7rl} ~,l »-" let 7rl be {Fm};;;=l' Then

II I(x) dIL"Yl (x) - I I(x) dIL(X) I


~ tllLm l(x)dIL"Yl(X) - L", l(x)dIL(X)I·

Because ,1 »- "
ElL (Fm)
the mth summand in the right member above does not
exceed 2IL(X) and so the whole sum does not exceed E. 0

4.17. If ILl and IL2 are regular complex measures, IL ~ ILl - IL2, and E >0
then for some (infinite) Borel partition {En}nEN of a Borel set E,

00

IILI (E) < LIlLI (En) - IL2 (En)1 + E.


n=l

For each n and some compact set Kn contained in En,

N
IILI(E) < LIlLI (Kn) - M2 (Kn)1 + 3E ~ IMI (U:=lKn) + 310,
n=l

whence IMI is inner regular.


Solutions 4.18-4.20 235

A similar argument, mutatis mutandis, shows IILI is outer regular.


o
4.18. If EI =f. E2 then 8B (x, EI) n 8B (x, E2) = 0 whence at most count ably
many 8B(x, E) have positive measure. Thus for each x, there is a sequence
1
{En(x)}nEN such that 0 < En(X) < - and IL(8B(x,En(X))) = O,n E N.
n
For each n and some finite set {Xnp}:':'l' X = U:':'l B (Xnp , En (Xnp)t.
If V is open and x E V then for some positive E, B(x, E)O C V. If ~ <E
m
then for some p, x E B (Xmp , Em (Xmp) t. If Y E B (Xmp , Em (Xmp) t then
d(x, y) < ~, i.e., B (X mp , Em (Xmp)t C B(x, E)O C V. Thus
m

is a base for the topology of X and IL (8 (Uk)) == O. 0


4.19. If E > 0, K(X) 3 K c U, and ILI(K) > ILI(U) - E (cf. 4.11) then
for some IK in C (X, q, K -< IK -< U. Thus

ILl (U) 2: Ix IK(X) dILl (X) 2: ILl (U) - E, ILn (U) 2: Ix IK(X) dILn(X)

limn-+ooILn(U) 2: lim
n-+oo}X
r
IK(X) dILn(X) = r IK(X) dILl (X) 2: ILI(U) -
}X
E

whence limn-+ooILn(U) 2: ILI(U),


If K(X) 3 K c U then Ix
IK(X) dILn(X) 2: ILn(K) and so

limn-+ooILn(K) :::; Ix IK(X) dILl (X) :::; ILI(U),

Hence if V is open and V :J U then ILl (V) 2: limn-+ooILn (U). If W is open


and W :J 8U then ILI(W) 2: limn-+ooILn (8U). Because ILl is regular

o = ILl (U) 2: limn-+ooILn (8U) 2: 0,


i.e., ILn (8U) == O. Hence ILn (U) == ILn(U) and ILl (V) 2: limn-+ooILn(U), The
regularity of ILl implies ILl (u) = ILI(U) 2: limn-+ooILn(U), 0
4.20. If E > 0 then for each x, there is an open set U(x) containing x,
sup {II (YI) - I (Y2)1 : Yl, Y2 E U(x)} < E,
and (cf. 4.19) IL (8U(x)) = O. For some finite N, U;;=l U (xn) = X. If
An ~f U (Xn) \ U:~\ U (xm) and Vn ~f U (Xn) \ U:~ll U (xm) then
236 4. Measure and Topology: Solutions

U is open, and F is closed. Hence

lim /Ln(U)
n-+oo
lim /Ln{F) = n---too
+ n-+oo lim /Ln{X) = /LI{X)
= /LI{U) + /LI{F) = n-+co
lim /Ln{U) + /LI{F)

and limn -+ co /Ln{F) = /LI{F) = 0. Thus if Yk E Vk , 2 S k S N, then

Ii f{x) d/Ln{X) - i f{x) d/LI (x) I


S liuf{x)d/Ln{X) - iuf{X)d/LI{X)1 + It f{x) d/Ln{x) I
S t, Ilk f{x) d/Ln{X) - f (Yk) /Ln {Vk)1

+ It, f (Yk) {/Ln (Vk) - /LI (Vk)) I


+ It, (f{Yk)/LdVk) - lk f{X)d/LI{X)) I
+ It f{x) d/Ln{x)l·

According to the choice and construction of the Yk and the Vk and since
limn -+co /Ln{F) = 0,

o
4.21. For each n, /Ln may be regarded as a continuous linear functional
on C {X, IC}. The uniform boundedness principle implies that there is a
finite M such that for all n in N, /Ln{X) S M, whence /LI{X) S M. The
argument in Solution 4.20 applies. 0
4.22. If x E E then x belongs to an interval of positive length and hence
each component of E is a nondegenerate interval. Hence E is a Borel set
that is the union of at most count ably many intervals. 0
4.23. a) Because x E 1I1R, [x] < X < [x] + 1, {x} E CO, 1), and in N there is a
least n{x) such that n{x)
1 }
< {x < n{x)' 2 def. {
If 8 = mf Y : Y E .6. x
( )}
> °
then 8 E(0,1), {8} = 8, nt8) < 8 < min {n~8)' n{8~ _ I} and
n(8)8 - 8 < '- < n(8)8 < 2, [n(8)8] = 1, {n(8)8} = n(8)8 - 1 < 8,
Solution 4.23 (cont.) 237

a contradiction. Hence 8 = 0 and if m, n E N and K ~ [{ ~x }] then

x = [mx] + {mx}, k{mx} < 1, kmx = k[mx] + k{mx}, {kmx} = k{mx},

k = 1,2, ... , K. Thus the K equally spaced numbers {mx}, ... , K {mx} lie
1
in (0,1). For N in N, there is an m such that {mx} < N whence K :::: N.
Hence .6(x) = [0,1]. (For an alternative approach, cf. Solution 4.36.)
b) The proof is based on the following derivation of Weyl's criteria for
equidistributivity.
For any sequence S ~ {x n } nEN in JR, if 0 ::; a < b ::; 1 then S is
equidistributed iff when f = X[a,b],

It follows that S is equidistributed iff for any step-function f,

If f is Riemann integrable and 10 > 0 there are step-functions It, h


such that It ::; f ::; hand fo1 h(x)dx - fo1 It(x)dx < 10. Hence S is
equidistributed iff for every Riemann integrable function f,

·
11m
n--+oo
I:~-1 f (Xk) =
n
11
0
f( x ) dX.

tributed then

(s4.2)

If 9 is Riemann integrable,jo1 g(x) dx ~f A, and 10 > 0 then:


i. there are step-functions UI, U2 such that U1 + 10 ::; 9 ::; U2 - 10 and
II u2 - u1111 < 210;
ii. in Coo ([0, 1],C) there are V1,V2 such that VI ::; U1,U2 ::; V2, and
Ilui -villI < 3€,i = 1,2;
iii. if Bi ~ fo1 Vi(X) dx, i = 1,2, P contains trigonometric polynomials
(PI, ¢2 such that IIVi - Bi - ¢illoo < 10.
238 4. Measure and Topology: Solutions

Hence IL:~-l:i (Xk) - Bi - L:~=l:i (Xk) I < €, i = 1,2. If (s4.2) ob-


tains for all ¢ in P it follows that

t g(x) dx =
Jo
lim
n-+oo
L:~=l 9 (x k ).
n
In sum, Sis equidistributed iff (s4.2) obtains for all ¢ in P.
If x is irrational and if Xk ~f {kx}, kEN then for each N in N, the
formula for the sum of a geometric series implies

n n 27rinNx 1
~ e27riNxk = ~ e27riNkx = e27riNx e , -,
L..t L..t e21rtN x-I
k=l k=l

whence IL:nk-l ne27riNxk I ~ neI 2 'N2 x-I I


7rt
--t 0 as n --t 00. o
4.24. Let M be sup {f.L(E) : E E 5}. Then since 5 is a monotone class,
ME f.L (5). If M = 0 then {M} = {J.t (0)} = f.L (5). If 0 < t < M and

s ~f inf {f.L(E) : E E 5, f.L(E) ~ t}

then for some sequence {En}nEN in 5, En ::::> En+l' f.L (En) ! s, and

f.L ( lim En)


n-+oo
~f f.L(E) = s.

If s > t then for some sequence {Hn} nEN in 5, Hn C H n+ 1 C E, f.L (Hn) ~ t,


and

r ~f sup{f.L(L) : L E 5,L c E,f.L(L) ~ t} = f.L (lim


n-+oo
Hn) ~ f.L(H).

If s - r > 0, A E 5, and A C (E \ H) then f.L(A) = 0 or f.L(A) = s - r, i.e.,


E \ H is an atom, in denial of the hypothesis. Hence s = r = t. 0
4.25. Fix fo in S and let K be supp (fo). Since K is compact, for some 9
in L, K -< g. Let I(g) be M, which may be taken as positive.
For € positive and s in S, let Us be the open set {x : sex) < €}. If
x E K then either fo(x) = 0, in which case x E Ufo, or foex) > 0, in which
case, by hypothesis, for some f in S, f(x) < fo(x) 1\ € and thus x E Ufo
Solutions 4.26-4.27 239

In sum, K C UfES Ufo Because K is compact, there is a finite subset


{h, ... , In} of S such that K C U~=l UJ;- If IE S then

hf ~f 1/\10/\··· /\ In E S, hfluf\K= 0, and hflK< 10 = €glK"

Thus 1 (hf) < 10M whence inf {I(f) : I E S} = o. o


4.26. Each I in Lu is lsc (cf. 2.105). If a E X, Na is the set of
neighborhoods of a, and I E S then

lim",=aF(x) = sup inf F(x)


NENa "'EN

~ sup inf I(x)


NENa "'EN

= lim",=al(x) = I(a)

whence lim",=aF(x) ~ F(a), F is lower semicontinuous, and thus F E Lu


(cf.2.105).
Because J is nonnegative, A:::; I(F). For

T ~f {g : gEL, for some I in S 9 :::; I},

F = V {g : gET}. If A < I(F) then for some h in L, h :::; F and


I(h) > A. But h = h/\F = h/\ V{g : gET} = V{h/\g : gET}.
Since {h} uTe L, the set {h - h /\ 9 : gET} is one to which 4.25
applies. Hence

A<I(h)=sup{l(h/\g): gET}:::;sup{l(g): gET}


= sup {J(f) : IE S} = A,
a contradiction. o
4.27. Since 1 is nonnegative, p,*(U) ~ sup{p,*(K) K E K, K c U}.
However Xu is lsc, whence Xu E Lu and, according to 2.105 and 4.25,

p,*(U) = p,(U) = 1 (Xu) = sup {I(f) : IE L, I:::; Xu}.

If Q < p,(U) then for some I in L, Q < l(f) and 0 :::; I :::; Xu. Furthermore,
for n in N, Kn ~ {x : I(x) ~~} is compact,

Kn c Kn+l C U, UKn = {x : I(x) > o} ~f W E 0, and


nEN

whence for some n, p, (Kn) > Q.


240 4. Measure and Topology: Solutions

Finally,

sup{JL*(V): VEO,K03VCU}:::;Sup{JL*(K): K03KCU}.

However if K 3 K C U then for some V in 0, U ::> V ::> V::> K. 0


4.28. Because JL*(E) :::; inf {JL*(U) : U E 0, E C U}, only the reversed
inequality needs attention, and then only if JL*(E) < 00. By definition, for
some sequence {gn}nEN in L u , gn ~ XE and I (gn) 1 JL*(E). Since each gn
is Isc, if f > 0, the set Un ~f {X : gn(X) > 1 - f} is an open set containing
E and I (gn) ~ JL (Un) ~ JL*(E). 0
4.29. a) Because for f in L, 1/\ fELl, it follows that 1 E M and hence
that A is a a-algebra. If U E ° then Xu E Lu (2.105). Hence for all f
in L, Xu /\ f E L C L1 whence Xu E M. In light of the equivalence of
Caratheodory and Daniell measurability, U E A. Since A is a a-algebra, it
°
follows that U F c A.
b) If E E 0, JL*(E) = JL(E),( cf. 4.28).
c) If E E °
then a) implies E is measurable and 4.28 implies the
desired conclusion. If JL(E) < 00 then f ~ XE ELl and if f > 0, there are
hand 9 such that h :::; f :::; g, h is usc, 9 is Isc, and

0:::; l(g - h) = l(g) - l(h) < f

(cf. 1.32). Because h is usc, so is k ~ h V o. It follows that 0 :::; k :::; f :::; g,


0:::; l(g - k) = l(g) - l(k) < f. For some sequence {Tn}nEN in L, Tn 1 k
and L 3 Tn /\ 0 ~f k n 1 k. If 0 < a < 1 then K ~f {x : k(x) ~ a} is
closed and for n in N, K c En {x : kn(x) ~ a}, whence K is a compact
subset of E. As a 1 0 JL(K) i JL(E). 0
4.30. If {KnJnEN C K(X), X = UnENKn, and E E A then

EnKn ~f En E A, UEn = E, and JL(En) < 00.


nEN

From 4.29c) it follows that when f > 0, En contains a compact set Bn such
f
that JL (Bn) > JL (En) - 2n . It follows that for large N, the measure of the
compact set U;;=l Bn is close to the measure of E.
If (X, A, JL) is a-finite and E E A then E is the countable union of
measurable sets of finite measure. Hence 4.29c) applies. 0
4.31. i. If )"(E) = 0 then for n in N, E can be covered by a sequence Sn
of intervals of total length 2- n . The set of all intervals in all the Sn is itself
a sequence of intervals, the sum of all their lengths is not more than one,
and each point of E belongs to at least one interval in each Sn-
Solution 4.32 241

Conversely, if E can be covered in the manner described and f > °


then removal of finitely many of the intervals can bring the length-sum of
the others below f while their union covers E.
ii. Let {Tn}nEN be an enumeration of Q n [0,1]. For k in N, cover Tn
with an open interval Ikn contained in (0,1) and of length not exceeding
2-(k+n). Then Uk ~f UnEN Ikn is open, Uk :::> Q n [0,1]' and A (Uk) ::; 2- k.
Hence E ~ nkEN Uk is a Gli contained and dense in [0,1]' A(E) = 0, and
E is a set of the second category. Because the complement of a dense Gli
is a set of the first category, E must meet every dense G{j.
On the other hand, if I is Riemann integrable then Cont(f), which is a
GIi, must be dense [otherwise A (Discont(f)) > 0]. Thus E meets Cont(f).
iii. Cover E in the manner described in i and let the set of all intervals
used be S. For each interval (a, b) in S, form the function

lab: IR :3 x f-t
{°X - a if -00 < x < a
if a ::; x ::; b
b-a ifb::;x<oo

Then I:(a,b)ES lab(X) ~f I(x) is majorized by the sum of the lengths of all
the intervals in S whence I is continuous and monotonely increasing.
If x E E let I k be the intersection of the first k intervals containing
x. If y E Ik \ {x}, the ratio lab(Y) - lab (x) is 1 for each of the k intervals
y-x
(a, b) used to create Ik and the ratio is nonnegative everywhere since each
lab is monotonely increasing. Hence I (y) - I (x) ~ k and so f' exists at
y-x
no point x of E. o
4.32. If a E Cont(f) and In(a) f+ I(a) then for some positive f and some
subsequence {fnk hEN

Ink (a) ~ I(a) + f or Ink (a) ::; I(a) - f.

In the first instance, for some positive 8, if a ::; x < a + 8 then I(x) <
I(a) +~ while

Thus A ({ x : I(nk(x) ~ I(x)}) ~ 8, kEN, a contradiction. The second


instance is handled, mutatis mutandis. o
242 4. Measure and Topology: Solutions

4.2. Haar Measure

4.33. If j, g, h E Coo (G, C) Fubini's theorem, Holder's inequality, the


Hahn-Banach theorem, and the translation-invariance of J1, imply

fa 1/ * g(x)h(x)1 dJ1,(x) :::; fa I/(Y)I (fa Ig (y-1X) 1'lh(x)1 dJ1,(X)) dJ1,(Y)


: :; fa I/(Y)I dJ1,(Y) ·lIglip . IIhllpl = II/IiI· IIglip . IIhllpl
II/ * gllp :::; II/Ill . IIgllp· (s4.3)

Since Coo (G, C) is II Ill-dense resp. II lip-dense in £1 (G,'x) resp. £P (G, ,X),
(s4.3) is valid for / in £1 (G,'x) and 9 in £P (G, J1,). 0
4.34. The function

is continuous, supp(F) C EE-l, and F(e) = J1,(E) > O. Hence on some


N(e), F(x) > O. Thus N(e) C supp(F) C EE- 1 . 0
def 1
4.35. If G = lR. and, for the Cantor set Co, E = "2 (Co u (-Co)) then
J1,(E) = ,X(E) = 0, E = -E, and E + (-E) = E + E, which consists of all
real numbers for which a ternary representation is 2:::=1 ~:, On = 0, ±1, ±2.
Hence E +E = [-1,1]. 0
4.36. Because 'lI' is compact, if G is not finite, G contains a sequence
{gn} nEN such that {hn ~f gng;;~l} is an infinite sequence converging
nEN
toe(=l): foreachN(e),N(e)n(G\{e})~0. Theset{h~ : k,nEN}
is a dense subset of G. 0
4.37. The formula aX. aY = a"'+Y shows that G is a (sub)group. If x E Q
then for some n in N, nx E Z, whence G is finite. If x E 1I1R then for n in N,
nx <t z, whence G is infinite and (cf. 4.36) is dense. 0
4.38. Owing to 4.36, G = H = 'lI'. If x E G then x 2 E H, whence
G:H=2. 0
4.39. e
i. If E G \ H then H n eH = 0 and G = Hl=JeH. Furthermore, if
88- 1 n eH ~ 0, i.e., if Pi E R, bi E H, Xi = Pibi, i = 1,2, and x1xi1 E eH,
then P1pi 1 E eH c G and so, owing to the nature of R, PI = P2. Thus
x1xi1 = b1bi 1 E H, i.e., x1xi1 E eH n H = 0, a contradiction. It follows
that 88- 1 n eH = 0. If £ is a measurable subset of 8 and J1,(£) > 0 then
88- 1 :J ££-1, which contains a 'lI'-neighborhood of e (cf. 4.34) and thus
Solutions 4.40-4.41 243

an element of the dense set (,H, a contradiction. It follows that the inner
measure of S is zero: T*(S) = O.
For x in '][' and some p in R, Xp-l ~f a E G. If x rt. S then a rt. H,
whence for some bin H, x = p(,b E R(,H = (,S. Thus

and so T* (SC) = O. The inner measure T* and outer measure T* are set
functions such that for each measurable set M,

T*(M n S) + T* (M n SC) = T(M),

whence T*(M n S) = T(M), in particular, T*(S) = 1 > 0 = T*(S).


ii. The set w-1(S) ~f 8 in JR. has properties analogous to those of S.

a) The set 8is nonmeasurable, A* (8) = 0, and A* (8) = 00.

b) The set 8 is thick and for every measurable subset M of JR.,

A*(M n S) = 0 while A*(M n S) = A(M).

iii. Ifp E P then -p+E = -p+ (p+E) = (-p+p) +E = E, whence


P is a subgroup of JR.. Because p. =f. 0, P is dense in JR.. If A(E) > 0
then for some n in Z, A (E n [n, n + 1)) ~ r > 0, whence for all pin P,
A (p + (E n [n, n + 1))) = A (E n ([p + n,p + n + 1))) Since P is dense in JR.,
choose ap in pn(l, 2]. Then the sets kp+(E n [kp + n, kp + n + 10), kEN,
are pairwise disjoint, of measure r, and contained in E. 0
4.40. If E E 21R \ SA, S ~f (E x JR.) U JR., and P, Q E S, there is a polygonal
path lying in S, connecting P to Q, and consisting of no more than three
line segments.
If y =f. 0, the section Sy ~f {x : {x, y} E S} = E and Fubini's theo-
rem implies S rt. SA' 0
4.41. Because UnENhnR = G, if RES and JL(R) = 0 then JL(G) = 0,
a contradiction. If JL(R) > 0 then RR- 1 contains some N(e) (cf. 4.34).
Since H is dense in G, some h in RR- 1 n H is not e. But then there are
in R elements rl, r2 such that rl = hr2' which, owing to the nature of R,
implies rl = r2, i.e., h = e, a contradiction. 0
[ Note s4.3: The existence of R in 4.41 is a consequence of the
Axiom of Choice. Solovay [Sol] showed if
Every subset of lR. is Lebesgue measurable
is adjoined to the Zermelo-Fraenkel axiom system (ZF) [Me] for
set theory, the result is as consistent as ZF itself. Because the
244 4. Measure and Topology: Solutions

Axiom of Choice leads to a denial of Solovay's axiom, the existence


of nonmeasurable subsets of lR cannot be deduced in ZF alone.)

4.42. If HE S and J.L(H) = 0 then G, as the countable union of cosets of


H is a null set, a contradiction.
If J.L(H) > 0 and: a) G is compact then J.L(G) = 00, a contradiction;
b) G is connected then HH- 1 = H contains some N(e) whence H is open,
hence also closed, a contradiction. 0
4.43. Since an infinite proper subgroup contains a countable infinite
subgroup, the proof for 4.42 carries over, mutatis mutandis. 0
4.44. If n = 0 then since G contains a dense, proper, infinite subgroup
H, e.g., Qm, the quotient group G/H ~f K is a vector space for which
there is a Hamel basis {kAhEA' If {gAhEA is a subset of G and such that
gAl H = kA' then {gAhEA is Q-linearly independent and its Q-span is a
subgroup R that is a complete set of coset representatives of H. Hence
4.41 applies.
If m = 0 then since ,][,n = lRn /Zn, K ~ Qn /Zn is a dense countable
infinite subgroup of the compact group ']['n. Hence 4.40 applies.
If mn =I 0, there are nonmeasurable subgroups in each factor of G and
their Cartesian product is, by virtue of Fubini's theorem, a nonmeasurable
subgroup of G. 0
4.45. In 4.44, when m = 0, n = 1 and K ~f Q/Z, the corresponding
nonmeasurable subgroup R of '][',then ¢-l(R) ~ S is a proper subgroup of
G. For {km}mEN an enumeration of K and ¢ (gm) ~ km, if x E G then for
some m in N and some r in R, ¢(x) = rkm . Hence

and so G = UmENSgm' Thus if S E S, there emerge the contradictions


J.L( G) = 0 or J.L( G) = 00 if G is compact resp. S = G if G is connected.
o
[Note s4.4: The question of the existence in a locally compact
group G of a subgroup H not in the completion S of 5 is subtle.
Because 5 is aR (K(G)), the latter may fail to contain G itself. In
[HeR], the structure theorem for abelian groups is used to show
that every infinite compact abelian group contains a nonmeasur-
able subgroup.

In work yet to be submitted for publication the writer es-


tablishes the same result via the use of maximal free subsets of
groups. Furthermore, it is shown that if G is an infinite connected
Solutions 4.46-4.50 245

locally compact group or if G is the Cartesian product of a set (fi-


nite or infinite) of finite-dimensional unitary groups U (n, q then
G contains a nonmeasurable subgroup.]

4.46. a) Haar measure is regular [Halm], whence E contains compact


sets Kn such that J.L(E) = J.L (UnEN Kn), i.e., E \ (UnEN Kn)ts a null set.
b) The group JRd, i.e., JR in the discrete topology, is a locally compact
group that is not a-compact because every a-compact set in JRd is countable.
o
4.47. If N E N(e) then the group H generated by U ~f N n N- 1 , i.e.,
U nEN un, is open, hence closed, whence H = G. 0
4.48. a) If J.L(H) > 0 then HH- 1 = H contains some N(e) whence H is
open and closed, a contradiction. b) Because H is the countable union of
compact sets, HE S. Hence a) applies.
c) From a) it follows that H is open whence e/ H is open. 0
4.49. a) If E is a compact subset of K then ¢-l(E) is closed. If
{UAhEA is an open cover of ¢-l(E), it may be assumed that each UA
is compact. Then {¢ (UA)}AEA is an open cover of E and thus contains
a finite sub cover {¢(UAJ}:=l whence F ~f U:=l UAn is a compact set
and H ~f ¢ (F n ¢-l(E) is a compact set such that ¢(H) = E. Hence
T = aR (K(H)) c ¢ (5).
b) Since G is a-compact, G E 5. If U is open in G and {Kn}nEN is a
sequence of compact sets such that G = UnEN Kn then

U = nun
nEN
(G\Kn)

whence O(G) c 5. If V is open in H then ¢-l(V) is open in G and thus


O(H) c T: 5 = aA(O(G)), T = aA(O(H)).
c) If G = JRd and H = JRd/Z then neither G nor H is a-compact but
T=5.
d) If G = JR2, H = JR, J.L = A2, V = A, ¢: JR2
{x,y} I-t x E JR, and
'3
E 1!1 { {x, x} : O:S x :S I}, then J.L(E) = 0 and v (¢(E)) = 1.
e) For the situation in d), if F is a nonmeasurable subset of (0,1] and
E is ¢-l n { {x, y} : x = y} then ¢(E) ~ T. 0
4.50. If ¢: G '3 x I-t ¢(x) ~f x/H E G/H = K is the quotient map then
G and K are Haar sets, E ~K, and F ~f ¢-l(E) is a subgroup of G. From
4.49c) it follows that F is not measurable. 0
246 4. Measure and Topology: Solutions

4.51. Because v{yE) = j.L{yEx) = j.L{Ex) = v{E), v is a Haar measure.


Furthermore,

j.L{Euv) = Ll{uv)j.L{E) = Ll{v)j.L{Eu) = Ll{u)Ll{v)j.L{E)


Ll{uv) = Ll{v)Ll{u) = Ll{u)Ll{v) E (O,oo),

whence Ll : G I-t (O,oo) is a homomorphism of G into the multiplicative


group lR+ ~ (O, 00).
If f E Coo (G, C) and x E G, the change of variable xa -+ y implies

fa f[aJ{x) dj.L{x) = fa Ll{a)f{y) dj.L{Y) = Ll{a) fa f{x) dj.L(x).

Since f is uniformly continuous it follows that Ll is continuous. 0


4.52. Since j.L(Gx) = j.L(G) = Ll(x)j.L(G) it follows that Ll(x) = 1. 0
4.53. a) If u E H then, owing to the left-invariance of p,

F{xu) = 1 f{xuy) dp{y) = 1 f (xuu-1y) dp(y) = F{x).

The continuity of F follows from the continuity of f. Furthermore, because


supp(f) is compact, so is supp{F) [= supp(f)/H].
b) If v E G and v / H ~f W (E K),

L(1 f {VX Y)dP{Y)) dr{z) = L F{wz)dr{z) = L F(z)dr{z)

= L(1 f{xy) dP{Y)) dr{z),

i.e., 1 : Coo{G,C) :3 f I-t fK UH


f{xy) dp{y)) dr{z) is a translation-
invariant functional. Owing to Daniell theory (cf. Section 1.2) and the
uniqueness up to proportionality of Haar measure, there is a constant k
such that 1(f) = k fG f{x) dj.L{x). It may be assumed that k = 1, whence
symbolically, dj.L(x) = dp(y)dr (xy/ H).
c) If u E H then u/ H is the identity of K and from b) it follows that
dj.L{xu) = Ll{u)dj.L{x) = dp{yu)dr{xuyu/H) = o{u)dp{y)dr{xy/H) and
Ll{u) = o{u). 0
4.54. Because Ll is a continuous homomorphism, as the kernel of Ll, U
is a closed normal subgroup. For a unimodular normal subgroup V and its
modular function "I, 1 = "I = Lll v whence V c U. 0
4.55. Since G is compact, it contains a limit point and so every point in
G is limit point. Because G is complete, G is perfect. If G is separable then
Solutions 4.56-4.57 247

G is metric and then #( G) = c. If G is not separable, it contains infinite


separable closed subgroups. 0
4.56. It may be assumed that M = 1. If J.L2 denotes product mea-
sure on S2 and a E S then J.L2(O((Sa)2)) = fsJ.L(xSa)dJ.L(x) = 1. For
r: S2 3 {x,y} 1--+ {y,x} E S2, it follows that J.L2 (rO ((Sa)2)) = 1 and fur-
thermore, O((Sa)2)nrO((Sa)2) =/-0. Thus (Sa)2 contains pairs {xa,ya},
{ua,va} such that {xa,xaya} = {uava,ua}, i.e., xa = uava, x = uav,
xaya = uavaya = ua, a( vaya) = a. Thus e ~ vaya E Sa and for z in
Sa, ze = z. Furthermore, zez = z2 whence ez = z. Thus for any a
in S, Sa contains an e that is a left and right identity. If xES then
xa E Sa, xa = exa, x = ex whence e is an identity for S. Thus for some w
in S, wa = e, i.e., S is a group [GeK]. 0
[ Note s4.5: The existence of J.L as described together with the
fact that S is a group implies that S may be endowed with its
Weil topology [We2].]

4.57. Let F be {A : A c S, A is closed, AS c S}. Because S E F, it


follows that F =/- 0. When F is partially ordered by reversed inclusion:
Ai ~ A2 iff Ai J A 2., a maximal chain C ~ {A-Y}-YEr enjoys the finite
intersection property. Thus n-yEr A-y ~f A =/- 0: A is a minimal element of
F and AS c A. If a E S then as c A, (as)S c AS. Since A is minimal,
as = A. If XES and a E A then ax E A, axS = A = as, and the
cancellation law implies xS = S. Similarly, Sx = S and so, for all a, b in
S, the equations ax = band xa = b have solutions (unique by virtue of the
cancellation law). Thus S is a group.
If x is near y then xy-i is near yy-i = e, whence x- i (xy-i) = y-i is
near x-ie = X-i. Hence x 1--+ x- i is continuous [GKO]. 0
[ Note 84.6: The hypotheses about compactness and the can-
cellation law cannot be dropped. For example, [0,1] is a compact
semigroup with respect to multiplication, fails to conform to the
cancellation law, and is not a group; (0,1] with respect to multi-
plication is a topological semigroup, conforms to the cancellation
law, but is not compact [even though its completion [0,1] is com-
pact].]
5
Measure Theory

5.1. Measure and Integration

5.1. Let J-L be a finite measure. If J-L is positive and

M ~f sup {J-L(E) : E measurable}

then for some sequence {En} nEN of measurable sets, J-L (En) i M. It may
be assumed that {En}nEN is a monotonely increasing sequence, whence
limn -+ co En ~f E is measurable and M = lim n -+ co J-L (En) = J-L(E) < 00.
If J-L is complex then IJ-LI is finite, IJ-L(E) I :s: IJ-LI(E), and the previous
argument applies. 0
5.2. a) Since E::'=1 lanl < 00, if E E 2N then E is empty, a finite set,
or a count ably infinite set and for any measurable partition {En}nEN of
E, E::'=1 J-L (En) converges. Furthermore, the triangle inequality for the
addition of complex numbers implies IJ-LI(E) = LnEE lanl·
b) The finite additivity of J-L on the ring of finite subsets of N is a
consequence of definition. If E:%1 an is conditionally convergent, then for
some permutation 7r : N ,....... N, E::'=1 a7r(n) fails to converge whence J-L (N) is
not defined. 0
5.3. a) If M ~f {E : E E aR (R) ,J-L1 (E) = J-L2(E) } then M is a monotone
class containing R whence M ::J aR (R).
b) If R ~f R ({ A : A = [a, b) n Q}) then for all A in R, #(A) = ~o
unless A =.0 If J-L1 def
= (; ( .
countmg = 2J-L1 t hen J-L1 = J-L2
measure ) and J-L2 def
on R. However, if r E Q then {r} = nnEN [r, r + ~) whence {r} E aR (R)

and J-L1 ({r}) = 1 < J-L2 ({r}) = 2. 0

5.4. 1 .1 ( ) def ' "


= - + z-,n E N, E c N, and J-L E = wkEEak then
If an def
2n 3n
(N,2 N,J-L) is a complex measure situation. Furthermore, N = UnEN{n},
while IJ-L(N)I < E::'=11J-L({n})I· 0
5.5. Decompose J-L into its real an imaginary parts and those into their
positive and negative parts: J-L = J-L1 + iJ-L2 = J-Lt - J-Ll + i (J-Lt - J-L2)' If
A E 5 and AcE then E = AU (E \ A) and IJ-LI(E) 2':: IJ-L(A) I· Hence
veE) :s: IJ-LI(E).

248
B. R. Gelbaum, Problems in Real and Complex Analysis
© Springer-Verlag New York, Inc. 1992
Solutions 5.6-5.9 249

On the other hand, if {En}nEN is a measurable partition of E then

L L
00 00

11-£ (En)1 ~ (I-£t (En) + 1-£1 (En) + I-£t (En) + 1-£2 (En))
n=l n=l

II-£I(E) ~ I-£t (E) + 1-£1 (E) + I-£t (E) + 1-£2 (E).


If Pi± are Hahn decompositions for I-£t, i = 1,2, then

II-£I(E) ~ 1-£1 (E n pt) + 1-£1 (E n P1-) + 1-£2 (E n P2+) + 1-£2 (E n Pi)


~~~. 0

5.6. Since 1-£ (U;::n Ek) ~ L~k 1-£ (Ek) 10, 1-£ (limn->ooEn) = O. 0
5.7. The map II : S 3 E f---+ 1-£ (J-1 (E)) is a finite measure and II «1-£. For
some integrable h, II(E) = IE h(x) dl-£(x) and if 9 is a bounded measurable
function, Ix g(x) dll(x) = Ix g(x)h(x) dl-£(x).
Since Xf-1(E)(X) = XE 0 f(x), if E E S then

Ix XE(x)h(x) dl-£(x) = Ix XE(X) dll = II(E) = 1-£ (J-1(E))


= Ix Xf- 1(E)(X) dl-£(x) = Ix XE 0 f(x) dl-£(x).

The approximation properties of simple functions and the dominated con-


vergence theorem apply. 0
5.8. First proof. Fatou's lemma implies Ix f(x) dl-£(x) = 0, whence
f(x) = 0 a.e.
Second proof. If F E S then limn->oo IF fn(x) dl-£(x) = O. Hence if
f(x) > 0 on a set of positive measure, there is a positive 8 and a set E of
finite positive measure on which f(x) ~ 8. Egorov's theorem implies that
E contains a subset F such that I-£(F) > ~ and fn ~ f on F. But then
limn->oo IF fn(x) dl-£(x) =I- 0, a contradiction.
Third proof. It may be assumed that {fn} nEN C L1 (X, q. The
hypothesis implies limn->oo IIfn II 1 = O. 0
5.9. a) The hypotheses imply sUPnEN IIfnl11 ~f M < 00. Hence if E > 0
and E€ ~ {x : x E E, Ifn(x) - fo(x)1 ~ E} then

[ Ifn(x) - fo(x)1 dx ~ [ +[ Ifn(x) - fo(x)1 dx;


lE lEE lE\EE
for large n, the first term in the right member above does not exceed 2EM
and the second term does not exceed E>..(E).
250 5. Measure Theory: Solutions

b) Since 9n ~f InAI~' I AI = I, the dominated convergence theorem


implies that if E E 5 then

If € > 0 and limn -+ oo IE


In (x) dJ1.{x) = IE I{x) dJ1.{x) + € then, via passage
to subsequences as needed,

Hence

lim [ In (x) dJ1.{x) = lim [ In{x) dJ1.{x) + lim [ In{x) dJ1.{x)


n-+oo Jx n-+oo JE n-+oo JX\E
~ [ l{x)dJ1.{x)+€+ [ I{x) dJ1.{x) ,
JE JX\E
a contradiction. o
5.10. If 1> € > 0, En ~f {x : Iln{x)1 ~ €}, and

A ~ [ Iln{x)1 d ( )
n Jx 1 + Iln{x)1 J1. x

then A n <f - En 1 + lin (x) I dJ1. (X ) + €J1. (X) <


Iln{x)1 - J1. (E) (X) If In m~s
n + €J1.. ~
0

then limn-+ DO An ~ €J1.{X).


Conversely, if An -40 as n -4 00 and J1. (En) +0 as n -4 00, there are
positive 8 and € and a sequence {nkhEoo such that nk -4 00 as k -4 00 and
J1. (Enk ) ~ 8. Furthermore,

and so

a contradiction. o
Solutions 5.11-5.14 251

5.11. If an -10 as n --+ 00, the convergence a.e. of r::'=1 an<pn{x)


implies limn ...... oo <Pn{X) ~ O. The dominated convergence theorem implies
limn ...... oo II<PnIl2 = 0, contrary to the orthonormality of {<Pn}nEN' 0
5.12. a) If E = 0, the conclusion is automatic. Since each <Pn is in
L2(X, JL), it follows that C ~f U:'=1 {x : <Pn(x) =I- O} is IT-finite and <P = 0
on X \ C. In the argument that follows it may be assumed that E c C and
that JL(E) > O.
Egorov's theorem implies that if S '" AcE and 0 < E < JL(A) < 00
then A contains some A€ such that JL (A \ A€) < E and <Pn ~ <P on A€. Since
{<Pn} nEN is orthonormal, the Schwarz inequality implies

[ 1<p(x)1 dJL ~ [ I<p(x) - <Pn(X) I dJL + [ l<Pn(x) I dJL


lA, lA, lA,
~ 1I(<p - <Pn) XA.lloo JL (A€) + viJL (A€),
whence <P is integrable. From Bessel's inequality it follows that for

r::'=IICnI2 ~ IIXA.sgn(<p(x))ll: ~ JL(A) < 00, whence limn...... oo en = O.


Since sgn(<p)<pn ~ 1<p1 on A€ it follows that fA, 1<p(x)1 dJL = 0 and thus <P ~ 0
on A€. Since JL (A \ A€) can be made arbitrarily small, it follows that <P ~ 0
on A. Moreover, E is IT-finite and so there are measurable sets An of finite
measure and such that E = U nEN An, whence <P ~ 0 on E.
b) If limnENvar( <Pn) < 00 then, since { <Pn} nEN is orthonormal, it may be
assumed that sUPnEN l<pn(a)1 < 00. The Helly selection theorem [Wi] im-
plies the existence of a subsequence, for convenience denoted again {<Pn}nEN,
such that lim n ...... oo <Pn(x) ~f <p(x) exists everywhere. Hence <p(x) ~ 0 and
from a) there emerges the contradiction

0= [
l[a,bj
1<p(xW dx = lim [
n ...... oo l[a,bj
l<Pn(x)12 dx = 1. o

5.13. The functions In: [0,1] '" x I--t sgn(sin2rr2nx),n E N, are in


Loo ([0,1],,x) and IIlnlloo == 1. Moreover, fol In(x) dJL == 0 while for n in N,
Iln(x)1 ~ 1. 0
5.14. For (JR,S'\,,x), if In = X[O,nj + (2 - ~J X(n,2nj,n E N, the require-
ments are met. 0
[ Note 85.1: For contrast consider (X, S, JL) ~ (JR, SA' ,x), and
f n def
= X[-n,nj· ]
252 5. Measure Theory: Solutions

5.15. Note that En :::) E n+1, n E N, and

11111t 2: f II(x)1 dJt(x) 2: bn+1Jt (En+1) 2: bnJt (En+1)


JE + n 1

111111 = ~ fen \En+l II(x)1 dx


N
2: L bn (Jt (En) - Jt (En+l ))
n=l
N
= blJt (El ) + L (bn - bn- l ) Jt (En) - bNJt (EN+1)
n=2
whence blJt (El ) + L~=2 (bn - bn- l ) Jt (En) :::; 2111111 < 00. It follows that
if kEN, there is an nk such that
1
k> L00

(bn - bn- l ) Jt (En) 2: (bnk +m - bnk ) Jt (Enk +m).


n=nk+l
1 .
Therefore k + bnkJt (Enk +m ) > bnk+mJt (Enk +m). Smce Jt (Endm) 1 0 as
-- -- 1
m - t 00, limm--->oobnk+mJt (Enk+m ) = limn--->oobnJt (En) :::; k' 0
5.16.

Ix II(x)IP dJt(x)

2: r
J{ '" : If("')I~€}
II(x)IPdJt(x) 2: EPJt({X : II(x)I2:E}). 0

5.17. Since Jt2a « Jt3 and Jtls 1.. Jt3, if Jt2a(E) > 0 then Jt3(E) > 0,
whence Jtls(E) = o. 0
5.18. a) If Jti = Jtia + Jtis is the Lebesgue decomposition of Jti with
respect to Jtj, i -=I- j, and Jtia resp. Jtis lives on Aia resp. A is , i = 1,2, then
(cf. 5.17) Aia n Ajs = 0,i,j = 1,2, Ais n Ajs = 0,i =I- j. Furthermore,
if E = Ala n A 2a , then Jti = JtiE + Jti(X\E) , JtiE 1.. Jti(X\E) , i = 1,2, and
Jti(X\E) 1.. Jtj(X\E) , i =I- j. If ddJtia ~f lij, i -=I- j, denote the Lebesgue-Radon-
Jtja
Nikodym (LRN) derivatives then, modulo a null set,

E={x : h2(X)hl(X)-=l-0}.

If JtlE(A) = 0 then 0 = Jtl (A n E) = fAnE h2(X) dJt2(X). Since


h2(X) > 0 a.e. (Jt2) on E, Jt2(AnE) = Jt2E(A) = O. An appeal to symmetry
completes the argument.
Solution 5.19 253

b) The following relations obtain either by hypothesis or by direct


deductions therefrom:

J.Li = J.LiF + J.Li(X\F) , i = 1,2, J.LiF -«: J.LjF, J.LiF -«: J.Lj, i #- j
J.Li(X\F) 1- J.Lj(X\F), J.Li(X\F) 1- J.LjF, J.Li(X\F) 1- J.Lj, i #- j.
Thus J.LiF and J.Li(X\F) are the unique LRN components of J.Li, i = 1,2,
whence J.LiE = J.LiF, J.Li(X\E) = J.Li(X\F) , i = 1,2. Hence

J.LI(E \ F) = J.LIE(E \ F) + J.LI(X\E)(E \ F)


= J.LIF(E\F) + J.LI(X\E)(E \ F) = 0 + O.
Symmetry implies J.LI(F\E) = 0 whence J.LI (E6F) = J.L2(E6F) = O. D
5.19. If J.LI 1- J.L2 then J.LI and J.L2 live on disjoint (measurable) sets Al and
A 2. If al,a2 E C and {En}nEN is a measurable partition of the measurable
set E then

L + a2J.L2 (En)1 = L
00 00

lalJ.LI (En) lalJ.LI (En n AI) + a2J.L2 (En n A2)1


n=1 n=l
:s (Iall IJ.LII + la21 1J.L21) (E),

whence lalJ.LI + a2J.L21 (E) :s (jalllJ.LII + la211J.L21) (E).


However, if {Eni} nEN is a partition of E n Ai, i = 1,2, then

is a measurable partition of E. Hence

+ a2J.L21 (E) 2: L
00

lalJ.LI lalJ.LI (Enl ) + a2J.L2 (Enl)1


n=l

L lalJ.LI (En2 ) + a2J.L2 (En2 )1


00

+
n=l

Since J.Li lives on Ai, i = 1,2, the last term in the right member above is
zero and the first two terms may be replaced by :E~=llall IJ.LI (Enl)1 resp.
:E~=lla211J.L2 (En2 )1·
Conversely, the argument in Solution 5.18a) may be repeated, mu-
tatis mutandis, to produce J.Lia, J.Lis such that

J.Li = J.Lia + J.Lis, J.Lia 1- J.Lis, i = 1, 2,


J.Lia -«: lJ.Lj I , J.Lis 1- lJ.Lj I, i #- j.
254 5. Measure Theory: Solutions

If J.1.ia lives on Ai, i = 1,2, let E be A 1 nA2. As in 5.18a), J.1.i = J.1.iE+J.1.i(X\E) '
J.1.iE <t:: J.1.jE, J.1.i(X\E) 1- J.1.j(X\E) , i =I- j, and

In view of the hypothesis,

(J.1.1E ± /-t2E)(E) = L h2(X) d 1J.1.21 (x) ± L !21(X) d 1J.1.11 (x)

= L (f12(X)!2I(X) ± !21(X)) d 1J.1.11 (x),

1J.1.1E ± J.1.2EI (E) = L Ih2(X)!21 (x) ± !21(x)1 dlJ.1.d (x)


= (1J.1.1EI + 1J.1.2EI) (E)
= L (lh2(X)llh2(X)1 + Ih2(X)1) dlJ.1.11 (x).
Since A above is an arbitrary measurable set, on E,

Since J.1.iE <t:: J.1.jE,i =I- j, if A is measurable then both l/-til (A) and
1J.1.21 (A) are zero or neither is zero. If 1J.1.11 (E) = 0 then J.1.1 1- J.1.2. On the
other hand, if 1J.1.11 (E) =I- 0 and

B~f{x : !21(x)=O}nE
C ~ {x Ih2(X)1 + 1 = Ih2(X) + 11} n E
D~f{x: Ih2(x)l+l=lh2(x)-II}nE

then

1J.1.11 (E \ (B U C)) = 1J.1.11 (E \ (B U D)) = 0


l/-til (B) = 1/-t21 (B) = 0
1J.1.11 (E6.C) = 1J.1.11 (E6.D) = O.

However, if a E C and lal + 1 = la + 11 then a ~ 0 and if lal + 1 = la - 11


then a ::; O. Hence h2(X) = 0 a.e. (1J.1.11) on E. In sum, J.1.I 1- /-t2, while
J.1.iE = 0 and /-ti = J.1.i(X\E) , i = 1,2. 0
5.20. a) The basic idea is to show that V'YEG !'Y' which a priori is not
measurable, can be replaced, via a properly constructed sequence {9n}nEN'
by VnEN9n'
Solution 5.20 (cont.) 255

If

E(-y, k, n) ~f {X : 2kn S !-r(X) } , (-y, k, n) E r X Z X N and

6(k, n) ~f sup J.£(E(-y, k, n)), !-rv-r' ~f !-r V!-r'


-rEr

then E (-y V 'Y', k, n) = E (-y, k, n) U E (-y/, k, n). Hence if

lim J.£(E(-Yknp,k,n))
p-+oo
= 6(k,n)

then UPEN E (-Yknp, k, n) = limp -+ oo E (-Yknl V ... V 'Yknp, k, n) ~f E(k, n) is


a measurable set and /l(E(k, n)) = 6(k, n).
If 'Y E r and J.£ (E(-y, k, n) \ E(k, n)) ~ to > 0, choose p so that

Thus

E('Y,k,n)
= (E(-y, k, n) \ E (-Yknl V··· V 'Yknp, k, n)) U E ('Yknl V ... V 'Yknp, k, n),
6(k,n) 2: J.£(E(-y,k,n))
2: J.£ (E(-y, k, n) \ (E ('Yknl V '" V 'Yknp, k, n)))
+ J.£ ((E (-Yknl V ... V 'Yknp, k, n)))
to to
2: to + 6(k,n) - 2 = 6(k,n) + 2'
and thus to = 0: J.£ (E(-y, k, n) \ E(k, n)) = O.
If (-y,n) E r X N then UkEZE(-y,k,n) = X whence UkEZE(k,n) = X.
For n fixed and k in Z, if A(k, n) ~f E(k, n) \ U~k+1 E(l, n) then

{k =f k/} '* {A(k,n) n A (k',n) = 0},

whence gn : A(k, n) 3 x 1--+ k; 1 is well-defined. Since UkEZ A(k, n) = X,


gn is defined on X. Furthermore, if 'Y E r,

{X : !-r(x) 2: gn(x)} = U({ x : !-r(x) 2: gn(x)} n A(k, n))


kEZ

C U (E(-Y, k + 1, n) \
kEZ
UE(l, n)) .
l=k+l
256 5. Measure Theory: Solutions

Since f.L (E('Y, k, n) \ E(k, n» == 0, it follows that

f.L(u (Eb,k+1,n)\ l=k+l


kEZ
UE(l,n»)) =0
and so f.L {x : 1"I(x) ~ gn(x) } = o.
The formula E ('Yknp, k, n) = E ('Yknp, 2k, n + 1) implies
== E (,knp, k, n) n E(2k, n + 1)
E (,knp, k, n)
E b2k,n+l,q, k, n) == E b2k,n+l,q, k, n) n E(k, n)

E(k, n) = UE b2k,n+1,q, k, n)
pEN

== U U E ('Yknp, 2k, n + 1) n E b2k,n+1,q, 2k, n + 1)


qENpEN

== U E b2k,n+l,q, 2k, n + 1) n E(k, n)


qEN

== E(2k, n + 1) n E(k, n).


Similarly, it follows that E(2k, n + 1) == E(k, n) n E(2k, n + 1), whence
E(2k, n + 1) == E(k, n). Consequently

U E(l, n) == E(2k, n + 1) \ U E(2l, n + 1)


00 00

A(k, n) = E(k, n) \
l=k+l
00

=E(2k,n+1)\ U E(l,n+1)
1=2k+l

u (( U
1=2k+l
E(l,n+1») \ ( U
l=k+l
E(2l,n+1»)).

Since

U E(2l,n + 1)
00

= E(2k,n + 1) \
l=k+l
= A(2k + 1, n + 1),
it follows that A(k, n) == A(2k, n + 1) U A(2k + 1, n + 1). Consequently,
gn(x) ~ gn+1(x) a.e., the measurable function limn-+ oo gn(x) ~ g(x) exists
a.e., and for all" 1"1 :s g(x) a.e. Where g(x) is not yet defined, let g(x) be
zero.
Solution 5.21 257

b) If his S-measurable and V'YErf'Y ~ h then

H~f U {x : h(x) ~9(X)- 2~}


mEN

C U {x : h(x) ~ 9m+1(X) - 2~ }.
mEN

The argument below shows that each summand in the right member above is
a null set, whence H is a null set (/1,). Since, for each n in N, UkEZ A(k, n) =
X, it suffices to show that for each k in Z,

JL ({ x : h(x) ~ 9m+l(X) - 2~ } n A(k,m + 1)) d~f JL(A) = O.


In A and off a null set in A,

1 k+1 2 k-1
- 9 m+1 (x) - -2m -- -
h(x) < -- -
2m+1 - -- -
2m+1 -
2m+1'

Furthermore,

A={x: h(X)~9m+1(X)}n(E(k,m+1)\ U
l=k+l
E(I,m+1))

= U ({ x : h(x) ~ 9m+l(X) - 2~ })
p,qEN

n {(E (,k,m+1,p, k, m + 1) \ E (r1,m+1,q, I, m + 1))},


whence it suffices to show each of the last summands above is a null set.
But on a typical summand,

a contradiction unless the summand is a null set. D


5.21. There is a sequence {{ a mn }mEN} nEN such that for each m,

0= amI < am2 < "', Emn ~f {x : amn ~ f(x)},

L L
00 00

amnJL (Emn \ Em,n+1) = (am,n+1 - amn ) JL (Emn) , (s5.1)


n=1 n=1
and such that as m - t 00, the left member resp. right member in (s5.1)
approaches fxf(x)dJL(x) resp. (JL x A)({ {x,y} : 0 ~ y ~ f(x)}). D
258 5. Measure Theory: Solutions

5.22. Note that Ix I(Y) d/-L(Y) is a constant. D


5.23. An application of 5.22 leads to the equality. D
5.24. Consider the measure situation ({O, I} ~f X, 5 ~f 2{O,I}, /-L) such
that /-L ({O}) = x, /-L ({I}) = 1 - x. If 1 ~f X{O} then

E(f) = x, E (J2) = x, and Var(f) = x - X2.

The range of Fn is {O,~, ... ,~} and

r
/-Ln ({ X ~f (Xl, ... ,Xn) : Fn(x) = ~ }) = (~)Xk(l- xt- k
Var(Fn) = ~ (~- X (~)Xk(l- x)n-k = X ~ X2 ~ 4~. D

[ Note s5.2: The last inequality occurs in S. Bernstein's proof


of the WeierstraB approximation theorem, viz.:

For 1 in C ([0,1], JR) and E positive, there is a positive 8 such


that
{Ix - YI < 8} =? {11(x) - I(Y)I < E}.

If n > sup {8- 4, 11~~oo} then since

it follows that

II(x) - Bn(x)1 ~f I'(X) - i;' (~) (~)Xk(l- x)n-kl


= Ii; (I(X) - 1 (~)) (~)xk(l- x)n-kl
~ IZ=I*-xl<n-a-I + 1Z=1*_xl~n-! I·
Solutions 5.25-5.28 259

Owing to the domain of k in the second summand of the right

membe, above, I~ -~12


n-'
21, whence

I/(x) - Bn(x)1 ~ E + 211/1100 E(~)xk(l- xt- k

<;, + 211/11= ~ (~n~/ (~)xk(1- x)n-k

~ E + 211/1100n! E(~- x) 2 (~)xk(l- x)n-k


~ E+ II/II~.
2n'
Thus 1 is uniformly approximable by the Bernstein polynomials
Bn.]

. (N,2N
5.25. ConsIder ,( ) . N:3 n
,I. 1 and En ~f{
- n } ,n E N.
f-t - , D
n
5.26. Otherwise there is a sequence {t n } nEN converging to zero and for
which limn_oo Ix Ix
1 (x, t n ) dJ.L(x) ::j:. limn_oo 1 (x, t n) dJ.L(x), in contra-
diction of the dominated convergence theorem. D
5.27. If

=
S(n, k) def n {x : I/(x, r) - I(x, 8)1 < kI}
r,8E(O,~)

S(n, k) ~f n {
p,qEQn(o,~)
x : I/(x,p) - I(x, q)1 < ~}

then S(n, k) c S(n, k). On the other hand, if x E S(n, k) then, owing to
continuity, there are r,8 such that I/(x, r) - I(x, 8)1 < ~. It follows that
S(n, k) = S(n, k) and so S(n, k) is measurable.
For each k, J.L(S(n, k)) -+ J.L(X) as n -+ 00. For a suitable choice of a
n
sequence {ndkEN' the measure of E ~f kEN S (nk' k) exceeds J.L(X) - E
and the convergence of I(x, t) on E is uniform. D
5.28. First example. If fn(x) = xn then limn_oo In (x) = X{l}' If
A(E) = 1 then for any E in (0,1),

A({X: xEEn(l-E,l]})=E, sup{/n(x) XEE}=l.


260 5. Measure Theory: Solutions

Second example. For n in N, if In = ..!:.X(.! 1] + nx(o.!] then


n n' 'n
limn-HX.l In = 0, whereas if :A(E) = 1, SUPxEE In (x) = n. D
5.29. Let 8 be the nonmeasurable set in 4.39. Let {rn}nEN be an
enumeration of Q n [0,1) and define 8 n to be (8 + rn)/'lL.. Then there is a
positive 8 such that 1> 8 = :A*(8) [=:A* (8n ), n EN].
For t in I n ~ [2- n -\ 2- n ), let It be defined by the equation

ft(x) = {Io if x E ~n and x


otherwIse
= 2n +1t - 1.
Since [0,1) = l:) nEN 8 n and (0,1) = l:)nENJn , if t E (0,1) then

#({x: It(X)~O})={l if2n+l~-lE8n.


o otherwIse
It follows that each It is a bounded measurable function different from zero
for at most one x in [0,1) and that if x E (0,1) then limt-to ft(x) = o. In
short, It ~. 0 as t ----+ O.
If :A*(D) < 8 then for each n in N, 8 n \ D ~ 0. Choose Xn in 8 n \ D.
As t traverses I n , 2n +1t -1 traverses [0,1) and there is in I n a tn such that
2n +1 t n - 1 = x n , whence ftn (x n ) = 1. As n ----+ 00, tn ----+ 0 and thus off D,
It 740:
Although limt-to It (x) = 0 for each x in (0,1), there is in (0,1) no
set D such that :A*(D) < 8 and as t ----+ 0, It(x) ~ 0 off D.
D
. K
5.30. a) If E > 0 then for some K, If k > 2'

Hence if 8 ~f 3~' p,(E) < 8, then

E E
and IE I/(x)1 dp,(x) = IE! + IE2 < K· 3K + 3 < Eo
b) Assume that if IE F and E > 0, there is a positive 8(E) such that

If E > 0, {fn}nEN C F, and IE?(lfnl,n) I/n(x)1 dp,(x) ~ E then

p, (E? (1/nl, n)) ~ 8(E).


Solutions 5.31-5.33 261

Since (X, 5, Jl) is nonatomic, E?:. (III, n) contains a subset IE such that
Jl (IE) = O~f) [< O(f)] (cf. 4.24). It follows that for all n in N,

f > h I/n(x)1 dJl(x) ~ nO;f) ,

a contradiction.
c) For (N,2N ,(), L1 (N,() is uniformly integrable since

{((E) < I} '* {((E) = O} '* {L I/(x)1 d((x) = o}.


On the other hand, if f > 0, then for any K, if k > max{K,f}, it follows
that I ~f kX{1} E £1 (N, () and JE~(lfl,k) = k > Eo D
5.31. a) Since Discont(1/1) c Discont(f) , III is integrable on every
bounded interval.
b) The improper Riemann integrals frr~J±(x) dx exist and so the im-
proper Riemann integral frr~ I(x) dx exists. The monotone convergence the-
orem implies frr~ I±(x) d'\(x) exist.
c) If S is a nonmeasurable subset of [0, 1] then 9 ~f Xs - X[O,lj\S meets
the requirements. D
5.32. Since I is uniformly continuous, if f > 0, there is in N an n such
that {Ix - yl <~} '* {1/(x) - I(Y)I < fl· If

mk ~f min I(x), Mk ~f max I(x)


~:-:;x:-:;~ ~:-:;x:-:; k!l
then Mk -mk < f and U~,:~ [mk' Mk] x [~, k : 1] meets the requirements.
D
5.33. The formula for the sum of a finite geometric series and Euler's
formula yield the following closed expressions:
1 N 1 1 _ i(2N+1)O
D N ((}) = _ ~ einO = _ e - iNO e .
2n ~ 2n 1 - e'o
n=-N

1 ei(N+!)O _ e-i(N+!)O
=
2n ei!O _ e-i!O

1 N 1
F N ((}) = 2n(N + 1) ~ Dn((}) = 2n(N + 1)
262 5. Measure Theory: Solutions

Since [0,211") :3 0 1-+ ei () ~f x E 'Jl' carries [0,211") bijectively onto 'Jl', the
results above may be converted to express DN and FN as functions on 'Jl'.
o
5.34. For I in A, the series E:'=-oo nineinx converges uniformly on
[0,211"] and defines a function 9 continuous on [0,211"]. If G(x) ~ f: g(t) dt
on [0,211"] then G(x) = -i E:'=-oo ineinx and 2.79 implies
00
n=-oo

Thus I ([0,211"], q, 1(0) = 1(1), and 11/1100 + 111'1100 :::; 1. Hence if


E C1
Ix - yl < f then I/(x) - l(y)1 < f. It follows that A is an equicontinuous
set offunctions contained in the unit ball of C ([0,1], q. The Arzela-Ascoli
theorem implies that II is compact. 0
5.35. For a trigonometric polynomial p : 'Jl' :3 t 1-+ E;:=-N cntn, let Lo(p)
be E;:=-N anCn· Then Lo is a linear functional defined and bounded on
a dense subset of C ('Jl', C) and so Lo has an extension to a bounded linear
functional L in (C ('Jl', q) *. The Riesz representation theorem yields the
measure JL conforming to the requirements. 0
5.36. a) Let ('Jl', 5,8, JL) be such that JL( {I}) = 1 and JL ('Jl' \ {I}) = o. Then
(5.3) is the Fourier-Stieltjes series for JL.
b) The inequality 11/*glll:::; fT(fTI/(s-lt)llg(s)ldr(s)) dr(t), Fu-
bini's theorem, and the translation-invariance of Haar measure r imply
III *gill:::; 11/111IIg111. Fubini's theorem then implies (5.4).
c)

aN(t) = N ~ 1 ~ Ctn Akik ) t n


N
= - N L (Ln * Ift n = AN * I(t).
1

+ 1 n=O
d) In consonance with the formula for the averages of the partial sums
of a Fourier series, let aN(t) represent the average of the first N + 1 partial
sums of a Fourier-Stieltjes series. Then, FN denoting Fejer's kernel,

aN(s) = i FN (S-lt) dJL(t)

IIaNl11 :::; i iIdJL(t) I FN (S-lt) dr(s) = IJLI ('Jl').

Direct calculation shows aN(t) = E;:=_NCn (1- J~l) tn. Thus if, for
some JL, E;:=-N Cntn is a Fourier-Stieltjes series then (5.5) obtains with
IJLI ('Jl') for M.
Solutions 5.37-5.39 263

Conversely, if (5.5) obtains for some M, then via the identification


1l' ~ [0,1), {Gn(t) ~ J~ O"n(s) dS} is a sequence of functions of uni-
nEN
formly bounded variation. Since Gn(O) == 0 the Helly selection theorem is
applicable. Thus there is a subsequence {G n ;} jEN converging uniformly on
1l' to a function G. If nj > Ikl, integration by parts yields

( 1 _ _lk+
~
_I 1_)C k
=110"n,.(t)e-27riktdt=Gn,.(1)+ 2ik
0 IT
11 0
Gn,.e-27riktdt.

The preceding calculation shows that as j - t 00, Ck = J;


e-27rikt dG(t), i.e.,
that the Stieltjes measure generated by G may be used for /-L 0
5.37. The construction of the Cantor set Co. for a positive a may be
carried out on any interval [a, b] in place of [0, 1]. If [a, b] ~f [O,~] and if
¢o. is the corresponding (monotonely increasing) Cantor function, let /-L be
the Stieltjes measure generated by ¢o.' 0
5.38. Let {xn} nEN be a dense subset of F. For 1 in Co (JR, C), let
Ln(f) be 1 (xn). Then L ~f 2:::'=1 ~: E (Co (JR., C))* and
1. The IILII :s;
Riesz representation theorem provides a Borel measure /-L such that for 1 in
Co (JR,C), L(f) = In~/(x)d/-L(x).
If U is an open subset of JR \ F and if K is a compact subset of U,
there is in Co (JR, C) an 1 such that K -< 1 -< U whence L(f) = 0 and thus
/-L(K) = 0: supp(/-L) C F.
If V is open and V n F =1= 0, there is in V n F an Xno and in Co (JR, C)
a 9 such that {xno} -< 9 -< V. Hence /-L(V) 2: L(g) 2: 9 ~::o) > 0 and so
supp(/-L) = F. 0
5.39. As a compact set in JRn , K is 1 (Co) for some 1 in C(Co,JRn ) (cf.
2.15). The Cantor function ¢o determines the measure

II: 513 ([0,1]) 3 E ~ ), (¢o(E».

For any set 8 in 5>.([0,1]), m(t) ~f ), (8 n [0, t]) varies continuously


with t. Hence if ),(8) > 0 then m(l) = ),(8) and m(O) = O. If a E (0, ),(8»
then for some to in [0,1], m (to) = a = ), (8 n [0, to]). It follows that
II is nonatomic and thus that /-L : 5j3 (JR n ) 3 E ~ II (J-1(E) n Co) is a
nonatomic measure on 513 (JRn ).
If UnK = 0 then 1-1(U) = 0, whence /-L(U) = O. The linear extension
F of 1 from Co to [0,1] is continuous. If V is open and V n K =1= 0 then
F- 1 (V) ~ W is open and W n Co =1= 0, whence II (W nCo) > 0 and so
supp(/-L) = K. 0
264 5. Measure Theory: Solutions

5.40. Let {xn}nEN be a countable dense subset of [0,1]. For each Borel
subset A of [0,1], if JL(A) ~f LXnEA 3: then JL (5/3) = Co· 0
5.41. Jensen's inequality applied to exp implies

exp ( [ f(x) dX) :::; [ exp (f(x)) dx.


J[O,lJ J[O,lJ

Rolle's theorem implies that if a :::; x then ea(x - a) + ea :::; eX and, since
exp is strictly monotonely increasing, equality obtains iff x = a. Hence if
a ~f I[o,lJ f(x) dx and f(x) =f. a on a set of positive measure (JL) then, on a
set of positive measure (JL), ea(f(x) -a)+ea < ef(x) and integration yields
exp (Iro,lJ f(x) dX) < IrO,lJ exp (f(x)) dx. 0
5.42. If F E F ([0, 1]) and 0 tJ- F then for some N(F) and some h in
C([O, 1], JR.), 0 tJ- N(F) and F -< h -< N(F). Hence 0 = h(O) ~ IrO,lJ hex) dx
and so supp(JL) = {O}. Thus if 0 :::; JL ({O}) ~f C :::; 1 then for f in
C ([0,1]' C), IrO,lJ f(x) dJL(x) = c· f(O). 0
5.43. For

A ~ {f : f E C ([0,1]' JR.) ,J(x) = f(1 - x), f(O) = f(l) = O},

the Stone-WeierstraB theorem implies that the span of {x f-+ sink 7rX } kEN

is II lloo-dense in A. Hence if f E A, I; f(x)dJL(x) = o. If E E 5/3 ([o,~])


then En (1 - E) = 0, and JL(E U (1 - E)) = 101 XeU(l-E) (x) dJL(x). The
integrand in the right member preceding is the II II 1-limit of a sequence of
functions in A: JL(E U (1 - E)) = O. 0
5.44. The linear span of {x f-+ cos k 7rX }kEN is II lloo-dense in

The argument in Solution 5.43 applies. 0


5.45. For the II II oo-closure B of span ( { x f-+ cos k 7rX }:=1)' the map
L : B 3 L~=l Ck cos k 7rt f-+ L~=l Ckak is extendible to an element L1 of B*
and, by virtue of the Hahn-Banach theorem, L1 may be extended without
increase of norm to a linear functional L2 defined on C ([0, 1], JR.). The Riesz
representation theorem implies the existence of a finite (possibly signed)
Borel measure JL such that for f in C ([0, 1], JR), L 2 (f) = 1[0,1] f(x) dJL(x).
o
Solutions 5.46-5.48 265

5.46. For J in F, the map f* : [0,1] '3 x t-+ J; J(t) dt is in C ([0,1], C).
Thus 5.30b) implies that for some K(l),

1f*(x)l:::; [ IJ(t)1 dt = [ + [ J(t) dt


J[O,I] J[O,I]\E? (f,2K(I» JE? (f,2K(I»
:::; 2K(1) + 1,

i.e., {f*} IEF is uniformly bounded.


If x:::; y, € > 0, and Iy - xIK(€) < € then

If*(x) - f*(y)l:::; [ +[ J(t) dt


J[x,Y]\E? (f,2K(€» J[x,y]nE? (f,2K(€»
:::; (y - x)K(€) + € < 2€
whence {f*} IEF is equicontinuous. The Arzela-Ascoli theorem applies.
D
5.47. If J.L E P, AI, A2 E S,B, AIlJA2 = [0,1], and J.L (AI) J.L (A2) > 0 then
J.L (E n Ai) . P .../.. . 1 2 d
J.Li: 5 ,B '3 E t-+ J.L(Ai) are III ,J.L I J.Li,~ = , ,an

Hence J.L is an extreme point of P iff for all E in S,B, J.L(E) is zero or one.
D
5.48. The linear span A of {xt-+ e nx }~=o is II lloo-dense C ([0,1], JR) and

L : A '3 2:;;=0 ane t-+ 2:;;=0 ant n is a nonnegative linear functional.


nx
If 9n E A, n E N, 9n ! 0, then, since L is nonnegative, L (9n) ! a ?: O.
Dini's theorem implies 119nll00 ! O. If kEN, there is an mk such that
a aL(l)
0:::; 9n :::; k if n > mk and thus a :::; L (9n) :::; -k- whence a = O.
If sup { IL(h)1 : hE A, Ilhll oo = 1} = 00 then for some {hn}nEN con-
tained in A, Ilhnll oo == 1 while IL (hn)1 ?: n. Hence ~ ~f Hn ~ 0 while
IL (Hn)1 ?: ..;n. = Hn + (n + 1) IIHnl1 00 E A, n E N, while
< 9n clef
However 0
n
gn ~ O. Hence for some subsequence {gnkhEN' 9nk ! 0 and L (9n k) ! 0
while

The first term in the right member is unbounded while the second converges
to zero as k ---+ 00, a contradiction. In sum, for some constant M and each h
in A, IL(h)1 :::; MHhll oo . Hence L may be extended to an L in C([O, 1], JR)*.
The Riesz representation theorem applies. D
266 5. Measure Theory: Solutions

L+
5.49. If J.LI is a measure and R ~f [a, b] x [c, d] C X then direct calculation
shows that J.LI(R) = (1 4xy) dydx. It follows that if 8 E S,a(X) then

J.LI (8) = Is (1 + 4xy) dydx, a formula that fails when °< a ::; b ::; 1 and

8 ~f ([0, a] x [0, b]) n X. A similar calculation reveals a contradiction for


the putative measure J.L2· 0

denoting I:
5.50. Integration by parts shows that if hex)
J(t) dt,
= ° off [a, b] then, F(x)

-1 J(t)h(t)dt = - [F(t)h(t)l~ -l b
F(t)hl(t)dt]

= lb F(t)h'(t) dt = lb h'(t) dJ.L(t).

Hence J.L ([a, b]) = I: F(t) dt and if E E S,a then J.L(E) = IE F(t) dt, whence
J.L «:
dJ.L
A and dA (x) = jX -00 J(t) dt. 0
5.51. a) The formulre

JR \ (x + E) = x + (JR \ E), E6.(x + E) = (JR \ E) 6. (x + (JR \ E))

lead to the conclusion.


b) If A(E)· A (JR \ E) > 0, the metric density theorem and translation-
invariance of A imply that for some p resp. q in E resp. JR. \ E and for all
sufficiently small positive a,

A(E n (p - a,p + a)) > 1.8a and A((JR. \ E) n (q - a, q + a)) > 1.8a.

=p-qthen
I f x def

A((JR. \ E) n (q - a, q + a)) = A((X + (JR. \ E)) n (x + (q - a, q + a)))


= A((X + (JR. \ E)) n (p - a,p + a)) > 1.8a

and if 2a < Ip - ql then (x + (JR. \ E)) n (E n (p - a,p + a)) = 0, whence

(x + (JR. \ E) n (p - a,p + a)) n (E n (p - a,p + a)) = 0,


2a ~ A(((X + (JR. \ E)) n (p - a,p + a))l:J(E n (p - a,p + a)))
= A(((X + (JR. \ E)) n (p - a,p + a))) + A((E n (p - a,p + a)))
> 3.6a,
a contradiction. o
Solution 5.52 267

5.52. a) Fejer's theorem implies that if f is continuous and periodic with


period 271" then f on [0,271"] is uniformly the limit of a sequence of trigono-
metric polynomials. Thus fn~. f(t) dJL(t) = 0 and the equality continues to
obtain if f is a.e. periodic with period 271" and fELl (lR, JL). Hence if
A E Sf3 and A + 271"n = A, n E Z then

A = UnEZ (A n [271"n, 271"(n + 1))) ~f UnEZAn


An + 271" = An+1
JL (UnEz(A + 271"n)) = JL(A) = L JL (An).
nEZ
Since XA(X + 271") = XA(X), it follows that JL(A) = fn~.XA(x)dJL = 0 and
so JL (U nEz (A + 271"n)) = O. If E E Sf3 and B ~f UnEZ(E + 271"n) then
B + 271"n = B, nEZ, and so

JL (U
nEZ
(B + 271"n)) = JL(B) = 0 = JL (U nEZ
(E + 271"n)) .

b) If {an}:=o C C, 2::=0 lanl = 00, and 2::=0 an = 0 define JL by the


· IOns
cond It . JL: 5 (3 3 E I--> { 0' " iftEhn"271"Z = 0 [If t h e conventIOn
.
L..J21rnEE an 0 erwlse
LnE0 an = 0 is adopted, fL(E) = 2:21rn~ an.]
If m E Z then fn~. e- ttm dJL(t) = 2:n=O an = O. On the other hand, if
E ~f {271"k}%,:0 then JL(E) = 0 and JL(E - 271"n) = 0 while
n-1
JL(E + 271") = -ao,··· ,JL(E + 271"n) = - Lak,n E N.
n=O
Thus if N > 0,
N
L JL(E + 271"n) = -Nao - (N - 1)a1 - ... - (N(N -1))aN-1
n=-oo
N-I ) N-I
=-N (
~an + ~nan.
268 5. Measure Theory: Solutions

It follows that S2N+1 t <


fails to converge as N -+ 00.
s. ° and S2N
-
is> ° N1
whence Ln:o J.L(E + 27rn)
0
°
5.53. a) Extend I so that I(x) = off [0,1]. There is a Borel set E on
which J.L lives and A(E) = 0. It may be assumed that {0,1} c E C [0,1].
For some N that is a null set (A), if x ~ N then I(x - y) as a function of
Y is Borel measurable whence fIR I(x - y) dJ.L(Y) exists and is finite a.e. (A)
iff Ix ~f fIR I/(x - Y)I dJ.L(Y) < 00 a.e. (A).
If En ~f {x : X E [0,1]' I/(x)1 ~ n}, n E N, then En E SA and fur-
thermore, A (En) i 1. Since {O, 1} C E,

[0,2] :::> En +E:::> En U (En + {1}) and En n (En + {1}) == 0 (A).


Hence 2 2 A* (En + E), A* (En + E) i 2, and

22 A* (UnEN
(En + E») 2 A* (U nEN
(En + E») 22,

i.e., A ~f UnEN (En + E) is measurable and A(A) = 2.


If x E A, say x E Eno +E, then for y in E, X-Y E E no ' I/(x-y)1 ~ no,
and so Ix < 00 a.e. (A) in [0,2]. If x E JR \ [0,2] and Y E E then x-y ~ [0,1]'
I(x - y) = 0, and Ix = 0.
b) The Fubini-Tonelli theorems imply that if I, g E L1 (JR, A) then
1* g(x) ~f fIR I(x - y)g(y) dy
exists for all x off a null set (A) and 1* g is
in L1 (JR, A).
The Lebesgue-Radon-Nikodym decomposition

J.L = J.La + J.Ls, J.La «: A, J.La 1. J.Ls, J.Ls 1. A


and a) reduce the problem to the study of f[o,1]I/(x-y)1 dJ.La(Y). If d~a ~g
and I, g are extended so that each is zero off [0,1] then each is in L1 (JR, A)
and fro,1]I/(x - y)1 dJ.La(Y) = fIR If(x - ylg(y) dy = III * g(x), which exists,
is finite a.e. (A). Furthermore, III * g E L1 (JR, A) (cf. 4.33). 0

5.54. a ) S·
mce ·
J.Ln IS complex, °< IJ.Ln I(JR) = an < 00.
def
I f bn 1
= -2-
def
nan
then L:'1 anbn = 1 and lJ.Lnl « L::"=1 bn lJ.Lnl ~f v.
b) If In ~f X[n,n+1] then J.Ln : 5,6 :3 E 1-+ fE In(x) dx is a nonnegative
measure and if v« J.Ln, n E N, then v (( -00, n]) == whence v = 0. 0°
5.55. For {Yn} nEN contained in JR, {Yn + E} nEN is a sequence contained
in S,6(JR). For S, the set of all such sequences,

a ~ sup JJ.L
l
(UnEN
(Yn + E») : {Yn + E} nEN E s} < 00
Solutions 5.56-5.57 269

and for m in N, there is a sequence {Ynm} nEN such that

fJ. (U
nEN
(Ynm + E)) > a - ~.
If {Xd kEN is an enumeration of {Ynm}m,nEN and

U(Xk +E) ~f A
kEN

then fJ.(A) = a. If G ~f ~ \ A,

then for x in ~,

If V2 (x + E) > 0 it follows that fJ. ((x + E) u A) > a, a contradiction. D


5.56. In Figure 85.1 it can be seen that if (at, a2) is near (b1 , b2) then
1fJ. (Q (at, a2)) - fJ. (Q (b 1 , b2 ))1 is the sum of the fJ.-measures of at most six
nonoverlapping rectangles with sides that are parallel to the vertical or
horizontal axes.
Owing to the results in 4.10 - 4.12, fJ. confined to a bounded rectan-
gle is regular. If {Rn} nEN is a monotonely decreasing sequence of closed
rectangles with sides that are horizontal or vertical and A (Rn) 1 0 then
limn->oo Rn is a point or a line segment, whence fJ. (Rn) 10.
If (bt, b2) approaches (at, a2), the nonoverlapping rectangles noted in
the first sentence constitute six or fewer families of rectangles partially
ordered by inclusion and each family contains a cofinal sequence converging
to a point or a line segment. Hence J is continuous. D
5.57. For t in [0,00), if

then fJ. (8Sd = 0 and fJ. ((0, 0)) = O. The argument in Solution 5.56 shows,
mutatis mutandis, that J : [0,00) :3 t I--> fJ. (St n E) is continuous. Since E
is bounded, for large t, J(t) = fJ.(E) whereas J(O) = O. Hence for some t a ,
J (ta) = a and F ~ Sta n E meets the requirements. D
270 5. Measure Theory: Solutions

lR

I
I
I
- - - --

(ai' a 2 )

• (bl' b 2)
Q(a l' a 2 )
------
I
I
I

~----------------------------------lR

Figure 85.1.

5.58. a) If K is compact then P,m(K) < 00, m E Z+, whence R,n is a-finite
(J.Lm) , mE Z+.
Since each open set in R,n is the union of countably many compact
sets, each open set is inner regular, hence regular, (P,m) , mE Z+. The set
of regular sets is a a-ring and contains all open sets whence every set in 5,8
is regular.
If K is compact and P,o(K) > M, (5.6) implies that for large m,
P,m(K) > M.
b) The argument in Solution 4.19 serves here as well. 0
5.59. If Um is open in R,m and Un is open in R,n then Um x Un is open
in R,m+n, whence 5,8 (R,m) x 5,8 (R,n) C 5,8 (R,m+n). On the other hand, if
Um +n is open in R,m+n then Um +n is the countable union of rectangles of
the form Xk=l [ak' bk).
m+n

The Fubini-Tonelli theorems imply that SA x SA C 5 A2 • If E is a


nonmeasurable subset of R, then A ~f E x {O} ¢. SA X SA but A2(A) = 0
and so A E 5 A2 • 0
5.60. Every subset of D ~ {(x, y) : x = y} in R,2 is a null set (A2)
and thus is in 5 A2 • In particular, if E is a nonmeasurable subset of R"
Solutions 5.61-5.64 271

F ~f D n { (x, y) : x E E}, and r' = {I} then F E 5>'2 while nevertheless


Pr,(F) =E ~ 5>.. 0
5.61. Let (Xi, 5i , J.Li) be (JR, 5,8,),) ,i = 1,2. Any rectangle contained in
~ ~f {(x, y) : x = y} in JR2 is a single point whence ~ is not the countable
union of rectangles of the kind described.
If (Xi, 5 i , J.Li) = (N,2 N,() ,i = 1,2, then 51 x 52 = 2N2 • 0
5.62. If A ~f p-1 « -00,0)) n K =I- 0, there is in Coo (JR n , JR) an f
such that 0 ~ f(x) ~ 1, f(x) = 1 at some point of A, and f(x) = 0
off A. Since supp(J.L) = K, In:f. n f(x)p(x) dJ.L(x) ~f -c < O. The Stone-
WeierstraB theorem implies that f! may be approximated by a polyno-
mial q such that" (J - q2) plK 1100 < 2J.L~K)' which leads to the conclusion

JRn (q(x))2 p(x) dJ.L(x) < -~ < 0, a contradiction. 0


5.63. Egorov's theorem implies that if € in (0,1 - a), there is in S>. an E
such that ),(E) > 1- € and on E, fn ~ f. Hence f is continuous on E and
since), is regular, E contains a compact set K such that ),(K) > 1- € > a.
o
5.64. i. If f E AC([a, b], JR) then:
a) The continuity of f follows by definition.
b) For some positive 8(1),

1 1
Hence for a unique N(l) in N, N(l) +1 < 8(1) ~ N(l)· If P is a
partition and IPI < 8(1) then varra,bj,P(f) ~ N(l) + 1.
c) If € > 0 then for a positive 8(€),

If )'(E) =
0 and € > 0, for some sequence {In}nEN of intervals,
E c UnENln and 2:~1), (In) < 8(€). Thus 2:::'=1), (f (In)) ~ €,
whence f(E) C UnEN f (In), ), (f(E)) ~ 2:::'=1), (f (In)) ~ €, and so
), (f(E)) = o.
Conversely if a), b), and c) obtain then b) implies

f(x) = var[O,xj(f) - (varro,xj(f) - f(x)) ~f g(x) - h(x),


272 5. Measure Theory: Solutions

i.e., I is the difference of two monotonely increasing functions 9 and h while


a) implies 9 and h are continuous (cf. 3.36).
Because 9 is monotonely increasing, there is a possibly empty but at
most countable set {1m} mES of pairwise disjoint closed intervals such that if
x < y and x and y do not belong to one of those intervals then g( x) < g(y):
9 is a bijection on the complement Q of the union of those intervals.
If 9 f{. AC([a, b], lR), for a sequence {ank' bnk }:;';;1 ,n E N, of finite sets
of pairs in Q2 and some positive E,

a ~ anI < bn1 ~ an2 < bn2 ~ ... ~ an,Kn- l < bn,Kn- l ~ anKn < bnKn ~ b
9 (ank) < 9 (bnk ) ,1 ~ k ~ K n , n E N
Kn Kn
L (bnk - ank) < 2- n , Lg (bnk) - 9 (ank) :2: E.
k=1 k=1

If An ~f U:;';;1 lank, bnk ] and A ~f limn_ooAn then A(A) = O. However


I(A) = g(A) - h(A), whence the translation-invariance of A and c) imply
A(g(A)) = O.
Since 9 is continuous and monotonely increasing, 9 ([ank' bnk]) are pair-
wise nonoverlapping compact intervals, 1 ~ k ~ K n , whence A (g (An)) :2: E
and A (limn_oog (An)) :2: Eo Owing to the bijective nature of 9 where it is
applied below,

whence limn_oog ([ank' bnk ]) = g(A), a contradiction of c) since the null


set A is carried by 9 to a set of positive measure. Thus 9 is absolutely
continuous; a similar proof shows h is absolutely continuous.
ii. If [a, b] = [0,1] and f is the Cantor function 4>0, then I is continuous
and monotonely increasing. However if E = Co, the Cantor set, then
A (f(E)) = 1 while A(E) = 0: a) and b) obtain while c) fails.
If I(x) = X(O,IJ(X) . x sin (~), 0 < a < 1, and x E [a,l] then

1f'(x)1 ~ 1 + 22 whence f is absolutely continuous on [a, 1]. Thus a) and


a
c) obtain. If n = 3,5, ... , and P is a partition determined by the points
2 2 4 n 1 .
0, - , ( ) , ... ,1 then var[O IJ p(f) ~ - " - k - - whence f IS not of
mr n-27r "7r~2 -1
k=1
bounded variation on [0,1]: a) and c) obtain while b) fails.
If 1= X{H then b) and c) obtain while a) fails.
iii. If E is Lebesgue measurable then owing to the regularity of A, there
is a sequence of {Kn} nEN of compact sets and there is a null set N such that
A = Nl:J UnEN Kn· Hence I(A) = feN) U UnEN f (Kn). From i it follows
that feN) is a null set and, from the continuity of f, that UnEN f (Kn) is
a union of compact sets. 0
Solutions 5.65-5.71 273

[ Note 85.3: The proof for iii goes back to first principles. On
the other hand, if E is Lebesgue measurable then E contains a
Borel set 8 such that E \ 8 ~ N is a null set (A). Then I(N) is
a null set and (cf. 1.13 - 1.16) 1(8) E SA whence I(E) E SA']

5.65. 1E Lip( 1) then 1 is absolutely continuous and for x in [a, b],


I: f'(t) dt = I(a), i.e., 1 is a constant, a contradiction.
If
I(x) = I(a) + 0
5.66. If I/(x) - f(y)1 :S: Mix - yl and E is Lebesgue measurable, the ab-
solute continuity of 1 implies A (f(E)) = IE
f'(t) dt :S: M IE
1 dt = MA(E).
o
5.67. The function ¢ : [0,1] 3 x f---' X + ¢o(x) is strictly monotone and
continuous whence ¢-l ~f 1 is strictly monotone and continuous and both
¢ and I, as homeomorphisms, are open.
Furthermore, A (¢ (Co)) = 1. Let E be a nonmeasurable subset of
¢(Co). Then SA 3 F ~f I(E) C Co and 9 ~f XF E M. If 9 0 1 ~f h then
h- 1 (1) = E, whence h ~ M. 0
5.68. In 5.67, F E SA and ¢(F) =E ~ SA. 0
5.69. The set A of all finite subsets A of E is a diset with respect to the
partial order: A -< A'iff A C A'. The net n : A 3 A f---' n A ~ XA is such that
for each A, n A E M, whereas n A converges to XE in IR x \ M. 0
5.70. Note that v «J.L. Let N resp. M be the v-essential supremum
resp. J.L-essential supremum of I. Thus {x : 1 (x) > M} is a null set (J.L)
and hence a null set (v). It follows that N :S: M. On the other hand,
. N
If =
< M then E def { x: 1(x) > N +
2 M} . ()
IS a null set v, whence

IE I(x) dJ.L(x) = 0 and so N ~ M ~ M, a contradiction. 0


5.71. See 2.27 and 2.28. A proof that exploits topology rather than set
theory follows.
Since A(E) > 0, there is a compact subset K of E and A(K) > O. The
Cantor-Bendixson theorem (2.28) implies that K is the union of a perfect
set P and a countable set N. Furthermore, P =f. 0 since otherwise, K is
countable and "\(K) = O. As a perfect subset of JR, P is of cardinality c.
Hence c ~ #(E) ~ #(P) = c. 0
[ Note 85.4: Sierpinski [Sier] showed that the stronger implica-
tion {>. * (E) > O} => {# (E) = c} is equivalent to the Continuum
Hypothesis (CH). Owing to Cohen's proof [Coh2] of the indepen-
dence of CH from ZF, the implication just cited is not derivable
in ZF.]
274 5. Measure Theory: Solutions

5.72. The Stone-WeierstraB theorem implies that the linear span A of


{[O,oo) 3 x ~ e- nx : n EN} is dense in Coo ([0,00), lR).
Moreover, since
1J.l1 is bounded, if f E Coo ([O,oo),lR)
then Jo f(x)dJ.l(x) = whence for
oo °
every compact set K, J.l(K) = 0. From 4.10 it follows that J.l (5/3) = {O}.
D

5.73. For n in N, the sequence { [2:' k; 1) } ~=o is a partition of [0,00)


and, since 1 - J.l ([0, x)) is monotonely decreasing and nonnegative,

E~+f; (1- J.l ([0, 2:))) 2- n = 1 (1-


00
J.l([O,x))) dx (~oo).

Abel summation, i.e., integration by parts with respect to counting measure,


yields

~ (1 - J.l ( [0, ~ ))) Tn = } ; kT n . J.l ( [ k ; 1,2: ))


+ K2- n . J.l ( [~ , 00))
f; (1- ([0, 2: )))
J.l Tn = J~oo (}; k2- n . J.l ( [ k 2~ 1,~ )))
+ KT n . J.l ( [~ ,00 ) ) .

If io,oo) xdJ.l(x) < 00 then (cf. 5.15) limK ...... ooK2- n . J.l ([~,oo)) = °
and so

}!..~ (~k2-n. J.l ([k; 1, 2:)) ) = 1 (1- ([0,


00
J.l x))) dx

=1 00
xdJ.l(x).

If r
J[O,oo)
xdJ.l(x) = 00 then J[O,oo)(I- J.l([O,x)))dx = 00. D

5.74. The following definitions and equations facilitate the discussion of


the question.
Solution 5.75 275

p~f {[~ ~)}oo


n
(s5.2)
2n ' 2 k=O

bk def
= J-L ({ y: 2kn ~ f(y) k + 1 }) , 1 ~ k
<~ ~ K (s5.3)

8k L
~f K-l bp = J-L ({ y: 2kn ~ f(y) < 2Kn }) '
p=k
o~ k ~ K - 1&(s5.4)

K-l
L akbk = a2 (81 - 82) + ... +aK-l (8K-l - 8K) (s5.5)
k=2

(s5.6)

If f E Ll ([0,00), J-L), then forlarge K and n, i.e., for small IFI and large
n, both members of (s5.5) approximate loco f(y) dy. Since f is bounded,
say f(x) ~ M, for fixed n, if K > M2 n then the right member of (s5.4) is

while 8 K = O. Furthermore, limn_co 8 1 ~ = O. Thus the right member of


(s5.6) approximates J[o,oo) J-L ({ Y 2n
: x ~ f(y) }) dy. 0
5.75. If

Jkn =
def k+ 1) , n E N, -lz0 ~ k ~ 2n -
[k2n ' ~ 1,

mkn~finf{f(x): XEJkn},Mkn~fsuP{f(x): xEJkn},


m~finf{f(x): O~x~I},M~fsup{f(x): O~x~l},
gkn : lR 3 Y {I I--t
o
?
if mkn y ~ Mkn
otherwIse.
then Gn(y) ~f Lk gkn(y) is measurable and counts the number of Jkn on
which the equation f(x) = y has at least one solution. Furthermore, Pn
denoting the partition created by the endpoints of the intervals Jkn,

LM gkn(y) dy = Mkn - mkn ~ oscJkn (I) and


1 M
Tn
Gn(Y) dy =L
k
OSCJkn (I) = var[O,I],Pn (I).
276 5. Measure Theory: Solutions

Since Gn ~ Gn+b limn..... oo G n ~f G exists and G(y) dy = Var[O,lJU).


On the other hand, G n ~ v whence G ~ v. If pEN and p ~ v(y) then
J::
for some n, Gn(y) ~ p. Since v is Nl!.loo-valued, G = v. 0
5.76. It may be assumed that a1 ~ a2 ~ ... ~ aN, and an ~ bn . If b1 < a2
then (bb a2) C [a, b] \ U:=l [an, bnJ, a contradiction. Hence b1 2: a2 and
similarly, if 2 ~ n ~ N then bn- 1 2: an. In sum, [an, bn] n [an+!, bn+!] -I- 0.
Since for some n, bn 2: b, it follows that 2::=1 (b n - an) 2: b - a. 0
5.77. Every line in R2 is a null set (A2). 0

5.78. The functions fn : [0,1] 3 x 1--+ 2:~=1 1 1,n E N, are


k2 1x - rkl"2

nonnegative, In i I, and 1 [O,lJ


fn(x) dx =
k=l
n
L.::
2(r: +(I-rk )!)
k2 . Since

maxo::;x::;1 (x! + (1- x)!) = J2, the monotone convergence theorem im-

plies 1[O,lJ
I(x) dx ~
00

k=l
J22
V2L.:: l/k 2 = _7r_ whence f(x) <
6
00 a.e. 0

5.79. Since A (UnEN (rn - l/n 2, rn + l/n2)) ~ 22::=1 l/n 2 = 7r2 /3, it
follows that R \ UnEN (rn - l/n 2, rn + l/n2) -I- 0.
For A ~f N\ {n2}:=2 ~f {an}nEN' it may be assumed that an < an+!.
For some sequence {tkhEN in Q, Itk -11 < ~, kEN. For the enumeration
ak
{rp}PEN of Q \ {td kEN , Sl ~f h, sp r
~f 0'-1 when p tJ- A, and when
1 < pEA, sp
is the first tk not yet chosen. Thus pEN is an enumeration {Sp}
of Q. Since

u (sp-!,Sp+!)
pEA p P
C[-1,2J, 1+L.::A((Sp_!,sp+!)) ~7r2/3,
p~A P P

it follows that R \ UPEN (sp - ~, sp + ~) -I- 0. o


5.80. If A({X2 : A (E"'2) = I}) > 1/2 then

A2(E) = 11 (11 A(E X2 ) dX2) dX1

2: r1 ( r
Jo J{ "'2 : .x(Ez2)=1}
1 dX2) dX1 > ~,

o
Solutions 5.81-5.83 277

5.81. By definition, if Al x A2 C E, Ai E 5)" i = 1,2, then (AI - A 2)


does not meet Q. Furthermore, if A2 (AI X A 2) > 0, it may be assumed that
A(AI) + A(A2) < 00. Then f : ]R. 3 xl ~ fIR XA 1 (Xl + X2) XA 2 (X2) dX2 is
continuous, nonnegative, vanishes off Al - A 2, and

Hence f is nonzero on a nonempty open set U contained in Al - A2 and


there emerges the contradiction: (AI - A 2) n Q =I- 0. D
5.82. For m in Z, n in N, if

if Xl E
m-1 m]
[-n-';:
[on [m: 1, : ] ~f (Pm-I,Pm] the value of fn is a convex (linear) com-
bination of f (Pm-I, X2) and f (Pm, X2)] then on (Pm-I,Pm], fn (Xl, X2) is
between f (Pm-I, X2) and f (Pm, X2). Because Pm depends on nand fX2 is
a continuous function of Xl, there emerges the following diagram in which
each arrow represents convergence as n --> 00 and ARB R C means B is
between A and C: B = mid(A, B, C).

f (Pm-I, y) R fn(x, y) R f (Pm, y)


1 (s5.7)
f (Pm-I, y) --> f(x, y) +- f (Pm, y)

The hypothesis implies that for each m, ]R.2 3 (X, y) ~ f (Pm-I, y), regarded
as a function on ]R.2, is Borel measurable whence f is Borel measurable.
D
5.83. In analogy with the method used in Solution 5.82, for n in N, E
contains a sequence {anm}mEN such that

A diagram similar to that in (s5.7) and an argument similar to that as-


sociated with (s5.7) but in which "Lebesgue measurable" replaces "Borel
measurable" concludes the proof. D
278 5. Measure Theory: Solutions

5.84. This time the device used in Solution 5.82 shows that

hn : R2 3 (Xl, X2) 1--+ In (g (X2) ,X2)


(a function on R2) is Lebesgue measurable and limn __ oo h n = h. 0
5.85. Tonelli's theorem implies that

If X2 =f. 0, f
JI[-ll] 2
1+ x~
IXII 2 dXl = In --2-' .
whence the double and Iterated
, Xl +X 2 x 2
integrals are finite. 0
5.86. Since I is continuous on [0,1]2 \ {(O, On and III 1, it follows that :s
I E Ll ([0,1]2, >'2), whence both integrals are the same as the integral of I
over [0,1]2. Since I (Xl, X2) = -I (X2' Xl), the double integral is zero. 0
I 1 1!-1
I : (0,1) 2 3
X l-
5.87. For (Xl, X2) 1--+ X2 2 ,

F (Xl) ~f 101 I (XI, X2) dX2 dXl


I -! 1
= { IXI - 21 if Xl =f. 2,
= 00 .f
I Xl = -1
2
10 1
F (Xl) dXl = 2V2.

Hence lEe (0,1)2, R) n Ll (0,1)2, >'2) and yet

o
5.88. For /-Ll : SA ([0, 1]) 3 E 1--+ IE x! dx and /-L2 = >.,
-d/-Ll () 1 d/-L2 ()
X =X2, - X =X _12,
d/-L2 d/-Ll
whence /-Li « /-Lj, i =f. j and each /-Li is totally finite.
Since d/-Ll :s 1, it follows that d/-Ll E L oo ([0, 1], /-L2). However if M > 0,
d/-L2 d/-L2
then /-Ll ({ X : ~~: (x) > M }) = /-Ld (0, M- 2 )) = 2~-3 > 0, whence

dd/-L
2 ~ L oo ([0, 1], /-Ld. 0
/-Ll
Solutions 5.89-5.93 279

5.89. If f
def
= dJ.L
d)' and °< X - a < x + a < 1, then

F(x) ~f J.L ([0, 1] n (x - a, x + a)) = ~ lx+a f(t) dt.


). ([0,1] n (x - a, x + a)) 2a x-a
As a ---> 0, F ~. f. D
5.90. For any Lebesgue measurable set E (in 1R), the metric density of
E is 1 a.e. on E. For E as described, the metric density of E is constantly
one-half. Hence E ~ SA· D
5.91. a) Since components are disjoint,

b) It suffices to prove the inequality for each of the finitely many compo-
nents {Dkh<k<K<N ofU~=1 I n . Each Dk is the finite union of interlocking
intervals dellotable (at, bd , ... , (a p , bp ) and such that

If

" (b· -a·) < )'(Dk ) "(b· -a.) < )'(Dk ) (s5.8)
~J J 2'~J J 2
j odd j even

then L1::;j<K (b j - aj) < ). (Dk) :::; L:1::;j<K (b j - aj), a contradiction.


Hence one of the inequalities in (s5.8) is false. D
5.92. If J.L as described exists then for some f in £1 ([0,1],),) and all
a in [0,1], J.L([O,a]) = Ioaf(x)dx. Since f = f+ -f-, it follows that
loa f+(x) dx = loa f-(x) dx whence for each Lebesgue measurable set E,
IE f+(x) dx = IE f-(x)dx, i.e., f+ == f-, f == 0, and J.L = 0, a contradic-
fu~ D
5.93. Let U represent a rotation in]Rn, i.e., with respect to an orthonormal
basis X for ]Rn, U is represented by a matrix Uxx ~ (Uij)~j:1 such that
UkxUxx = id, and det(Uxx) = 1. In particular,

U (B(x, r)O) = B (U(x), r)t, ). (U (B(x, r)O)) = ). (B (U(x), r)t) .

Since). is regular, if E E SAn there is a sequence {B (Xmn' rmnt} m,nEN


of open balls such that E c Vm ~f UnENB(xmn,rmnt, Vm+1 C Vm ,
280 5. Measure Theory: Solutions

and oX (Vm) 1 oX(E). However, U(E) c U (Vm) = UnEN B (U (xmn) ,rmnt


whence oX (U(E)) = oX(E). D
5.94. Let be an enumeration of IQ and for m,n in N, let Jmn
{rn}nEN

be (rn - 2-(m+n+1) , rn + 2-(m+n+1)). Then Wm ~f UnEN Jmn is a dense


open set containing IQ, N m ~f IR \ Wm is nowhere dense, oX (Wm) = 2- m ,
and W ~f n mEN Wm is a dense Go that is a null set (oX). Furthermore,
IR \ W = UmEN N m , a set of the first category. D
5.95. If fELl (IR, oX) then 9 = f * XE and since XE ELl (IR, oX), it follows
that 9 E L1 (IR, oX).
Conversely, if IIgl11 < 00 then the translation-invariance of oX and the
Fubini-Tonelli theorems imply

1 9(x)dx= 1(l xE (t)f(X-t)dt) dx= (1 f(x)dX) (l xE (t)dt)

IIgll1
whence °: :; f
JIR f(x) dx = oX(E) < 00. D
5.96. Since J.t (IR) = 1,

fb : IR 3 x f--t J.t (( -00, b) + x) [= J.t (( -00, b + x))]

is, for every b in IR, a monotonely increasing function such that

lim fb(X)
x~-oo
= 0, lim fb(X)
:£---+00
= 1.
Hence for all b, fIR fb(X) dx = 00 = oX (( -00, b)). Furthermore,

J.t([a, b) + x) = J.t([a + x, b + x)) = J.t(( -00, b + x)) - J.t(( -00, a + x)),


lim J.t([a,b) +x)
a-+oo
= a-+oo
lim J.t([a+x,b+x)) = 0,
lim J.t([a,b)+x) = lim J.t([a+x,b+x)) =0.
b-t-oo b-t-oo

If En = 0,;;;
def [ 1) then En = U. k=O
m- 1 ( k ) .
Em.n + mn

J.t (En + ~) m-1 J.t (Em.n + ~ + ~)


mn
Hence L:PEz ---''----~'"''-'-
mn
= "~ "~ mmnn mn For
k=O pEZ
each k,

"
J.t (Em.n + ~ +~)
mn mn = L
J.t(Em.n+~)
mn
~ mn mn mn mn
pEZ pEZ
Solutions 5.97-5.98 281

whence 2:
P
EZ
It ( En
mn
+l
mn
) 1
= m - = - ---+
mn
1
n
1 IR
It (En + x) dx as m ---+ 00.

Thus A (En) = ..!:. = [ It (En + x) dx. A similar calculation shows


n JIR

The map v : 5/3 :3 E I--' fIR It(E + x) dx is a measure such that for all n in
N, A ([a, a+ ~) ) = v([a, a+ ~) ) whence v= A. D

5.97. For k in N, UrEQn B (r,~) ° = JRn whence for each k there is an


rk such that Am [T- (B (rk' ~) 0)] > O. If T(x) EB(rk' ~) ° then
1

Hence T- (B (
1 0, ~) 0) _T- (B (0, ~) 0) contains a neighborhood
1

of 0. However

whence T- (B ( 1 0, ~) 0) contains a neighborhood of 0. Correspondingly,


if T(y) = z then T- 1 (B (Z' ~) 0) :J Y + T- 1 (B ( 0, ~) 0) which con-
tains a neighborhood of y. Thus if U is open in JRm then for some sequence

{zp,: } P pEN
in Qm x Q, U = UPEN B (Zp,: )0 and so T-l(U) is open,
P
i.e., T is continuous.
[Since T(x+y) == T(x)+T(y), it follows that for n in N, T(nx) = nT(x)
and, more generally, for s in Q, T(sx) = sT(x). If a E JR, T(ax) = aT(x)
because T is continuous. J D
5.98. Let Em be {x : am(x) = O} and let E be UmEN Em. Each Em is
the union of lOm-l pairwise disjoint intervals, each of length lO-m. Hence
282 5. Measure Theory: Solutions

each of Em, E, [0, 1] \ E is in Sa and [0,1] \ E = I-l(k). Furthermore,

(:or
E = Ell!! (E2 \ E l ) l!! (E3 \ (El U E 2)) l!! ...

A(E) = 1~ ~ = 1, A (I-l(k)) = °
1- 1 (1) = (El \ E2) U (El n E2 n E3 \ E 4 ) U··· E S,a

A (1-1(1)) = (110 - 1~2) + (1~3 - 1~4) + ... = :1'


[1 1
Hence I E M, I ~ 0, and 10 I(x)dx = 11 because A (I-l(k)) = 0. o
5.99. a) If mEN C E and limm--too Xm = x, let the decimal repre-
{X m }


sentatlOn bn I I 1
L..m=l IOn' If x - Xm < 1QP then bp = 2 or 7 whence
for x be ,,",00
x E E, i.e., E is closed.
b) Since [0,1] is connected, E is closed, and E =1= [0,1]' E is not open.
c) #(E) = # ({2, 7}N) = C.
d) Since En (.28, .7) = 0, E is not dense. (Alternatively, E is closed
and E =1= [0,1].)
e) Since E is closed, E is Lebesgue measurable. In fact, E is like the
Cantor set Co and A(E) = 1 - .8 2::=0(.2)n = 0. 0
5.100. Define a sequence {an} nEN as follows:
°
Each an is or k and these occur alternately in blocks of size
1,2,22,23, ... , i.e., 0, k, k, 0, 0, 0, 0, k, k, k, k, k, k, k, k, ....
,,",00 an -.- AN(x,k,m) 2
If a = L..m=l mn then on the one hand, hmN--too N 3'
· AN(X, k, m) 1
wh ereas 1ImN--too N = 3' Hence a E E.
~T
If p E L'I, X
def
= ,,",00
L..m=l
€n(x)
-n-' an
d !: ()
p x = {I° ift€p(x)
h'
=k h
t en
m 0 erwlse
Ip E M and A N (·, k, m) E M because AN(X, k, m) = 2::=l/p(x). Fur-
thermore, E E SA because

[0,1] \ E = {X : J~oo AN (XJ.v,k, m) eXists} E SA'

For x in [0,1]' if 8 > ° and 2:~-1


- m
€n(:) is the m-ary representation

for x, let p be such that ~ < 8. The argument showing E =1= 0 and used
mP
on the m-marker places following the pth, shows as well that there is in E a
y such that Ix - yl < 8. Hence E is dense and since km
m- 1
= ~ ~ rf. E,
~mn
n=l
E is not closed.
Solutions 5.101-5.104 283

For x, 8, and p as above, if y = 2:~=1 fn(X)


mn
+ ~ ~ then y ¢. E
L.J mn
n=p+l
and Ix - yl < 8, whence [0,1] \ E is dense and so E is not open. D
5.101. In each of the intervals deleted in the process of constructing a
Cantor-like set, there can be constructed a Cantor-like set, and this process
may be repeated without surcease. The Cantor-like sets so created are
pairwise disjoint, their union is dense in the underlying interval, and the
union may be constructed so that its measure is any number in [0,1).
In each interval In, n + 1), nEZ, construct a countable dense union
Sn of Cantor-like sets such that A(Sn) E (0,1) and 2::=-00 A (Sn) < 00.
If E = UnEZ Sn then A(E) < 00. Every nondegenerate interval (a, b) con-
tains an interval deleted at some stage of the construction of the sequence
{Sn}nEN' Such a (deleted) interval contains a Cantor-like set of positive
measure and so 0 < A(E n [a, b]) < b - a. D
5.102. The set E of Solution 5.101 serves. D
5.103. If

Fmn =
def [m-l m)
~'2n
n
,1 ~m~ 2 ,nEN,
E def p.
k = FlkU 3k U •·· UF2"-1,k

then A(Ek) == ~. If l = k + P then A(Ek nEd < ~A(Ek)' whence if


nk ~ nk+l, kEN, then A (nkEN En,,) = o. D

5.104. The Gk are pairwise disjoint. Hence A (limn--+ooAn) ~f A (Goo) = 0


(cf. 5.6). Furthermore,

Hk ~f { n
p=l
Anp : 1 ~ nl < n2 < ... < nk }
= {x : x belongs to at least k of the An} ::> Hk+1,

and Gk = Hk \ Hk+1 whence Hk, G k E S>..


Since An \ (UkEN (Gk nAn)) C Goo and

it follows that
00 00
A(An)=LA(GknAn), LA(An) = L A(GknAn). (s5.9)
k=l n=l k,nEN
284 5. Measure Theory: Solutions

[The double series in the right member of (s5.9) converges absolutely and
thus the order of summation is of no consequence.]
Since Gk C UP1<"'<Pk Apl n··· n A pk , it follows that

Thus

n=1 n=1 Pl <"'<Pk-l


n~{Pl "",Pk-d

Each term in the right member above, e.g., >. (G k n Arl n··· nArk nAn),
occurs exactly k times, namely when n = rk and PI = rl,.·. ,Pk-I = rk-I,
when n = rk-I and PI = rl, ... ,Pk-I = rk, etc. Hence (s5.9) implies
<Xl <Xl <Xl

o
n=1 k=1 n=1

5.105. If a> 0 then for some k in N, 0 < (V2 _l)k ~f m + nV2 < a,
whence S ~f { m + nV2 : m, n E Z} is dense in R
If a < band E > 0 then for some a and f3 in S, a < a < b < f3 and
f3 - a < b - a + E. Hence

As E ~ 0, the last terms above approach zero, whence


f =I- 0 on a set of positive measure, it may be assumed that for some positive
f: f(x) dx = O. If

8, >. (E>(J, 8)) > O. However, E>(J, 8) may be covered by a countable union
of intervals and for at least one, say J, fJ f(x) dx > 0, a contradiction.
o
5.106. If E ~f {x : If(x)1 = a < I} is not a null set (r) then

IaN (J, xo)1 :::; i FN (y-IXO) If(y)1 dr(y)

:::; { + ( FN (y-IXO) If(y)1 dr(y)


lE j.[,\E
< { + ( FN (y-IXO) dr(y) = 1,
lE 11l'\E
a contradiction.
Solutions 5.107-5.109 285

a contradiction. Thus f(x) = 1 a.e. or f(x) = -1 a.e.


When 'f is regarded as [0,271") then f : 'f 3 x I-t x corresponds to
9 : [0,271") 3 0 I-t ei8 . Then O"N(g,O) = g(O), IIglloo = 1. There is no
constant c such that g( 0) ~ c. D
[ Note 85.5: The sequence {FN}NEN of Fejer kernels is an
example of an approximate identity in that for any f in L1 ('f, r),
limN_oo IIf - FN * fill = O.
If G is a locally compact group and J.L is Haar measure, let U
be the set of all open neighborhoods of the identity of G. Partially
ordered by reversed inclusion: U' )- U iff U' c U, U is a diset.
A general form of an approximate identity for L1 (G, J.L) is a net
n : U 3 U I-t nu ELl (G,J.L) such that nu(x) = 0 off U, nu ;::: 0,
and Iinul11 = 1. For such a net nand f in L1 (G,J.L), nu * f
II Ill-converges to f.]

5.107. Let 9 : Q n (1/4,3/4) I-t Q n [1/4,3/4] be a bijection. Extend g-l


to [0,1] to a map

X if x ~ Q n [1/4,3/4]
G: [0,1]03XI-t { g-l(X) ot herW1se
.

Then G : [0, 1]1-t [0,1] is a bijection and f ~f G- 1 meets the requirements.


D
5.108. The construction in Solution 5.101 may be modified to produce
Cantor-like sets Can in [n, n+1) so that L:nEZ >. (CaJ = 1. If A is a Cantor-
like set in [a, b], the corresponding Cantor-like function gA is monotonely
increasing and maps A onto [0,1] and hA : (a, b) 3 x I-t tan (~9A(X)) maps
A n (a, b) onto (0,00). Hence if, for each n in N and for every Cantor-like
set A constructed in [n, n + 1),

f(x) = {~A(X) if x E A
otherwise

and a < b then J: f(x) dx = 00. D

5.109. If Ekn ~f f-1 ([~, k; 1) ) and f n def


=
,",00
L..Jk=-oo
k
2n XE kn then
o 5:. f - fn 5:. 2- n and fn i f· D
286 5. Measure Theory: Solutions

5.110. If U is an open subset of IR then (g 0 I) -1 (U) = f- 1 (g-l (U)).


Since g-l(U) is Lebesgue measurable, there is a Borel set A and two null
sets N 1, N2 (oX) such that N1 c A, An N2 = 0, g-l (U) = (A \ N 1) u N 2.
Since f is continuous, f-1(A) E Sj3(IR) while f- 1 (Ni ) E SA(IR), i = 1,2.
Hence (g 0 1)-1 (U) = (I-1(A) \ f- 1 (N1)) U f- 1 (N2) E SA· D
[ Note 85.6: Compare the result above with 5.67 where the
hypothesis about the Lebesgue measurability of sets f- 1 (N) for
null sets (oX) is dropped.]

5.111. The metric density theorem implies that if x is outside a null subset
• oX * (A n (x - d, x + d)) . *
N of A then hmd>O,d ..... O 2d = 1. Smce oX (A) > 0,
A \ N =f. 0 and for some positive d, oX* (A n (x - d, x + d)) > (). 2d. D
5.112. For the map

9 :X 3 x f-t {Of (x) if If(x)1 ::; 1111100


otherwise

f(x) == g(x) and sUPx Ig(x)1 = Ilflloo. D


5.113. The map h : [0,1] 3 x f-t oX ({ y fey) 2:: x}) is a monotonely
decreasing function and
limxioo hex) 10.
°: ;
h ::; 1. Furthermore, limxl-oo hex) i 1 and

If x> ao ~f sup {x : hex) 2:: ~} then hex) < ~. On the other hand,

h (ao) = oX (n
nEN
{Y : fey) > ao - ~ }) 2:: ~.
Finally, if a1 < ao and h (a1) 2:: ~, the monotone character of h implies
that for x in (a1' ao), hex) 2:: ~, whence b) fails for a1. D
5.114. The Stone-Weierstrafi resp. Fejer theorem implies that for 9 in
C ([0, 1], 1R), I ro,1] f(x)g(x) dx = o. The Daniell theory of integration then
implies that if f = u + iv, the functionals

Iu : C ([0,1], 1R) 3 9 f-t u(x)g(x) dx


JrO,1]
[

I v :C([0,lj,IR)3gf-t [ v(x)g(x)dx
JrO,l]
are both zero and their unique extensions are zero. Hence if E E SA then
!r0,1] f(X)XE(X) dx = 0 whence f(x) == o. D
Solutions 5.115-5.120 287

5.115. If F(u) ~f I (e U ) , G(V) ~f 9 (e V ) then F, G E L1 (JR, A) and

f I/(XY)9 (~) I
J(O,oo) y
Y dy=
A{f F(u+v)G(-v)dv=F*G(u). 0

5.116. > o. If to > 0, there is a positive A


It may be assumed that 119111
such that fn~\[_A,AjI9(x)1 dx < ~. If 0 < b < 6~ then for some N in N,

{m,n>N}=?{A({X: I/m(x)-ln(x)l2:b})<611;111}.

Since Ifni :S 191 a.e., {fn}nEN C L1 (JR, A). If

Emn ~f {X : I/m(x) - In(x)1 2: b},

then

111m - In 111 :s f + f + f I/m(x) - In(x)1 dx


JIR\[-A,Aj J[-A,AjnE mn J[-A,Aj\E mn
to to to
<"3+"3+"3=f.

Hence {fn}nEN is a II Ill-Cauchy sequence. Let h be its II Ill-limit. There is


a subsequence {fnkhEN such that Ink ~. h whence h == I· 0

5.117. For some n in N, A (E n [-n, n]) ~ A(F) > 0 and so it may be


assumed that 0 < A(E) < 00 and hence that XE ELl (JR, A). Consequently
I : JR 3 x ........ fIR XE(2x - Y)xE(Y) dy is continuous, nonnegative, and zero
off ~(E + E) which is a subset of E. Furthermore, I (~x) = XE * XE(X),
whence if I = 0 then xil = O. Plancherel's theorem implies IlxEII~ = 0
while IIxEl11 > 0, a contradiction. Thus I is positive on an open set U,
which must be a subset of E (d. Solution 5.81). 0
5.118. a) Every subset of the Cantor set Co is in SA, # (Co) = c, whence
2' 2: # (SA) 2: 2'. 0
5.119. Since JRn is separable, there is a sequence {B(an,rn)}nEN such
that E C UnEN B (an, rn), i.e., E = UnEN E n B (an, rn). 0
5.120. On the one hand, # (S{j) = c (cf. 1.6), but # (SA) = 2' (> c)
(cf. 5.118). If E E SA \ S,B, nothing need be proved. If E E S,B then
#(E) = c (d. 2.29). Since A(E) = 0, every subset of E is in SA and since
# (2E) > c = # (S{j), E must contain a set that is not Borel measurable.
o
288 5. Measure Theory: Solutions

5.121. If E E SA, there are null sets (>.) Nt, N2 and in S,a an A such
°
that E = (A \ N 1 ) u N 2 • Thus P, (N1 ) = p, (N2 ) = and S,a C SI" whence
EES w 0
5.122. If In(x) == X[O,l) (x) . x + 2X{1} and lo(x) = x,x E [0,1J then each
In is monotonely increasing, In ~. 10, 10 is continuous, but

lim In(1)
n->(X)
t- 10(1). o

5.123. a) If {gn}nEN C Af and gn IIJl g then there is a subsequence


{gnkhEN such that gnk ~. g whence Ig(x)1 ~ I(x) a.e.
b) If gn ~f 11\ n,n E N, then {gn}nEN C A f . If Af is II Ill-compact
then {llgnI11}nEN is bounded whence I E L1 ([0,1],>'). 0
5.124. If 8 C [0,1], s ¢. SA2 ([0, 1J2), and

E ~f (8 x {O}) U ({O} x 8)

°
then >'2(E) = and so E E SA2 ([0, 1J2). On the other hand, if F ~f E + E
and x E E then Fx = 8 ¢. SA. Since >'*(8) > 0, Fubini's theorem implies
F ¢. SA2 ([0, 1J2). 0
5.125. For E the Cantor set Co, E +E = [0,2]' whence Co contains a
Hamel basis B. If 8 ~f UrEQ(r + B) and 8 1 ~f 8 U (-8) then >'(81 )
and 8 1 = -81 . Let 8 n+1 be 8 n + 8 n (= 8 n - 8 n ), n E N. If
= °
X ~f L abb E 8 n , ab E Q,
bEB

then x is a sum of not more than 2n - 1 members of 8 1 and hence at most


2n- 1 of the coefficients ab are not zero. Since Co + Co = [0,2], it follows
that UnEN 8 n = JR. If each 8 n is measurable then one of them, say 8 no ' has
positive Lebesgue measure: >'(8no ) > 0. Hence for some M in N,

If KEN and 2n o < K, K SUP1<k<K Irkl < 2- M , Q \ {O} ::) {rt, ... , rK} (a
K-element set) then for each K-::element subset {b 1 , ••• , bK} of B,

K
L rkbk E U \ 8 no +1 = 0,
k=l

a contradiction.
Solutions 5.126-5.128 289

Thus some Sn tt SA and if Sn1 is the first such Sn then nl > 1. Hence
Sn1-1 E SA and Sn1- 1+Sn1- 1 = Sn1 tt SA· (This result was communicated
to the writer by Harvey Diamond and Gregory Gelles.) 0
5.126. Owing to the Riesz representation theorem, each f.Ln may be re-
garded as an element Fn of (Ll ([0,1], A))* [= LOO ([0,1], A)) and a) implies
that sUPnEN IlFnlloo ~f M < 00.
Let S be the Q-span of the set of characteristic functions of open
subintervals with rational endpoints in [0, 1) . Then b) implies that the
technique of proof of the Arzela-Ascoli theorem leads to a subsequence
{FnkhEN such that limk-+oo Fnk(f) exists for each I in the countable 11111-
dense set S. If 9 E Ll ([0, 1], A) then for I in S,

IFnk(g) - Fnl(g)1 :::; IFnk(g) - Fnk(f)1 + IFnk(f) - Fnl(f)1


+ IFni (I) - Fnl (g)l·
°
If f > then for suitable I, the first and third terms of the left member
above do not exceed 3 whereupon for large k the second term does not
f

f
exceed 3· 0

5.127. a) °: :; x+ 1 1
In(x) = nIx n XE(t) dt :::; n- = 1.
n
b) If a < b then

Iln(b) - In(a)1 = n 11 lbb+! -


a+! a
I
XE(t) dt :::; 2n lb
a
XE(t) dt :::; 2n(b - a).

If al < b1 :::; a2 < ... :::; ak < bk < ak+! < ... :::; am < bm , f > 0, and
2::;;'=1 (bk - ak) < 2: then 2::;;'=1 lin (bk) - In (ak)1 < f.
C) The Fundamental Theorem of Calculus (in the context of Lebesgue
integration) implies that In ~. XE.
d) Since E C [0,1) and In(x) = °off [-~, 1], it follows that

( Iln(x) - XE(x)1 dx =
JR
/1-1 Iln(x) - XE(x)1 dx

and the dominated convergence theorem implies In 1I...!l1 XE. o


5.128. If [p,q) c [c,d) and r = 1- 1 (p),s = 1- 1 (q), then

f.L ([P, q)) ~ ( f'(x) dx = I(s) - I(r) = q - p = A ([P, q)).


Jf- 1 ([p,qj)
290 5. Measure Theory: Solutions

Since f' E £1 ([a, b], >.), and J.L coincides with>' on all intervals, J.L = >.. 0
5.129. Let I be cPo, the Cantor function, and let A be Co, the Cantor
set. 0
5.130. a) Since IE BV, there are two nonnegative monotonely increasing
functions II, h such that I = II - hand IL I~ are finite a.e. The LRN
theorem implies Ji(x) :::: li(O) + J;
II(t) dt, i = 1,2.
b) It may be assumed that 1(0) = 0 [otherwise the argument applies
to 9 ~f I - 1(0)]. If II (x) ~f J; 1f'(x)1 dx and h
~ II - I, then both II
and h are monotonely increasing absolutely continuous functions. 0
5.131. Let {{Jnkh<k<2n-l} be the systematic enumeration of the
- - nEN
intervals deleted in the construction of the Cantor set Co. If

then I is continuous on [0,1] \ Co, whence a.e. Since I is unbounded, there


is no function 9 continuous on [0,1] and such that I ~ g.
On the other hand, the function I ~ Xn ~ 1, but I is continuous
nowhere. 0
5.132. If E, FE SA' 1= XE, and 9 = XF, then

{
lR
Ay = E0
ify:::;O
if 0 < y :::; 1 and
if 1 < y
l I(x)g(x) dx = >. (E n F),

whereas
>.(F) if y ::; 0
h(y) = { >'(EnF) if 0 < y:::; 1 and
o if1<y

1 00
h(y) dy = 11 h(y) dy = >. (E n F) = l I(x)g(x) dx.

If {Enh<n<N<oo, {Fm }l<m<M are two finite sets of pairwise disjoint


measurable sets~ if 0 < al < ..-: <-aN, and 0 < b1 < ... < bM then for the
. sImp
nonnegatIve . I f'
e unctlOns I def L...n=1 anXEn an d 9 def
= ""N L...m=1 bmXF""
= ""M
lR if Y :::; 0
El U···UEN if 0 < y :::; al
E1 U ... UEN-l if al < Y :::; a2

if an-I < y ::; aN


if aN < y
Solution 5.133 291

and
~~=1 bmA (Fm) if y :::; 0
~:'::=1 bmA (En n Fm) if 0 < y :::; a1
~:.~::~ bmA (En n Fm) if a1 < y :::; a2
h(y) =

~~=1 bmA (E1 n Fm) if aN-1 < y :::; aN


o if aN < y.
Integration by parts shows 1000 h(y) dy = IR
f(x)g(x) dx.
The general case is handled via monotonely increasing sequences of
simple functions that are approximants to f and g. 0
5.133. a) If M ~f Ilglloo < 00 then If(b) - f(a)1 :::; Mlb - al.
Conversely, if JR[0,1] ':) f E Lip(1) then f E AC, whence for some
9 in L1 ([0, 1], A) if 0 :::; x < y :::; 1, f(y) - f(x) = I:
g(t) dt. If 9
is not essentially bounded, it may be assumed that for all n in N and
A(En)~fA({x: g+(x)~n}»O.
Thus nA (En) :::; IEn g+(x) dx. If a> 0 then En is contained in an open
set Un such that A(Un ) < A (En) (1 + a). As an open set in JR, Un is the
union of at most count ably many pairwise disjoint open intervals (ank' bnk):
Un = Uk (ank' bnk ).
Let K be the Lipschitz constant for f: If(y) - f(x)1 :::; Kly-xl. Then,
owing to the fact that fJ, : E 1--+ IE g+(x) dx is a measure, and fJ, «: A, for
all sufficiently small positive a,

nA (En) (1- a) :::; 1 Un


g+(x) dx = L [
k lJnk
g+(x) dx
,
= L f (bnk) - f (ank) :::; KA (Un) < KA (En) (1 + a)
k

whence n :::; K 11 + a, and a contradiction emerges as a -+ O.


-a
If f E e[0,1], the argument above may be applied to the real and
imaginary parts of f.
b) If f E Lip(l) and fn ~f f,n E N then {fn}nEN serves.
Conversely, if limn--+ oo fn(O) = f(O), limn--+ oo Var[0,1] (f - fn) = 0, and
the Lipschitz constants K (fn) form a bounded set then for x in [0,1]'

If(x) - fn(x)1 :::; var[0,1] (f - fn) + Ifn(O) - f(O)I,

whence fn ~ f. If {(am' bm)}1<m<M is a finite set of pairwise disjoint


intervals contained in [0,1], and K ,;; sUPnEN K (fn) then for a set
292 5. Measure Theory: Solutions

of pairwise disjoint intervals,

M M
L lIn (bm) - In (am)1 :::; K L Ibm - ami,
m=1 m=1

an inequality that persists as n --+ 00. D


[ Note 85.7: Absent the boundedness of the set {Kn}nEN' the
limit of the In can fail to be absolutely continuous. The cause of
the difficulty is the possibility that for a function g, var[O,I] (g) can
be small while the Lipschitz constant for 9 can be large owing to
a steep but short rise at just one point.

The reader is urged to approximate the Cantor function ¢o,


which is not absolutely continuous, by a sequence {In} nEN of con-
tinuous piecewise linear functions such that

n~oo
lim var[O 1] (fn - ¢o)
'
=0

while the Lipschitz constants K (fn) form an unbounded set.]

5.134. If Il exists then IIlII < 00. If IE C([O,l],C) \ BV([O, 1],C) then
for the putative Il, IIlI = 00. D
5.135. If a :::; b then Ilg ([a, b)) = III ([a, b)) . (f(b) + I(a)), whence
Ilg « Ill'
Ilg([a,b))
Asb!a, III () () ( ) () dll gl _.
([ a, b))=lb+la--+la+O+la=-d D
III x-a
5.136. When the Daniell construction is applied to the Riemann integral
in Coo(lR, C), among the results is Ll (lR, >.). D
5.137. The Schwarz inequality implies L2 ([0, 1], >.) C Ll ([0,1], >.) and
that for I in L2 ([0, 1], >.), II: I(x) dxl2 :::; Ib - al I: I/(x)12 dx. The mono-
tonely increasing function 9 : [0,1] 3 x 1-4 I: I/(t)12 dt serves.
Conversely, for a 9 as described,

1
_1_ [b I(X) dX
b - a ia
I.I-1- ia
b- a
[b I(Y)dyl :::; (g(b) - g(a)).
b- a

For all a off a null set (>.), as b --+ a, the limits on both sides exist whence

I/(aW :::; g'(a), 10 1


I/(x)12 dx:::; 10 1
g'(x) dx :::; g(l) - g(O) < 00

and so I E L2 ([0, 1], >.). D


Solutions 5.138-5.141 293

5.138. The Schwarz inequality implies

IIGII~ = 1111 x
g(t)dtI2 dx::; 11 (1 (1 X
12 dt)
X
19(tWdt) dx

: ; 11 xllgll~ ~ IIgll~
dx = < Ilgll~· o

5.139. By definition, 10 I(x) dx


fl 1 oc
= "3 ~ n"3
(2)n-l . If Ixl < 1

__
I-x
1 oc
=~xn,
L...J
n=O
( 1)'
x =
1-
1X)2 = ~ nx (1 _
oc
n
-
1
,

whence f I(x) dx = 3. o
1[0,1]
5.140. For a fixed Borel set B and /-tB : 5,8 (JR) 3 A I--t /-t (A x B), /-tB «: v.
Thus the LRN theorem implies there is in £1 ([0, 1], v) an IB such that for
each Borel set A, /-tB(A) = fA IB(X) dv(x). For x fixed, the function IB(X)
has the required properties. 0
5.141. 1ft, sEA and u ~f t-s then lim n--+ oc e icnu exists. Since A(A) > 0,
there is a positive a such that A - A ::> (-a, a) and so for all u in (-a, a),
limn --+ oc e icnu ~f g(u) exists, Ig(u)1 == 1, and gIC-a,a)E M.
i 00,
l
If {cn}nEN is unbounded, it may be assumed that Icnl whence
q . eicnq - e icnP
if [p, q] c (-a, a) then as n -+ 00, e'cnu du = . approaches

l
P ZCn
q
both zero (because Icnl i 00) and g(u) du (by virtue of the bounded
convergence theorem). Thus 9 ~ 0, a contradiction since Ig(u)1 == 1. Hence
for some finite M, Icnl ::; M.
If lim n --+ oc Cn does not exist, it may be assumed that for some b in
(0, ~ ), if m =I- n, then !em - Cn I 2:: b. If

. (a57l")
mm "2' 12M . (
< lui < mm 57l" )
a, 6M

then
O<d=mm
def . (ab 5b7r)
4'24M <IC m -
lui
Cn l' 2 <6'
57l"

°< . sm
d . (Cm-Cn)U)
< sm 2 <"21
294 5. Measure Theory: Solutions

whereas leiCm'U - eiCmUI = 12iei(Cm~Cn)" sin Cem ~ en) u) I~ 2 sin d, a con-


tradiction of the convergence of { eicn U } nEN. 0
5.142. For all a off a null set (A),

lim -b-1
b-+a - a a
lb I(x) dx = I(a) and lim -b-1
b->a - a a
lb I/(x)IP dx = I/(a)IP.
b#a b#a
Holder's inequality implies

_ 1_rb I/(x) Idxl


P
~ elb _ alp-1 J: I/(x)IP dx
1 b-aJa Ib-ai P

= e 1b ~ a lb I/(x)IP dxl·

Thus I/(a)IP ~ el/(a)IP and, since 0 < e < 1, 1 == o. o


5.143. There is in (0,1) an a such that

{o < x < a} '* {I/~)I ~ 2(1/'(0)1 + 1) ~ K}.


Hence
I(x) I K. 1 1.
I-~-
X2
~! if 0 < x < a, I/(x)1 ~ 11/11oe,
X2
~ ~ ~ if a ~ x ~ 1.
X2 a2

Thus r
J(O,l)
Ig(x)ldx ~K r
J(o,a) X2
d~ + 1 ~ all/lioe < 00.
a2
o
5.144. If 1 E C ([0,1], q then the two changes of variable
k r
t --+ s + -,
n
0~ k ~ n - 1 and s --+ -
n
lead to the equations

L'f
I(t)g(nt) dT(t) = I: 1
n-l

k=O n
!:±.!
n I(t)g(nt) dT(t)

=
n-l

k=O
r 1 (k)
I:.!.n J[O,l] ~ + - g(r) dr.
n n
(s5.1O)

For large nand r in [0, 1], ~ is near zero and thus


n
Solutions 5.145-5.147 295

is small, whence the right member of (s5.1O) is near

{ (I:
i[O,l] k=O
I (~)
n n
.!.) g(r) dr ~ { ({ I(p) dP)
i[O,l] i[O,l]
g(r) dr.

If IE £1([0,1]'>..) there is in C([O,l],C) an h that is lillI-near to


I· 0
5.145. It suffices to prove the result for a singleton set {f}. Let {En} nEN
be a sequence of measurable sets such that J1, (En) < 2-(n+1). It follows
that Un>m En ~f Fm :::) Fm+b J1, (Fm) :::; 2- m , and gm ~ XFm III! 0 a.e.
Consequ-;;ntly, IE""
II(x)1 dJ1,(x) :::; Ix
gm(X) dJ1,(x) ! o. 0
[ Note 85.8: The conclusion in 5.145 may be interpreted as
saying that if I E £ 1 (X, J1, ), then the map

v: S 3 E f-+ Ie I(x) dJ1,(x) ~f veE)


is a complex measure. J

5.146. Since gn(x) ~f max{!t(x), ... ,ln(x)} is a monotonely increasing


sequence of integrable functions, 9 ~f limn---+ oo gn E £1 (X, J1,). Since In:::; 9
and In ~. 0, limn -+ oo Ix
In (x) dJ1,(x) = o. 0

5.147. a) Fubini's theorem and the equation.!. =


x
roo e- xt dt (if x > 0)
io
yields lR Si:X dx = 1 (l
00 R
e-xtsinxdx) dt. Repeated integration by
parts and the bounded convergence theorem lead to

{R sin x roo dt 7r
J~ooio ~dx= io 1+t2 =2·

b) If I(x) sinx If
={ ~ . x ¥= ~ then - f" ~ 0in [2n7r, ( 2n + - 1)]7r,
1 otherWIse 2
n E N, i.e., -lis convex there, whence I(x) = II(x)1 ~ ~ (1 - 2:7r) ~ o.
Thus

1° 00
II(x)ldx ~ -
2
2: 1(2n+!)7r (
00

7r n=l 2n7r
2n7r)
1- -
X
dx

~ ~ (1 - 4n In (1+ 4~) ) . (s5.11)


296 5. Measure Theory: Solutions

1) (_1)k+1
Since 1 - 4n In ( 1 + -4
n = 1- E~l
- k (4n )k - l' it follows that if n 2:: 2,
the nth term in (s5.11) exceeds ~
8n
- ( 3)2 whence the last member in
4n
(s5.11) diverges. 0

5.2. Probability Theory

5.148. If A and B are independent then

P (A n (0 \ B)) = P(A n 0) - P(A n B) = P(A)(l- P(B))


whence A and 0 \ B are independent and conversely. For the case of n sets,
mathematical induction serves. 0
5.149. a) If E is a Borel set in JR and B E S then

if{O,l}cJR\E
if {I} c E and {O} ct. E
if {O} c E and {I} ct. E .
if {O, I} c E

If {En}l<n<N c S,8 (JR), {Fnh<n<N C S, then 5.148 may be applied to


n1~n~N XF~ (En). - -
b) If {Aj}1~j9 are Borel sets in JR then h-;l (Aj) ~f Cj c 0 and
wE C j iff

(lj1 (w), ... , /iK j (w)) E g-;l (Aj) ~f Bj (a Borel subset of JRKj) .

If each gj is a linear combination of products of characteristic functions of


pairwise disjoint measurable sets, the result follows from the independence
of S. Any gj is the limit of a sequence {gjn} nEN of functions of the type
just described. Thus if

then p(nJ=lCjn) = nJ=lP(Cjn). As n -+ 00, limn_ocCjn = Cj and


the equation remains valid when each Cjn is replaced by Cj . 0
5.150. a) If P(A) = 0 then p(AnB) = 0 = P(A)P(B). If P(A) = 1 then
P(A n B) = P(B) = P(A)P(B).
b) Since f (w) is constant a.e., for every x, P (1-1 (x)) is either one or
zero and a) is applicable. 0
5.151. The formula (5.7) is valid if f and 9 are characteristic functions of
measurable sets because XAXB = XAnB·
Solutions 5.152-5.153 297

If I ~f L::'1 aiXA; and 9 ~f L:;=1 bjXBj are independent, it may be


assumed that the Ai resp. B j are pairwise disjoint and that the ai resp. bj
are pairwise different. Then every pair {Ai, B j } is independent and

= ~ aibj
i,j=l
1 fI
XA;(w)dP(w) 1 fI
XBj(w)dP(w)

= In I(w) dP(w) In g(w) dP(w).

Hence (5.7) is valid for simple functions.


When I and 9 are integrable, they are limits of sequences of simple
.
functlOns. For Fkn def= { w: 2kn :::; I ()
w < ~,- }
k+1 2n :::; k :::; 2n , n EN,

the paradigms for the functions in the sequence approximating I are

Since I and 9 are independent, the functions In and gn are independent.


Hence

In In (w)gn(w) dP(w) = In In(w) dP(w) In gn(w) dP(w),

an equation that persists as n ~ 00. o


5.152. a) It suffices to show that if 0 ¥A, BeN \ {1} then A and B
are not independent. Since P(A) = L:nEA 2- n!, P(B) = L:mEB 2- m!, and
P(A n B) = L:kEAnB 2- k!, it suffices to show that if a! + b! = c! + d! then
{a, b} = {c, d} (whence if m, n ~ 2, there is in N no p such that p! = m!+n!).
1) It may be assumed that a :::; b, c :::; d. If b < d then d! > 2b!,
d' d'
whence a! + b! < 2' + 2' = d! < c! + d!, a contradiction. Arguments based
on symmetry conclude the proof.
p! p!
2) If p! = m! + n! then as in 1), m! + n! < 2" + 2" = pI, again a
contradiction.
b) Because neither I nor 9 is constant a.e., a) applies. 0
5.153. Consider first the case where
298 5. Measure Theory: Solutions

and I(x) ~ XX[a,b](X) ~f F[a,b] (x). If 9 is not constant a.e. there is


in [0,1J a Borel set A such that °
< ,\ (9- 1 (A)) < 1. It follows that
[x - c, x + cJ = 1-1 ([x - c, x + cD and, since I and 9 are independent,

,\ (9- 1 (A) n [x - c,X + cD =,\ (9- 1(A)) .


2c

The metric density theorem implies that the left member above approaches
zero or one for all x off a null set. The right member is neither zero nor
one, a contradiction: 9 must be constant a.e.
For the general case, k(x) ~f 1-1 (x) . Xf([a,b]) (x) is well-defined. If
h ~f kol then h = F[a,b] and for any Borel set A, h- 1(A) = 1-1 (k- 1(A)).
Since I and 9 are independent, 5.150b) implies hand 9 are independent
whence the conclusion of the preceding paragraph applies. 0
5.154. a) The set T ~f {9"Y ~f 1'"( - In I'"((w) dP(w)} \ {O} is inde-
,"(Er
pendent, span(S) = span(Tl:J{l}), and fn9'"((w)dP(w) == 0.
If #(T) = 1 then dim (S) :s; 2. That trivial case aside, let P be the set
of all possible products of at least two elements of T:

P ~f {9,"(1 ..... 9'"(n : 2:S; n < 00,9,"(; =1= 9'"(j if i =1= j } .

Then, according to 5.151, if P ~f n~=1 9'"(k E P and h E T,

K
[ h(w)p(w) dP(w) = II [ 9'"(k (w) dP(w) [ h(w) dP(w) = 0.
1n k=11n 1n
b) If 9'"( E T then P9,"( is either an element of P (if all the factors of
P are different from 9'"() or 9'"( is a factor , say 9'"(1' of p, in which case
P9,"( = 9;1 n~=2 9'"(k· In either case, P9'"( is the product of pairwise different
independent functions and so I n P9,"((w)dP(w) = 0, i.e., Pc T.L. Since
InP(w)dP(w) == 0, Pc S.L. Because T is independent, {O} ¥P and if r
is infinite, so is P. 0
5.155. a) The range of ro is {a, 1} and if n > °the range of Tn is {a, ±1}.
It follows that

°1 if {a, ±1} n A =
if {a, ±1} n A = {1}
0
2
,\ (r;I(A)) =
1
- if{0,±1}nA={-1}
2
1 if {a, ±1} n A ::J {±1}.
Solution 5.156 299

Direct calculation shows that the rn are independent and indeed, e.g., if
0< n1 < n2 < ... < nk, that). (r;;-/(l) n r~l(l) n··· n r;;-k1(1)) = 2- k.
b) The function Fn(x, t) ~f n::.=o
(1 + rm(x)rm(t)) is a sum of terms
of the form Wp(x)Wp(t). In other words, Fn(x,') may be regarded as a
one-parameter fanlily of elements in some finite-dimensional span of some

1) [ ] ( )__
of the Walsh functions. Mathematical induction implies that when x E
k+
[ 2kn ' ~ n+1
C 0,1, Fn x,t -- 2 X[~,W) and

1 [0,1]
f(t)Fn(x, t) dt = 2n+1 j w f(t) dt.
~
2 (s5.12)

k;
If f E (span (W))1- then for all k, n in N, the right member in (s5.12) is zero.
Thus for every dyadic subinterval J ~f [2:' 1) of [0,1], fJ f(t) = 0,
whence f(t) == 0, i.e., (span (W))1- = {O}. D
5.156. If :E:=1 P (An) < 00 then for each positive E, there is an n(E) such
that P (U~n(E) An) < E. Hence

(s5.13)

[The implication (s5.13) obtains even if S is not independent.]


Assume the implication

(s5.14)

is valid. Then the preceding paragraph shows

(s5.15)

is also valid. Thus (s5.14) implies the validity of

(s5.16)

whence, owing to (s5.15), (s5.16) yields

(s5.17)
300 5. Measure Theory: Solutions

Thus all conclusions hinge on the truth of (s5.14).


Note that

nU
00 00

limn ...... 00 An = An
n=l m=n
1 - P (ITilln ...... ooAn) = P (0 \ limn...... ooAn)

= nl!..~ P(.on (0 \ Am))

= n-+-oo
lim lim P
M-+oo
(n M
m=n
(0 \ Am)) . (s5.18)

Since S is independent,

lim P
n-+oo
(n M
m=n
(0 \ Am)) = lim lim rrM P (0 \ Am)
n--too M--+oo
m=n
M
= n--+oo
lim lim rr (1 - P (Am)).
M-+oo
m=n

However, if limM ...... oo I1~=n (1 - II (Am)) exists, it does not exceed one and
it is different from zero iff ~:=n P (Am) < 00. In the current circum-
stances, (s5.18) implies the validity of (s5.14). 0

5.3. Ergodic Theory

5.157. Since E E S, owing to the assumptions about T, it follows that


T- 1 (E) E Sand J.l(E) = J.l (T (T-l(E))) = J.l (T- 1 (E)). 0
5.158. If Am ~f {x : maxn~m sn(x) > O} then

Am C Am+1 ,m E N, U Am = A. (s5.19)
mEN

For each x in X the Riesz lemma (cf. 3.1) is applicable to the set

Let D(x) be the set of distinguished indices. Then

nED{x) nED{x)
Osn~N-l O~n~N-l
Solutions 5.159-5.160 301

Hence if Bn ~f {x : n E D(x)}, 0:::; n:::; N -1, then

N-l
L
n=O
1 Bn
f (Tn(x)) dl-£(x) > O.

Since T(E) =E and

Bn = {x: max
n-l<p:O:;N-l
(f (Tn(x)) + ... + f (TP(x))) > o}
= T-(n-l) AN-(n-l)

it follows that

lnnE f (Tn(x)) dl-£(x) = iN-(n-llnE f(x) dl-£(x)

0:::;
N-l
L
n=O
!
A N _(n_1)nE
f(x) dl-£(x) =
N+l
L
k=2
!
AknE
f(x) dl-£(x).

The relations noted in (s5.19) imply

Hence 0 :::; 'L.f.=o Ik -+ r f(x) dl-£(x) as N -+ 00. o


+1 JAnE
5.159. The results in 5.158 apply to the situation in which f- a replaces
f and -Sn - a replaces -
Sn
. 0
n n
5.160. a) The equations

F(T( X )) -- -1' f(T(x)) + ... + f (TN (x))


ImN--+oo N
_ -1"-
- ImN--+oo
(NN+ 1) . N + 1
SN+1(X) _ f(x)
N
= F(x)
and analogous equations for F lead to the conclusion.
b) Since F :::; F,

{x : F(x) < F(x)} c U {x F(x) < r < S < F(x) } ~ U Crs'


r,sEIQi r,sEIQi
302 5. Measure Theory: Solutions

The argument in a) shows each C rB is T-invariant, i.e., T{CrB ) = CrB.


Furthermore, CrB = C rB nAB since otherwise, sUPnEN Sn{x) :s: S and then
n
limn..... oo sn{x) :s: s, a contradiction. Hence from 5.159 it follows that
n

If - f replaces f above, it follows that fe rs f{x) dp,{x) :s: Tp, (CrB ) whence
p, (CrB ) = 0 and F{x) ~ F{x).
c) For every E in 5,

Since fELl (X, p,), if € > 0 and p,{E) is small then p, (T-k{E)) is also small
and each term averaged in the rightmost member is less than €. It follows
that the average itself is small and that {Sn} is uniformly integrable.
n nEN
o
5.161. ·
Smcea.e.
- Sn
---> F and SInce
· - {Sn}
IS unl10rm
.£ Iy m .
. t egrable,
n n nEN
Egorov's theorem implies that for each positive €, there is in 5 an E. such
that p, (E.) < € and Sn ~ F on X \ E •. Hence if € is small and both m and
n
n are large then both

[ Ism{x) - sn{x) I dp,{x) and [ Ism{x) - sn{x) I dp,{x)


JE. m n JX\E. m n

are small. Hence {Sn}


n nEN
is a II Ill-Cauchy sequence and for some g in
L l ( X,p, ) , -Sn II--->IiI g and g =. F. T hus
n

as n ---> 00. o
[ Note 85.9: The approach using 3.1 in 5.158 - 5.161 is that
of F. Riesz. He used 3.2 to prove the differentiability a.e. of a
monotone function in ]RIR. J
Solution 5.162 303

5.162. For generality, assume 5) is a Hilbert space and U : 5) 1-+ 5) is


unitary. Let M consist of all x such that L(x) exists. Then 0 E M (# 0)
and it is shown next that the subspace M is II II-closed in 5).
II II .
If M :J {xn}nEN and Xn -+ x then for m m N,

L~=o un(x) _ L;;=o Un(x) I


I M+1 N+1

~ I L~-o ~m;~ - xm ) I + I L;;-o ~n~~ - xm ) I

+ II L~~ U; iXm
) - L;;-J.r~n1(x m
) II.
Since U is unitary, for large m, the first and second terms in the right
member above are small and, by definition, the third term is also small,
i.e., x E M: M is II II-closed.
The following assertions and proofs lead to the conclusion.

i. Ifx = U(x) then x E M since x =


LN
ni/+1
un (x)
.
ii. If x E 5) then x - U(x) E M since

as N -+ 00.
iii. If x E M then y ~f U-1(x) EM since

L;;=O un (y) _ L;;:Ol Un- 1 (x) N


N+1 - N ·N+1·

iv. If x E M then U(x) E M since

as N -+ 00. Thus U (M) = M.


v. If x E M~ then U(x) E M~ since if y E M [= U (M)] then for some
z in M, y = U(z) and then (U(x), y) = (U(x), U(z)) = (y, z) = o.
Hence if x E M~ then x - U(x) E M n M~, whence x = U(x),
x E M n M~, x = O. In sum, M = 5). 0
304 5. Measure Theory: Solutions

5.163. For 1 : 1r E Z f-+ Z and for all z in 1r,

~~:~ 1 (zk) { I - zn if z =1= 1


n = n(l- z)
1 ifz=l
whence

lim ~~:~ 1 (zk) = {O if z =1= 1


n----+oo n 1 if z = 1 .
If 1 is a polynomial, it follows that

lim ~~:~ 1 (zk) = { 1(0) if z =1= 1 . (s5.20)


n----+oo n 1 if z = 1

Fejer's theorem implies that (s5.20) is valid for any 1 in C (1r, q and the
Riesz representation theorem shows that J1zo is a discrete measure concen-
trated at zero resp. one according as Zo =1= 1 resp. Zo = 1 and J1z o (1r) = 1.
D
6
Topological Vector Spaces

6.1. The Spaces LP (X, 1-"), 1 ~ p ~ 00

6.1. Because Bf.(f,O) is a-finite, E = limn->co En for some sets En of


finite measure. Hence

~ a.
h[ I(x) dl-"(x) = [ I(x) dJ.t(x) = lim [ I(x) dl-"(x)
h n->coh.

However, if X ~f JR., 5 ~f 2iR, I-"(E) = {O


00
if E is ~nite,
otherwIse
1== 1, and I-"(E) < 00 then IE I(x) dl-"(x) = 0 while Ix I(x) dl-"(x) = 00.
o
6.2. For (JR.,S.x,>'), if In ~ ~X[-n2,n2],n E N, the requirements are met.
o
6.3. Since Ix
(1 - In(x)) dl-"(x) = IE (1 - In(x)) dl-"(x), the dominated
convergence theorem applies. 0
6.4. If I as described exists then I ~ 0 a.e. (1-"1). If {En}nEN is a
measurable partition of X and if 1-"1 (En) =1= 0, Holder's inequality implies
(1-"2 (En)~:1 ~ [ (f(x)t dl-"l(X) and so
(l-"dEn)) lEn
~ (1-"2 (En))P < II/IIP ~f a < 00.
~ ( (E) )P-l - P
n=1 1-"1 n

Conversely, if, for every measurable partition {En}nEN of X,

then {1-"2(E) > O} =} {1-"1(E) > O} whence 1-"2 « 1-"1 and for some I in
Ll (X, 1-"), 1-"2(E) == I E I(x)dl-"l(x). If 0 < bn < bn+1 i 00 then

305
B. R. Gelbaum, Problems in Real and Complex Analysis
© Springer-Verlag New York, Inc. 1992
306 6. Topological Vector Spaces: Solutions

is a measurable partition of X and

- bnJll (E)
Jl2 (En) > n, ( (Jl2(E
(En) t >- VnJll
))p-l LV (E) ~ VnJll
n, L..J LV
n <
(E) _ a
Jll n n=l

whence IIfll~ ~ a. o
6.5. When Fn ~f {x bn ~ If(x)1 < bn+1} then E:'=l bf.Jl (Fn) is, for
appropriate choices of the bn , near IIfll~. On the other hand, Ebn+l C Eb n ,
and Fn = Ebn \ E bn +1 • Thus

L ~Jl (Fn) = L bf. (Jl (Eb n) - Jl (Ebn+l)) .


00 00

n=l n=l

For large N, E:'=N+1 bf.Jl (Fn) is small and

N N-l
L 1I;.Jl (Fn) = L (~+1 -11;.) Jl (Eb n) + biJl (EbJ - bj.Jl (EbN+l) .
n=l n=l

According to 5.15, limN_oo bj.Jl (EbN) = O. Because bl may be chosen


to be zero it follows that E:'=l (~+1 - bf.) Jl (EbJ is a good approximant
to IIfll~. The mean value theorem implies that for some ()n in (bn , bn+l ),
Io
~+l - bf. = p()~-l (b n+1 - bn ), i.e., that IIfll~ = p oo t p - l Jl (Et ) dt. 0
6.6. For some {En}nEN' Jl(En) < 00 and E",(g,O) = UnENEn.
Assume p = 1 and that IIglioo = 00. Some Eno contains pairwise
disjoint subsets Fm, mEN, such that Jl (Fm) > 0 and

m+l>sup{lg(x)1 : XEFm}>inf{lg(x)1 : XEFm}~m.

If amJl (Fm) = -
m; and h(x) = E:-l
- am sgn (g(x)) XFm (x) then

L m
[ 00 1 2
JJ Ih(x)1 dJl(x) = 2 = : and
x m=l
1
L L
[ 00 00

JJ g(x)h(x) dJl(x) ~ mamJl (Fm) = m = 00,


X m=l m=l
a contradiction.
When 1 < p < 00, if Jl (E",(g, 0)) < 00 and IE (g, 0) Ig(x)IP' dJl(x) = 00
#
then Igl is unbounded. If E > 0 and

G m ~ {x : mE~ Ig(x)1 < (m+l)E},m=0,1,2, ... ,


Solution 6.7 307

then each G m is measurable and k ~f ~:=1 mqG m ~ Igl < k + E. The


theory of extrema in the differential calculus shows that if z > 0 then
(1 + z)P'
-'--_-'-::- <_ 2P -1, whence
I

1 + zpl

Because J.t (BI_(g, 0)) < 00, ~:=1 (mE)PI J.t (G m ) ~ ~:=1 am = 00. Abel's
theorem (cf. 2.54) implies that if Am ~f ~::1 ai then

(mE)P'-1
Hence if h = ~:=1 sgn(g) Am XG", then

a contradiction. Hence if J.t (Ei- (g, 0)) < 00 then 9 E LP' (X, J.t).
If J.t (Ei-(g, 0)) = 00, the preceding discussion implies that

1 En
I def
Ig(x)IP dJ.t(x) = bn < 00.

J)
{

X
Ih(x)IP dJ.t(x) = L B'bE. < 00, J)(
00

m=1 X
g(x)h(x) dJ.t(x) =
00b
L i.
m=1
= 00,

a contradiction.
Ifp> 1, X ~ {0,1}, J.t({0}) = 00, J.t({1}) = 1, and 9 == 1 then
LP (X, J.t) = L1 (X, J.t) = LOO (X, J.t). If hELP (X, J.t) then h(O) = 0,
whence h E L1(X, J.t), and Ix
g(x)h(x) dJ.t(x) = Ix
h(x) dJ.t(x) , whereas
Ix Ig(x)IP' dJ.t(x) = 00. 0
6.7. For each finite T, 6.6 implies I~T Ig(x)IP' dx < 00. Holder's in-
equality implies that LT : LP (JR, >.) 3 h 1----+ I~T g(x)h(x) dx is a continuous
308 6. Topological Vector Spaces: Solutions

linear functional on LP (JR, A) and the general theory of £P spaces shows that
1

IILTII = (I':T Ig(x)IP' dX) 17.


For all h in £P (JR, A), limT--+coILT(h)1 < 00.
The Banach-Steinhaus theorem implies that for some finite M and all pos-
itive T, IILTII S M. 0
6.S. An informal proof can be given using approximants to integrals by
finite sums and appropriate passages to the limit.
A formal proof uses Holder's inequality and Fubini's theorem, viz.:

[ II f(x, y) dJL(x) I
P
dv(y)

P1
S [(lIf(X,y)dJL(X)I - IIf(X,y)dJL(X)I) dv(y)

S Ixy If(x, Y)III If(x, y)1 dJL(Xf-1 d(JL x v)(x, y)

SI ( [ If(x, Y)III f(x, y) dJL(Xf-1 dV(Y)) dJL(x)

[ If(x, Y)III f(x, y) dJL(X)IP-1 dv(y)

S r
(jylf(x,y)lPdv(y) ) 1.
p.
[ Ir
[jxf(x,Y)dJL(X) IPI (P-1) dv(y) 1;r
([ II P
f(x, y) dJL(x)I dV(Y)) *S I ( [ If(x, y)IP dV(Y)) *dJL(x). 0
I
6.9. In any Banach space x, II-convergence implies weak convergence
because ifx E X,x* E X* then l(x,x*)1 S IIx*lIllxll.
If Xn ~ x in £1, the Banach-Steinhaus theorem implies that for some
M, Ilxnll S M < 00. Furthermore, the substitution Yn ~f Xn - x reduces
the discussion to the case of weak convergence to O. It follows that, Xnk
denoting the kth component of x n , for each k, limn --+ co Xnk = O.
If Xnll)lO it may be assumed that IIxnl 1 ?:: € > O. Hence for some
sequences {nm}mEN and {km}mEN'
kl k2 3
Llxnkl<~, L IXnkl> 5€,n>n 1
k=1 k=k 1 +1
Solutions 6.10-6.11 309

It follows that if x~k = sgn (Xnk) , nm < n ~ nm+1, k m <k ~ km+1' then
x~ ~ {x~khEN E (£1r = £00 and l(xn,x*)1 ~ E/5,n EN, a contradiction.
D
[ Note 86.1: The reader might wish to suggest why the phrase
"sliding hump" is associated with the discussion above.]

6.10. If

o if x ~ 0

1 if 1 ~ x

and f. > 0, then for suitable constants p, q, r, sand p', q', r', s',

if x = 0
g(x) ~ [p+g,B(rx+s)]· [P' + q',B(r'x + s')] = {~ if Ixl ~ 1/2 '

o ~ g:S 1, and L g(x) dx < E.

Since 9 E Coo (:R, C), for some nonnegative gn in Coo ([0,00), C),

H ence f def
= ",,00 h'
L.m=1 gn meets t e reqUIrements. D
6.11. To each JLn there corresponds the linear functional

and the hypotheses imply r ~f {Gn}nEN is a subset of the unit ball B (0, 1)
of LOO (X, JLo), the dual space of L1 (X, JLo). However, B (0,1) is compact in
the topology a (Loo (X, JLo) ,L1 (X, JLo)) whence r contains a subsequence
weakly* convergent to some Goo in B (0, 1). The corresponding measure
JLoo is what is required. D
[ Note 86.2: A less perspicuous but more general solution of
6.11 is based on the natural embedding: X '---+ X**, valid for all
Banach spaces X.]
310 6. Topological Vector Spaces: Solutions

6.12. If f E Ll ([0, IJ, A) and f 2: 0 then [O,IJ = l:JnENEn and

to,I] f(x) dx = ~ in f(x) dx


00 00 1 1 00

2: ~)n - I)A (En) 2: n: I.: . nA (En) 2: '2 I.: nA (En) .


n=2 n=2 n=2
Conversely, if f is measurable, f 2: 0, and L:~1 nA (En) < 00 then

1[0,1]
f(x)dx:::; fnA(E n ).
n=1
o

6.13. The differentiability of f at any point other than zero follows from
the holomorphy in C \ {O} of the factors of f. The difference quotient at
zero is x sin (~) which approaches zero as x -+ O. Furthermore,

if x ¥0
otherwise

If 0 :::; x :::; ~2 then 0 :::; ~x


rr
:::; sinx :::; x, whence if x 2 > ~rr then -;. <
x
~2'
Thus

12xsin (:2) 12: 2X , rr!211 = Irr~ I,


I -~x cos (~)
x
1 :::; ~ and 1!,(x)l2: (2 _ ~) ~ > 0.7,
2 Ixl rr Ixl Ixl

whence f' 1- Ll (JR, A). 0


6.14. If f = 2X[0 !)
'3
+ Xl! a)
3' 3
+ 1.5X[a IJ 3'
then F 2: f a.e. while f is not
monotone. 0
6.15. For some sequence {In}nEN of pairwise disjoint closed intervals,
r
JJ n
If(x)1 dx ~f an> 0 whence L::=1 an < 00. It may be assumed that
~oo ! ~oo -! def
L..m=1 aJ < 00. Then .L..m=1 an 2 XJn = 9 meets the requirements. 0
6.16. For E ~f E=(f, 1) and F ~f [0, IJ \ E, if A(F) = 0 then f == XE.
If A(F) > 0 then for some m in N,

A ({ x : f(x) 2: 1 + ~ }) + A ({ x : f(x) < 1- ~ }) > O.


Solution 6.17 311

If the first summand in the left member above is positive, then

11 f(x) dx = 11 (f(x)t dx i 00,

a contradiction. Hence f(x) ::; 1 a.e., the second summand in the left
member above is positive, and fn ! 0 on F. Thus

The last summand converges to zero as n -+ 00 whence f ~ XE, a contra-


diction.
If f(x) eO
a.e. then, e.g., if f(x) = e2'1l"ix, then (f(x)t dx = O. f;
If f must be lR-valued and yet f(x) eO
a.e., then P ~ 0 whence for
some measurable E, P ~ XE. If E± ~ E= (f± , 1) then f = XE+ - XE_,
which is XE for no E. 0
6.17. It may be assumed that f is lR-valued. Thus for all real a,

l a
a +1
f(x)dx = 0

and so if bE (0,1) then

a+b+1 la+b+1 l a+1 l a+1


l f(x)dx= - f(x)dx= - f(x)dx
a+l a+b a+b a+b
a+1 la+b ) l a+b
=- ( la - a f(x)dx = a f(x)dx

bl1 a
a+b
f(x) dx = l1
b
a+l
a+b+1
f(x) dx

whence f(x) ~ f(x + 1).


For k in N, if

Nk ~ {x : f(x) = f(x + 1) = ... = f(x + k - 1) # f(x + k)}


then>. (Nk) == 0 and on 1R \ UkEN Nk ~ S, f(x) == f(x + n), n E N, i.e.,
f(x) ~ f(x + n). Off a null set N in S, f(x) = limn->oo n Jxx+~ f(t) dt. If
a E S\N and U ~f UZ=o (a+k,a+k+~) then U is open, >'(U) = 1,
a+.!.
and fu f(x) dx = 0 = n fa n f(x) dx -+ f(a) as n -+ 00. Thus f(x) ~ O.
o
312 6. Topological Vector Spaces: Solutions

6.18. If E c M n BI(f, 0) and )"(E) ~ m > 0 then it may be assumed


that 1 fJ. E and thus either I(x) == 0 on E or Ixlnl/(x)1 i 00 a.e. on
E. The latter alternative and Lebesgue's monotone convergence theorem
imply IE Ixlnl/(x)1 dx i 00, whereas 1M Ixlnl/(x)1 dx $ 1, a contradiction.
D
6.19. In every closed interval J let C Jn be a Cantor-like subset such that
)"(CJn ) > )"(J)-.!.. Then Un ~ J\CJn is open and Un = J. Furthermore,
n
as n -+ 00, ).. (Un) -+ 0 whence

1 J
I(x) dx = n--+oo
lim 1
Un
I(x) dx = n--+oo
lim (
JU n
I(x) dx =0

and so I == o. D
6.20. On the one hand F(x) ~ I: I(t) dt is absolutely continuous and
F' == I. On the other hand,

.!. {d (f(x + h) _ I(x)) dx = F(d + h) - F(d) _ F(c + h) - F(c) .


h Jc h h
Hence for all c,d off a null set, F'(d) = F'(c), i.e., I(d) = I(c), and so I is
constant a.e. D

6.21. Because 1I~loo $ 11/111. it follows that

IIU[t1- ,nloo = 11](8). (e it -1)1100 $ltI 2,

1](8)1·leit - 11 $ It1 2, 8, t E JR, and 1](8)1·1 eit t- 11 $ Itl·

Thus when It I is near zero, 1](8)1 is small, whence 1==0. D

6.22. If I(x) = 0 off [a,b] and Q(x) ~f I~oo Iq(t)ldt then, since 1/12 is in
C80 (JR, C), integration by parts is applicable and leads to the equation

L Iq(x)I·I/(xW dx = -l b
(i'(x)l(x) + l(x)1 (x)) Q(x) dx.

Because 11'1 ·111 + 1/1·111 $ 1/12 + 11'12 and Q(x) i IIqlll. it follows that

L Iq(x)I·I/(xW dx $l b
Q(x) (1/(xW + 1/'(x)1 2) dx

$ Ilqlll lb I/(x)12 + 1/'(x)12 dx,

i.e., IIqlll may serve for C q • D


Solutions 6.23-6.26 313

6.23. It suffices to prove the result when -00 < a < b < 00 and I = X[a,b]
since the result for an arbitrary I follows by appealing to simple functions
that are II Iit-approximants to I·
If A ~ {x : a~x- ~ ~ b} then T(f)(x) = XA(X). Since

A= [a-~
2'
b-Jb2+4]
2 U
[a+~
2'
b+Jb2+4]
2 '

it follows that l T(f)(x) dx = A(A) = b - a = l I(x) dx. o


6.24. The characteristic function of any finite interval [a, b] contained in
[0,1] is the II Ill-limit of bridging functions in Coo ([0,1], C). It follows that
if °~ a< b~ 1 then b~ alb I(x) dx = b~ alb g(x) dx. As b a, the --t

equation becomes I ~ g.
If I(x) ~ g(x) and if h in Coo ([0, l],lR),

11 I(x)h(x) dx = 11 g(x)h(x) dx. o

6.25. If F(x) = It I/(s)1 ds then


rt I(s)g(s) dxl ~ M11rt ~1/(s)1 ds = MF(s)l~ + Mit F~) ds
I11 S S I S

~ MF~t) + Mll/lll (1 - ~)
~ MI~lll + Mll/lll (1- ~) = MII/lll.

Division by t followed by passage to the limit as t --t 00 yields the result.


o
6.26. Because I E Ll ([0,1]' A), I is the II lit-limit of a sequence of step
functions. However, if J is an interval then {x : xn E J} is the union of
at most two intervals. Hence if s is a step function then sn(x) ~ s (xn) is
also a step function whence In EM.
Furthermore, if a E (0,1) then

r I/n(x)1 dx = 10r + 11 x~-ll/(x)1


10
l

n a
dx ~f I + II.
314 6. Topological Vector Spaces: Solutions

Because I is continuous at zero, for some positive a, I is bounded in [0, a),


whence the inequality 1-..!:. < 1 insures that I < 00. On [a, 1] the integrand
n
in II is dominated by a!;-11/(x)l. D
n
[Note s6.3: The proof above that In is measurable goes back to
first principles. The measurability of In follows also from 5.109.]

6.27. Since Jo11In(x)1 dx = J::+1I/(x)1 dx, it follows that

as N,K - 00. D
6.28. By hypothesis, I E L1 (X,J-L) and it follows that 11/1100 : : : M,
whence ingn,lg E L1 (X, J-L), n E N. It may be assumed (via the use of
subsequences as needed) that In ~. I, gn ~. 9 (whence Ingn ~. Ig)·
Since lIng - Igi ::::: 2Mlgl, IIlng - Igll1 - 0 as n - 00. Furthermore, from
IIlng - Ingnll1 ::::: M IIg - gnll1' it follows that
IIlngn - Igll1 ::::: IIlngn - ingl11 + IIlng - Igll1 - 0

as n - 00. D
6.29. For all t in [0,00), 1- e- t ::::: t, whence 1- e- fn ::::: In. D
6.30. If a) and b) obtain and b > 0, Egorov's theorem implies that AJ!.

:2'
2

contains a measurable subset B such that JA~ \B Iln(x)1 dx < n E N,


and lIn - Iml ~ 0 on B. Hence

IIln - 1m II 1 ::::: r
1R\A~
+ r
1A~ \B
+ r I/n(x) - Im(x)1 dx
1B
b b r
:::; "2 + 22 + 1B I/n(x) - Im(x)1 dx.

Because convergence is uniform on B, for large n, m, the third term in the


last line above is small whence for some 9 in L1 (JR, >..), IIln - gill - 0 as
n - 00. Since a subsequence {Ink hEN converges to 9 a.e., I == g.
Conversely, if IIIn - I II 1 - 0 as n - 00 and if f > 0 then for some
measurable set E€ and an no, if n ~ no then

r
1R\E.
I/(x)1 dx < -2f ,
1R
r I/n(x) - l(x)1 dx < 2 f2 '
Solutions 6.31-6.32 315

Furthermore, E€ contains a measurable subset A€ of finite positive


measure and such that if n 2: no, flR\A. Ifn(x)1 dx < 21:3 whence a) obtains.
Because fn IIJll f, for some finite M and all n, IIfnll1 ~ M. For
measurable Band TB : L1 (IR,A) 3 9 1--+ fB g(x) dx, TB(g) ---+ 0 as A(B)---+
O. The uniform boundedness principle implies that for some K, IITBII ~
K < 00 and ITB (lfn!)1 ~ ITB (If!)1 + K Ilfn - fill' If b > 0, there is an no
such that if n 2: no then

IIfn - fill ~ Kb , lim sup ITB (Ifn!)1


).,(B)---+O n~no
~ ).,(B)---+O
lim ITB (If!)1 + b = b.

Hence lim)"(B)-+o sUPn ITB (lfn!)1 = 0 and b) obtains. o


6.31. Because

SN(X) i f(x) if -1 < x ~ 0


ISN(X)I ~ SN( -x) i f( -x) if 0 ~ x < 1,
the dominated convergence theorem applies. o
def X
6.32. For F(K) = SUPx~K f(x)'

[ Ign(x)1 dx = [ +[ Ign(x)1 dx
J[O,l] J[O,l]n{ x : 19n(x)I~K} J[O,l]n{ x : 19n(x)I<K}

~ F(K) [ f (Ign(x)l) dx + K ~ F(K)M + K < 00,


J[O,l]n{ x : 19n(x)I~K}
whence {gn} nEN C L1 ([0,1], A). If a > 0 then for all sufficiently large K,
a
F(K) < 8(M + 1)' Hence

IIgn - gml11 ~ [
J[O,l]n{x: 19n(X)-9m(x)19K}
+ [ 2. Ign(x) - gm(x)1 dx
J[O,l]n{x: 19n(X)-9m(x)I>2K} 2
~f I + II.
Since J: f (Ign(x)!) dx ~ M,

[1 f(lgn(x)1 V Igm(x)l) dx = [ f (lgn(X)!) dx


Jo J[O,l]n{ x : 19n(x)I~19m(x)l}
+ [ f (Igm(x)!) dx
J[O,l]n{ x : 19n(x)I<19m(x)l}
~2M.
316 6. Topological Vector Spaces: Solutions

Because Ign(x) - gm(x)1 ::; 2 (lgn(x)1 V Igm(x)l) and I is monotonely in-


creasing, it follows that I Cgn(x) ~ gm(x)l) ::; I (lgn(x)1 V Igm(x) I) and

so for all n,m, 2F(K)1 Cgn(x) ~ gm(x)l) dominates the integrand in II.

On the other hand, II ::; 2· 2MF(K) < i. Owing to bounded convergence,

as n, m -+ 00, I -+ O. Hence for large n, m, IIgn - gm 111 < a. If gn II-+111 h then


a subsequence gnk ~. h whence 9 == h. 0
6.33. LEMMA. FOR ANY BANACH SPACE X AND A SEQUENCE {Xn}:::o
CONTAINED IN X, xn ~ Xo IFF FOR SOME K, IIxn II ::; K < 00 AND FOR
ALL x* IN A DENSE SUBSET D OF X*, (Xn' x*) -+ (Xo, x*) AS n -+ 00.
PROOF: If Xn ~ xo, the uniform boundedness principle implies that
for some K, IIxnll ::; K < 00.
Conversely, if y* E X*, some x* in D is near y* and the condition
IIxnll::; K < 00 implies I(xn,y*) - (xo,y*)1 is small for large n. 0
a) =* b). In the current context, for any step function s,

lim [In(t)s(t) dt = [
n->oo lIT{ lIT{ 10 (t)s(t) dt.
Because the set of step functions is dense in LP' (JR., >.), In ~ 10.
b) =* a). The general lemma just proved implies the result. 0
6.34. If {In}nEN c A and IIln - 1111 -+ 0 as n -+ 00, then a subsequence
converges a.e. to I. Hence III ~ 1 a.e. and so A is II Ill-closed.
Let In be the nth Walsh function (cf 5.156). Then A J {fn}nEN. If
Jo
E E S.e, Bessel's inequality implies l XE(x)ln(x) dx -+ 0 as -+ 00. Since n
the span offunctions of the form XE is dense in LOO([O, 1], >.) and IIlnlll ::; 1,
it follows that In ~ 0, whereas ¢. A. ° 0
6.35. Because Z is a Borel set, S E SA. Furthermore,

[II cos7r/(x)ln dx = [ + [ Icos 7r/(x) In dx,


lo 1s l[O,lJ\S

Icos7r/(x)1 = 1 iff XES, and Icos7r/(x)ln -+ 0 as n -+ 00 iff x E [0,1] \ S,


whence the bounded convergence theorem implies the result. 0
6.36. a) If IIlnlloo ::; M < 00 and h n ~ In * 9 then

Since IIg[x+y] - g[x] 111 ~ 0 as y -+ 0, it follows from the Arzela-Ascoli


theorem that {h n } nEN contains a uniformly convergent subsequence.
Solutions 6.37-6.40 317

b) For fn as described, the map [O,27r] '3 () 1---+ eiIJ permits the identifi-
cation of'][' with [O,27r). Concordantly, the discussion is reduced to consid-
eration of Fn((}) = 2cosn(},0 ~ () < 27r. Because

the Riemann-Lebesgue lemma implies h n ~ O. D


6.37. The formula f(t)
~
= -1 jn f(x)e-' x dx0t
implies
27r -n

1(t + s) - 1(t) = ~ r e_itxe-iSX - 1 f(x) dx.


s 27r l-n s

The dominated convergence theorem permits the passage s ---> 0 and shows

l' exists and 1'(t) = ~ r


l-n
itx
_ixe- f(x) dx. Similar calculations show
27r
that for all k in N, j(k) exists and j(k)(t) = 21 jn (_ix)ke-·tx f(x) dx.
0

7r -n
The Riemann-Lebesgue lemma implies the conclusion. D
6.38. For some h in Coo (JR, C),

h i= 0, hex) = h( -x), and supp(h) n supp (1) = 0.


Repeated integration by parts shows that if 9 = h then
t 2 g(t) E Co (JR, C), 9 E L1 (JR, C) nCo (JR, C), 9 = h
(g * ff= h1 = 0, 9 * f = O.
D
6.39. Because f-;-g = j. g, it follows that in the present context, j2 = [.
Since 1 E Co (JR, C) it follows that 1
= O. The uniqueness theorem for
Fourier transforms implies f ~ O. D
6.40. If 9 d~ f· XE then IIg111 = 1 whence Igl ~ 1. If, for some real
a, g(l) = eia then 1 = fIRg(x)e-i(x+a) dx = flRg(y - a)e- iy dy = g[-aj(l).
Because Ilg[-ajlll
= 1 and g[-aj 2: 0, the problem is reduced to showing
that if f 2: 0 and IIflll = 1 then 1(1) i= 1.
If 1(1) = 1 then flRf(x)cosxdx = 1. If

En ~f {x : X E E,cosx < 1- ~} ,n E N, and lin f(x)e-iXdxl < 1


318 6. Topological Vector Spaces: Solutions

then 1 En
f(x)cosxdx ~ (1-~)r f(x)dx andlEn

1 = r + r f(x)dx lRrf(x)cosxdx
lEn lR\En
=

= r + r f(x)cosxdx
lEn lR\En

~ (1-.!.)
n
1 + r f(x)dx
En lR\En

whence in f(x) dx ~ (1 - ~) in f(x) dx and f(x) == 0on En. Because


~ = (Un2:2 En) U (U {27l'k}) , f == 0, a contradiction.
kEZ D
6.41. It may be assumed that Ilflll = 1. If [(to) = eia f:. 1 then to f:. 0
and thus if b ~f !.!:.. then, in a calculation like that in 6.40, it follows that
to
- def
f[b] (to) = g(to) = 1.

Because I/gl/1 = I/fl/1 (= 1) and 9 ~ 0, the problem is reduced almost to


that in 6.40. The approach in this case is to replace as defined in 6.40 En
= { x: costox ~ 1 -;;:
by Fn def 1} . D

6.42. Because v ('Jl') = 1, Ii Xdv(x)1 ~ IT Ixl dv(x) = v ('Jl') = 1.


In the pattern of 6.40, 6.41, if i e-iaxdv(x) = 1 then instead of En
or D
En, Gn ~
- { x.. In
~e -ia X <
_ II}
-;;: serves. D

[ Note 86.4: When v above is a discrete measure its support is


a finite or countable set. The conclusion when supp(v) is a finite
set is that the v-center of gravity lies inside the unit ball of C or
on 'Jl', and in the latter case iff supp(v) is a single point.]

6.43. If {f,g} c Coo(G,C) then f * 9 E Coo(G,C) and Fubini's theorem


implies I/f*glll ~ I/flh· I/glh· If hE LP'(G,J.l) HOlder's inequality and the
translation-invariance of J.l imply

whence f * 9 E U(G, J.l) and I/f * gllp ~ IIfll1 ·IIgllp· D


Solutions 6.44-6.47 319

6.44. If hELP' (G, f.L) then Fubini's theorem and Holder's inequality
imply

Ii (nu * I(x) - I(x)) h(x) df.L(x) I: :; Ilhllp' i II/[y-I] - Illp nu(y) df.L(Y)
Iinu *I - Illp :::; i II/[y-I] - Ilip nu(y) df.L(y).

Because II/[y-I] - Ilip -t 0 as y -t e, the net U 3 U ~ Iinu *I - Illp


converges to zero. 0
6.45. Because, for all I in L1 (G, f.L), lin * I - fill -t 0, it follows that

Iini - ~Ioo = II~L .lin - 11100 o. -t o


1
fIR e- = J7r shows that Ct =
2
6.46. a) The formula X dx r=;.
V7rt
b) Choose C nso that for gtn ~ cngtX_[~,~], IIgtnl11 = 1, n E N. Thus
1
for n in N, 9 ~ n is a particular instance of nu in 6.44. If 0 <t < n - and
n'
I E L1 (JR, A) then

Ilgt * I - 1111 :::; I (gt - g~,n) * 1111 + Ilg!;,n * I - 1111


: :; Ilgt - g~,nI11 111111 + Ilg~,n * I - 1111 . (s6.1)

For large n, the second term in the right member of (s6.1) is small and if
o < t < ~,
n
the first terms is small as well. 0

6.47. As shown in 6.37, i (JR, C). The formulre there show that
E Off
Ijtk) I :::; 2nkll/l11. If, for some b, f(b) =f. 0, let the Kth order Taylor formula
for ibe
jtk)(b)(t _ b)k
L
K
k! + RK+1(t).
k=O
Then for some a between band t,

as K - t 00. Hence i is entire and because supp (i) is compact, i = o.


Thus I = o. 0
320 6. Topological Vector Spaces: Solutions

6.48. If 9 ~ 0 then Tg (L2 (JR,A)) = {O}, which is 11112-compact.


If g#:-O and I E L2 (JR, A) n L1 (JR, A) ~f V, then Tg(f) E V whence
T;(j) is defined and T;(j) = 9·1. Owing to the Plancherel theorem, the
Fourier transform confined to V is a II 112-isometry that is extendible as an
isometry to L2 (JR, A). Thus if the II 112-closure Tg (B(O, 1)) of Tg (B(O, 1))
is II 112-compact then its image A under the Fourier transform is also II 112-
compact.
However, A = {g. i : IE B (0, I)}. Because 9 #:- 0, 9 =1= 0 and,
since 9 is continuous, for some a, b, a < b and on [a, b], 191 2 ~ d2 > O. It
may be assumed that [a, b] = [0,1].
If In E C80(JR,Q and In(x) = 0 off [2-(n+1),2- n ] then {fn}nEN is
orthonormal and the Plancherel theorem implies {Tn} nEN
is orthonormal
and for some h n in L2 (JR, C), h;, = In and {h n }nEN is also orthonormal.
Consequently,

Thus neither A nor Tg (B (0, 1)) is II 112-compact. o


6.49. For x in [1,00), the Kronecker function

8x (Y) ~f X{x}(y) ELl ([1,00), /1).

If FE (L1 ([1,00), /1))* and F (8 x ) ~fax then

lax I :s: IIFII . 118x 111 = IIFII . x.


Conversely, if M < 00 and lax I :s: M . x then

F:L 1 ([1,00),/1)3/f--+ L I(x)·a x


xE[l,oo)

is in (L1 ([1,00),/1))* and 11F11:s: M. Thus F E (L1 ([1,00),/1))* iff

sup { IF ~x) I : x E [1,00) } < 00. o

[Note s6.5: In the context above, (L1 ([1,00),/1))* is a proper


subset of LOO ([1,00), /1).]
Solutions 6.50-6.52 321

6.50. If p ~ 1 and I E L1 (JR, A) then (cf. 6.43) Tf : LP (JR, A) 3 9 I---; 1* 9


maps LP (JR, A) into itself and IITf(g)llp :::; 111111' Ilglip. Thus if

9 : JR 3 Y I---; { ISin (F) I if Y #0


IYI'
o otherwise

is in LP (JR, A) then 1* 9 E LP (JR, A).


1
However, the substitution y I---; - shows that
z

Ilgll~ = 11 + {
-1 JIR\[-l,lj
Isin ziP IzI 3p-4/2 dz.
Izlp

The first integral is finite iff p > ~ and the second integral is finite iff p < 2.
Hence 1* 9 E LP (JR, A) for all I in L1 (JR, A) iff ~ < p < 2. D
[ Note 86.6: Actually Cohen [Coh!] showed that if B is a
Banach algebra containing a bounded approximate identity then
each x in B may be written as a product xy of elements of B.
From this [HeR] it follows that if V is a Banach space that is
also a B-module then B . V is a closed subspace of V. That
general result implies that if G is a locally compact group then
L1 (G, p,) * V (G, p,) = LP (G, p,).

It is a consequence of the results just cited that if I E V(G, p,)


and n is an approximate identity in L1(G, p,) then n *I IIJr I.]
6.51. Two continuous functions must differ on a nonempty open set,
hence on a set of positive measure, whence if I and 9 are continuous and
I ~ 9 then I = g.
On the other hand, for 0: in (0,1), let S in LP ([0, 1], A) be the equiv-
alence class containing the characteristic function of a Cantor-like set Co..
If 9 is continuous and 9 E S then g(x) = 0 on a dense set whence 9 = 0
whereas Ilgllp = 0: > O. D

6.52. For R ~f {(u,v) : u < v < __u_, -1 < U <


u+1
o} the map

T: R 3 (u,v) I---; T«u,v)) def (v


= ( u + l)(v + 1), V;-+1 /u+T)
322 6. Topological Vector Spaces: Solutions

is a bijection between Rand (0,1)2. Note that for (x, y) in (0,1)2,


x
u = xy - 1, v =- - l.
y

The absolute value of the Jacobian determinant for T is 12(1 ~ v) I·


Thus the problem is to find the values of p in [1,00) and for which

The integral above is finite iff

lim
tl~1
i t
f (11~u-1
u
1
lul p (l + v)
dv ) du

_ 1·
-
ETO
1m
if t
Iln(l +u)1 d
IuPI u < 00. (s6.2)
tL-1

If -1 < u < 0, Iln(l + u)1 ::; ~:=1 I~n. Hence if p = 1, the right
2
member of (s6.2) does not exceed ~ . Because the integrand is nonnegative,
I E L1 ([0, IF, A2).
. Iln(l+u)1 > Iln(l+f)1
On the other hand, If 1 < p then on (t, f), lul p lul p
whence the right member of (s6.2) is unbounded as f i 0. D
6.53. If I E Coo (G, q then

III - I[nlil p ::; III - l[nlIL)Q . [A (supp(f))] t

which approaches zero as n ---+ e. Because Coo(G,C) is dense in LP(G,p,),


it follows that for any g in £P(G, p,), Ilg - g[nlil p ---+ as n ---+ e.
In particular,
°
D

6.54. According to Solution 6.53 for some strictly monotonely decreas-


ing null sequence {an}nEN' if Ibnl < an then 11/[bnl - III: < 2- n . Thus
~:=1 11/[bnl - III: < 00. The monotone convergence theorem implies

L
00

II (x + bn ) - l(xW < 00 a.e. D


n=1
Solutions 6.55-6.57 323

6.55. If 0 < An(A) . An(B) < 00 and I ~f XA, 9 ~f XB, the result is
directly verifiable.
If I ~ XA, the Bk are pairwise disjoint, 0 < An(A)'Il~=l An (Bk ) < 00,
= 0 :::; b < b < ... < bK, and 9 def
b def
o 1 2 L..tk=l bkXBk t h en
= "i;"'K
ift>b+K

F(t) ~f Lt I(x) dx = { ~('~p ~n (A n Bkl

L~=l An (A n B k )
Hence, by Abel summation,

The general case can be handled via successive approximations by


monotonely increasing sequences of simple functions in which first, I = XA
and 9 E LP' (JR n , An), second, I is a simple function and 9 E LP' (JR n , An),
and third, I E LP (JR n , An) and 9 E LP' (JR n , An). 0
6.56. If P ~ C[x] is viewed as a subset of G([O, 1], C), the map
K K
T: P 3 Lbkt k 1-+ Lakbk E C
n=l k=l
is, by hypothesis, a bounded linear functional defined on a dense subset of
LP ([0, 1], A). Hence T may be extended without increasing its norm to an
element of (£P ([0, 1], A))*, i.e., of £P' ([0,1], A)). 0
6.57. a) An extreme point must lie on 8B(O, 1) because if 1I/IIp < 1 and

Ilgllp = 1 then I
__ { (1 -
1 1
°
11/11p) + III lip 1I~lp if I i=
.
°
2g + 2(-g) if 1= 0
When p > 1, I E LP(X,I-£), 1I/IIp = 1, g, h E B(D, 1), I, g, h are three
points in B(O, 1), a E (0,1), and 1= ag + (1 - a)h then

and, according to the criterion for equality in the Minkowski inequality, for
constants A, B, not both zero, Ag + Bh ~ O. It follows that

(aB - A(l - a))h = -AI if A i= 0 and ((1 - a)A - Ba)g = -BI if B i= O.


324 6. Topological Vector Spaces: Solutions

If A =I- 0, f is a multiple of h. Thus Ilhll p = 1 and for some (J in~, f = ei8 h,


IIglip = 1, and a = la + ei8 - 11. If cos (J =I- 1 then a = 1, a contradiction,
and if cos (J = 1 then e i8 = 1, f = h, and yet another contradiction. If
B =I- 0, a similar argument leads to a similar string of contradictions. In
c
sum: p = 8B(O, 1).
b) When p = 1, (X, 5, {l) is nonatomic, f E LMX, {l), and IIflll = 1,
it may be assumed that, e.g., for some positive a, A ~f {x : f(x) ~ a}
has finite positive measure and thus may be decomposed into a union of
disjoint measurable sets of positive measure: A = A 1 lJA2. For some b,e
such that 0 < b, e < a and b{l (At) = elL (A2) and if

f + b on Al { f - b on Al
gl ={ f - e on A2 , g2 = f + e on A2 ,
f elsewhere f elsewhere

then gl =I- g2, ~91 + ~92 = f, IIg1/l1 = IIg2111 = 1, and f is not an extreme
point.
When p = 1, (X, 5, {l) is nonatomic, and f is an extreme point then
If I is also an extreme point. [PROOF: If g, hE B(O, 1), a is in (0,1), and

If I = ag + (1 - a)h

then f = a . sgn(f)g + (1 - a) . sgn(f)h, sgn(f)g ~ sgn(f)h, and 9 ~ h on


E ~f E",,(f, 0). However,

1 = IIflll = lelf(x)1 d{l(x) = a Ie g(x) d{l(x) + (1 - a) Ie h(x) d{l(x)

Ie g(x) d{l(x) = Ie h(x) d{l(x) = 1

1 ~ IIglll ~ lie g(x) d{l(x) I = 1, 1 ~ IIhlll ~ lie h(x) d{l(x) I = 1,


whence 9 ~ h ~ 0 off E, i.e., 9 ~ h, a contradiction.]
In sum, if f is an extreme point in Ll(X,{l), If I is an extreme point in

{0
L~(X, {l), an impossibility.
Hence [; _ if p = 1 and (X, 5, {l) is nonatomic
p- 8B(O,I) ifp> 1 .
o
[ Note 86.7: If X contains atoms C1 can fail to be empty, e.g.,
in £~ ({O, I} ), C1 consists of four points.]

6.58. If A = B then A 1- = B1-. Conversely, if A 1- = B1-, it follows that


(A1-) 1- = (B1-)1-. Because A C (A1-)1-, if f E (A1-) 1- \ A then for some 9
in A1-, Ix g(x)f(x) d{l(x) = 1 whereas Ix f(x)g(x) d{l(x) = O. 0
Solution 6.59 325

6.59. a ) The functionf :[ ) :7 t f-+ ( 1 + t P) ach'!eves Its


0,00 . mInIma
.. I
1+ t P
value 2I - p « 1) when t = 1 and is strictly monotonely decreasing resp.
increasing in [0,1) resp. (1,00).
If either x or Y is 0, the desired conclusion is automatic. If (6.1) is
valid when °< IIYII ~ II xii ~ 1 and if Z ~f II~II then

If (6.1) is false then for some positive E and vectors Xn, Yn,

Ilxnll= 1, IIYnl1 ~ 1, Ilxn - Ynll 2: E


lim II ~ (xn + Yn) liP = l. (s6.3)
n--+oo ~ (1lxnll P + IIYnII P )
If, in (s6.3), IIYnl1 ~ q < 1 then, owing to the triangle inequality and the
strict monotonicity of f in [0,1), for some p in (0,1),

contradicting (6.1).
Thus it may be assumed that IIYnl1 ---t 1 as n ---t 00 and hence if
Yn II
Zn = IIYnll then Zn - Yn
def
°
II ---t as n ---t 00. Consequently for large n,

Ilxn - znll 2: E and, to boot, limn--+ oo II (X n ; Yn) II = 1, contradicting the


uniform convexity of (Y, II II)·
Homogeneity considerations like those treated above show that the
conclusion in a) is equivalent to the following statement: If x, Y E Y and
°
max{llxlI, Ilyll} > then

b) In Hilbert space 5), e.g., in C, the theorem of Apollonius, i.e., the


parallelogram law,
326 6. Topological Vector Spaces: Solutions

is valid. Thus for X,y in 5), if Ilxll, Ilyll ~ 1, E E (0,2], and Ilx - yli ~ E then
x+yll
Il -2- ~ V~
1- "4 def
= 1- 8(E). Thus 5), e.g., C, IS. umformly .
convex.
For f,g in B(O, 1) if Ilf - gllp ~ E > 0, let E be

{t : If(t) - g(tW ~ ~ (If(t)IP + Ig(t)IP) ( ~ ~ max {If(t)IP, Ig(t)IP}) }.


Owing to a), (s6.4) applies for each t in E. Hence on E,

I(f(t) +2 g(t)) I
P
<
-
(1- 8P (max{lf(t)I,lg(t)l}
If(t) - g(t)1 )) ((If(t)IP + Ig(t)IP))
2

~ EP.
and so (
JX\E If(t) - g(t)IP dJ.L(t)
2
If

max {llf(t)IP dJ.L(t) , llg(t)IP dJ.L(t)} < 2:: 1

then llf(t) - g(t)IP dJ.L(t) < ~, in contradiction of the hypothesis, i.e.,

max {llf(t)IP dJ.L(t) , llg(t)IP dJ.L(t)} ~ 2:: 1.

Another application of (s6.4) as applied in a) reveals

Ix {~(lf(t)IP + Ig(t)IP) - (~If(t) + g(t)I)P} dJ.L(t)


~ l {~(lf(t)IP + Ig(t)IP) - (~If(t) + g(t)IY} dJ.L(t)
> { 8 ( If(t) - g(t)1 ) ((If(t)IP + Ig(t)IP)) dJ.L(t)
- JE max{lf(t)I,lg(t)l}
p 2

~ 8p (m~~j(DI~~~{~)I}) ~ max {llf(t)IP dJ.L(t) , llg(t)IP dJ.L(t)}


>8 ( If(t) - g(t)1 ) ~
- p max{lf(t)I,lg(t)l} 2P+2·

Hence if f, g E B(O, 1) and Ilf - gllp ~ Ethen

6.60. If Ilfoli p = 0, the conclusion is automatic. If Ilfoli p > 0, it may


b e assumed t h at £or a 11 n, II!n II p >. If gn ~f ~.
- °
Ilfollpfn th
Ilfnllp en gn
f
JO,

Ilgnllp = lifo lip·


Solutions 6.61-6.63 327

Egorov's theorem implies that for m in N, and some Em, gn ~ 10 on


1
Em and J-t (Em) > J-t(X) - -. However, because
m

(r
1

ilgn - lollp = + ( Ign(t) - 10(tW dJ-t(t») P


JEm JX\Em
1

~ (fem Ign(t) - 10(tW dJ-t(t») P


1

+2 ( ( I/o(tW dJ-t(t») P , (s6.5)


JX\Em

it follows that for large m the second term in the right member of (s6.5) is
small and then for large n the first term is small. Thus gn IIJr 10'
However,

11/0 lip . II In - lollp ~ 1III/oilp In - 10 Il/nllpll p + 11/0 II In lip - 11/0 lip lollp

= Il/nllp ·llgn - lollp + Il/ollp '1ll/nllp -ll/ollpl· 0

6.61. For a subsequence {ENkhEN' XEnk ~. I· 0

6.62. Because 11/11~t~ ~f an+1 ~ 11/1100an , limn->oo an+1 ~ 11/1100 while


1
an
Holder's inequality implies 1I/IIn ~ J-t(X)n(n+l) II/lIn+!' Hence

_ l'1m ~
1·1mn -> 00 an+1 > 1 = 11I11 00' o
an n->oo J-t(X):n

6.63. If L1 (X, J-t) = LOO (X, J-t) and inf {J-t(E) : E E S, J-t(E) > O} =0
for some sequence {En}nEN in S and a sequence {kn}nEN in N,

If Fn ~f U:=n Em then Fn :::> Fn+1' J-t (Fn) > 0, J-t (Fn) 1 O. All the pairwise
disjoint sets G n ~ Fn \ Fn+! have positive measure. Hence there emerges
the contradiction 2::=1 2nXG n E L1 (X, J-t) \ Loo (X, J-t). Thus

inf {J-t(E) : E E S, J-t(E) > O} > O. (s6.6)

If there is no atom in X then (s6.6) is denied.


328 6. Topological Vector Spaces: Solutions

If {An} nEN is an infinite sequence of pairwise different atoms then


either Z:::'=1 /L (An) = 00, in which case every nonzero constant function is
in Loo (X, /L) \ Ll (X, /L), a contradiction, or Z:::'=1 /L (An) < 00, in which
case for some sequence {nkhEN'

yet another contradiction.


Hence for some N in N, the finitely many atoms in X may be listed:
AI"'" AN. Since (s6.6) implies /L (X \ U:=1 An) = 0, if 1 E Ll (X, /L)
then °
1 is constant a.e. on each An, 1 ~ off U:=1 An, and
N
1= LI(An)XAn , i.e., dim(Ll(X,/L)) =N.
n=1

Conversely, if Ll (X, /L) is finite-dimensional, there is a (necessarily fi-


nite) maximal linearly independent set {XEn} :=1 of characteristic functions
of measurable sets of positive measure. Then X\ U:=1 En is a null set and,
modulo a null set, X is the union of the pairwise disjoint sets

U Em, 1
n-l
En =
def En \ ~ n ~ N.
m=1

°
If some /L (En) = then XEn E span ( {Am };;'-==\).
Furthermore, each En is
an atom, since otherwise there are least N + 1 linearly independent char-
acteristic functions of nonnull sets. It follows that Ll (X, /L) = Loo (X, /L).
o
6.64. The linearity of Tf follows from its form as an integral. Furthermore,
ITf(g)1 ~ 1llll1llglloo. 0

6.65. If 1 E L 1 (JR, A) and -k(z) def (z) -


= k Vi ,I(v) def
= 1 Vi (z) .
then, m the
notation of 6.46,

Hence if k is regarded as an element Tk in (Ll (JR, A))*, i.e., Loo (JR, A),
then for all t in (0,00),
(s6.7)
Solutions 6.66-6.68 329

Because {gthE(O,oo) is an approximate identity, as t - 0, (s6.7) becomes


Tk (1) = 0, i.e. Tk is the zero functional. Hence k ~ °and so k ~ 0. 0
6.66. a) Because f *9 = 9 * f it suffices to prove S(f * g) = S(f) * g. By
hypothesis,

S(f) * 9 = iS(f) (r1x) get) dr(t) = i (S(f))[t-1] (x)g(t) dr(t)

= is (I[t- 1 ]) (x)g(t) dr(t).

The last integral is the limit of approximating sums of the form

which converge to S (i f (r1x) get) dr(t)) = S(f * g).


b) Because §(J)n = i S(f)(t)t- n dr(t) , if gn(t) ~f tn, then, owing to
the translation-invariance of r,

§(J)n = x-nS(f) * gn(x) = i §(J)n dr(x) = i x-nS(f) * gn(x) dr(x)

= i x- n (f * S (gn)) (x) dr(x) = fnS(g;,)n ~f anfn. 0

6.67. For any /1, if (L oo (X,/1))* = L 1(X,/1) then L 1(X,/1) is reflexive,


B(O, 1) is (J' (L1(X, /1), (L1(X, /1))*)-compact, and the Krein-Milman the-
orem implies that the weak closure of the convex hull of £1 is B(O, 1). In
the present context, L1(X, /1) = L1 ([0,1],),) and £1 = 0 (cf. 6.57) whence
L1([0, 1],),) is not reflexive and so (Loo(X, /1))* (X, /1). "1-£1 0
An alternative solution below makes no use of the Krein-Milman the-
orem.
The map T : C ([0, 1], C) :3 f f-+ f(O) is a functional of norm one and,
via the Hahn-Banach theorem, can be extended to a f in (L oo ([0, 1],),))*.
If (Loo ([0, IJ, ),))* = L1 ([0,1]')') then for some h in L1 ([0,1],),) and all f
in C ([0,1], C), f(O) = ( h(x)f(x) dx. Hence ( hex) dx = 0, on every
J~~ J~~
closed subinterval [a, bJ of [0,1], whence h ~ 0. On the other hand, if f == 1
then f(O) = f[o,1] 1 . hex) dx = 1, a contradiction. 0
6.68. Let F be the equivalence class determined by f. If {fdf=1 C F,
Ek ~f {x : fk(X) = f(x)}, and E ~f nf=1 E k , then )'(E) = 1 and on E,
330 6. Topological Vector Spaces: Solutions

Ik(x) = I(x), 1 ::::; k ::::; K. If {cPk}~=1 is a partition of unity (subordinate


to some open cover {Nd~=1 of [0,1]) then on E,
K K
L cPk(x)lk(x) = I(x) L cPk(X) = I(x),
k=1 k=1
i.e., E~=I cPk!k E F. (The same kind of argument shows F is convex.)
The map
h [] vx if t = x
x: 0,1 3 t f-+ { 9x(t) if t =I- x
is in F and hx(t) is continuous. For n in N and x in [0,1] let x lie in a
1
neighborhood Nn(x) such that OSCNn(x) (h x ) < ~. Because [0,1] is com-
..
pact, for some n m N, If Jnk def(k lk- + -1) ,k = 0,1, ... , n, then
= - - -,n n n n
OSCJnk (h~) < ~.
Let {cPnk}~=o be a partition of unity subordinate to {Jnd~=o and
let Hn be E~=o cPnkhl<.. Because each hx is continuous and each cPnk is
continuous, each Hn isncontinuous.
It is shown next that: a) each x in [0,1] is contained in a neighborhood
N(x) such that OSCN(x) (Hn) <~; b) for all m,n in N, all x in [0,1], every
n
neighborhood U(x) of x, and some Y in U(x), Hm(Y) = Hn(x).
a) If x E [0,1] and Sex) ~ {k : x E Jnk} then # (S(x)) ::::; 2 and
x lies in a neighborhood N(x) such that N(x) n (UkfiS(X) Jnkf) = 0. If
YI, Y2 E N(x) then

Hn (Yi) = L cPnk (Yi) h~ (Yi) ,i = 1,2


kES(x)
IHn (YI) - Hn (Y2)1::::; L cPnk (YI) Ih~ (yd - h~ (Y2)1
kES(x)
+ L
kES(x)
cPnk (Yd cPnk (Y2) ·Ih~ (Y2)1·

If A ~ sUPkES(x) Ih~ (Y2)1 = 0, the last summand above is zero. If A > °


1
and for some positive b, diam(E) < b then sUPk OSCE (cPnk) < nA. It may
1 1
be assumed that diam (N(x)) < b, whence IHn (Yd - Hn (Y2)1 < - + -.
n n
b) If Gn ~f {x : Hn(x) = I(x)} then A (nnEN Gn ) = 1 whence it
follows that G ~f {x : Hn(x) = I(x), n EN} is dense in [0,1]. For a
given U(x), choose Y in the nonempty set G n U(x).
Solutions 6.69-6.70 331

For x in [0,1] and a positive to,

IHm(x) - Hn(x)1 ~ IHm(x) - Hm(Y)1


+ IHm(Y) - Hn(Y)1 + IHn(Y) - Hn(x)l·

Hence if !, ~ < :. and if U(x) is such that


m n 3
to
OSCU(x) (Hm) ,0SCU(x) (Hn) < 3"

then for some Y in U(x), Hm(Y) = Hn(Y). It follows that

ifm,n >~. Thus {Hn}nEN is a I\l\oo-Cauchy sequence and so for some g,


Hm ~ g, 9 is continuous and f ~ g. D

6.2. Hilbert Space S)

6.69. If 1 a def
Izi < -, 2 def {
= {an}nEN E £ (N), and b z =
1 }
-- then
2 n- Z nEN

Thus fa(z) ~ (a, bz) = L::'=l ~


n-z
is holomorphic in D(0))°.
2
If the

span of imC'Y) ~ ,* is not II 112-dense in peN) then for some nonzero a,


fa(z) == 0 on [-~,~], whence fa(z) == o.
On the other hand,

. {}0 U N, ~oo ~ ~oo ~


whence for all k III Lm-l- n k+l = 0 and al = - wn-2 - n k+l. As
k --+ 00, there emerges: al = O. Induction shows that for all n in N, an = 0,
a contradiction. D
6.70. If K is a II 112-compact subset of £2 (N) and if FN .fr 0 on K then
K contains a sequence {km ~f {kmn }:'=l}OOm=l such that for some positive
a and each m in N, L:n~m Ikmn l2 ~ a 2 • Because K is compact it may be
332 6. Topological Vector Spaces: Solutions

assumed that for some k ~f {kn }:'=l in K, IIkm - kll2 -+ 0 as m -+ 00,


2 a2
whence for each n, kmn -+ kn as m -+ 00. For large no, En~no Iknl < 24

and for large mo, IIkm - kll~ < ;6 if m ~ mo. Thus if m ~ mo + no,
2

a contradiction. o
6.71. Because all functions considered are in L2 (R?, A), all integrals given
as well as those introduced below exist. From Green's theorem it follows
that for all positive R,

[ (f(x,y)6.g(x,y) - 6.f(x, y)g(x, y)) dxdy


iB(O,R)

= 1 (f(x,y)89 (X,Y) _g(x,y)8 f (X,Y)) ds


l'aB(O,R) 8n an
= [ (f(x,y)89 (X,Y) _g(x,y)8 f (X,Y)) dy
iaB(O,R) 8x 8x
+ (g(X, y) 8f~~ y)) _ (f(X, y) 8g~ y)) dx.

A typical estimate for the integrals in these equations is

[
IiaB(O,R) f(x, y/g(x, y) dyl
8x

: :; 127< If (R cos 8, RSin8)1./ 8g (RCO~:, Rsin8) /.IRsin8 1 dB.


Thus, A(R) denoting the right member above,

[ If(rcos8,rSinfJ) 8g(rco~8,rSinfJ) 1rdrd8 = [R A(r) dr


iB(O,R) x io
:::; IIfll2 ·11 ~! 112
Hence for some {Rn}nEN' Rn i 00 while A (Rn) 10. Similar estimates for
the other three integrals show that

I[
iB(O,R)
(f(x,y)6.g(x,y) - 6.f(x, y)g(x, y)) dXdyl
Solutions 6.72-6.74 333

is dominated by the sum of four quantities, each depending on n and ap-


proaching zero as n --+ 00. 0

6.72. For example, 'Y : [0,1] 3 t 1-+ X[O,t] E L2 ([0,1],'>') serves. 0


6.73. a) If 0 ~ a ~ b< 00, the Schwarz inequality implies

I/n(b) - In(a)1 ~ lb 1/~(x)1 dx ~ Mlb - al!

I/n(x)1 ~ { I/n(1)1 + Mix - 11! ~f 0 ~ x ~ 1


1/x If x> 1

whence In E Lip (1/2) and II In II <Xl ~ 1 + M, n E N.


For each interval [0, k], kEN, the Arzela-Ascoli theorem implies that
{In} nEN contains a subsequence {fkn} nEN uniformly convergent on [0, k]
and such that {fkn}nEN :J {1k+l,n}nEN' Thus S ~ {gn ~f Inn} nEN con-
verges everywhere.
b) If f > 0 and kE > 2, there is an N such that for {gn}nEN as in a),

Igm(x) - gn(x)1 < {~/22 if 0 ~ x ~ k and m,n > N


<- if k < x.

c) Because

IIgm - gnll; = { + (<Xl Igm(x) - gn(x)12dx


i[O,k] i k

~ k IIgm - gnll<Xl + 1<Xl 2/x 2 dx,

for large k and all m, n, the second term in the right member above is small
and for large m and n, the first term is small.
In sum, {gn} nEN is pointwise, uniformly, and II 112- convergent. 0

6.74. Since l y
_Y I/(x,y)ldx ~ (lY _y If(x,y)1 2 dx )! (2y)"2, it follows that
1

On the other hand, if A > 0 then for all y near zero,

1
"2
y
(l Y

-Y
If(x,y)ldx )2 A2
~ 8'
Y
334 6. Topological Vector Spaces: Solutions

which implies that IIfll2 = 00, a contradiction. 0

6.75. If to > 0 and En ~f {x : Ifn(x)1 ~ to > O} then )..(En) -+ 0 as


n -+ 00 and

6.76. It may be assumed that fn ~f t::.::,f - g~' O. Because

foX Ig(t) - t::.hf(t) I dt::; (L Ig(t) - t::. h f(t)1 2 dt) "2 Ixl!,

it follows that for a, x off a null set,

1 X
g(t)dt = nl~~lx n (f (t+~) - f(t)) dt

= lim
n~oo
nhrx+1.
n f(t)dt- lim
n~oo
nl a
a+1.
n f(t)dt
= f(x) - f(a). 0

6.77. a) Because IITfll~ = fo1 1fox f(t) d{ dx::; fo1 xllfll~ dx = ~llfll~,
it follows that IITII ::; 2-!.
b) For P: L2 ([0, 1],),,) :;) h I---t fo1 h(x) dx, P E [L2 ([0, 1], )..)], p 2 = P,
and P (L2 ([0, 1],),,)) = C. Because

(Tf,g) = fo1 (foX f(t)dt)9(X)dX= fo1 (1 1g (X)dX)f(t)dt


= fo1 [fo 1g (X)dX- fot9(X)dX]f(t)dt=(J,(P-T)9)

it follows that T* = P - T.
c) If {fn}nEN C B(O, 1) then

ITf(x)1 ::; x! ::; 1 and ITfn(x) - Tfn(Y)1 ::; Ix - YI!·

The Arzela-Ascoli theorem implies {Tfn}nEN contains a uniformly conver-


gent subsequence S and, because).. ([0,1]) = 1, S is II 112-convergent. 0
6.78. a) Because).. ([0, 1]) = 1, II 112 ::; II lIex" whence a II Ilao-limit point of
S is a II 112-limit point of S.
Solutions 6.79-6.81 335

b) Owing to a), the graph of the inclusion map

L: G([O, l],q :::> S 3 f ~ f E L2([0, 1],~)

is closed whence L is continuous: IILII exists. Hence from a) it follows that


II 112 ~ II lloe ~ IILII . II 112.
c) The map Ly : S 3 f 1--+ f(y) E C is a II lloe-continuous linear
functional and so for some K y, ILy(J)1 ~ Kyllflloe ~ KyMllfl12' The
Hahn-Banach and Riesz representation theorems imply that for some ky in
L2 ([0, IJ, oX), Ly(J) = fo1 ky(x)f(x) dx. D

6.79. For 9h ~f f[h] - f[-h], (§h)n = 2iln sin nh whence, owing to Parse-
val's equation, for some constant G1 ,

Ilghll~ = 4 L Ilnl 2 sin2 nh ~f 4 Lr~sin2nh ~ G1 Ihl1+ a .


nEZ nEZ

def k N . 2 1
If kEN, N = 2 ,h = 7r
2N' and "2 ~ Inl ~ N then sm nh ~ "2 and so for
some constant G2 , I:2 k - 1 :"OlnI9 k r;, ~ G 2- (1+
2 k a ). Owing to the Schwarz
inequality, for some constant G3 ,

L rn~
2k-l:5lnl:"02k

D
6.80. xe 271"inx is in the
For all n in Z \ {O}, the function gn : [0,1] 3 x 1--+

(vector) subspace A. Hence if h E A~, n E Z \ {O}, and H(x) ~ xh(x)


then fin = 0, i.e., H is a constant. Unless h = 0, h 1:. L2 ([0,1], oX), a
contradiction: A~ = {O}. D

6.81. If f(t) == t, then


o
1 1

n+2
1
tnf(t)dt = - - , n E Z+.
Conversely, the WeierstraB approximation theorem implies that if

11 (J(t) - t)t n dt == O,n E Z+,

then for any continuous function g, fo1(J(t)-t)g(t) dt = 0. Since G([O, 1], q


is II 112-dense in L2([0, 1], oX), it follows that f(t) ~ t. D
336 6. Topological Vector Spaces: Solutions

6.82. If 0 < J.L(E) < 00 then, owing to the monotone convergence theorem
and the Schwarz inequality, a.e. on E,
00

n=l
L: Iln(x) -
00

(J.L(E))-! In+1(x)1 < 00


n=l

L: Un(x) -
00

In+1(X)) = nl!.~ (ft(x) - In (x)) .


n=l

Hence limn --+ oo In (x) ~f gE(X) exists a.e. on E.


Because S ~f UnEN {x : In(x) =I O} is cr-finite, for some {Em}mEN'
J.L (Em) < 00, mEN, and S = UmENEm . Corresponding to each Em there

is a gEm ~f h m and In ~. 2::=1 h m ~f I. Because {fn}nEN is a II 112-


Cauchy sequence, its II 112-limit must be I· 0
6.83. For S, Em as in 6.82, the fundamental inequality is

~
~
IIlnll~
2
< 00.
n=l n

The reasoning used in 6.82 applies, mutatis mutandis. o


6.84. Bessel's inequality implies that for x in Sj, (x, x n ) ---t 0 as n ---t 00.

o
6.85. a) The map P : C ([-1,1], C) " I I-> {x I-> I(x) +/(-x)} is
such that for I in C([-1, 1], C), IIPI1I2::; 111112 and P is extendible to an
element, again denoted P, of [L2 ([-1,1], A)). Furthermore, p 2 = P and
P(S) = S whence M = P (L2 ([-1, 1], A)).

b) If I E L2 ([-1,1]' A) and I rv ~ao + f:an cos mrx + bn sin sin mrx


n=l
then because the sine function is odd, and the cosine function is even,

PI rv
1
2ao + L: an cos mrx.
00

n=l
Hence the set of trigonometric polynomials of the cosine functions consti-
tute an orthonormal basis for M. Each cosine function may be uniformly
approximated on [-1,1] by a pin C [x 2 ]. The Gram-Schmidt process en-
dows M with an orthonormal basis consisting of polynomials. 0
Solutions 6.86-6.87 337

6.86. a) If E > 0 and U ~f {x I(X,Yk)1 < E, 1:::; k:::; K} is a weak


. E
neIghborhood of 0, for some m, I(xm , Yk)1 < 3,1 :::; k :::; K, and for some
E
n, n> m and I(Xn,Yk)1 < 3m whence Xn + mXn E U.
b) If Fe B(O, R) and Xm + mXn E F then 1 + m 2 :::; R2, whence for
some M in N, m < M and m < n :::; M - 1 or n > M - 1 ~ m. If a > 0,
Y ~f 2a E;;;;:;
X m , and U ~f {x : I(x, Y)I < a}, then

(Xm + mxn,y) = 2a > a


and so F n U = 0.
c) If Zk ~f x mk + mkxnk ~ 0 then for some K, IIzkll :::; K whence for
some M, mk :::; M. Hence for all large p,

o
6.87. a) Let Xn - Yn be Zn. If XES) and Ilxll :::; 1,

L L
00 00

X ~f aonXn = aOn (Yn + zn)


n=1 n=1

Thus WI ~f E:=I aonZ n exists and Ilwdl :::; r. Induction produces a se-
quence {wd kEN such that

n=1 n=1

x = ~ (~apn) Yn +wP+1
~ (~lapnI2) :::; p~o (~lapnl'laqnl)
00

00

L(s + 1)r 8 < 00.


8=0
338 6. Topological Vector Spaces: Solutions

ror A Pn
D
= ",p
L.."p=O apn , A n def
def
= 1·lmp~oo A ·
Pn eXIsts, = { APn } nEN
both Ap def
and A ~f {An}nEN are in £2 (N), and Ap 1I-!2 A. Thus

IIX - t, AnYn11 ~ IlwPHl1 + L~H ApnYnl1


+ l i t (APn - An)Ynll· (s6.8)

The third term in the right member of (s6.8) does not exceed

Similar calculations show that the second term in the right member of
(1 + (L:'=NH c2n p}
! 1

(s6.8) does not exceed (L~N+IIApnI2r . Be-


cause Ilwp+III - 0 as P - 00, X = L:'=l AnYn.
IfL:'=1 bnYn = 0 then Ibnl·IIYnll - 0 as n - 00. Because IIYnl1 is near
Ilxnll (= 1), it follows that bn - 0 as n - 00. Hence L:'=l bn Ilznll < 00
and L:'=l bnzn = L:'=l bn (xn - zn) + L:'=l bnzn = L:'=l bnxn ~f Z
exists. Hence IIzI12 = L:'=llbn I2 ~ (L:'=llbn lllzn I1)2 ~ (L:'=llbnI2) . r2
and if L:'=l Ibn l2 is positive, there emerges the contradiction: 1 ~ r2 < l.
b) The argument in the last paragraph of a) applies, mutatis mutandis.
o
6.88. Each ¢"( may be expressed in terms of the vectors in'll:

and 11¢"(11 2 = LOE~ la"(oI2. If 80 is fixed and, as, varies over r, a"(oo == 0
then 'l/Joo is II II-approximable by finite sums L~,o boo"(a"(o'l/Jo in each of which
the coefficient of'l/Joo is zero. Thus lI'l/Joo 112 is approximable by finite sums
L',,(, 0 bOo,,(a,,(o ('l/Jo, 'l/Joo) each of which is zero, a contradiction.
Hence to each 'l/Jti, there corresponds a ¢"( such that a"(ti =1= 0 whence
#(1),.) ~ #(r). A similar argument shows #(r) ~ #(1),.). 0
Solutions 6.89-6.91 339

6.89. a) From the Schwarz inequality it follows that T E [L2 ([0, 1], ,X)]
and IITII ~ IIKII2.
b) If {cPn}nEN is a complete orthonormal set for L2 ([0, 1]''x) and

II 112
then Lm,nENamncPncPm -t K. The maps

M,N (I
TMN : L2 ([0,1],'x) :7 f f-+ L 10 amncPm(X)cPn(y)f(y) dy
m,n=1 0

are such that IITMN - Til ~ Lm~M or n~N lamn l2 . 0


6.90. Let {cPn}nEN in 5) be orthonormal. Then IlcPm - cPnll = DmnV2. The
balls B (cPn, 0.5) are pairwise disjoint and their union is a subset of B (0, 2).
Arguments based on homothety show that for some positive r, B (0,1)
contains infinitely many pairwise disjoint translates of B (0, r). Hence if It
is translation-invariant and It(B(O,r)) < 00 then It(B(O,r)) = O.
If 5) is separable, every open set is the countable union of translates of
open balls of radius not exceeding r whence every open set, in particular 5)
itself, is a null set, Le., It is trivial.
If 5) is not separable, it contains a separable infinite-dimensional (Hil-
bert) subspace 5)1. Each ball BSjl (xo, r) of 5)1 is 5)1 n B (xo, r). If It is a
translation-invariant measure for 5) and 0 < It (B(O, 1)) < 00, the formula

Ito (BSjl (xo, r)) ~f It (B (xo, r))

can be used to determine a translation-invariant outer measure It* in 5)1


and therefrom a translation-invariant measure Ito in 5). But then

0< /-Lo (BSjl (0,1)) = It (B (0, 1)) < 00,


a contradiction. o
[ Note 86.8: If nontrivial measure is taken to mean that every
ball of (finite) positive radius has (finite) positive measure, the ar-
gument used above and reduced to its first paragraph shows that
5) cannot be equipped with a nontrivial (Borel) measure. Further-
more, the argument remains valid when translation-invariant is
replaced by unitarily invariant.]

6.91. Because the diameter of a set in 5) is translation-invariant, pP is also


translation-invariant. Every closed set is pp-measurable (cf. 4.8), whence
each ball is pp-measurable. If 5)1 is an n-dimensional subspace of 5) and
340 6. Topological Vector Spaces: Solutions

r > 0 then S)1 n B(O, r) ~f BfJ1 (0, r) is the n-dimensional ball and 4.6
implies: pn (BfJ1 (0, r» ~ An (BfJ1 (0, r» > O. Thus from 6.90, it follows
that pn (B(O, r» = 00, n E N, and 4.5 implies pI' (B(O, r» == 00. 0
6.92. The Schwarz inequality implies that if IE S) then 11/111 ::; 11/112. If
kEN and

then IIIkl12 i 00, lIikl11 10.


def ,",00
If I = L..Jk=l k-s Ik then I E S), whereas
11

(ik, I) = k ~ f n~k
n=l ns
~ k~ roo
11
dx 13
(x+k)s
8k~
= - t 00
5(1 + k)~

as k - t 00. If r > 0 and k is large then

Ik E {I : I E s), II/liI < r} \ {g : 9 E S), (g, f) < 1} .

Hence there is no nonempty lillI-ball B(O,r)O contained in the a(S),S)*)-


neighborhood {g : 9 E S), (g, f) < 1}. 0
6.93. For T : S) 3 x ~f {xn}nEN t--+ T(x) ~f {Xn} ,
n nEN

T = T*, T(S) = D, and T (M-L) = A.

If A is not dense in S), for some nonzero y in A -L, whence for all z in M -L ,
(T(z),y) = (z,T(y» = O. ThusT(y) E (M-L)-L = M. SinceMnD = {O},
T(y) = 0, and so y = 0, a contradiction. 0
6.94. If I E S then
00 00 00
11/1100 ::; L lanl ::; L n lanl ::; 1, II/II~::; L lanl2 ::; 1.
n=l n=l n=l

If S contains the sequence {Jk(X) '" 2:::'=1 aknsin2mrxhEN then via the
standard selection algorithm and subsequent diagonalization, there emerge
sequences {AJ iEN' {Aij} j EN' etc., such that

· l'
1
.1m
Z--HX>
aki1 def
= aI, J-+oo J
def
.1m ak i .2 = a2,
Solution 6.95 341

etc. exist. If

and if p > r, then for large N,

Because 2::'=N+l n~ < L ~~,


oo
the second term in the right member

above does not exceed ~ whence {gp}PEN is a II 112-Cauchy sequence with


II 112-limit say, g. Furthermore, limp->oo bpn ~f bn , n E N, exist, and

L bn sin 2mrx.
00

g(x) '"
n=1

Because 2:;;=1 n Ibnl = limp->oo 2:;;=1 n Ibpnl :::; 1, 2::'1 n Ibnl :::; 1, whence
9 E S and so S is II 112-compact. 0
6.95. If J;
gl (t) dt + Cl = J;
g2(t) dt + C2 then setting x at zero shows
Cl = C2. Thus J;
(gl (t) - g2 (t)) dt == 0, and so gl ~ g2: T is well-defined.
If {/,g} is in the closure of 9 then for some sequence {fn}nEN in S,
In(x) = J;
gn(t) dt + Cn and {fn'gn} ....... {/,g} as n ....... 00 whence

IIln - 1112 + IIgn - glb ....... 0 as n ....... 00.

If E ~ { (x, t) : 0:::; t :::; x :::; I} and K ~f XE then IIKII2 = 2-~, and

I(x) = r
1[0,1]
K(x, t)g(t) dt + c.

As in 6.89a), it follows that IIln - Cn - Um - cm)1I 2 :::; 2-~ 119n - 9mlb.


Furthermore,

ICn - cml = lien - cm l 2 :::; IIln - Cn - Um - cm )II2 + IIln - Imll2


:::; T~ IIgn - gmll2 + IIln - 1m 112
·
W h ence 1lmn->oo def.
Cn = C eXIsts.
342 6. Topological Vector Spaces: Solutions

If F(x) ~f I: g(t) dt then


F(x) - fn(x) + en = 11 K(x, t) (g(t) - gn(t» dt

Ilf - (F + e) 112 ::; IIf - fnll2 + IIfn - (f - en)1 2 + len - el


::; IIf - fnll2 + T! IIg - gnll2 + len - el·
Hence f = F + e, Tf = 9 and g is closed. o
6.96. If {fn};;=1 is an orthonormal subset of M then

E ~{ x : t. lfn (X)1 2 =I- 0 } =I- 0

d f fk(X)
and for any fixed x in E, ak(x) ~ ---=---~--'-1' 1 ::; k ::; N, are
(L:;;=1Ifn(x)1 2) 2
well-defined.
Thus for t in X,

It. an lin I,; c (Ix (t. .. l)' l) !


(x (tl (x l/n(8 dM('

~ c (t.10,.(Xl1') I ~ c,
an inequality valid for each (x, t) in X2. In particular it follows that when
x = t, 1L:;;=1 an(x)fn(x) 12 == (L:;;=1Ifn(x)1 2) ::; C 2. Owing to the or-
thonormality of {fn};;=1' Ix L:;;=1Ifn(x)1 2 dJL(x) = N::; C 2JL(X). 0
6.97. a) Because IIfgll2 ::; IIfiloollgll2, IIT,II ::; IIfiloo.
< 1111100 ~f M < 00 and En ~f {x : If(x)1 2: M - ~ }
If IIT,II
then JL (En) > O. If, for each n in N, £2 (X, JL) \ {O} contains an hn such
that hn(x) ~ 0 off En then it may be assumed that IIh n ll 2 = 1 whence
IIT,II 2: M - ~ and so IIT,II = M, a contradiction. Thus for some no,
n
£2 (X, JL) \ {O} contains no h such that h( x) ~ 0 off Eno' Hence Eno
contains no measurable subset of finite measure, i.e., Eno is an infinite
atom.
Conversely, if E is an infinite atom then for any h in £2 (X, JL) \ {O},
XEh ~ 0, whence IIXEhll2 = 0, i.e., IITxEIi = 0 < IIXElioo = 1.
Solutions 6.98-6.99 343

b) If En ~f {x Ik(x)1 > n} then En::) En+l and p, (En) > 0, n E N.


If p, (En \ En+!) = 0 for all but finitely many n, then k(x) = 00 on a
measurable set F of positive measure and because X contains no infinite
atoms, F contains a set H of finite positive measure. Then XH E L2 (X, p,)
and kXH tj. L2 (X,p,).
Thus it may be assumed that for all n, p, (En \ En+d > 0 and each
En \En+l contains a subset Hn of finite positive measure. If a;p, (Hn) =~
n
and 9 ~f 2:::=1 anXHn then 9 E L2 (X,p,) and on Hn, k(x)g(x);::: ~
whence IIkgl12 = 00. D
6.98. Intuition suggests that Tf is surjective iff for all h in L2 (X, p,),
7 E L 2 (X, p,). As given, the criterion is meaningless because -1 may fail
to be defined. The following paragraphs deal with this difficulty.

°
If 0 < p,(E) < 00 and f- 1 (0) ~ E then T f cannot be surjective because
i= XE E L2 (X, p,) and if fg = XE then f(x)g(x) ~ 0 both on E and
on X \ E, a contradiction. On the other hand, if E is an infinite atom
and hE L2(X,p,) then h(x)IE~ O. Thus if f E LOO(X,p,) and f!x) is
essentially bounded off E then Tf is surjective.
In light of the preceding paragraphs the following criterion suggests
itself:
Tf is surjective iff for every a-finite set E, II f~E 1100 < 00, in the
sense that p, ((fXE) -1 (0)) = 0 and f~E ((fXE) -1 (0)) = o.

Indeed, if Tf is surjective, E is a-finite, and II f~E 1100 = 00 then as in


Solution 6.97b), for some 9 in L2 (E, p,), f g tj. L2 (E, p,). Extend the
XE
domain of 9 to X so that g(x) = 0 off E. Then 9 E L2 (X, p,). Because T f
is surjective, for some h in L2 (X, p,), Tf(h) = g. Because g(x) = 0 off E,
fh = 9 = fXEh and f g = h tj. L2 (X, p,), a contradiction.
XE
Conversely, if h E L2 (X, p,) and for every a-finite set E, II f~E 1100 < 00
then E ~f {x : h(x) i= O} is a-finite and 9 d~f 7E L2 (X,p,), whence
Tf(9) = h. D
6.99. a) If 9 i= °
).. is not a constant.
and Tf(9) = )..g then for all t, (t - )..)g(t) = 0, whence
b) For all f in Loo ([0,1], )..),

Tf (8(1)) = f8(1) = 8 (Tf(l)) = 8(f),


344 6. Topological Vector Spaces: Solutions

whence if 8(1) ~f 9 (in L2 ([0,1], A» then 8(J) = g/. If h E L2 ([0, 1], A)


then (h V 0) 1\ (n· 1) + (h 1\ 0) V (-n· 1) ~f h n ~. hand h n 1I~2 h, whence
8 (h n ) = gh n 1I~2 h and ghn ~. gh and so gh E L2 ([0, 1], A).
If 9 ~ L OO ([0,1]' A) then for all n in N,

A({X: n:$lg(x)l<n+l})~an>O.

If ,",00 Ikn l 2 < ,",00 n 2 1kn l 2 _ d h ~f ,",00 kn then


L.tn=l 00, L.Jn=l - 00, an - L.Jn=l XEn
~ ~ ~
h E L2([0, 1], A) while g/ ~ L2 ([0, 1], A), a contradiction. It follows that 9
is in L oo ([0,1]' A) and 8 = Tg • 0
6.100. a) See Solution 6.91.
b) For A in C, and some nonzero x ~f {xn}nEN'

whence A = ano and then Xn = 0 if an #- A.


c) For some subsequence {ankhEN' limk_oo ank ~f a exists and is not
zero. If Xk ~ {Onnk}nEN then 8 ~f {Xd kEN C B(O,I) whereas Ta(8)
contains no II 112-Cauchy sequence. 0
[ Note 86.9: Contrast the result in b) above with the result in
6.99a).]

6.101. The conclusion follows from the equation

o
6.102. Because IIxn -xmll 2 = IIxnll2 + IIxmll2 - 2iR(x n ,xm), it follows
that limn _ oo IIxn - xmll :$ 1 + 1- 2 = O. 0
6.103. For fixed y, Ly : Sj 3 x 1--+ 8(x, y) is a continuous linear functional,
whence for some Zy in Sj, Ly(x) = (x, Zy). Moreover, T : Sj 3 Y 1--+ Zy is
linear and because I(x, Ty)1 :$ IIxil . IIYII, it follows that IITII :$ 1. 0
6.104. If IIx -yll = a > 0 and Z ~f x-y then (x-y,z) = a2 . For any
u and positive €,

N (u; €) ~f {w : I(w, z) - (u, z) I < €}

is a (weak) neighborhood of u and

o
Solutions 6.105-6.107 345

[ Note 86.10: There is a stronger result: The weak* topology


of SJ or of the dual X* of any Banach space X is normal. The
sequence of ideas in the proof is the following: a) The unit ball
of X* is a (X*, X)-compact; b) X* is a-compact in the topology
a (X*, X); c) X* is a (X*, X)-LindeI6f; d) in Lindel6f spaces regu-
larity and paracompactness are equivalent; e) every paracompact
space is normal. J

6.105. If dim (SJ) ~f n (in N) then the weak and II II-induced topologies
are the same and SJ is the complete metric space en, a set of the second
category.
If dim (SJ) = 00, W is weakly open, and x E W, then for some w in
W, IIwll ~f 1 + b > 1 and for some z in B(O, 1), (w, z) > 1 +~. Then
U ~f { U : I(u,z) - (w,z)1 < ~} is a weak neighborhood of w, whereas
if v E B(O, 1) then lv, z)1 ::; 1. Hence

b b
I(w,z) - (v,z)1 ~ l(w,z)I-I(v,z)1 ~ 1 + 2 -1 = 2'

i.e., UnB(O, 1) = 0. Thus WnU is a weak neighborhood ofw, WnU c W,


and (W n U) n B(O, 1) = 0. In sum, B(O,l) is weakly nowhere dense.
A similar argument shows that for all nonnegative R, B(O, R) is weakly
nowhere dense. Hence SJ = Un EN B(O, n), a countable union of weakly
nowhere dense sets. 0
6.106. Let P be the orthogonal projection of SJ onto M:

P = p2, IIxl1 2 = IIPxl1 2 + Ilx - Px11 2 , Px ..L (x - Px).

Then the Hahn-Banach theorem and the preceding equalities imply

sup { l(z,x)1 : z E M.l n 8B(O, 1) }


= sup {I(z,x - Px)1 : z E M.l n 8B(O, I)} = Ilx - Pxll
inf {llx - yll : y E M} = Ilx - Pxll· 0

6.107. If gEM .1 then h(t) =


def (
g,/[t]
)
== 0 == ~h(n) while

whence g( -n) == 0 and so 9 = 0. o


346 6. Topological Vector Spaces: Solutions

[ Note 86.11: The result in 6.107 is the simple version of Wie-


ner's Tauberian theorem. The general form asserts that if G is a
locally compact abelian group and I E L1 (G,JL) then

span ({I[xj : xEG}) =L1 (G,JL)

1
iff is never zero. The distinction between L2 (G, JL) and L1 (G, JL)
is important and provokes a very much more complex proof. Wie-
ner's original proof was hard analysis. The modern proofs use
functional analysis, particularly the general theory of commutative
Banach algebras [Loo].]

6.108. Because S includes the set A of all infinitely differentiable functions


that together with all their derivatives vanish at both zero and one, S is
dense.
If {fn}nEN C S, In IIJ2 I, D (In) IIJ2 9 and h E A, then it may be
assumed that In ~. I, D (In) ~. 9 and thus

11 D (In)(x)h(x) dx = In(x)h(x)l~ -1 1
In(x)D(h)(x) dx

= -1 1
In (x)D(h)(x) dx

11 g(x)h(x) dx = l(x)h(x)l~ -1 1
l(x)D(h)(x) dx

= -1 1
l(x)D(h)(x) dx. (s6.9)

If J; g(t) dt ~f G(x) then


11 g(x)h(x) dx = G(x)h(x)l~ -1 1
G(x)D(h)(x) dx

= _11 G(x)Dh(x) dx. (s6.1O)

Since (s6.9) and (s6.1O) obtain for all h in A, it follows that I ~ G, i.e., I
is differentiable a.e. and D(I) ~ g. 0
. def8(f(x),y)
6.109. For x m JR, g(x) = 8x ,and some a(x,y),

Tx(Y) ~f (f(x) - f (xo) ,y) = 9 (xo, y) (x - xo) + a(x, y) (x - xo) .


If x =I=- Xo then

ITx(Y)1 1 :s: Ix - xol! . Ig (xo, y) + a(x, y)1 -+ 0 as x -+ Xo·


Ix - xol 2
Solutions 6.110-6.112 347

The uniform boundedness principle implies that for x near Xo and some M,
liT., II 1 ~M < 00. Thus
Ix - xol 2
1
11/(x) - I (xo)11 = sup IT.,(y) I ~ liT., II ~ Mix - xol 2 . D
ilyil=l

6.3. Abstract Topological Vector Spaces

6.110. If {x,x*} E (V \ {O}) x (V* \ {O*}) then the map

8 xx ' : V 3 Y 1--+ (y, x* ) x

is compact because dim (im (8xx *)) = 1. For T as described and all y,
T8xx '(Y) = (y,x*)T(x) = 8 xx .T(y) = (T(y),x*)x. Because x* =F 0, For
(T(y), x*) def
some y, (y,x*) =F 0, whence for all x, T(x) = (y,x*) x = cx. Since
T(x) is y-free, so is c. D
6.111. The uniform boundedness principle applies twice, once to the
sequence {Tn(v)}nEN (in W regarded as a subset of W**) and then to
{Tn} nEN (in [V, W]). D
6.112. If x, y E V and x* E V* then

(T(x + y), x*) = (x + y, 8 (x*))


= (x, 8 (x*)) + (y, 8 (x*))
= (T(x),x*) + (T(y),x*)
= (T(x) + T(y), x*).
Because the equations above are valid for all x* in V*, T is additive. Similar
calculations show T is a not necessarily continuous endomorphism of V and
8 is a not necessarily continuous endomorphism of V* .
Furthermore, if

Ilxn - xII + Ilx~ - x*11 + liT (xn) - yll + 118 (x~) - y*11 -+ 0

as n -+ 00 then for all x* in V*,

(T (x n ) ,x*) = (xn' 8 (x*)) -+ (x, 8 (x*)) = (T(x),x*)

as n -+ 00. Hence y = T(x). Similarly, for all x in V,

(x, 8 (x~)) = (T(x), x~) -+ (T(x), x*) = (x, 8 (x*))


348 6. Topological Vector Spaces: Solutions

as n -+ 00 and so S (x*) = y*. The closed graph theorem applies. 0


6.113. If X ~f {x,},H and Y ~f {YO}OEA are Hamel bases for V then for
each 'Y, x, = L~EA a,oYo a finite sum. If 80 is fixed and, as 'Y varies over
r, a,oo == 0 then YOo = L~,o boo,a,oYo = L~EA cOooYo, a contradiction of
the linear independence of Y. The rest of the argument given in 6.88 may
be repeated, mutatis mutandis. 0
6.114. Because V is infinite-dimensional, the Gram-Schmidt biorthogonal-
ization process leads to the construction in V x V* of an infinite biorthogonal
sequence {{ Vn, v~}} nEN' i.e., (vn' v;;") = 8mn . Furthermore,

is also a biorthogonal set {{Yn'Y~}}nEN and if SeN, LnEsYn II 11-


converges.
For {r n} nEN an enumeration of Q and for each t in JR, an infinite
subsequence St ~f {rnk(t)}kEN converges to t. If t =1= t' then St n St' is
empty or finite. If {t m } ~=1 is a finite set then, because St m is infinite,
St m \ U m'i-m St m , =1= 0.
l~m'~M

= L.-rnESt Yn. I Z = L.-m=1 amZtm =


· a Zt def
For each t, t h ere IS '" f def ",M 0
and rn ESt", \ U m'i-m St m , then (z, y~) = am = 0, 1 ~ m ~ M. Thus
1~m'~M

Z ~f {ZthEIR is C-linearly independent and may be extended to a maximal


C-linearly independent set - a Hamel basis H for V. Furthermore, owing
to 6.113, c ~ #(H) = # (H). 0
6.115. Let S ~f {An} nEN be a countable infinite subset of A. Then
L oo x>'n
--.".-"---;-:- converges and represents some Y in V. Hence for some
n=1 2n Ilx>'n II
finite subset 8 of A, Y = L>'EO a>. (y)x>.. If An E S\8 and if a>'n is continuous
then

a>'n(Y) = a>'n (L:


>'EO
a>. (y)x>.) =0

= a>'n (~ 2n I~:>.n II) = 2n 1I~>'nll '


a contradiction.
Hence for every countable infinite subset S of A, A contains a finite
subset 8 such that for every>: in S \ 8, aX is discontinuous. 0
6.116. a) Both Co (N) and £1 (N) are Banach spaces, each of cardinality
c. Owing to 6.114, each has a C-Hamel basis of cardinality c. Any bi-
jection between Hamel bases for Co (N) and £1 (N) may be extended to an
Solutions 6.117-6.122 349

isomorphism T : Co (N) ~ £1 (N) and ker(T) = T- 1 (O) = {O}, a closed


set.
However, (Co (N)) * = £1 (N) is II Ill-separable while (N))* = £cc (N)
W
is not II IIcc-separable. Hence T cannot be a continuous isomorphism [even
though ker(T) is closed].
b) By the same token, T- 1 is an isomorphism of £l(N) onto co(N). If
T- 1 were open, T would be continuous. 0
6.117. For some w in W, IIx - wll > d(x, W) - a. The Hahn-Banach
theorem assures the existence in V* of a z* as described. 0
6.11S. If # exists then for some {v, v*} in V x V* , (v, v*) = 1.
But then (v#, v*) = 1, (v, iv*) = i, and (v#, iv*) = (v, iv*) = -i. On the
other hand, (v#, iv*) = i ((v#, v*)) = i. 0
6.119. If v* E M and v E M1. then (v, v*) = 0 and the linearity of
( , ) implies span(M) C (M1.)1.. If v* E (span(M))w* [the weak* closure
of span(M)], a> 0, and v E M1., then

{y* : l(v,y*-v*)I<a}n(M1.)1.
::::> {y* : I(v, y* - v*)1 < a} n span(M) =1= 0.

Thus for some y* in (M1.)1., I(v, y* - v*)1 = I(v, v*)1 < a. Because a
is an arbitrary positive number, I(v, v*)1 = O. Thus v* E (M1.) 1. , i.e.,
(span(M))w* C (M1.)1..
For a (V, V*) resp. a (V*, V), each of the dual pair {V, V*} is the dual
of the other. If v* E (M1.)1. \ MW*, the Hahn-Banach theorem implies
(Mw*)1. contains a v such that (v, v*) = 1, a contradiction. 0
[ Note s6.12: Even when V is a Banach space and V =1= V**,
for a (V, V*) resp. a (V*, V), V and V* are locally convex and a
dual pair and each is the dual of the other [Ko, Sch].]

6.120. Because M C (M 1. ) 1.' if v E (M 1. ) 1. \ M, according to the Hahn-


Banach theorem, for some v* in M 1., (v, v*) = 1, whence it follows that
v¢.( M 1. ) 1.' a contradiction. 0
6.121. For any nonreflexive Banach space V, e.g., Co (N), the canonical
embedding V"--t V** makes V a proper subspace of V**. Thus V1. = {O}
and (V1.)1. = V** ¥V. 0

6.122. a) It may be assumed that K =1= 0 and that 0 E K. If v ¢. K,


the Hahn-Banach theorem implies that for some positive numbers a, band
some v* in V*, (B(v,a),v*) C (-oo,-b) while v*(K) C [0,00). Thus
{y : I(y - v, v*)1 < ~} n K = 0, whence v ¢. KW, the weak closure of
K. Because K C KW, it follows that K = KW.
350 6. Topological Vector Spaces: Solutions

b) By definition K c KW. Because K is convex, weakly dense in K W,


and II II-closed it is, per the previous paragraph, weakly closed: K = KW.
D
6.123. For B ~f {WI, ... , WN} a basis for W and a ~f (a1,"" aN) in
eN, T : eN :3 a. ~ 2::=1 an WnEW is a linear bijection and

The open mapping theorem implies T is open whence T- 1 is continuous.


Hence if limk_oo Vk = v then limk_oo T- 1 (Vk) ~f a exists and T(a) = v,
i.e., v E W, W is closed.
If W E W then W = 2:~=1 In (w)W n. The Hahn-Banach theorem
implies that each In (in W*) may be extended to a v~ in V*. The map
P: V :3 v ~ 2::=1 V~(V)Wn is an idempotent in [V, W]e and

6.124. If I is continuous then ker(f) = 1-1(0) is closed. Conversely, if


ker(f) is closed, it is a closed subspace M of V. If M = V then I = 0
and I is continuous. If M ¥ V for some v in V \ M, I(v) = 1. If z E V

then Y ~ z - I(z)v E M. If, for some Y' in M and a in C, z = Y' + av


then I(z) = I (y') + al(v) = 0 + a . 1 = a, whence y' = y. The Hahn-
Banach theorem implies that for some v* in M J.., (v, v*) = 1. Because
(z, v*) == I(z), v* = I and I is continuous.
If ker(f) is not closed then I is not continuous and so I =I O. If
ker(f) = M =I V then for some v in V \ M, I(v) = 1 and for some v*
--def

in M J.., (v, v*) = 1. The argument of the preceding paragraph shows that
v* = I. D
6.125. Let {Yj}f=l be a basis for WjM. If xjjM = Yj then {Xj}f=l
is a basis for a closed subspace N in Wand M n N = {O}. The direct
sum U ~f V EB N with norm II II' : U :3 {v,n} ~ Ilvll + Ilnll is a Banach
space and S ~f T EB id is in [U, W]. If z E Wand z/M ~ 2:f=l ajYj
then z - 2::=1 ajXj EM, i.e., for some W in M, z = W + 2::=1 ajxj and
for some v in V, T(v) = W whence z E S(U). Thus S: U ~ W is open.
If {IDdkEN C M and limk_oo IDk = IDa then because S is open, for
some sequence {{ Vk, nd } ;:0=0 in U, {Vk' nk} ---+ {va, no} as k ---+ 00 and
S({vk,nk}) = IDk = T(Vk) +nk,k E Z+. Hence nk E MnN = {O} and
so IDa E M, i.e., M is closed.
Solutions 6.126-6.128 351

On the other hand, if {X>.hEA is a Hamel basis for the infinite-dimen-


sional Banach space W, let >.' be such that the coefficient functional a).'
is not continuous (cf. 6.124). Then M ~f ker (a).') is not closed although
dim (WjM) = 1. D
6.126. a) If V* is separable, V* \ {O} contains a dense set {v~} nEN and V
contains a set S ~f {vn}nEN such that Ilvnll ~ 1 and I(vn, v~)1 ~ IIv~II-!.
n
If v E V\span(S) then for some v*, in (span(S))~, (v, v*) = 1. There
is a sequence {ndkEN such that V~k ---+ v* as k ---+ 00 whence

Thus Ilv~k II ---+ 0 as k ---+ 00, whereas v* =f. O. Hence V \ span(S) = 0 and
V is separable.
r
b) Although £1 (N) is separable, (.el (N) = £00 (N) is not. D
6.127. Let Tbe the II II-induced topology of V*. Since 0' (V*, V) c T
it follows that O'A(O'(V*, V)) c O'A(T). By virtue of Alaoglu's theorem,
every B (v* , r) is 0' (V* , V)-compact and because V* is II II-separable, every
II II-open subset of V* is the countable union of balls, i.e., the countable
union of 0' (V*, V)-compact sets. D
6.128. The construction depends on two observations:
a) If M is a proper subspace of a topological vector space V and U is
a nonempty open subset of M, then for x in U, -x + U is an open subset
containing O. If Y E V then for some nonzero t, ty E -x + U c M, i.e.,
M = V. Thus no such U exists: M contains no nonempty open set.
b) If M is a closed proper subspace of V and v tJ- M there is an open
set U such that v E U C (V \ M). On the other hand, if v EM, U is open,
and v E U then the previous argument shows that the open set U \ M is
not empty whence M is nowhere dense. In sum:
A proper subspace contains no nonempty open set and a proper
closed subspace is nowhere dense.
Let {vn}nEN be dense in V. Then

{B (vn,rmt : 0 < rm E Q,m,n EN} ~f {Ad kEN

is a sequence of open sets. If Yl E V \ {O} then Al \ span (Yd =f. 0. If


Yl,' .. , Yn are linearly independent and

Yk E A k- 1 \ span (Yl, ... ,Yk-d , k = 2, ... , n,


then some Yn+l is in An \ span (Yl,· .. ,Yn) (=f. 0). The sequence {Yn} nEN
is linearly independent and because every open set is the union of some of
the A k , it follows that {Yn}nEN is dense. D
352 6. Topological Vector Spaces: Solutions

6.129. For the maps

TN: V* 3 v* 1-+ {(V1' V*), ... , (VN' v*) ,O,O, ... } ,N E N,


T: V* 3 v* 1-+ {(Vn,V*)}nEN E 1:1 (N):

a) because IITN (v*)111 :S: (2::;;=lllvnll) Ilv*ll, TN E [v*,1:1 (N)]; b) since


II(TN - T) (v*)111 = 2:::'=N+11(vn, v*)I, II(TN - T) (V*)1I1 -+ as N -+ 00
for all v* in V*; T (v*) E 1: 1 (N); c) the uniform boundedness principle
°
implies T E [v*, 1: 1 (N)] .
If a > 0, M ~ SUPN IITNII, and {an}nEN E Co, then for some N',
lanl < 2~ if n > N'. If Ilv*11 :S: 1 and p, q > N' then

6.130. a) For x in V and r in (0,1), induction provides a sequence


{xnkhEN such that Ilx - 2::{;=1 r k - 11lxllxnk II < rKllxll· It follows that
T (1!1 (N)) = V.
°
b) Hence if E > and x E V then for some a ~f {an}nEN in 1: 1 (N),
IIal11 < IIxll + E and T(a) = 2:::'=1 anXn = x, i.e.,

In other words, if a E 1:1 (N) then the quotient norm of a/ker(T) does not
exceed IIT(a)ll. Because Ilxll :S: 2:::'=llan l·llxn ll :S: Ila111' IIT(a)11 does not
exceed the quotient norm of a/ker(T). 0
6.131. For the maps SN : V 3 v 1-+ V + Tv + ... + TN-lv, if N < M
IITIIN d f
then IISMV - SNvl1 :S: 1 _ IITII' whence limN_co SNV ~ Sv exists. fur-

thermore, S E [VJ, IISII :S: 1 _ ~ITII' and S(id - T) = (id - T)S = id. The
desired solution is x = Sy. 0
6.132. Themap.~ T . 02 (1!.1)
1'1 3y ~f - TY
- { Y1,Y2, ... } 1-+ { Y2,Y3, ... } ~f
is such that (Sx, y) == (x, Ty) whence T = S*. Furthermore, because
II(S*txll; = 2::~n+1IxkI2, it follows that (S*tx IIJl2 O. On the other
hand, if en ~f {onm}mEN then IIen l1 2 == 1 while (S*t en+1 = e1, whence
II(S*tll == 1. 0
6.133. As a closed subspace of a Banach space, M is itself a Banach
space. As observed in Solution 6.128, if M ¥=Mn , n E N, then each
Solutions 6.134-6.136 353

Mn is nowhere dense and so M is a Banach space of the first category, a


contradiction. D
[ Note 86.13: If a vector space V over an infinite field IK is
the union of finitely many subspaces V!, ... , Vn then for some no,
V = Vno [GeO].]

6.134. If V is a Banach space and E::l Ilxnll < 00 then

{txn ~f YN}
n=l NEN

is a Cauchy sequence and has a limit.


Conversely, if (6.2) is valid and {vn}nEN is a Cauchy sequence, for
some subsequence {vnkhEN' E%"=l Ilvnk - Vnk+111 < 00. It follows that
limk-+co (v nl - Vnk) exists whence limn-+co v n exists. D
6.135. a) If x E Conv(A) then for some vectors V!, ... , VN in A and some
numbers h, ... ,tn in [0,1], E:=l tn = 1 and v = E:=l tn v n. If E > 0 then
for some vectors al, ... ,aM in A, A C U~=l B ( am, ~) o. For each n, some

am n in S ~f {am}~=l' and some b n in B (O,~) 0, Vn = am n + b n whence


N
V = En=l tnamn + "",N def
L.m=l tnbn = a + b. Furthermore, a E Conv ( Sand
)

bE B (O,~) o. As the convex hull of a finite set, Conv(S) is compact and

Conv(S) contains vectors CI, . . . , Cp such that Conv(S) C U:=l B (cp'~) 0

whence Conv(A) C U:=l B (c p , Er,


i.e., Conv(A) is totally bounded and
thus its closure K is compact.
b) Because K is II II-compact and x* is II II-continuous, the maximum
value of I(x, x*) I IK is achieved on K.
c) The map F : C (A, q 3 9 f-+ g(x) is a positive functional in
(C (A, q) * and the Riesz representation theorem implies that for some
complex measure I-tx, F(g) = fA g(y) dl-tx(Y) and Ill-txll = IIFII. Moreover,
IIFII = 1 and F (x*) = fA (y, x*) dl-tx(Y)· D
6.136. If b) obtains and w E W then

Bw : V 3 v f-+ B (v, w) E Z

is continuous, whence for some constant Kw and all v, IIBw(v)1I ::; Kwilvil.
The hypothesis that B is continuous separately in each argument and the
closed graph theorem imply that T : W 3 w f-+ Bw is continuous. Hence
for some constant C, IIBwll ::; Cllwll and so IIB(v, w)1I ::; Cllvll . IIwll.
Thus b) implies a), which implies B is jointly continuous, which implies
N· D
354 6. Topological Vector Spaces: Solutions

6.137. Because f is differentiable, if w* E W* and v, h E V then for the


map
9 : IR 3 t I---T (f(v + th), w*) ,
and an a depending on v + hh, t2 - tl and h, and approaching zero as
It2 - til -+ 0,

Hence g' exists everywhere and g' == 0, i.e., 9 = g(O) = (f(v), w*). If f is
not constant then for some x and h in V, f(x + h) -j:. f(x). The Hahn-
Banach theorem implies that for some w* in W*, the corresponding 9 is
not constant, a contradiction. 0
6.138. If T is II II-continuous, Xn ~ 0, and T (x n ) ~ 0, it may be
assumed that for some positive a, all n, and some x* in V*, I(T (x n ) , x*)1 2:
a, whereas (T (x n ) ,x*) = (xn' T* (x*)) -+ 0 as n -+ 00, a contradiction.
Conversely, if (6.3) is valid, the equation (v, S (v*)) ~f (T(v), v*) de-
fines S uniquely. Because II II-convergence implies weak convergence,

as n -+ 00. It follows that S (v*) E V*. The argument in Solution 6.112


shows that both S and T are II II-continuous. 0
6.139. It may be assumed that x* -j:. 0. There are sequences {an} nEN'
{mn}nEN' and {Yn}nEN such that

an 1 0, mn E M, d (x, M) > IIx + m nll - an


IIYnll ~ 1, I(Yn,x*)1 2: IIx*lI- an> O.

Then
(x,x*)Yn M
x- E
(Yn, x*)
d(x M) < l(x,x*)1 < l(x,x*)1
, -1(Yn,x*)I-lIx*ll-an
l(x,x*)1 = I(x + mn,x*)1 ~ Ilx*ll· (d(x,M) + an)
I(x, x*)1 _ a < d(x M) < I(x, x*)1 .
IIx*11 n - , - IIx*ll- an

o
6.140. a) The sequence space (function space)

Vi ~f { a ~f {an}nEN : ~ anxn II II-converges }


Solutions 6.141-6.142 355

normed according to the formula Iiall' ~f sUPN 112::=1


anxnll is, owing to
the uniform boundedness principle, a Banach space and

L anxn ~f x
00

T : VI 3 a f-t E V
n=1

is a continuous isomorphism between Banach spaces. Hence T- 1 is an


2 IIT-l II'
(x)
isomorphism and for all n, l(x,x~)1 ::; Ilxnll
b) The biorthogonality relations (xn'x;") = 8mn flow from the unique-
ness of the sequence {an}nEN in x = 2::=lanxn. If {xn'x~}l.:J{y,y*}
is biorthogonal then (xn'Y*) == 0 whence (x,y*) == 0 = (y,y*) = 1, a
contradiction. Hence B is maximal.
c) and d) The idempotency of each SN and Pn is also a consequence
of the uniqueness of the sequence {an} nEN associated with x. Furthermore,
IISn(x)11 and IlPn(x)1I ::; 2I1 T - 1 (x)II'·
e) The uniform boundedness principle applies. D
6.141. Each SN is a linear combination of the linear maps Pn (cf. 6.140),
each of which is continuous because each x~ is continuous.
If (6.4) obtains and y E L then for some N, y = SN(y) = S~(y). If
v E V then for some sequence {YkhEN in L, Yk IU Y whence
(s6.11)

The second term in the right member of (s6.11) is zero and the third term
does not exceed M IIYk - yll, whence X is a basis.
If X is a basis then span(X) is dense and 6.140e) obtains. D
6.142. If X is a basis then no Xn is 0, L is dense, and (6.5) obtains for
K = sUPN IISNII: a), b), and c) obtain.
Conversely, if a), b), and c) obtain, let LN be span ({Xn }:=I).
Then
{xd is linearly independent. If XN ~f {X n }:=1 is linearly independent and
X N +1 is linearly dependent, there are coefficients aI, ... ,aN+! such that

N+! N+!
L anxn = 0, L lanl > 0
n=1 n=1

a contradiction. Hence X is linearly independent and there is a contin-


uous linear idempotent RN : V 3 v f-t RN(V) E LN (cf. 6.123). The
proof in Solution 6.123 depends on the Hahn-Banach theorem used in
356 6. Topological Vector Spaces: Solutions

the extensions of the coefficient functionals. These extensions can be cho-


sen coherently so that RNRN+NI = R N .
If Ilxll = 1, a E (0,1), and N E N then for some N' greater than N
and some coefficients a1, ... , aN',

Furthermore, because Ilxll = 1,

N'
~ IIRNII·a+M I>nXn
n=1

~ IIRNII . a + M (a + Ilxll)

IIRNII ~ Mi~:a).
In sum, IIRNII ~ M,N E N.
°
If v E V and a > then for some N in N, any N' greater than N, and
for some y in LNI, Ilv - yll < M a . Hence
+1
IIRNI(V) - vii ~ IIRNI(V) - yll + Ily - xii = IIRNI (v - y)11 + Ily - vii
(M + 1)a
< M+1 =a,

Hence the maps x'N: V 3 v f---+ bN(v),n E N are in V* and {xn}nEN is a


~~. 0
6.143. If X is a weak basis for V then for each v in V, there is a
unique sequence {an}nEN such that E:=1 anXn ~ v. The argument for
6.141 may be used to define a norm II II' on the sequence space Vi. The
uniform boundedness principle applies and the proof given for 6.141 may
be repeated, mutatis mutandis. 0
Solution 6.144 357

6.144. For the associated coefficient functionals x~, if En ~f 2n l~x;;'11 and

llYn - Xn II < En, n EN, the following argument shows that Y ~ {yn} nEN
is also a basis (cf. 6.87).
If Zn ~ Xn - Yn and va E V then for some r in (0,1),

00 00

L Ilznll'llx~11 = r, L l(vo,x~)I'llznll < r Ilvoll,


n=l n=l


1.e., = ,",00
Y def *) .t
L ..m=l Va, xn Zn eX1S s.
(

Furthermore,

=L =L
00 00

Va (va, x~) Xn (va, x~) (Yn + zn)


n=l n=l

=L +L
00 00

(va, x~) Yn (va, x~) Zn ~f U1 + Vb


n=l n=l

whence, L denoting span(Y), U1 E £, and IIv111 ~ r Ilvoli. Similarly,

By induction there can be defined a sequence {vkhEN such that

Thus limk--+oo Ilvo - uk+111 = 0, i.e., span(Y) = V.


For a finite sequence {a n }:=l' the biorthogonality of {xn,x~}nEN im-
plies the fundamental inequality

t lanl·llznll = (t. sgn (an) Ilznll X~) (t anxn)

~ r II~anxnll. (s6.12)

If m ~ n then, because
358 6. Topological Vector Spaces: Solutions

(s6.12) implies B ~f IIE~=1 xnll :::; _1_II~Ykll


1-r ~
~f 1-r
~. Because X is
k=1
a basis, for some K, IIEZ'=1 akxkll :::; K IIE~=1 akxkll, whence

II~akYkll:::; II~akXkll + ~ak IIZkl1


:::; II~akXkll +rll~Xkll
:::; K II~akXkll + r II~Xkll = (K +r) II~akXkll

:::; ~ ~; II~akYkll·

Thus Y is a basis by virtue of 6.142. o


6.145. When V ~f qx] is regarded as a subspace of the Banach space
C ([0,1], q then {Pn(X) ~f xn} is a countable Hamel basis for V.
nEZ+
o
6.146. Let G be {0,1}N regarded as a compact group with respect to
component-wise addition (modulo 2). For normalized Haar measure in G,
the map T : G 3 9 ~ (E1. E2,"') f-+ E:'=1 ~: E [0,1] is measure-preserving
and continuous with respect to the product topology for G. Moreover, Tis
bijective off a null set in G.
a) The maps TN : G 3 9 f-+ E:=1 En (X, X~) Xn ~f TN(g)(X) are such
that for each gin G, TN(g) E [V], and for each x, TN(g)(X) is a continuous
function of g. Hence TN(g) is a Bochner measurable vector-valued map
and both C 1(x) ~ {g : limN,N'-+ oo IITN(g)(X) - TNI(g)(x)1I = O} and its
T-image C(x) ~f T (C1 (x» are measurable subsets of G.
b) For Z in V, B(z) ~f {g : limN--->ooTN(g)(z) exists} is measurable
and if Z ~f {Zm} mEN is dense in V then B ~f nmEN B (zm) is measurable
and C1 ~ nXEV C1(x) C B.
However, if 9 E B, x E V, M ~f SUPNEN IITN(g)ll, and a > 0, there is
a
a Zm such that Ilzm - xii < 2M' Then
a
IITN(g) - TNI(g)11 :::; II (TN (g) - TNI(g» (zm)11 + "2'
Because 9 E B, limN,N,---+oo II(TN(g) - TNI(g» (zm)11 = 0, whence 9 E C1 .
Hence B = C 1 and both C 1 and C ~f T (Ct) are measurable.
Solutions 6.147-6.148 359

For 9 in C1 and x in V let x(g) be E~=1 En (x,x~)xn. Then for h in


C1, x(g + h) = x(g)(h) whence C1 is a dense subgroup of G and so C is
dense in [0,1).
c) If KeN, a K-cylinder Z in G is a set determined by a set of subsets
A k , k E K: {g : {n E K} =:} {En E An} }. If K is finite Z is a finite cylinder
and if K = {n,n + 1, ... } then Z is a I n . If E is measurable and a I n for
each n while F is an arbitrary finite cylinder then f..L(E) = f..L(E)f..L(F). The
a-algebra of measurable subsets of G is generated by the finite cylinders,
whence f..L(E) = (f..L(E))2, whence f..L(E) = 0 or f..L(E) = 1. Thus f..L(Cd = 0
or f..L (C1 ) = 1 and .A(C) = 0 or .A(C) = 1.
d) If .A(C) = 1 then f. L (Cd = 1 = f..L(G). Because C1 is a measurable
subgroup of G, it follows that C1 = G whence C = [0,1). D
a
6.147. a) If -
n
= 1- -q1 + r then n,q,r,qn - qa,nqr
dcl
= d> 0 and

qn - qa =n- nqr =n - d, n = 1,2,


r
JB(O,b)
Ik(x - yW d.An(x) ~ cq r
JB(O,b)
Ix - ylqa-qn d.An(x)

~ cq r
JB(y,l)O
+cq 1 (B(O,b))\B(y,1)°
Ix - ylqa-qn d.An(x). (s6.13)

Because d > 0, if n = 1, the first term in the right member of (s6.13) is

r
JB(y,l)O
Ix - yl-1+d dx = 11 lul-1+d
-1
du = ~ < 00.
d

If n = 2, the first term in the right member of (s6.13) is

1B(y,l)O
Ix - yl-2+d d.A2(X) = 12"'11
0 0
r-1+d drd() 271" < 00.
= -d

Because 9 : B(O, b) \ B(y, 1)° 3 x 1--+ Ilx - yll-n+d is continuous and


bounded it follows that the second term in the right member of (s6.13) is
finite, n = 1,2.
b) The argument in a) shows that k E £q (IR n , .An) ,n = 1,2, and since
q > 1, from 6.43 it follows that for all I in £1 (IRn' .An), k * lEU (IRn, .An)
and Ilk * Illq ~ 11/111 ·llkll q· D
6.148. The relations Ilalll = lal·1I/1I and III +gll ~ 11/11 + Ilgll are ensured
by the basic properties of sup. Furthermore, if IIIII = 0 then I is a constant
and because 1(0) = 0, I = 0, whence II II is a true norm.
If {In} nEN is a Cauchy sequence in Ea then
360 6. Topological Vector Spaces: Solutions

whence limn->oo In{s) ~ I{s) exists. Furthermore, if sf:. t then

I/{s) - l{t)1 = lim I/n{s) - In{t)1 < lim Il/nll < 00,
Is - tl a n->oo Is - tl a - n->oo

whence I E Ea.
Finally,

I/{s) - I{t) - (fn{s) - In{t))1 .


11m I/m{s) - Im{t) - (fn{s) - fn{t)) I
Is - tl a m->oo Is - tl a
:::; m->oo
lim Ilfm - Inll,

whence Ea is complete. o
6.149. As in 6.148 it follows that II II is a norm, whence II II' is also a
norm. If fn is the function such that the graph of y = fn{x) is that shown
in Figure s6.1 then Ilfnll = n, IIfnlloo = .!.,
n
Il/nll' = n + .!..
n

y-axis

Line tangent at 0 to graph of y = .f" (x)

Horizontal asymptote for graph of y = j~ (x)

(0, ~) -------------------------------

o x-axis

Figure s6.1.
o
6.150. Because K is a bounded closed subset of a finite-dimensional space,
K is compact. 0
Solutions 6.151-6.153 361

6.151. The relation L1 (JR, A) nco (JR, C) ::) Coo (JR, C) and the II 1100-
denseness of Coo (JR, C) in Co (JR, C) show that L1 (JR, A) nCo(JR, C) is 111100-
dense in Co (JR, C).
If f(x) == vk: exp (- x;) then j(t) = vk: exp ( - t;), whence
L1 (JR, A)~, which is a subalgebra of Co (JR, C), contains a separating sub-
algebra generated by the translates of j The Stone-Weierstrafi theorem
applies. 0
6.152. a) The map B : P~ 3 {p, q} I--> L(p· q) defines a positive defi-
nite inner product. The Gram-Schmidt orthonormalization process applied
to the sequence {gk(X) ~f xk}n produces for Pn a B-orthonormal basis
k=O
{fk}~=O· If Pn+1 ~ gn+1 - L~=o L (gn+1 . fk) fk then L (Pn+1 . Pn) = o.
b) If Pn+ 1 has v real zeros, then 0 $ v $ n + 1. If v = 0 then n + 1
is even, say n+ 1 = 2m, and there are real numbers {rj,sj};:1 such that
Pn+1 (x) = n7=1 ((x - rj)2 + s3), a sum of squares of polynomials, whence
L (Pn+1) > 0, whereas, as found in a), L (Pn+1 . 1) = O.
Thus 1 $ v. If the v real zeros ofPn+1 are {Zj };=1 then n+1-v is even,
say n + 1- v = 2m, and Pn+1(X) = n;=1 (x - Zj)· n:1 ((x - ri)2 + sr).
If m?: 1 then
v v m

Q(x) ~ Pn+1(x)· IT (x - Zj) = IT (x - Zj)2. IT ((x - ri)2 + sn,


j=1 j=1 i=1

a sum of squares of polynomials, whence L( Q) > o. On the other hand,


because deg (n;=1 (x - Zj)) < n, a) implies L (Q) = O. Hence m = O.
c) If a) and b) hold for P:+1, deg (Pn+1) = n+ 1, and L (Pn+1 . P n ) = 0,
it may be assumed that the leading coefficient of Pn+1 is 1. Thus

deg (P:+1 - Pn+1) $ n


0< L ((Pn+1 - Pn+d 2)
= L (Pn+1 (Pn+1 - Pn+1)) - L (Pn+1 (Pn+1 - Pn+l)) = 0 - 0

whence Pn+1 = Pn+1· o


6.153. For n in N and fn(x) ~f nx[O 'n.!.J' IIfnlll = ..!.. Hence for any gin

(L! ([0,1]' A) r, lim n ---. oo gUn)


Ifsup!EB(O,1) IgU)1 = 00
= O.
2 n

then for some sequence {kn}nEN in B(O,l),

g (k n) ?: n. Owing to the continuity of g, Vn $Ig ( ~) I- 0 as n - 00,


362 6. Topological Vector Spaces: Solutions

a contradiction. Hence for some constant K and all I in V, IgU) I ::; KII/II!. 2
If [a, b] c [0,1],

IIX[a,b]II !
2
= (b - a)2 and Ig (X[a,b]) 1 ::; K(b - a)2

whence 9 (X[a,b]) is a count ably additive interval function that may be ex-
tended to a measure p on S{3 and p «>.. Consequently, for some h in
£1 ([0,1], >.) and all I in V, gU) = J01I(x)h(x) dx. In particular, ift E [0,1],
9 ( Un)[t] ) = nit
t+'!'-
n hex) dx ---> ° as n---> 00. Hence h == ° and g(V) = 0.
o
6.154. Viewed as the set of all £2 (X, p)-valued functions on the set

Y =
def {
1,2, ... ,n }

endowed with counting measure (, V is a generalized £4 (Y, (), denoted


£4 (Y,(,£2(X,p)) to emphasize the range of the functions. The Holder
and Minkowski inequalities and the completeness of £2 (X, p) ensure that
V is a Banach space.
If 9 ~f {gl,'" ,gn} E £~ (Y, (, £2(X, p)) then the map

n
Fg : £4 (Y, (, £2(x, p)) :7 I ~f {h, .. . ,In} r-+ L Uk, gk)
k=l
is in V*. Conversely, if F E V* then, (0, I, 0h denoting the vector for
which only the kth component I is nonzero, the map

hk : £2(X, p) :7 h r-+ F (0, I, 0h

is in (£2 (X,p))* [=£2(X,p)] and F(h, ... ,ln) = L.~=lUk,hk)' The


4 l!.
Holder and Minkowski inequalities imply that IIFII = (L.~=1 Ilhk Iii) 4.
o
6.155. a) By virtue of the open mapping theorem, T is open and thus for
some positive r, T(B(O,r)) :J B(O, 1). Hence, en denoting {8nm }mEN' for
some Xn in B(O,r), T(xn) = en,n E N. IfT(x) = {an}nEN then

L L anXn ~f y exists, z ~f x - Y E ker(T)


00 00

lanl'llxnll < 00,


n=l n=l
Solutions 6.156-6.159 363

whence the map Q : V :3 x ~ L~=l anxn is continuous and linear. Because


Q (xn) = Xn it follows that Q2 = Q, i.e., that Q is a continuous idempotent.
Thus id - Q ~f P is also a continuous idempotent and P{V) = ker(T).
b) Because x = P(x) + Q(x) ~f U + v and P(V) n Q{V) = {O}, the
direct sum Z ~f ker(T) EB im( Q) is defined. As the image of a projection,
Q(V) is closed, hence a Banach space, and F : Z :3 {x, y} ~ x + Y E V is
a continuous surjection, whence F is open. D
6.156. If K = 1, it may be assumed that Yl ~ M. The Hahn-Banach
theorem implies that for some y* in M ~, (Yl, y*) = 1. By definition, each y
in Ml ~f span (Yl, M) is for some a in C and some m in M, uniquely of the
form y = aYl + m. If {zn}nEN C Ml and Zn = anYl + mn --+ Z as n --+ 00,
then an = (zn, y*) --+ (z, y*) ~f a and mn = Zn - anYl --+ Z - aYl as
n --+ 00. Because M is closed, mE M whence Ml is closed. Mathematical
induction implies the result for any finite K. D
6.157. a) If sm(x) ~f L~=llanl'lfn(x)1 then Sm :::; Sm+! and {sm}mEN
is a lillI-Cauchy sequence with a limit S in Ll(X,/L). Hence it follows that
F(x) ~f L~=l anfn(x) exists a.e., FE Ll(X,/L), and 11F111 :::; Ilslll.
b) If an = rneiIJn and fn(x) = Ifn(x)1 eicPnex) , 0 :::; On, ¢n < 211" then
(6.6) holds iff for all n in N, On + ¢n(X) == 0 (mod 211").
c) The equation (6.6) holds for all sequences {fn}nEN iff at most one
an is different from zero.
d) The equation (6.6) holds for all sequences {an} nEN iff at most one
f n differs from zero a.e. D
6.158. Because V and Ware separable, #(V) = #(W) = c. From 6.114
it follows that the cardinality of a Hamel basis for V or W is c. Any bijection
between Hamel bases for V and W may be extended to an isomorphism of
V and W. D
6.159. Because II lloo-convergence is uniform convergence, V is closed.
The convexity of V follows from direct calculation.
Because 1 = IIo! f(x) dx - It f(x) dxl :::; IIflloo, each f in V is outside
B(O,1)°. Furthermore, if {j, t, 17 > 0 and 17 = ~ + (j(1 + t), the function

l+t ifO<x<~-{j
- -2
_l;t(x_~) if~-{j<x<~+{j
2 2
-(1 + t) if~+{j<x<l
2 --
364 6. Topological Vector Spaces: Solutions

is in V and Ilf€llex> = 1+ € whence inf {llfllex> :f E V} = 1. However, if


9 E V and Ilgllex> = 1 then

1°! 1 -
g(x) dx ::; -,
2 !2
1
i 1
g(x) dx ::; -,
2
1!° g(x) - i!2
1
g(x) dx ::; 1

and equality obtains iff g(x)l[o,!]= 1, g(x)I[!,l]= -1, which is impossible


if 9 is continuous. 0
6.160. By definition, Mu(x) ;::: 0 = Mu(O). If 0 =I- t = Itiei9 E C
then x E aU iff tx E Itlae i9 U (= ItiaU because U is circled). Thus
Mu(tx) = ItIMu(x).
If 13, -y > 0, x = j3u E j3U, Y = -yv E -yU then Mu(x) = 13, Mu(y) = -y,
and, because U is convex,

x + y = (13 + -y) (j3! -y u + 13 ~ -y v) E (13 + -y)U.

Hence Mu(x + y) ::; 13 + -y = Mu(x) + Mu(y). o


6.161. a) ~ b). If € > 0 and for all v in N(O), p(v) < € then when
x-y E N(O), the triangle inequality implies Ip(x) -p(y)1 ::; Ip(x-y)1 < €.
b) ~ c). Because U = p-l (( -00,1» and p is continuous, U is open.
The basic properties of a seminorm assure that U is convex and circled.
x
c) ~ d). If Mu(x) = 13 and a > 13 then x E aU, whence - E U,
a
whence p (;) < 1, whence p(x) < a, whence p(x) ::; 13 = Mu(x). If

p(x) = 13 and a > 13 then p (~) < 1, whence ~ E U, whence x E j3U,

whence Mu(x) ::; 13 = p(x).


In sum: p::; Mu ::; p.
d) ~ a). If € > 0 and x E

"2Y
then Mu (x) = p(x) ::;

2<€ whence p
is continuous at O. 0
6.162. If II II induces the topology of E then U ~ {x : II xii ::; I} is a
convex circled neighborhood of O. The set U is bounded because if r > 0
2
then -B(O, r) ::) U.
r
Conversely, if N (0) is bounded and convex then for some circled neigh-
borhood W of 0, W + We N(O) and Conv(W) ~ U c N(O). Hence U
is also bounded. The Minkowski functional Mu is a seminorm p. If x =I- 0
then, because E is a Hausdorff space, for some neighborhood Z of 0, x (j. Z.
1
Because U is bounded, for some positive A, U c AZ, i.e., >..U c Z whence

x (j. ~U and so Mu(x) = p(x) > ~ > 0, i.e., p is a norm, say II II. Because
U is bounded anJ U = B(O,l), it follows that for any neighborhood Y
Solutions 6.163-6.167 365

of 0, there is a positive .x such that B ( 0, ~) C Y: the II II-topology is


stronger than the given topology. On the other hand, for each n in N, there
1
is an N(O)n such that nN(O)n C uo, whence if x E N(O)n then Ilxll < -,
n
i.e., P is continuous at O. Thus B(O, 1)0 is open (cf. 6.161) and so the
II II-topology is weaker than the given topology. D

6.4. Banach Algebras

6.163. Because nkx - x = n (n k- 1 - x) + nx - x, the result follows by


induction. D
6.164. a) If z = rei8 , r < 1 then I(z) = 2:~1 anz n = 2::=1 anrnein8.
The Cauchy formula implies that for n in Z,

~
ior
27r
while fin = 9 (e i8 ) e- in8 dO. Because I is uniformly continuous
v21l"
on D(O, 1), it follows that fin = ~an. Note that an = 0 if n < o.
V 21l"
b) Because 9 is continuous, Fejer's theorem implies that 9 is the uniform
limit of a sequence {Pn} nEN contained in C [e i8 ]. The maximum modulus
principle implies that III - Pnll oo ---- 0 as n ---- 00. D
6.165. For PI: D(O, 1) 3 z t--t z, if h(Pl) = Zh then h(pf) = zh and so
for any polynomial p, h(p) = P (Zh). Hence 6.164b) applies. D

6.166. Direct calculation shows that IIr*lloo ~ (~/~17)!. Consideration


1

of Maclaurin's series for eX shows that limn -+ oo (~!) n = O. D

6.167. Let M be ker(h), a closed maximal ideal. If, for each a in X


and some la in M, I(a)(a) =f. 0 then in some N(a), la(x) =f. O. How-
ever, X is compact and UaEX N(a) ::> X, whence for some {aih~i~n<oo'
U~=1 N (ai) ::> X. Then Iial = laJai and 0 < 2:~=llla.l2 ~ I E M,
whence 71 E A(X) and 1 EM, a contradiction. Thus for some ah in X and

for all I in M, I (ah) = o. Furthermore, J ~f {k : k E A(X), k (ah) = O}


is a proper ideal containing M, i.e., J = M. Thus hi : A(X) 3 9 t--t 9 (ah) E
C and h have the same kernel, i.e., h = hi. D
366 6. Topological Vector Spaces: Solutions

[Note 86.14: If X is locally compact the result is still valid. The


argument bears on the construction of the algebra Ae obtained by
adjoining a formal identity e to A:

Ae ~f { Ae + J : A E C, J E A} .
The crucial point is the existence of a bijection between M(A)
and M (Ae) \ {A} [Loo].]

6.168. a) The translation-invariance of /-l implies

J--;-g(O) = i (iJ(y-1x)g(Y)d/-l(Y)) (o,x)d/-l(x)

= i ( i f(z)g(y)(o, yz) d/-l(Y)) d/-l(z)

=i J(z)(o, z) ( i g(y)(o, y) d/-l(Y)) d/-l(z) = 1(0)9(0).

b) If h(f) -:f. 0 and g E Ll(G,/-l) then J * g[x] = J[x] *g and so

h(f)h (g[x]) = h (J[x]) h(g)) whence Oh(X) ~f h~(;J) is J-free. Further-

h (J[xy]) (h (J[X])) (h (J[y])) .


more, h(f) = h(f) . h(f) ,I.e., Oh(XY) = Oh(X) ·Oh(Y)·
Because 10h(X)1 ::; ~~)' Oh is a bounded function of x. If 10h(X)1 > 1
then 10h (xn)1 i 00, a contradiction. Hence 10hi ::; 1. On the other hand,
Oh(X)Oh (x-I) = oh(e) = 1 whence 10hi = 1.
The Riesz representation theorem implies that for some (3 in L 00 (B, /-l),
h(f) = Je J(x)j3(x) d/-l(x). If n is an approximate identity for A, then
h (n[x]) = h(n)oh(x). Because n * g - g IIJl 0, it follows that

Ih(g) (h(n) - 1) I::; lin * g - gil ~ 0,

i.e., hen) ~ 1 and, for h fixed, h (n[x]) ~ Oh(X). Hence for any J in
Ll(G, /-l),

[ J(X)Oh(X) d/-l(x) = lim [ J(x)h (n(x-l)) d/-l(x)


le le
(1
n

= li;n i J(x)n (X-ly) j3(y) d/-l(Y)) d/-l(x)

le * n)(y)j3(y) d/-l(Y)
= lim [ (f
n

= i J(y)j3(y) d/-l(Y) = h(f). 0


Solutions 6.169-6.172 367

6.169. If G is discrete then X{e} is the identity for A(G). Conversely, if u


is an identity for A(G) and for each neighborhood U(e), nu =~f I-£(~)xu
then U f--+ nu is an approximate identity. Thus u * nu = nu 1I-!1 u. Hence

u(x) = 0 off {e}, whence I-£(e) > O. Thus, since the measure of every
compact set is finite, a set is compact iff it is empty or finite. Note that G
is locally compact. 0
6.170. Let u be an identity modulo I. If x E I and Ilu - xii ~f Ilzll <1
t h en w def ,,",00 n·
= - L.m=l Z eX1sts an
d

(u - x)w - (u - x) - w = zw - z - w = O.

Since x, xw, uw - WEI, it follows that u E I, a contradiction. Hence the


distance of u from I is at least one and so u ¢.1, in particular, 1 ~A.
The continuity of the algebraic operations implies that if x, y Eland
z E A then x + y, zx Eland, as a proper subalgebra of A, 1 is an ideal.
o
6.171. Let I be the set of bounded functions in A(G). If f E A(G) and
bEl, then 9 ~f f*b E A(G) and Ig(x)1 ~ Ilbll oo ·lIg111 < 00, whence I is an
ideal. Because every f in A(G) is the limit of simple functions and these
are in I, 1 = A(G). 0
6.172. Generally, if T, S E [A] and T S - ST and S commute, the endo-
morphism ~ : [A] 3 T f--+ TS - ST is a derivation on [A]. Because TS - ST
and S commute, ~2(T) = 0 whence by Leibniz's rule, ~k (Tl) = 0 if
k > t. On the other hand, ~2 (T2) = 2 (~(T))2, whence by induction and
Leibniz's rule, it follows that ~n (Tn) = n! (~(T))n. However if M is the
norm of the operator ~, then lI~n (Tn) II ~ MnllTlin whence

lim II
n-+(x)
(~(T)t II~ ~ lim MW;II = 0,
n~oo n! n

i.e., ~(T)is a generalized nilpotent in [A].


For x in A, let x' denote D(x) and let Rx denote the endomorphism
A 3 z f--+ xz.
If ~ is the map D f--+ DRx - RxD, the conclusion above is that ~(D)
(= Rx ') is a generalized nilpotent in [A] and thus that x' is a generalized
nilpotent in A. Since A is commutative the result follows. 0
[Note 86.15: The argument above permits the interesting con-
clusion that Coo (~, C), regarded as an algebra A with respect to
the natural definitions of addition and multiplication, cannot be
endowed with a norm II \I with respect to which A is a Banach
algebra.
368 6. Topological Vector Spaces: Solutions

Indeed, if A is a Banach algebra with respect to some norm


II II then the Banach algebra [A] of continuous endomorphisms of
A consists of bounded endomorphisms T : A 3 f I-> T(f) E A
such that IITII ~f sup { IIT(f) II : IIfll = I} < 00. The maps

D:f I-> D(f) ~ f' and, for 9 in A, Rg : f I-> gf

are in [A] and D is a derivation: D(fg) = D(f)g + f D(g).


Because DRg - RgD = R g, and RgRg' = Rg,Rg, it follows
that Rg and DRg - RgD commute.
Fix to in JR and let ¢to be the functional that maps each f in
A into f'(to).
Because f I-> f(to) and f I-> f(to + ~) are in [A, q, they are
II II-continuous. Thus for n in N,

Ln : A 3 f I-> f(to + ~2
n
- f(to) ~f Ln(f)

are II II-continuous. For all f in A, Ln (f) ---+ ¢to (f) as n ---+ 00,
whence each ¢to is a II II-continuous linear functional. The closed
graph theorem [Rud] implies that D is II II-continuous: DE [A].
If t E JR, the evaluation map E t : A 3 f I-> f(t) E C is in
[A, q whence IEt(f)1 = If(t)1 ~ Ilfll· Thus Ilflloo ~ Ilfll· Hence
in the context of 6.172
1 1

II(g'tll n ~ II (g't II~ = 119'1100


whence g' = 0. Because 9 is an arbitrary element of A, it follows
that D = 0, a contradiction [SiW].
An alternative to the argument in the last paragraph stems
from the characterization of the set N of generalized nilpotents of
a commutative Banach algebra B as the radical 'R(B) of B, i.e., N
is the intersection of the kernels of all algebraic homomorphisms
of B into C: N = 'R = nhE[B,q ker(h) [Be, Loo]. When B = A,
among the elements of [B, q are the evaluation maps E t described
above. Hence the generalized nilpotent g' vanishes at each t in JR,
i.e., g' = 0, and the contradiction achieved earlier is repeated.]
COMPLEX ANALYSIS:

SOLUTIONS
7
Elementary Theory

7.1. Geometry in C

7.1. A circle on E is the intersection of E with a plane IT for which the


equation is a~ + brJ + c( = a 2 + b2 + ~. The plane IT and E intersect iff
a 2 + b2 + c2 ~ 1. The equation e
+ 1]2 + (2 = 1 and the formulre for the
coordinates of e(~, 1], () ~ (x, y) lead to the equation

(a 2 + b2 + c2 _ c) (x 2 + y2) - 2ax - 2by + a2 + b2 + c2 + c = °


representing a circle in C or, if a 2 + b2 + c2 = c, a straight line in Co The
latter circumstances imply that IT passes through (0,0,1). The reasoning
is reversible and leads from a circle in C to a circle on E \ {(O, 0, I)} or from
a straight line in C to a circle passing through (0,0,1) on E. 0
7.2. a) Because 6.(pqr) = {ap + (3q + ,r : a, (3" ~ 0, a + (3 +, = I},
t
the convex hull of the set {p, q, r}, (6. (pqr ) corresponds to the supplemen-
°
tary conditions ath > and the sides [pq] resp. [qr], resp. [rp] of 6.(pqr)
°
correspond to the supplementary conditions, = resp. a = resp. (3 = 0. °
b) If 1 > r > max{lpl, Iql}, choose z so that 11 - zl = VI - r2 ~ r1
and Izl = r. The lines {l,z} resp. {1,z} are tangent to Co(r) at z resp. z.
Moreover, 1f. sED ( ) then R(s ) ~ -
O,r 1 +-r def
= K . 1
l-r
°
Let 2(}o denote the size of the angle L (z 1 z). Then < 200 < 7r and if
s E 6. (zz 1) then for some ¢ in [7r - 00 , 7r + 00 ] and pin (0, rl], l-s ~f pei </>.
Thus

Isl 2 = 1 + p2 _ 2pcos¢ ~ 1 + pcos(}o _ 2pcos Oo ~ (1- PC~(}o) 2


p cos (}o 2 def
and so I-lsi>
2
, R(s) ~ --0-
cos 0
= K 2· Because

6.(pql) c D(O,r) U 6. (zzl)

it follows that R(s) ~ max{Kl,K2 } ~f K(p,q). o


7.3. a) If Tl ~ Tabed then T1(z) = z iff z is a solution of the quadratic
equation cz 2 + (d - a)z - b = 0. There is no solution iff c = d - a = =I- b. °
°
[If c = d - a = b = then T1 = id and every z is a fixed point.]

371

B. R. Gelbaum, Problems in Real and Complex Analysis


© Springer-Verlag New York, Inc. 1992
372 7. Elementary Theory: Solutions

b) If Tl fixes no point in C then Tl fixes (0,0,1) in E.


c) If TlT2 = T2Tl then TlT2 (Zl) = T2 (Zl) whence T2 (zt) = Zl, e.g.,
Zl = Z2. Furthermore, T2Tl (Z3) = Tl (Z3) whence 0:) Tl (Z3) = Zl or (3)
Tl (Z3) = Z3. Each of 0:) and (3) implies the contradiction Z3 = Zl.
[Note s7.1: When X is a set and G is a group, G is said to act on
X iff for 0:, (3 in G and x in X, o:(x) E X and o:«(3(x)) = (o:(3)(x).
If 1. G acts on X, 2. 0: fixes exactly one point, and 3. (3 fixes
exactly two points then 0:(3 =1= (30:. The argument is in no way
different from that given in c).
On the other hand, in the context of c), an alternative (ele-
mentary?) proof can be constructed on the basis of the following
directly verifiable statements and references:

i. For T ~f Tabcd, T(z) = z iff cz 2 + (d - a)z - b = 0 [Solution


7.3a)].
ii. (d l - al)2 + 4blCI = 0 =1= (d 2 - a2)2 + 4b2C2 =1= o.
iii. If S = Tpqrs then ST = TS iff

[Solution 7.12b)]. ]

d) If Tl (and hence T 2) has no fixed point then Cl = C2 = 0 and each


Ti is a translation whence TlT2 = T 2T l . If Tl and T2 have fixed points and
they are shared then CIC2 =1= O. Since if 0: =1= 0 then Tabcd = T(aa)(ab)(ac)(ad) ,
it may be assumed that CI = C2 = 1. Because a fixed point of Ti is a zero of
Z2 + (di - ai) Z - bi it follows that d l - al = d2 - a2 ~f 8 and bl = b2 ~f b.
Direct calculation then shows that TlT2 = T 2T 1 .
e) Since IT'(z)1 = Icz+dl- 2, it follows that the equation of the isometric
circle is Icz + dl = 1. The center is at - ~ and the radius is I~I· D
7.4. Direct calculation shows that

(W - WI) (W2 - W3) ~f Pw + Q ~f M(w)


(w - W2) (WI - W3) Rw + S
= (z - zt) (Z2 - Z3) ~f pz + q ~f N(z)
(z - Z2) (ZI - Z3) rz + s

establishes a bijective correspondence between the extended z-plane and


the extended w-plane, and that

w = (Sp - Qr)z + (Sq - Qs) ~f az + b ~f T(z).


(Pr - Rp)z + (Ps - Rq) cz + d
Solutions 7.5-7.7 373

1
It follows that N (ZI) = 0, resp. N (Z2) = 0 whence M (T (ZI)) = 0 resp.
1
= 0 and so W = WI resp. W = W2· If Z = Z3 then N(z) = 1
M (T (Z2))
whence M(w) = 1. Because the maps M,N are Mobius transformations
the equation M(w) = 1 has the unique solution w = W3.
If Z is a Mobius transformation and Z (Zi) = Wi,1 ::; i ::; 3, then
ZT- I (Wi) = Wi whence ZT- I is a Mobius transformation with three fixed
points and so ZT- I = id. In a word, T is unique. D
7.5. For Z as in Solution 7.4,

ZT- I ((T(q), T(r), T(s))} = (Z(q), Z(r), Z(s)) = (0,00,1)


whence

X (T(p),T(q),T(r),T(s)) = ZT- I (T(p)) = Z(p)) = X(p,q,r,s). D

7.6. a) Direct calculation shows that Iz - pi = klz - ql is the equation of


a circle Ca(r) resp. a straight line L according as k :-:J. 1 resp. k = 1. When
_ kip - ql _p- k 2q _ P
k :-:J. 1 then r - 11 _ k 2 1 ' a - 1 _ k2 ' and p - q . When k = 1 and ¢ is
the unique point in [0,27r) such that p - q = Ip - qlei 4> then L is the image
,* of lR under the map, : lR :3 t 1--+ p; q + te i ( 4>- ~) and again p = qP.
For a given K and a p not in K, the preceding formulre permit the
determination of q so that Iz - pi = klz - ql is the equation of K. Then
W E T(K) iff T-I(w) E K, i.e., iff IT-I(w) - = IT-I(w) - k or pi ql .
equivalently, when (cp + d)(cq + d) :-:J. 0,

I w - T(p)
w - T(q) -
I-I ++ I· cq
cp
d
d
k
.
(s7.1)

When 0 = cp +d then cq +d :-:J. 0 and the corresponding equation is


Iw - T(q)1 = lap: bl. A similar formula obtains if cq + d = O. Note the
special case in which k = 1.
b ) Note that (s7.1) shows that each of T (q), T (p) is the reflection of the
other in T(K), a relationship valid as well when (cp+d)(cq+d) = O. D

7.7. If T(U) = U and z E au <!;bf K then IT(z)1 ::; 1 and if T(z) E U then
z E T-I(U), a contradiction. Hence T(K) c K. Since the same argument
applies to T- I , it follows that T(K) = K. In particular, T-I(O) and
T- I (00) must be reflections of each other in K. If a = 0 then T- I (0) ¢. U,
a contradiction; if c = 0 then T is a translation of a dilation or contraction
and cannot map U onto U unless b = 0 and lal = 1.
374 7. Elementary Theory: Solutions

Because T -l( 0) = --b and T -1 (00) . follows that U


d It
= --, b
'3 - - =
def
a c a
d I a n Z-adef an
a and -- = =. Thus T(z) = - - . --_- = --. <I>a(z). Because
c a c 1- az c
'9 Z - a
IT(I)1 = 1, it follows that for some () in [0,211"), T(z) = e' - - =
I-nz
o
7.8. When Z - a = Iz - ale i 4>, 0:::; 4>
< 211", reference to Figure 7.1 shows
r2 r2
that k = 1 12 whence zP = a + =--=. 0
z-a z-a
7.9. Since the cross-ratio is invariant with respect to Mobius transforma-
tions and since any three points on a circle or a line can be mapped via a
Mobius transformation into any three points on the same circle or line, it
follows that if the equation is valid, its validity is independent of the choice
of Zl, Z2, Z3'
Let Zl, Z2, Z3 determine the circle Ca(r). The equations below obtain
by virtue of the Mobius transformations involved.

x (Z,Zl,Z2,Z3) = X (z - a,Zl - a,Z2 - a,Z3 - a)


=x(z-a,~,~,~)
Zl - a Z2 - a Z3 - a

= X (_ r2 _, Zl - a, Z2 - a, Z3 - a)
z-a

= X (_r2 _ +a,Zl,Z2,Z3)'
z-a

Hence if X (Z',Zl,Z2,Z3) = X(Z,Zl,Z2,Z3) then (z'-a) (z-a) = r2.


When the three points are on a line L, Z2 may be chosen to be 00
, - -
. t h e context 0 f t he problem, -
whence, In Z -- Zl Z - Zl
- = =--=:, wh'ICh'ImpI'leS
Z3 - Zl Z3 - Zl
that for any zion L, Iz - zll = Iz' - zll, i.e., L 1- [z,z'j. However, (s7.1)
implies as well that ~ (Z' - Zl) = _~ ( Z - Zl ), i.e., that z' and Z lie in
Z3 - Zl Z3 - Zl
opposite half-planes determined by L.
All arguments are reversible. 0
7.10. If (), 4> E [0,211"), P - q = aei9 , and q - r = bei4> then

p- q = l!.e i (9-4»
q-r b

which is real iff () - 4> = O. o


7.11. a) See Solution 7.5.
Solutions 7.12-7.14 375

b) Let R be as in Solution 7.5. Then

{X (Z1' Z2, Z3, Z4) E lR.} {:} {R (Z1) ~ r E lR.}


{:} {Zl, Z2, Z3, Z4 E R(lR.)} . o

7.12. a) Direct calculation and the identity det(AB) = det(A) det(B)


shows that Mo is a group with respect to composition. Furthermore,

b- ad
__e_+~ ifel:O
Tabcd(Z) ={ ez + d e
a b
-Z+- 1'f e= 0
d d

b) For a given Tabcd in Mo it may be assumed that ad - be = 1.


The map M : Mo 3 Tabcd f--+ (~ :) ~M (Tabcd) [in 8L(2, C)] satisfies
M (Tabcd 0 Tpqrs) = M (Tabcd) . M (Tpqrs).
e) Owing to a), it suffices to verify that each Tab and To is the compo-
sition of an even number of reflections.
i. If b I: 0, let L1 and L2 be two lines perpendicular to the segment Ob
and such that L1 passes through 0 while L2 passes through b/2. Then
for each Z in L 1, reflection in L1 leaves Z unchanged and reflection in
L2 yields Z + b. Thus for any Z in L 1, TOb(Z) is the composition of
two reflections applied to z. Since # (L 1) > 3, 7.4 implies TOb is the
composition of two reflections.
ii. If () > 0 then TeiB ,0 is the composition of two reflections, the first
about a line L1 through 0 and the second about a line L2 through 0
and making an angle of size () /2 with L 1 .
iii. If a > 0 then Tao is the composition of a reflection in Co(1) followed
by a reflection in Co (Va).
iv. Reflection in {z : ~(z) = O} followed by reflection in Co(1) is the
same as To.
o
7.13. ~ (a(x + iy) + b) = (ad - be)y 0
e(x + iy) + d (ex + d)2 + y2'
7.14. A reflection in Ca(r) followed by a reflection in a line through a
reduces the problem to the case in which Cb(S) C D(a, r)o. If b = a, there
376 7. Elementary Theory: Solutions

is no problem. If b =f. a, the composition of Toa and Tl.o further reduces


the problem to the cases where Cb(S) C U and Ca(r) ~ Co(l). Finally,
7.4 may be used to transform Cb(S) into some Co (s') while C o(l) is left
invariant. 0
7.15. Since e rei9 = ercoslJeirsinlJ, the limit exists iff cos() < 0, i.e., iff
7r 37r
- < () < -. 0
2 2

7.2. Polynomials

7.16. Because

1 + z - (1 + n)zn + zn+l
1-z
1
< 1 then for large n, 1(1 + n)zn - zn+ll < - - 1, whence, for
if Izl ::; r
r
large n, IPn(z) I > o. 0
7.17. Note that when a = 0 the 2n zeros of fn(z) are, for k = 0, ... , 2n-1,
b...L
2n e
i(2k+l)"
2n •
Telr
hlmagmary
··· ···ff
part s are posltIve 1 0 <
_ k <_ n - 1.

If a =f. 0 then 1Z2n + b - f n (z) 1 = 1az 2n - l l. For all sufficiently small


positive E and all sufficiently large positive R, the contour r consisting of
the horizontal diameter and the upper half of the circumference of D(O, R)
is, by abuse of language, a simple closed rectifiable curve and the bounded
component C of C \ r contains all the zeros of z2n + b that lie in the
upper half-plane TI+ ~f {z : ~z > o}. For sufficiently large positive R,
bl
if Izl > R then laz 2n - l l < Iz2n + and RoucM's theorem implies that n,
the number of zeros of z2n + b in C is the same as the number of zeros of
fn(z) in C. 0

7.18. ForninN,letAnbe{z: nz=n,zED(O,~)}. It follows

that UnEN An = D (0, ~), whence for some n', # (AnI) = c and some z'
in D (O,~) is a limit point of AnI. Hence f(n/)(z) = 0on AnI and so

f(nl)(z) = 0 on U. 0
7.19. It may be assumed that ao = 1 whence
(1- z)f(z) = 1- ((1- ad z + ... + (an-l - an) zn + anZ n+l )
d~ 1- g(z).
Solutions 7.20-7.24 377

If Z E D(O, 1) \ {I} then Ig(z)1 < 1 and so 1 - g(z) =I- 0, i.e., fez) =I- O.
Furthermore, f(l) > 1. D
7.20. The number of components of E(a) is the number of pairwise
different zeros of p - a. D
7.21. a) If zEn, there are n numbers Wi(Z) such that p(Wi(Z)) = z.
Thus if few) ~f I1~=1 (w - Wi(Z)) + Z then
n
J'(w) = LIT (w - Wj(Z))
i=l # i

J' (Wi(Z)) = IT (Wi(Z) - Wj(Z)) =I- O.


#i

For each i, there is a positive ri such that D (Wi(Z), ri) n D (Wj(z), rj) = 0
if i =I- j while, by virtue of the inverse function theorem, f is injective on
Wi ~ D(wi(z),rit. Furthermore, f(Wi ) ~f Vi is open and
n
f (Wi(Z)) = Z E V ~ nVi,
i=l

n
whence is open.
b) The inverse function theorem implies also that Wi E H(n), whence
q(z) = 'L~=1 q (Wi(Z)) is holomorphic in n. D

7.3. Power Series

7.22. When so(z) = 1, sn(z) = 1 + Z + ... + zn, n E N, Abel summa-


tion implies 'L~=o cnz n = 'L~';:; (cn - cn+d sn(z) + CNSN(Z). If Z E K,
ISn(z)1 ~ ~,n E N, and the WeierstraB M-test applies. D
7.23. The Arzela-Ascoli theorem and Cauchy's formula imply there is a
subsequence {fmk hEN converging uniformly on every compact set to some
f in e. From Cauchy's formula it follows that fez) = 'L~=ocnzn. The
same argument shows that every subsequence of {fm}mEN contains a sub-
subsequence converging on every compact set to f. If fm ..p f on some
compact set K then for some subsequence {fml } , and some positive 8,
k kEN
SUPzEK Ifm~ (z) - f(z)1 ~ 8, in contradiction of the preceding sentence.
D
7.24. Because sUPk>n IClkC2ki ~ sUPk>n \elkl' sUPk>n IC2kl, Rc ~ R C1 R c2 ·
On the other hand, direct calculation shows that an example as required is
378 7. Elementary Theory: Solutions

provided when

1 if n is even { 0 if n is odd
Cl n ={0 otherwise ' C2n = 1 otherwise '

in which case RCl= RC2 = 1 < 00 = Rc· D


7.25. a) When z = x + iy, Inzi = le(x+iy)lnnl = e x1nn . Hence if x> 1,
L::=lln-zi = L::=l e- x1nn = L:~l n- x < 00. It follows that «(z) is
holomorphic in n ~f {z : ?R(z) > 1}. Furthermore, if y = 0 and x 1 1
then «(z) i 00 whence 1 E S(). Thus the coefficients Cn may be calculated
by termwise differentiation of ,",00 ~:
= L:~l (-1)k(ln n)k /n 2
Ck
L.m=l nZ k!
b) Since 1 E S() and «(z) is holomorphic in n, it follows that Rc = 1.
D
7.26. The Gutzmer coefficient estimateL::=o Icn 2 l r2k ::; (M(r; 1))2 and
the hypothesis imply that Cn = 0 if n =I- k. D
7.27. If Cn ~ 0 and 1 is not a singularity of f then the power series
f(n) (1/2)
L::=o , (z - 1/2t converges for some z in (1,2). However, since
n.
Cn ~ 0,

The left member of (s7.2) is L::=o cnz n , which diverges when Izi > 1, a
contradiction.
If, more generally, ?R (cn ) ~ 0 then for g(z) ~f L::=o?R (cn ) zn, it fol-
g(n) (1/2)
lows that L:~o , (z - 1/2t diverges when z > 1. On the other
n.
hand, g(n) (1/2) = ?R (J(n) (1/2)), whence L::=o f(n) \1/2) (z _1/2)n di-
n.
verges. D
Solutions 7.28-7.32 379

7.28. According to 7.27, if Ra = 1 then 1 E 8(1). Hence R(a) = 1+8 > l.


If 8 < t, the same argument implies 1 + 8 E 8(1), a contradiction. 0
7.29. The hypothesis implies # (8(1) n T) < 00, whence for some a on T,
some N(a), and some M, if z E Un N(a) then 12:;;=0 Cnznl ::; M, N E N.
It follows that Icnznl ::; 2M, n E N, for all such z. 0
7.30. The Maclaurin series coefficients for Pk are in Z. Furthermore,
2::=0 nzn = z (_1_)' = ( z )2 and direct calculation shows that
z-l 1-z
00 z(z + 1)· .... (z + k -1) def Qk(Z)
for k = 2,3, ... , 2:n=on k z n = (1- z)k+1 = (1- z)k+1
whence Qk E N[z] and deg (Qk) = k. Thus

~ n k+1zn = z( Qk(Z) )'


~ (1- z)k+1
n=O
(1 - z)Q~(z) + (k + l)Qk(z) def Nk(z)
=Z (1 - z)k+2
= Z ..,...,...--"-7+-:-;::-
(1 - Z)k+2·

Induction shows that NkP+1)(z) = (1 - Z)Qi;+2)(Z) + (k - p)Qi;+1) (z),


whence if k 2: p then NkP +1)(0) = Qi;+2) (0) + (k - p)Qi;+1) (0) > O. Since
Nk E Z[z], it follows that Nk E N[z]. 0

7.31. If 0 ::; r < 1 then, as noted in 7.30, 2::=o(n + l)rn = (1 ~ r)2.


Hence for m in N, the absolute values of the functions in K are uniformly
bounded in D (0,1 - ~) ~f Bm. The Cauchy integral formula implies
they are equicontinuous on D(O, 1). It follows that the Arzela-Ascoli the-
orem is applicable to any sequence {lnIB} m nEN
in K. Thus K contains
a double sequence {gmn} m,nEN such that {gmn} nEN C {gm+1,n} nEN and
for each m, {gmn}nEN converges uniformly on Bm. The diagonal sequence
{gnn} nEN converges uniformly on each compact subset of U. 0
i() L(
7•32 . a ) When z -- e, ) _ 2· i(2n+1)~
z, n - ~e
,,0
2 L.Jk=n-1
sin (n - k - 1/2) ()
n_k .
According to 3.60, the sequence

. () sin 2()
{ sm sin 3() sin(2n - 2)() sin(2n - 1)0 }
+ - - + - - + ... + + --'------'-
2 2 n n nEN

is uniformly bounded on [-71",71"]. It follows from the maximum modulus


theorem that the sequence {L(z,n)}nEN is uniformly bounded on D(O,l).
1 71"2
b) Because 2::-1
- 2"n = -6 ' f E A(U). On the other hand, if the terms
of the series for f are rearranged to form a power series 2::=0
cnz n then
380 7. Elementary Theory: Solutions

E:=o Icnl ;::: E~l :2 (E~). k=l


Since E~=l ~ ;::: In(k + 1), it follows that
2(2nL k1) ;::: ~2In2 whence E:=l !enl =
n2 00. o
k=l

7.33. IfzED(a,r)OcOthenforxinX, (~
gx -z n=O gx -an
=I:( /~-a))n+1.
The series converges uniformly on compact subsets of D(a, r)O and Fubini's
theorem validates the equation

r dM(X)
Jx g(x) - a =
r I: (z - a)na)n+1 dM(X)
Jx n=O (g(x) -
~ r )ndef~ )n
= L...JJ~ ( (x)dM(X) (
_ a)n+l z - a n = L...J C
(
Z - a. 0
n=O X 9 n=O

7.34. If s = minlzl=r IJ(z)1 (> 0) and Iwl < s then Rouche's theorem
implies that N(f) = N(f - w) = 1 in D(O,r)o. If g(w) is the unique
solution in D(O, r)O of J(z) - w = 0, the principle of the argument implies
g(w) is given by the formula in (7.1). The holomorphy of 9 follows from
the equation

g(w) = ~ (2:i lZI=r (~~~~~ dZ) wn,


which is valid if w E D(O, s)o. o
8
Functions Holomorphic in a Disc

8.1. General Results

8.1. If I E H(U) then 1'(0) = 0 and 1'(0) = 1 according as 1'(0) is


calculated by using the zeros of lor the values of lin where I (lin) 1= O.
If 9 E H(U) then g'(O) = ±1 according as g'(O) is calculated by using
9 (lin) for n odd or n even.
If hE H(U) then h(z) = z when z = 1/(2n),n E N, whence h(z) == z
although h(z) 1= z when z = 1/(2n + 1), n E C. 0
8.2. a) If 0 ~ r < 1, there is a Zr such that M(r; f) = II (zr)l. Because
o ~ r2 < r, if Izl = r then M (r2; I) = sUPlzl=r (z2) ~ I/(z)1 whence
II I
M (r2; I) ~ M(r; f), in contradiction of the maximum modulus principle
unless I is a constant.
b) If u ~f 'iR(f)andv ~f c.;s(f) then I = v 2 + iv. The Cauchy-Riemann
equations imply -4v2vy = vy . If Vy ~ 0 then v2 = -1/4, a contradiction
unless I is a constant. 0
8.3. For r fixed in [0,1), {In (re i6 )} nEN is a II Ill-Cauchy sequence con-
verging to I (re i6 ).
If In (re i6 ) ~f 2:::'=0 anmrmeim6 for functionals Fm in L1 ([0,211"], ..\)*,
i.e., in Lco ([0,211"], ..\), anmrm = Fm (fn) and for all m, limn.-co anm ~ am
exists.
Furthermore,

{O ifk<O
lim Fk (fn) = Ik =
akr k otherwise .
A

n.-co

Cauchy's formula implies that if 0 < r < s < 1 then

The right member above converges to zero as m, n _ 00. Thus if 0 < r < 1
then for some 9 in H(U), InID(o,r)~ 9 and Fk(g) = limn.-co Fk (fn) = akrk,
whence 9 = f. 0

381

B. R. Gelbaum, Problems in Real and Complex Analysis


© Springer-Verlag New York, Inc. 1992
382 8. Functions Holomorphic in a Disc: Solutions

8.4. For z in D(O,r)O and some m in Z+,

L akzk ~f zmh(z), ao =I- 0,


00

l(z) = zm
k=O

and m = pn + q, p E Z+, °: ;
1I (D(O,s)O), h = gn and 1 = zq (zPg(z)t.
q < n. Then for some s in (O,r) and gin
D
8.5. Because F(z) ~f (z - a)l(z) ~ E:=odnz n is entire,

F(z) = ~ cnzn
z-a L..J
n=O

en E~- dial F(a)


whence - - = a -~ --t a - - = a as n --t 00 D
cn+! E~!o dial F(a) .
8.6. Since 1f'(z)1 is bounded and continuous in U, the criterion for
Riemann integrability (cf. 3.45) implies that for each () in [0,271"),

exists. Because 1 (re iO ) - 1(0) = J[o,re iO ] f'(z) dz, it follows that

ifO::;r<l

otherwise

D
8.7. The Schwarz reflection principle implies that for some function 1
holomorphic in a region n containing Au U, Ilnnu= Ilnnu. The identity
theorem for holomorphic functions implies Ilnnu= Ilnnu= 0. D

8.8. The function 1-1 is holomorphic in the open set n ~f I(U). Thus
h ~f 1-1 0 g E H(U) and h(U) c U. Because h(O) = 0, the Schwarz lemma
implies h(D(O,r)) c D(O,r). D
Solutions 8.9-8.12 383

8.9.
. shows
InductiOn TINn=O (1 + z 2n) = ",2
L..tk=l+
N 1 _l k
z. o
8.10. The convergence of each product in U is assured since
00 00

:L Iz2nl + :L I 2(2n-l)I < Z 00


n=O n=O
in U. Thus {f,g} c H(U).
Since (1- z2(2n-l)) = (1- z2n-l) (1 + z2n-l), induction shows that
a proper pairing of the factors in J9 leads to partial products

(1 - z4)
(1 - z6) (1 - z8)
(1 - Z12) (1 - Z16) (1 _ z20)
(1 - z24) (1 - z32) (1 - Z40) (1 _ z48)

The estimates

PN ~f g (1 + IUn!} ~ exp (t,IUnl)


IPn -11 ~ 1ft (1 + un) -11 ~ IpN - 11

imply that the products listed above converge to one. It follows that
J(z)g(z) = l. o
8.11. If such an J exists then

:L cnrneinO, °~ r < l.
00 00

J(z) = :L cnz n =
n=O n=O

Fejer's theorem applied as in Solution 6.164 shows that g(O) ~f e- iO is


the uniform limit on [0,271"] of the averages {O'N} NEN of the partial sums
of 2::=0 cne inO . On the other hand, for en : [0,271"] :3 0 1-+ einO ,

(g, en) = 10 r27r


e- i (n+1)O dO = 0.

Thus (g, O'N) = 0, N E N, a contradiction. o


8.12. For any arc A ~f {z : z = eiO,O ~ a < 0 ~ b < 271"}, J(A) is an
arc contained in oD(D, K). The Schwarz reflection principle implies that
384 8. Functions Holomorphic in a Disc: Solutions

for some region n containing D(O, 1) and some F in H(n), F(z)lu= J(z).
Thus for some N in Nand {Zk};:-'=l contained in D(O, 1),

G(z) ~f F(z) IT
k=l
(1 - ZkZ)
Z - Zk

is holomorphic in some D(O, R)O properly containing D(O, 1). Hence for
some h in H (D(O, R)O), G = eh. Furthermore, IG(z)1 = K on 1[', whence if
h ~ u + iv then on 1[', ell is a constant. Thus u'" = u y = 0 on 1[' and the
Cauchy-Riemann equations imply v'" = Vy = 0 on 1['. Hence h' = 0 on T.
The maximum modulus principle implies h' == 0 and so h and hence G are
constants. 0
8.13. Because 0 J(T), for some r in (0,1), J(z) =I- 0 if
tt Izl ~ r. Thus
1 1271" f' (re i9 ) .
Rouche's theorem yields N = -2' J ( '9) ire,9 d().
1rZ 0 ret
o
8.14. a) Since F(()) ~f J (e ilJ ) is continuous, FE L2 ([0, 21rj, ,x), and

~ =
Fn {con if n E Z+ .
otherwise

Hence I::'=o Icn l2 = IIFII~ < 00.


b) Cauchy's formula and the orthogonality of { ein9 } nEZ imply that if
z E U then

J(z) =~
21rZ
1 Izl=l w -
J(w)
Z
=~ r271" F(():
21r 10 1- e- z 9
d() (s8.1)

1271" F(())e in9 d() = 0, n E N, and

~ r27r F(())ei9 z
21r 10 1 - e,9 Z
d() = ~
21r
f: 10r
n=l
271" F(())e in9 zn = O. (s8.2)

The result follows from (s8.1) and (s8.2), after addition and simplification,
from the formula - 1 3?
21r
(ee'it +- zz)
-'-t- = Pr (() - t).
c) i.

L L rnein(-t) + L rneint
00 00 00

rlnleint =
n=-oo n=l n=O
re- it 1 1 - r2
= 1 - re- it + 1 - re = 1 - 2r cos t
it + r2
= 2nPr (t)
1 - 2r cos t + r2 = (1 - r) 2 + 4r sin2 ~.
Solutions 8.15-8.18 385

71'
ii. The result a) applies when J(z) == 1. iii. If 0:::; r < 1 and It I :::; "2 then

71'(1 - r)
Thus Pr(t) < 71' 2(1 - r )2 + 4t 2 and

71'(l-r) .fll<7I'
71'2(1-r)2+4rt 2 1 t -"2
Pr(t) < { 1 -r 71' . o
71' (1 + r2) if "2 < It I < 71'

8.15. Cauchy's formula implies J(a) = 2~ 127r J (a + seit ) dt whence

r J(a)sds = 2r2 J(a).


Jo o

1
8.16. If r E (0,1) then 1'(0) = -2
71'Z Izl=r z
.1
J(:) dz, whence 11'(01 :::; ~.
r
o

L cnrneinO
00

J (re iO ) =
n=O

L + i L (bn cos nO + an sin nO)


00 00

= (an cos nO - bn sin nO)


n=O n=O
= u (re iO ) + iv (re iO ) ,
and kIck = J(k)(O). By appeal to Euler's formula and the observation that
127r eimO dO = 0 when m f 0, it follows that

- 1 127r u (re iO) e -ikO dO Ck rk


= --. o
271' 0 2

8.18. a) Cauchy's formula applies.


b) From 8.17 it follows that

1 {27r .
-C n = 71' Rn Jo (M - u(r, 0)) e- mO dO.
386 8. Functions Holomorphic in a Disc: Solutions

Because M - u(r, 0) ~ 0,

2 1211" 2M 1211" 1 1211"


lenl:=:;-
rrrn
(M-u(r,O))dO=-
0 -- u(r,O) dO.
rn 0 rrrn 0 o

8.19. a) The map, : [0,2rr] 3 t


211"r
f (re it ) is such that
r211"
f-t ,* = f (8D(0, r))
and so L(r) = 10 b'(t)1 dt = r 10 If' (re it ) I dt. On the other hand,

Cauchy's formula implies If'(O)1 :=:; -


1 1211" If' (re't)
. I dt. It follows that
2rr 0
L(r) ~ 2rrr 1f'(0)1.
b) The Cauchy-Riemann equations and the area theorem based on
Jacobian determinants show that

The orthogonality of the system {e int } nEZ leads to the stated formula.

c) Cauchy's formula implies (J'(0))2 = -1 1211" (J' (re't))


. 2dt whence
2rr 0

If' (0) 12 :=:; ~


2rr 10
r211" If' (re it ) 12 dt
A(r) = for (fo211" If' (se it ) 12 dt) sds
~ 2rr for If' (0) 12 s ds = rrr21f' (0) 12. o

8.20. If fez) = 2::=0 cnzn then 12(r) = 2::::0 Icnl 2r2n. o


def fez)
8.21. Because g(z) = zn-1 E H(U) and

Ig (re iO ) I :=:; (r + ~)n-1 if r < r + E < 1,

it follows that g(U) c D(O, 1). Schwarz's lemma implies Ig(z)1 :::; 14 0
8.22. Since f- 1 E H(U), for some nonzero C1,
Solutions 8.23-8.26 387

in U, whence g(z) =1= 0 in U. Thus for some h in H(U), g(z) = eh(z) and
def h(z}+2h-i
gk(Z) = ze ,1 ::; k ::; n, meet the requirement. The Fundamental
n

Theorem of Algebra implies that I(z) = (g(z»n has no more than n solu-
tions. 0
8.23. Because 1(0) = lim n--+ oo I (~), 1(0) E lR. If I(v) (0) ~f Cv E IR
when 0 < l/ < m then IR '3 I (xn) - ~:-o cvx~ ---+ I(v+l) (0) as n ---+ 00.
- - X~+l
o
1
8.24. Cauchy's formula implies that if 0::; r < 1 then Iln(z)1 ::; -Ir-z
--I
in D(O,r)o. Thus on every compact subset K of U, {lln(z)IIK}nEN is
bounded and {In (z) IK } nEN is equicontinuous. The Arzela-Ascoli theorem
implies that some subsequence {Ink hEN converges uniformly on every com-
pact subset of U. Thus

Illnk - Inllll::; { +( link (re i6 ) - Inl (re i6 ) Ir drdO ~ I + II.


1D(O,r) lU\D(O,r)
Owing to the uniform convergence on D( 0, r), I is small for large k, l. Be-
cause fu lin (re ith ) IrdrdO ::; ~, it follows that for r near one, II is small
for all k, l. 0
8.25. If such an I exists, # (Z(J) n U) is finite since otherwise, for
some sequence {zn}nEN' IZnl ---+ 1 while I (zn) == 0, in contradiction of the
hypothesis. Hence for some ro in [0,1), if ro ::; Izl < 1 then I(z) =1= O.
It follows that each singularity of 9 ~f 7- in U is a pole (g is meromor-

phic in U) and the finite set peg) n U may be listed: peg) n U ~f {Pk}~=l'
Then g(z) . n~=l (z - Pk) ~f h(z) E H(U). Furthermore, if IZnl ---+ 1 as
n ---+ 00 then h (zn) ---+ 0 as n ---+ 00, whence h may be extended to a func-
tion h in C (D(O, 1), <C) and he'll') = {O}. The maximum modulus theorem
implies h = 0, a contradiction. 0
[ Note s8.1: The power series ~:=l zn! defines a function I in
H(U). For q in Q and t in [0,1), I (re q271"it) i 00, i.e., by abuse
of language, for a dense set of radii, 1* = 00. The result above
shows that, nevertheless, for some sequence {zn} nEN contained in
U, IZnl ---+ 1 while lim n--+ oo II (zn)1 < 00.]

8.2. Applications of Mobius Transformations

8.26. a) Let 0 be {w : R(w) > O}. Then ¢(w) ~f


1- w is holomorphic
l+w
in 0 and ¢(O) c U. Hence 9 ~f ¢ 0 I E H(U), g(O) = 0, and g(U) c U.
388 8. Functions Holomorphic in a Disc: Solutions

The Schwarz lemma implies Ig(z)1 ::; Izl, i.e., I ~ ~ ~~;~ I < Iz/. Hence

I~ ~ :~~;~: I ::; Izi from which the conclusion follows.


b) From a) and the Cauchy formula for the coefficients it follows that
. 1
If 0 < r < 1 then Icnl ::; --,-
1211" -
1+r
- 1 1 1+r
d() = -, - - - . The methods
271"n.rn 0 1 - r n. rn 1 - r
of the calculus show that minrE(O 1) ~ 1 + r is achieved when
, rn 1 -'- r

O< r = -1 + y'1 + n 2 def


= rn < 1.
n
Because

r~ ~ (1 _~) n ~ e-l,

1 - rn = 1 + -1
n
+ M n
= 1 + x - V~
1 + 2" def 1 + x 2 , and

l+x-Jl+x2~ (1- ~)x,O<()<x,


+ 1 ()2

it follows from Stirling's formula that

as n ---. 00. D

8.27. a ) If a E U then I<Po (z )1 = 1 on 'Jr. Thus TIn J(z)iF. ()


def
= g Z
().
IS
k=1 ":f!Zk z
holomorphic in U and g(U) cU. Hence IJ(z)1 ::; TI~=II<pZk(Z)I·
b) From a) it follows that (8.1) obtains. If z E U then I1z - _Zk I < 1.
- Zk Z

If Z ~f r E (0,1) then direct calculation shows that

Note that TI~=1 (1 - JLk) > TI~~1 (1 - JLk) 1 A ~ O. Furthermore,


A ::; TI~=1 (1 - JLk) ::; exp ( - 2:~=1 JLk), whence if 2:%"=1 JLk = 00 then
Solutions 8.28-8.31 389

A = O. Hence if '2::;::1 (1 -Izkl) = 00 then for z in (0,1), TIkEN Ilf>zk 1 = O.


Thus I(z) = 0 on (0,1), whence 1 == o. 0
8.28. Direct calculation shows that if Izl ~ r < 1 then

The argument used in 8.27b) shows that B E H(U) and since the absolute
value of each factor in (8.2) is in [0,1), B E H oo . 0
8.29. It may be assumed that a i- O. If g(z) ~f If>a 10
If>_a(z) and
0

C ~f If>a(b) then If>-a(c) = b, C E U, g(O) = 0, and g(U) c U. Hence


Schwarz's lemma implies Ig(z)1 ~ 14 Since g(c) = c itfollows that g(z) == z.
Hence z = If>a 0 1 0 If>_a(z), i.e., id = If>a 01 0 If>-a. Thus

i.e., I(z) = z. 0
[ Note 88.2: If a i- 0, {±sgn(a)} is the set of If>a-fixed points,
and each is on 11'. Thus if a i- 0, U contains no If>a-fixed point
while zero (in U) is the unique If>o-fixed point (cf. 9.9).
The function 9 may be regarded as the If>a-conjugate of 1 in
the o-semigroup of self-maps of U.]

8.30. a) If a E U, then I(a) ~f b E U. If g(z) ~f If>b 0 1 0 If>_a(z)


then g(U) c U,g(O) = 0. The Schwarz lemma implies Ig(z)1 ~ Izl and
, ,11'(a)1 (1-laI 2)
Ig (0)1 ~ 1. However, Ig (0)1 =
1-I/(a)12 .
b) If 1 ~f ~ then a) implies 11'(0)1 ~ 1 -1/(0)12. 0

8.31. For some a in U, I(a) = 0, and so if g(z) ~f 1 (If>_a(z)) then


9 E H(U), g(U) = U, 9 is injective, and g(O) = O. Schwarz's lemma implies
that for z in U, Ig(z)1 ~ 14 Furthermore, Schwarz's lemma may be applied
to the inverse g-l of g: for all z in U, (z) 1 ~ 14 Thus
Ig-1

Yet another application of Schwarz's lemma shows that for some 0 in [0, 21T),
g-l(z) = eiOz. Because g-l(Z) = If>a (J-1(z)), it follows that

o
9
Functions Holomorphic in a Region

9.1. General Regions

9.1. a) The function g(z) ~f I(z) - I'(a)z is such that g'(a) = 0 whence
9 is not injective near a, i.e., near a there are two points b, e such that
g(b) = gee), whence the conclusion.
b) If I is never zero in n 1 then 7
E H (n1). The maximum modulus

theorem implies M > 0, III : : : M, and 171 : : : ~. Thus M ::::: III : : : M:


I/(z)1 is a constant in n1 . The open mapping theorem implies I(z) is a
constant. 0
[ Note s9.1: If a
11
E U \ {O} and R = 1= then I(z) =
def Zn - a
_
a 1- az n
is in H (D(O, R)O), has n zeros in D(O, 2)°, and I/(z)1 == 1 on'lI'.]
9.2. For each w in I(n) and for some z in n, w = I(z). The hypothesis
implies that the formula hew) ~f g(z) uniquely defines h and that

h 0 I(z) = g(z).

If I' (a) i= 0 then h' (a) exists and is ~: ~:~. The trivial case in which I is a
constant aside, if I'(a) = 0 and bn -+ I(a) as n -+ 00, there is a sequence
{an}nEN stich that an -+ a, I' (an) i= 0, and I (an) = bn . It follows that
h (b n ) = 9 (an) -+ g(a) whence I(a) is a removable singularity of h, i.e., h
may be extended from a function holomorphic on I(n) \ {! (1'-1 (o))} to
a function holomorphic on I (n). 0
9.3. If n d~f D(O, 2)° \ {O}, I(z) ~f ~,
z
{Pn}nEN C qz], and Pn ~I on 'lI'

then 0 = Jlr Pn(z) dz -+ Jlr dzz = 27ri. 0


9.4. For a fixed in n and z in n, there is a rectifiable simple arc, such
that ,(0) = a and ,(I) = z. Because n is simply connected,

h(z) ~f 1 k(f'(w),g'(w)) dw

is well-defined and h'(z) = k (f'(z),g'(z)). o


390

B. R. Gelbaum, Problems in Real and Complex Analysis


© Springer-Verlag New York, Inc. 1992
Solutions 9.5-9.9 391

9.2. Regions n Containing D(O, 1)


9.5. For some positive 8, if Izl < 1 + 8 then f(z) = E:=o cnz n and for all
real 0, E:=o cnein8 E JR. Thus if Cn ~f an + ibn then

L (bn cos nO + an sin nO) = 0


00

n=O

and the series converges uniformly on R Hence an == bn == en == O. 0


9.6. According to the principle of the argument, the winding number
with respect to zero of the curve 'Y : [0,1] :3 t 1--+ f (e 21l"it) is n. Thus, as t
ranges over [0,1], O(t) ~f arg (J (e it )) increases by 2mr. Hence cosO(t) =0
at least 2n times. 0
[ Note s9.2: If f has nonsimple zeros in U, f is near an F
having simple zeros in U and in number N equal to the sum of the
multiplicities of the zeros of f. The number of zeros of !R(F(z)) on
Tis 2N and as F approaches f the zeros of !R(F(z)) on T approach
the zeros of !R(f(z)) on T. Hence with the proper interpretation,
the result above obtains when the hypothesis of the simplicity of
the zeros of f is dropped.]

9.7. The zeros of 9 in n are isolated whence all the singularities of £


9
are removable: £ ~ h may be regarded as an element of H(n). Because
9
If(z)1 :::; 1 in n, Ih(z)1180= If(z~1180 :::; 1 and the maximum modulus
theorem implies the result. 0
9.8. a) If r E [0,1) and Izl :::; r, for some €(z), 1€(z)1 < 1 and
f(z) = 1'(0) + €(z)z,
fn(z) = f(zn) = f'(O)zn +€(zn)zn, and

FN(Z) = 1'(0) (\-_z:) z + €(zn) ( \ ___Z:) z.

Thus IFM(Z) -FN(Z)I ~ 0 on D(O,r) as M,N -> 00. It follows that


·
1ImN-+oo F N = ,",00 f def F eXIsts.
L ..m=l n =
.
b) Cauchy's formula implies Ffv ~ F'. 0
9.9. According to Brouwer's fixed-point theorem 3.75, D(O,l) contains
an a such that f(a) = a. Because f(U) c f(D(O, 1)) C U, a E U. From
8.29 it follows that there is only one such a in U. 0
392 9. Functions Holomorphic in a Region: Solutions

9.10. For the curve 'Y : [0,211"] 3 t t-t I (e it ) E D(O, 1), if ( E (1r \ 'Y*),
there is a neighborhood N ~f D«(,r)O that does not meet 'Y*. For any a in
the nonempty set N n U, the index of 'Y with respect to a is

Ind (a)
"(
~f -1-1 ~
211"i "( Z - a
= _1_
211"i
r
10
27r
'Y/(t) dt
'Y(t) - a
= _1_ r
27r
211"i 10
l' (e it ) ieit dt = _1_
I (e it ) - a
r(f(z) - a)' dz .
211"i 1"1: I(z) - a (s9.1)

Since a is in the unbounded component of <C \ 'Y*, it follows that Ind"( (a)
and thus (s9.1) implies a f{. I(U), a contradiction.
=
0
°
9.3. Other Special Regions

9.11. When rn 10, In(z) ~ I (rnz), and

Km ~ {z : ~ ~ ~(z) ~ ~(b + 1), 1~(z)1 ~ 1~(b)1 ~ ~(b + 1) }


then {fn}nEN is uniformly bounded on Km and Vitali's theorem implies
there is a sequence {Ink hEN converging uniformly on Km. Iteration of
the (Arzela-Ascoli) procedure used in proving Vitali's theorem leads to a
subsequence, say {gp}PEN' converging uniformly on each compact set Km·
Because Irnb - rnll 0, the result follows. 0
9.12. There is a Mobius transformation ¢ : {w : ~w < M} ~ U and
9 ~f ¢ 0 I is bounded near zero. Hence zero is a removable singularity of 9
and ¢-1 0 9 = f. 0
9.13. If, for each r in (rl' r2) and some Zr on Co(r), I (zr) = then
1-1 (0) contains a point of condensation Zoo in A (0, rl : r2) 0. It follows that
°
°
ICz) = in A(O, rl : r2)O, a contradiction. 0
9.14. If R > R - E > r + 'f/ > r, the maximum modulus theorem
implies that for some FE in H (D(O, R - E)O), Pn ~ FE. If z E D(O, R)O
°
and Izl > then for some positive E, Izl > r + E. However, if E < 8 and
R> R-8 > r+8 then Fo(z) = FE(Z) in D(O, R-E). Hence if F(z) ~f FE(Z)
then the preceding sentence assures the value of F (z) is independent of the
choice of E. Because Pn ~ I on every compact subset of D(O, R)O, zero is a
removable singularity of F. 0
9.15. Any z in n is contained in a subregion n1 for which the maximum
modulus theorem implies I cnt} c Uc. Hence I(n) c Uc. 0
9.16. Note that E is not a region. Each component C of E is a (convex)
region and {)C consists of finite or infinite line segments. The Schwarz
Solutions 9.17-9.18 393

reflection principle yields an entire function Fc such that Fclc= flc. If


0 1 and O2 are components and Of n 0i ~f B =f. 0 then FCl (z) = FC2 (z)
on B, whence FC1 = FC2. 0

9.17. Since fez) = :E:=-oo cnz n in A(O, r : R)O, for p(z) ~f :E~~-oo cnzn
in H (A(O, r : (0)°) and q(z) ~ :E:=o cnz n in H (D(O, 1)°), f = p + q on
A(O, r : 1)°. Thus 8.25 applies to q and for some s in (r, 1), p is bounded
in A(O,s: 1). 0

9.18. If f(x + iy) ~f u(x, y) + iv(x, y), the Cauchy-Riemann equations


imply that on any nondegenerate interval (p + iq, r + is1contained in n, one
of the following four situations obtains:

a)
Ux > 0, Vy > 0
, b)
Ux < 0, Vy < °
uy < 0, Vx > 0 uy < 0, Vx >
,
°
Ux < 0, Vy < 0 U x > 0, Vy > 0
c) , d)
uy > 0, Vx < 0 uy > 0, Vx < 0

Ifp ~ r, q ~ s, and a) obtains then for Vet) ~ v(pt+r(l-t),qt+s(l-t)),

Vel) - V(O) = v(p, q) - vCr, s) = 11 V'et) dt

= (p - r) 11 vx(pt + r(l - t), qt + s(l- t» dt

+ (q - s) 11 vy(pt + r(1- t), qt + s(l - t» dt

> O.
The preceding argument can be repeated in fifteen other situations,
which, together with that above, exhaust all possibilities and are depicted
in the following display.
Ux Vy --t
+ + + +
uy Vx - + + - - + + -
p~r q~s 7+ +7 -7 7-
p~r q'5:.s +7 7- 7+ -7
p"5.r q~s -7 7+ 7- +7
p"5.r q"5.s 7- -7 +7 7+
The homolog of V is a function U and the entries 7+, -7, etc. describe
the circumstances of the pair {U(l) - U(O), Vel) - V(On: e.g., 7+, means
U(l) - U(O) E lR. and Vel) - V(On > O. In each situation,
f(p + iq) =f. fer + is). 0
394 9. Functions Holomorphic in a Region: Solutions

9.19. There is a Laurent series representation g(z) = L::'=-oo cnz n . If


the series contains infinitely many nonzero terms then zero or infinity is an
essential singularity of 9 and Picard's theorem implies 9 is not injective.
The Fundamental Theorem of Algebra implies

g(z) = Co + CIZ or g(z) = C-l + Co.


z

Because 9 is injective, C-ICI i:- O. If Co i:- 0 then 9 ( - ~) = 0 or

9 ( - :) = 0, each a contradiction. D

9.20. If I(a) = 0 and I =1= 0 then for all z in some A(a,O: r) contained in
I'(z) I~(z) _
n, I(z) i:- 0 and thus ~z-al=r I(z) dz > 0, whereas ~z-al=r In(z) dz = 0,
a contradiction.
If n ~f U \ {O} and hn(z) ~f zn then each hn is never zero in n while
hn ~ 0 on compact subsets of n.
If {gn} nEN is a sequence of functions in H(n) and gn ~ 9 on compact
clef . u clef
subsets of n, then each In = e9n IS never zero, In --+ I = e9 on compact
subsets of n, and I is never zero in n. D
9.21. If the conclusion is false, there is some neighborhood V(b) and a
subsequence {Ink hEN such that each Ink is never zero in V. From 9.20
and the equation I(b) = 0, it follows that 1==0 in V(b) whence also in n,
a contradiction. D
9.22. If a E n then I(z) = L::'=o cn(z - a)n and Re 2:: ~(a). Hence if
I(z) = L::'=o bn(Z - (a + I))n then I(z + 1) = L::'=o bn(z - a)n = 21(z)
whence bn = 2cn , n E N. Hence ~(a+I) :S Rb = Re. It follows by induction
that Re = 00. D
9.23. a) For rR ~f {z : zEn,lzl=R}nn, if zEn and R > Izl,
Cauchy's theorem implies I(z) = ~
27rZ
(lR + (
-R JrR
I(t) dt). For large R
t - z
and t in r R , It-zl = Itl·jI - ~ j2:: ~. Thus the second integral is majorized
by 27r M R-r, which converges to zero as R --+ 00.
b) If I(z) = ez2 then I E H(n), I is not constant, and I (ilR+) C lR.
D
9.24. a) Because I and 9 are continuous on Co(R), 1* g(z) exists. Since

Cauchy's theorem implies 1* g(z) is independent of R.


Solutions 9.25-9.27 395

b) As the limit of Riemann sums, f * 9 E H (n).


c) As rational functions, fez - w) and g(w) may be written as follows:

fez - w) = LK (Mk
k=l
L km
A
m=l (z - W - O!km)
/3krn
)
;

Hence Cauchy's formula implies that f * 9 is rational. o


9.25. The series representations

which are locally valid, imply h(z) = (z - a)-km (eo + ... ). o


9.26. Let A be Res(f, a). o
9.27. Thefunctiong~f focJ>_aisinA(A(O,O: l)D)andg('JI') cR Hence
if g(z) = I:~=-oo cnz n then Schwarz's reflection principle implies that for
some G in H(C \ {O}), G(z)lu\{o}= g(z). Thus it may be assumed that
g(z) = I:~=-oo cnz n , a representation valid in C \ {O}.
For each () in JR,
00 00

n=-oo n=-(XJ

L
00

g(e- i9 )= cnein9=g(e-i9),
n=-oo

whence Cn = C- n . Thus

L
00

9 (e i9 ) + 9 (e- i9 ) = 2cn cosn()


n=-oo

L ~ (en) cos
00

= 2CO + 2 n() ~f k(()),


n=l

an element of C([O, 271"], JR). The orthogonality properties of the system

{sin n(), cos n()} nEZ+

imply Cn
1
= 271" J0 r 7r
k( ()) cos n() d(), i.e., each Cn is real and Cn = C- n·
In sum, if z =1= 0, g(z) = 2CO + I:~=1 Cn (zn + z-n). Induction shows
396 9. Functions Holomorphic in a Region: Solutions

that for some Pn in c[z], (zn + z-n) = Pn (z + z-l) ,n E N. Because


{z + Z-l : z #- o} = C, it follows that for some entire function F, if
z #- 0 then g(z) = F (z + Z-l). Finally note that 1= go <I>a· 0
9.28. Vitali's theorem applied in an No(a) implies In ~ 0 on compact
subsets of No(a). If bE 0 then 0 contains a polygonal path 7r connecting
a to b. For a set {Nk}~=o of open neighborhoods, their union covers 7r,
and Nk n Nk+1 #- 0,0 ::; k ::; K - 1. The identity theorem for holomorphic
functions implies In(z) ~ 0 on each compact subset of each N k • Hence
limn -+ oo In(b) = O. Vitali's theorem applies again. 0
9.29. a) Note that ~ is not a region. For ~± ~ ~ n 11±,

o 1 + iz
maps (~+) conformally on 11+ and 9 : 11+ 3 z f-t - - . -maps 11+ confor-
1- zz
mally onto U. Hence

h.~ {/Og(z) ifzE~+


. 3zf-t log(-z) ifzE~_

is in H(~) and h(~) c U. If F E H(U) and FIE= h then limz-+! h(z)


exists. Direct calculations show that

limh
y!O
(~+
2
iV) #- limh
yiO
(-21 +iV) .
b) Morera's and Cauchy's theorems applied to F show that F E H(U).
o
9.30. The region 0 ~f C is simply connected, whereas if I(z) = eZ then
1(0) = C \ {O}, which is not simply connected. 0
9.31. If I(z) = ce.-z then limlR(z)-+oo I (lR(z)) = 0, while I (x + i) i 00
as x i 00. 0
9.32. If f: > 0 then {z : z E 0, I/(z)1 > M + f:} ~f A€ is open.
If a E F n C then for all z in some nonempty D( a, r)O, I/(z) I < M + f:.
If a E F \ C, i.e., if a = 00, then for some positive R and all z such that
Izl > Rand z E 0, I/(z)1 < M + f:. In sum, F is contained in an open set
that does not meet A€ whence A~ c o. Because 00 tJ- A~ it follows that A~
is compact. Hence the maximum modulus theorem obtains for I on A€.
However, if z E 8A€ then

I/(z)1 ~ M + f: because If(z)1 > M + f: in A€ and


If(z)1 ::; M + f: because z E 0,
Solution 9.33 397

Le., II(z)1 = M + €, and so II(z)1 == M + € in A€. Thus A€ is a relatively


open and closed subset of O. Since 0 is a region, A€ = 0 or A€ = 0.
The former alternative contradicts the hypothesis re the behavior of 1 near
000 0. 0
9.33. If 1 E H(O) and I(z) ¢. 0 on 0, assume I(a) i= O. Then 1(0)
contains some closed disc D(f(a), r). The geometry of the situation implies
that for some w on oD(f(a),r), Iwl > II(a)l· 0
10
Entire Functions

10.1. Elementary Theory

10.1. Because
def
( ) = J(z) - J(O) <'
gz Ec.
z
and Ig(z)1 is bounded, Liouville's theorem implies 9 is a constant. Since
limlzl->oog(z) = 0, J(z) == J(O). 0
10.2. Because J(O) = 0, 10.1 applies. o
to.3. Because J(z) = ~::01 cnz n + zm ~~=m cnz n- m ~f g(z) + zmh(z),
it follows that limr->oo M(r; g) = 0 and so limlzl->oo h(z) = O. Since h E E,
h=O. 0
def J(z)
10.4. It may be assumed that 9 :f=. O. If h(z) = g(z) then for some
positive r, Ihl is bounded in 0 ~f C \ D(O, r). Hence every z in Z(g) n 0
is a removable singularity and h E H(O). If {am} ~=1 is the (finite!) set of

zeros of 9 in D(O, r) then G(z) ~f J(z) n~=~~:)- am) E E and for some

K, IG(z)1 ~ KlzIM+k. From 10.3 it follows tha~ G E C[z] whence L is a


9
rational function. 0
10.5. Because J E E and IJ(z)1 ~ Alzla, = 0 and
it follows that J(O)
hence 9 ~f J(z) E E. If a < 1 then limlzl->oo Ig(z)1 = 0 whence 9 = J = O.
z
J() ,,[a]-l n
If a ~ 1 and J(z) ~f ~~=o cnz n then h(z) ~f z - "':;[:]0 CnZ E E
and h is bounded, whence a constant. 0
10.6. If a E ({Zn}nENr and Wn = (_l)n then for no J, J(zn) = Wno
Hence a necessary condition is that ({Zn}nENr = 0.
On the other hand, if ({Zn}nENr = 0, the WeierstraB product theorem
ensures the existence of a 9 such that 9 (zn) == 0 and g' (zn) =I- O. Thus near
Zn, g(z) = Cn1 (z - zn) + Cn2 (z - Zn)2 + ... and Cn1 =I- O. Consequently, if
Pn(z) = (Wn ) then near Zn,
Cn1 Z - Zn
g(z)Pn(z) = Wn + dn1 (z - zn) + ....
398

B. R. Gelbaum, Problems in Real and Complex Analysis


© Springer-Verlag New York, Inc. 1992
Solutions 10.7-10.11 399

The Mittag-LefHer theorem implies the existence of a meromorphic function


h such that the principal part of h at Zn is Pn. It follows that f ~f gh E e
and f (zn) = wn. 0

10.7. Because f(lR) C lR., each Cn in fez) d~f ~::'=oCnzn is real. Since
f(ilR.) C ilR., C2n == O. 0
10.8. Because f(x+iy) = ~(f(x+iy»+iSS(f(x+iy» ~ u(x, y)+iv(x, y),
it follows that u(x, 0) = ~::'=o anx n resp. v(x,O) = ~::'=o bnxn , the series
converging on lR.. The corresponding series ~::'=o anz n resp. ~::'=o bnzn
converge on C, represent 9 resp. h in e and fez) = g(z) +ih(z) on lR.. 0

10.2. General Theory

(2n + 1}IT d f d f eiz +2e- iz


10.9. a) Because cos 2 = O,n E Z, fez) ~ cosz ~ is
~ 1
transcendental. If a E C and then b = a ± va
def
2 - 1 =I- 0 and b + b = 2a

whence if () E [0,2'lT) and b = Ible i8 then

ei {-i{ln JbJ+8)) = band cos( -i(ln Ibl + ()) = a.


b) For such an f, f (C) is not dense, in denial of the Weierstrafi-Casorati
theorem.
c) Such an f is conformal, fCC) is a simply connected proper subregion
of C and C \ fCC) consists of more than one point. Thus Picard's theorem
implies f is a constant function, a contradiction (cf. 9.30). 0

10.10. If fez) ~f ~;:=o CnZ n , CN =I- 0, and M > 0 then for some positive
T,
. > T then ~n=O
If Izl
I
N-l leN I
cnz n- N < -2- Iand
/zN2CN/ > M. It follows that
If(z)1 > M if Izl > T.
If f is transcendental then 00 is an essential singularity of f and the
Weierstrafi-Casorati theorem implies that if E > 0 then for some Zn, IZnl > n
and If (zn)1 < E. 0
10.11. If IZnl i 00 and f (zn) = 0, n E N, then for any polynomial p,
f (zn) P (zn) == o.
Thus it may be assumed that # (Z (f» < 00 and hence for some posi-
tive M, if Izl > M then fez) =I- o.
If, for some sequence {zn}nEN' IZnl ~ n, If (zn)1 < IZnl-n, and P E C[z]
then for some v in N, some K p, and all large Izl, Ip(z)1 ::; Kplzlv. Hence
If (zn) p (zn)1 ::; IZnl-n . Kp IZnr, which converges to zero as n -+ 00.
If there is no sequence {zn}nEN as described above then for some N
in N and for all z in C \ D(O,N)O, If(z)1 ~ Izl- N , i.e., if Izl ~ N then
400 10. Entire FUnctions: Solutions

IZN J(Z)I :::: 1. Picard's theorem applied to the entire function zN J(z) is
contradicted. 0
°
10.12. For b in C for some a in C, g(a) = b. If g'(a) =I- then 9 is univalent
near a and for some positive r, some h in H (D(b, r)O), and all w in D(b, r)O,
g(h(w)) = w. Then for k in (0, r) and some function € : D(O, 8)° I-t C such
that limz-+o €( z) = 0,

F(b + k) - F(b) = 9 (h(b + k)) - g(h(b))


= g'(h(b))(h(b+ k) - h(b))
+ €(h(b + k) - h(b))(h(b + k) - h(b)),

whence F'(b) exists and is g'(a)h'(b).


°
If g'(a) = then for some positive 8, some m in N, and some p univalent
and holomorphic in D(a, 8)°, g(z) = b + (p(z))m. Furthermore, for some
positive t, p(D(a,r)O) ~ D(O,t)O ,and in H(D(O,t)O) and some univalent
h, p(h(w)) = w. The preceding argument, mutatis mutandis, applies and
shows F'(b) exists. 0
10.13. Because p - q = ef (e 9 - f - 1) and e9 - f E £ and is never zero, Pi-

whence the polynomial p - q must be the constant zero.


°
card's theorem implies that e9 (z)-f(z) -1 = has infinitely many solutions,
0
°
10.14. If the equation w 3 + w 2 + a = fails to have at least two roots
then a =I- 0 and there is one triple root, say r, and so 3r 2 + 2r = O. Since
2 4
a =I- 0, r =I- 0, whence r = -3' But then a = - 27 and

3 2 4 2 1
w +w - 27 = ( w + 3) 2 (
w- 3) '
a contradiction, whence the equation always has at least two roots.
Hence if -a (j. (J3 + j2)(C) then J(z) omits two of the roots of

in denial of Picard's theorem. o


10.15. a) If IJ(c)1 = a and c (j. E~ then D(c, r)O nEa = 0 for some positive
r. Hence in D(c, r)O, IJ(z)1 :::: a. The minimum modulus theorem implies
J is a constant. But then Ea = 0 = E~, whereas Picard's theorem implies
E~ =C.
Conversely, if c E E~ the continuity of J implies IJ(c)1 ::::: a.
b) Because C is bounded, Ea is open, and C is a region, C contains
a sequence {zn}nEN converging to some c and IJ (zn)ll infzEc IJ(z)1 ~f m
and m < a. Because J is continuous, IJ(c)1 = m, i.e., c E C. If z(f)nC = 0
then m > 0 and the minimal modulus theorem is denied. 0
Solutions 10.16-10.21 401

[ Note 510.1: If I(z) ~ eZ then Z(f) = 0 and for each a, every


component of Ea is unbounded.]

10.16. Let 11[0,1] be g. Because 9 E C OO ([O,l],C), the coefficients in the


Fourier series L:~=-oo cne21Tint for 9 satisfy, for each k in N, inequalities of
the form lenl ::::; ~~. Hence the series represents I on R, and, owing to the
cited inequalities for the coefficients, converges uniformly on each compact
subset of C: for some F, FIIR= IIIR· D
10.17. The set {m + nv'2 : m, n E Z} is dense in R (cf. 4.23). Since

I (Z + m + nv'2) = I(z),
it follows that I(z) == 1(0). D
10.lS. Picard's theorem implies that for some sequence {zn}nEN' IZnl i 00
and II (zn)1 ~ n. The broken line connecting Z1 to Z2 to ... , is 'Y* for some
'Y. D
10.19. The substitution w 1-+ ue i= in the integral shows that for k in Z,
2mn
I (ze i~") = e i~" I(z). Because I(z) = L:~=o iro,z] w n! dw, it follows that
I is transcendental and so 00 is an essential singularity of I. Thus I is not
a
injective. Since 1(0) = 0, if (j. I(c) then f:. 0 and a i~"
kEZ
{e a}
rt I(c),
in denial of Picard's theorem. D
10.20. Because e f = 1- e9 and e f is never zero, e9 is never one. But then
9 omits all the values 2mri, in contradiction of Picard's theorem unless 9 is
a constant, in which case I is a constant. D
[ Note 510.2: Fermat's conjecture asserts an impossibility: if
{x, y, z} C Z \ {O} and n E N \ {1, 2} then xn + yn f:. zn:
{{{x,y,z} C (Z \ {O})} 1\ {n E (N \ {1,2})}} => {xn + yn f:. zn}.
To the writer's knowledge the conjecture has remained unresolved
since 1665.
Corresponding to a triple {x, y, z} in (Z \ {0})3 is a triple
{F, a, H} of entire functions that never vanish. Corresponding to
N \ {1, 2} is the larger set Co From 10.19 it follows that
{{{F,a,H} c £} 1\ {F,a,H f:. O} 1\ {a E C}}
=> {Fa + aa f:. Ha}.]

10.21. For some g, I = e9 and e 9 (z)-z is transcendental unless it is a


constant, which is the case iff g(z) - z = a, a constant. In that event,
I(z) = eae z .
402 10. Entire Functions: Solutions

If g(z) - z is not a constant then I(z) + e Z = e Z (e 9 (z)-z + 1), and as


in Solution 10.19, the second factor of I vanishes infinitely often. 0
10.22. If I is transcendental, 00 is an essential singularity of I. Because
1-1 (a) is finite and nonempty, Picard's theorem implies # (J-l(b)) is infi-
nite, in denial of the hypothesis. 0
10.23. From 10.21 it follows that I E iC(zj and since I is injective, for
some nonzero a and some b in C, I(z) = az + b. 0
10.24. It may be assumed that 1(0) = 0. The inverse function theo-
rem implies I is locally injective. Thus for some positive rand 8, some
H (D(O, 8)°), and some h, h (D(O, 8)°) C D(O, r)O and for z in D(O, r)O,
h(f(z)) = z. For any Wo and some zo, I(zo) = woo Then I ([O,zo])
is a curve-image connecting °to woo The segment [0, zoj can be cov-
ered by finitely many open discs D (Zn, rnt in each of which I is invert-
ible: for some D (f (zn), 8 nt, some h n in H (D (f (zn) , 8 nt), and all z in
D (z, rnt, hn(f(z)) = z. The monodromy theorem implies that for some
F, F(f(z)) = z. In short, I is injective. [In particular, by virtue of 10.22,
for some constant a, I (z) = az.j 0
10.25. If p E iC(zj and Z(p) = Z(f) then £ E C. 0
P
10.26. a) Because C \ I(c) contains a nonempty half-space, Picard's
theorem implies I is a constant.
b) If 9 ~f -i<I>_l and

hew) ~f In Iwl + iO, w ~f Iwlei 7l"1i, 0 E (0,1)

then goh ~f I conformally maps U onto n ~f {w : °< ~(w) < 1}. 0


10.27. a) If I (A 2) n Ll =1= 0 then A2 n 1-1 (L l ) =1= 0 and so A2 n L2 =1= 0, a
contradiction. Hence the connected set I (A 2 ) is contained in C \ L l , hence
in one of AI, B l , say AI. Similarly, the connected set I (B 2) is contained in
one of AI. B l . If the open set I (B 2) is contained in Al then I(c) n Bl = 0
in denial of the WeierstraB-Casorati theorem. Thus I (B 2 ) c B l •
The Schwarz reflection principle implies that if ZPi is the reflection of z
in Li then (f(z)t ' = I (ZP2). Hence if, e.g., I (A 2) is a proper subset of Al
then I (B 2) ¥Bl and thus I(c) fails to contain two points, in contradiction
of Picard's theorem.
b) If I (L 2) "1=L1, it may be assumed that I (Z2) Al for some Z2
E

in L 2. Thus some neighborhood Nl (f (Z2)) is contained in AI. For some


neighborhood N2 (Z2), I (N2 (Z2)) C Nl (f (Z2)) C AI. On the other hand,
since Z2 E L 2, N2 (Z2) n B2 =1= 0, whence I (N2 n B 2) c I (B 2) C Bb in
contradiction of the preceding sentence.
Thus I (AI) = A 2, I (Bl ) = B2, and I (L2) = Ll .
Solutions 10.28-10.29 403

c) There are linear transformations z f---+ pz + q resp. w f---+ rw + s that


carry L2 resp. Ll onto JR. in the z-plane resp. w-plane. With respect to
these transformations, the map J corresponds to a map F while the half-
spaces Al resp. Bl correspond to the half-spaces {z : <S(w) > O} resp.
{w : <S( w) < O} and the half-spaces A2 resp. B2 correspond to the half-
spaces {z : <s(z) > O} resp. {z : <s(z) < O}. By this mechanism, the
problem is reduced to consideration of F, which maps the upper resp. lower
half of the z-plane onto the upper resp. lower half of the w-plane and JR.
onto R
Hence for F(x + iy) ~ U(x, y) + iV(x, y), Vy(x,O) = Ux(x, 0) ;:::: 0, i.e.,
R(x) ~f F(x + iO) is a monotonely increasing function. Furthermore, R is
strictly increasing since otherwise F is a constant on a bounded infinite set
and hence is a constant.
If either limx=ooF(x) < 00 or limx=_ooF(x) > -00 then F omits
infinitely many real values in denial of Picard's theorem. Thus F assumes
each real value precisely once.
Picard's theorem implies that if F is transcendental then F assumes
every real value, save at most one, infinitely often, whereas F assumes
every real value precisely once. Thus F E Clz]: F(z) = Z=:=oCnz n . Each
coefficient en is real, and, since F is monotonely increasing on JR., deg(F) is
odd. For each real r, F(x) = r has precisely one real solution. If deg(F) ;:::: 3
then F' vanishes at most finitely often, whence for some real r and a,
F(a) = r and F'(a) =1= 0: the multiplicity of the zero a is one, i.e., there are
three or more zeros and only one is real. The nonreal zeros (which occur in
conjugate pairs) are mapped by F onto nonreal numbers, i.e., not onto T,
a contradiction. Thus for some positive a and some real (3, F(z) = az + (3.
Correspondingly, for some a, bin te, J(z) = az + b. 0
10.28. The set { (u, v) : u =1= v 2 } is an open set and the complement of
an infinite set. Hence if J ~ u + iv and u =1= v 2 , Picard's theorem implies
J is a constant. 0
[ Note sI0.3: In several of the preceding items, the substitution
z f---+ .!.w reduces the discussion to the behavior of J at 00 to the

behavior of J (~) ~f g( w) near zero, an essential singularity of


g. In such instances the (weaker) Weierstrafi-Casorati theorem
serves instead of Picard's theorem.]

10.29. Note that k ~f !!:. is defined on Z(J) and k E H(C \ Z(g)). For
g
some (3, (3 and k assume the same values with the same multiplicities on
404 10. Entire Functions: Solutions

Z(f), i.e., if a E Z(f), /(z) = (z - a)P (ao + ... ), and ~i:j ~f A, then near
a,

,8(z) - A = (z - a)P (b o + ... ) and -


h(z)
- A = (z - a)P (Co + ... ).
g(z)
h -,8g def
Hence - / - = a E £, and a/ + ,8g = h. D

10.3. Order of Growth

10.30. a) If Izl < R e , the series converges, whence lenznl - 0 as n - 00.


Thus maxnEz+ lenznl exists and is for some no, Jcnoznol. Furthermore,

b) If 0:::; n < vf(r) then

rn (ICv/(r) Irv/(r)-n -Icnl) ~ O. (s1O.1)

Because both factors in the left member of (s1O.1) are nonnegative, the
form of each factor implies that it increases as r increases. Thus (s1O.1)
remains valid as r increases, i.e., if 0 < r < r' < Re and 0:::; n < vf(r) then

As a monotone Z+ -valued function, v is a step-function. If ro is a point


of discontinuity and r ! ro then (s1O.1) implies vf(r) ! v (ro).
1

c) If n E N then for some m greater than n, Cm =I- O. If r > (II~:II) m:=n


then Icml rm > lenl rn, whence for each n there is an r such that vf(r) > n.
d) InlnM(r;g) = InlnM(ar;f) .lnar.
lnr lnar lnr D
InlnM(r; f)
10.31. a) If E > 0 then for large r, I < p(f) + E, whence
nr
M(r; f) < exp (rp(f)+E). The maximum modulus principle implies that for
all z, 1/(z)1 < exp (lzlp(f)+E). Hence w(f) :::; p(f).
On the other hand, if IJ < p(f) then for some {rn}nEN' rn i 00 and
M (rn;!) > exp (r~) whence IJ < w(f).
b) For large r, InM(r;f) < krp(f) whence 7(f) :::; v(f). If IJ < v(f)
then for some {rn}nEN' rn i 00 and M(rn;f) > exp(lJr~(f)), whence
IJ < 7(f).
Solution 10.32 405

c) The Gutzmer coefficient estimate implies vf(r) :::; M(rj J), whence
(J) :::; p(f).
If ((f) = 00 then p(f) :::; ((f). If ((f) < (3 < 00 then for large r,
1

Icnl rn :::; J.tf(r) < exp (r.8) and if r = (~) If and n is large,

n ( e{3r.8) 11
Icnlr < ~ .

If 2e{3 ~f 6 then

[c5r .B] 00

M(rj J) :::; I: Icnl rn + I: lenl rn


n=O n=[c5r.BJ+l

I:
00

< (6r.8 + 1) J.tf(r) + T11 ~f I(r) + II(r),

-1. InlnM(rj J) < Inln(I + II)


Im r -+ oo Inr - Inr .

Note that limr -+ oo II(r) = o. When 0 < x :::; y, the inequality x + y :::; 2y
yields for large r,

In(I(r) + II(r)) :::; In2I(r) and


In I(r) :::; In (26r.8) + In J.tf(r) = In 26 + (3ln r + InJ.tf(r).
Furthermore, if some Cm =I 0 then from 10.30c) it follows that for large r,
J.t(r) > !em Irm. If p > 0 then M(rjpJ) = pM(rj J) and J.tpf(r) = pJ.tf(r). It
follows that p(pJ) = p(f) and (pJ) = ((f). Hence it may be assumed that
!em I > 1 and so In J.t f (r) > mIn r. If m > {3, the estimate applied earlier
yields

In 26 + (3ln r + In J.t f (r) :::; 2ln J.t f (r) and


InlnI(r)
---'--'- < In2 + InlnJ.tf(r) ,
lnr - lnr
whence ((f) ;::: p(f). o
10.32. In what follows, f is the symbol for the function under discussion.
a) M(rj f) = eTrP •
b) InlnM(rj f) = r.
c) Note that f is entire since the Cauchy-Hadamard formula for the
radius of convergence is lim n-+ oo ~
qn
= 00. Furthermore,

-3
InlnJ.tf(r) = 2lnlnr + In 4lnq'
406 10. Entire Functions: Solutions

whence p(f) = o. d) Direct calculation shows

Zlp,,(r) = [ar O ] , /-tp,,(r)=exp(r O ) .

Because the coefficients in the power series representation of Fa are positive,

a monotonely increasing function of r, whence

)_ 1· In In M (r; Fa)
p(F.
o- 1m 1 .
r->oo nr

From 10.31 it follows that p (Fa) = a. Furthermore, if € > 0 then for large
r,

-dnr < lnlnM(r;Fo) -alnr < dnr and


-e In M (r; Fa) e
r < < r.
rO

D
10.33. a) Since the series converges iff Izl < 1, Zlf(r) == O. b) Because
r .
the factors - decrease as n lDcreases, the absolute values of the terms
nO
increase so long as ro 2: 1, after which the absolute values decrease. Hence
n
Zlf(r) = [r i ]. D
10.34. a) If a> (J" then M(r; I) ::; era, whence p(f) ::; a and so p(f) ::; (J".
If 6> 0 and p(f) = (J" - 6 then limr->oolf(z)I!lzl=r::; e 1z1 0"-6 in denial of the
definition of (J".
b) To show ~ 2: p(f), it suffices to assume ~ < 00 in which case if € > 0
then for some M(€) and all n in N, lenl < M(€)n-~+'. The last inequality
provides a comparison between lenl and the coefficient of zn in the series
for FHe ~f Fa. Thus M(r; f) :S leol + M(€)Fo [(ae)-i r]. From lO.30d)
it follows that p(f) :S a (= ~ + f).
On the other hand, by virtue of the Cauchy estimates, if r > 0,

len I < r- n M(r; I).


If p(f) < 00 and € > 0 then for some M 1 (€),
Solutions 10.35-10.37 407

[
n
the minimum achieved when r = (p(f) + €)
p(/)+.
_1 1. Hence ~ S p(f) +€,
i.e., ~ S p(f), an inequality that is valid as well if p(f) = 00.
1 - def eJ1l
c) As in b), for a finite M(€), if", = ep(f)limn-+oo Icnl n < 00 then
n ) - p(J)
for n in N, \cnl S M(€) ( ep(f)(", + €) . Consequently,

and, since the order of Fp(f) is p(f) and FpC!) is of type one of that order
(p(f)), it follows that T(f) S ",.
Again, as in b), Cauchy estimates imply that if T(f) < 00 then for
any positive r and € and some finite M 1 (€), Icnl S Ml(€)r-neCr(f)+€)rP(/).

The right member is minimal when r = (P(f)(TZJ) + €)) -pffi. Hence

n
Icnl S Ml(€) ( ep(f)(T(f) + €)
)"fm and", S T(f) + €. o
10.35. Stirling's formula for n! and 10.34 imply p(f) = .!...
0:
Similarly,
T(f) = 0:. 0
10.36. a) If J(z) ~f 2:~o cnz n then 10.34 leads to

( ) _r
p g - Imp-+oo
(p + k) In(p + k)
-In \cpl
_ -1' plnp (p + k) In(p + k) - (J)
- lmp-+oo - 1n ICp I p 1np - P .

b) Because f' (z) = 2::=1 nCnz n- 1 ~f 2:~=o dnz n , 10.34 applies to


h h 1 -1' -In(n+1)-lnlcn+ll (n+1)ln(n+1) f
s ow t at p (J') - lmn-+oo (n+1 ) 1nn+1( ) . nlnn , rom
which it follows that p (f') = p(f).
c) From 10.34 and a calculation similar to that in b),

T
, 1-.-
(f) = -hmn-+oon «n + 1) ICn+ll) .e.

r
n
ep

= e~limn-+oo n: 1 ((1 + n)~ (n + 1) (lcn +1I*') ~


= eP(f)T(f). 0

10.37. a) It may be assumed that p(f) S p(g).


408 10. Entire Functions: Solutions

If p(f)
< p(g) and E > 0 then for large r,
exp (rP(g)-€) - exp (rp(f)+€) < M(r; g) - M(r; f) :::; M(r; J) + M(r; g),

< exp (rp(f)+€) + exp (rP(g)+€), and

~exp (rP(g)-€) :::; M(r;f) + M(r;g) < 2exp (rP(g)+€),


whence p(f + g) = p(g).
If p(f) = p(g) and J+g is not a constant, then p(f+g) :::; p(f) [= p(g)].
However, if J(z) = eZ,g(z) = -e Z + z then p(f) = p(g) = 1, whereas
p(f + g) = 0 < p(f) [= p(g)].
b) Arguments similar to those in a) show that if p(f) < p(g) then
p(f g) = p(g) whereas if p(f) = p(g) and J 9 is not a constant then the
sharpest general inequality is p(fg) :::; p(f) [= p(g)].
c) If, for some F in [0,00][0,00]2, p(fog) = F(p(f),p(g)) then: a) when
J(z) = z and 9 is arbitrary, F(p(f),p(g)) = F(O,p(g)) = p(g); b) when
J(z) = g(z) = eZ then 00 = F(p(f), p(g)) = F(O,O). There is no such
F. 0
10.38. If, for a subsequence {nkhEN and a positive E, -1 n lk I ~ E then
n cnk
Icnkl (e~ )Pk ~ 1, whence the series representing J does not converge in
n ~ {z : Izl > e~ }. 0

10.39. For In(z) ~ 2::=ocm (zn)m ~f 2::=oaknZkn, 10.34 and 10.36


imply p (fn) = np(f), valid even if p(f) = 00.
Note that 7(f) is defined only iff 0 < p(f) < 00. When 0 < p(f) < 00,
from 10.34 it follows that 7 (fn) = 7(f). 0
10.40. If limn---->ooa n ~f a < 00 and J(z) = z, the requirements are
fulfilled. Thus it may be assumed that an --+ 00.
1
If the power series 2::=0 cnz n is such that len In --+ 0 as n --+ 00 and
limn---->oo Icnnni = 00 then J(z) represented by the power series meets the
an
conditions.
def [1
If (3n = an, Kn = Inn -
def In an
Tn -1, and

~f {O if m rJ. {(3n}nEN
Cm -
e- Kf3
n n if m = (3n ,

then Icm I~ = { 0
e-Kn
if m rJ. {(3n} nEN and thus
if m = (3n
M(n; f) > - J(n) f3n
cf3n n-
--''---'--'- ->- = e -Knf3n+f3n Inn-Ina n

= ef3n 1 00. o
Solutions 10.41-10.43 409

10.41. Because M(r; f) ~ era, it follows that p(f) ~ 0:. From 10.34 it
nlnn
follows that if E > 0 then for all large n, -In Icnl = p(f) ~ 0: + E. Since

1 E E:,
~
0
n
for large n, lenin < 1, -lnlenl > 0, whence Icnl ~ n--;;.

10.42. It may be assumed that lanl ~ la n+11 and that 1(0) = 1 (whence
la11 > 0). If r > 0, the result in 8.27a) may be reformulated in the current
context as follows:
If 1 E H (D(O, r )0) and Z(f) n D(O, r)O = {an} :=1 then

If E > 0 then for large r,

_1_ < 1 < M(r; f) < exp (rP(f)+f)


lanl n - la1a2'" ani - rn rn

exp (rP(f)+f) ( n ) pu"J+<


The minimum of rn is achieved if r = p(f) + E '

whence Ia~ I < (e(p(f~ + E)) P(J1+<. Hence if 8> E then

L0011- I P(f)+6
<00. o
n=l an

10.43. ~ exp (~ n(l~ n)2 ) , it follows that p(f) ~ 1.


Because M(r; f)

1 }oo deC
On the other hand, Z(f) = { n(ln n)2 n=2 = a. When 0 < 8 < 1, the

integral test for convergence of 2::=2 (1 1 )2 leads to consideration of


n S nn 8

J 2R_-;:-d_x----c-;;- - X1-s 100 +28 R


x S (lnx)28 - (lnx)2s 2
r x-s
12 (lnx)2s+1
dx
.

If the integral in the left member is convergent then so is the integral in


right member, whereas the first term in the right member is unbounded as
R --t 00. Thus v(a) = 1 and so 10.42 implies p(f) ~ 1.
The same argument implies that for
410 10. Entire FUnctions: Solutions

M(r;fN):::; exp(r'1]N). Thus f(z) ~f PN(z)fN(Z) and, since p(PN) = 0, if


€ > 0 then for large r, M(r; f) < '1]€ exp (r'1]N). Thus In M{r; f) :::; € + r'1]N
and, since limN->oo '1]N = 0, T(f) = O. 0
1
10.44. If Izl :::; 2 then
Zp+l )
E(z,p)=exp ( -p+l - ... ,

lzIP+! )
IE(z,p)1 :::; exp ( p+ 1 + ...

IzIP+! ) ( Izls )
< exp ( 1 _ Izl :::; exp 1 _ Izl ,and
IE(z,p)1 < exp (2IzIS).

For large r,

IE(z,p)1 < (1 + r) exp (r P (~+ ... +rp~l ) ) < 2rexp (~rp) .

Hence, since P < s, for large r, say r 2: R, IE(z,p)1 < exp (2IzIS). On the
other hand, if Izl E [~, R] then for some K, IE(z,p)1 :::; exp (KlzIS). 0

10.45. Because Z (~ ) = {O, 1,~, .. -}, 10.42 implies that P (~ ) 2: 1.


The estimate in 10.44 implies that if 1 < s :S 2 then

whence InM (r; ~) :S lnr +,r + A(s) 71';s. The estimate used earlier

leads to P (~ ) :S s: P (~ ) = 1.

Because In r +,r :S In M (r; ~ ), it follows that T (~ ) = 00. In sum,

11
00 ( z) resp. e
1 + nOn n)2
az 1
resp. f(z)'

are all of order one but their types of that order are 0 resp. a resp. 00.

o
Solutions 10.46-10.47 411

10.46. a) If p = 0 then E(z,p) - 1 = z. If p > 0 then as an entire


function, E is representable by a power series: E(z,p) ~f 1 + cnpz n . I:::l
Furthermore, E'(z,p) = -zP exp [I::=1 ~], whence
{ -O ifl::;n::;p
cnp < 0 ifn > p

Since E(I,p) = 0, it follows that


00 00

1= - L cnp = L ICnpl
n=p+l n=p+l

L
00

I(E(z,p) - 11 ::; Izlp+l ICnpl = Izlp+l.


n=p+l

b) From a), the definition of the exponent of divergence, and the general
convergence properties of infinite products, it follows that

G(z)
def
= IT
00
E (
Z
an' 8(a) ) E £

and that z!~Zlz) E £ and is never zero, whence is of the form eg(z). 0

10.47. a) The convergence properties of infinite products and the defini-


tion of exponent of divergence imply the result.
b) For R positive, let n(R) be # (Z(f) n D(O, R)O). Then in

E (zn'z 8(z) )def(


= z) exp [Pn(z)] ,
1 - Zn

Pn is a polynomial and deg (Pn ) = 8(c). Thus

Direct calculation and the maximum modulus principle imply

M(2R; J) ~ M (2R; qR) .


412 10. Entire Functions: Solutions

In D(O,R), ifn > n(R), In (1- ~) may be determined so that

and then for

qR(Z) = e hRCZ ). The definition of o(c) implies that for large n in the infinite
series above, as in the discussion of the WeierstraB factorization theorem,
2RoCc)+1
the terms are dominated by the terms of 2::=1 Icn loCc )+1 . Thus in D(O, R),

lR (hR(Z)) < InM(2R; f) and according to 8.19, if hR(Z) ~f 2::=0 bmzm


In M(2R; f) - lR (b o) . . .
then Ibml < 2 Rm ,m E N. Direct calculatiOn shows that if
n > p then

bn = gCn)(o) _ ~ _1_ and


n! ~ ncn
k=nCR)+1 k

IgCn) I(0) I <- 2 InM (2R;Rn


f) -lR (b o)
+
~ _1_.
~ I In (sI0.2)
n. k=nCR)+l Cn

If p+ E,n then InM(2R;f) < (2R)P+€ and since n > /\'(c) , as R ~ 00,
both terms in the right member of (slO.2) approach zero. It follows that if
n > p, gCn)(o) = 0, i.e., g is a polynomial p and deg(p) ::; [pJ. D
10.48. a) The results follow from 10.34.
sin7rzdef
b ) Because -_-- ( (0.
= h z) E E and Z h) = ,it follows that for some k
J(z)
in E, h = ek • From 10.43 it follows that k is a polynomial and deg(k) ::; 1:
k(z) = ao + alz. Furthermore, h is an even function whence al = 0 and
thus e ao = 1, i.e., h == 1. D
10.49. For

Mo ~f 2 max Ig(x)1 and


xE[-l,l]

Mn ~ M n- 1 + 2 max Ig(x)l, n E N,
xE[-n-l,n+l]
Solution 10.49 (cont.) 413

if G(x) ~ MI + 21xl when 0 :::; x :::; ~, and, according as

1 3)
2n + 2n +
xE ( - 2 - ' - 2 - ,resp.xE
( 2n + 3 2n + 1)
--2-'--2- ,

G(x) ~ G (2n; 1) 4= ( Mn _ G (2n; 1)) (x 2n; 1) 4=

then G > Igl 2:: g, G(x) = G(-x), G is a monotonely decreasing resp.


increasing on (-00,0] resp. (0,00), continuous, and piecewise linear.
If a)
H (x) ~ {G(O) if -00 < x :::; 0
1 G(x) if 0 < x < 00 '

b) n in N, c) kn in 2N, d) n < kn < kn +b and e) (n: 1) k


n
> HI (n + 1)
then

whence FI (z) ~f L::=I (n: 1) k


n
E E.
For
H (x) ~f {G(x) if -00 < x :::; 0
2 G(O) if 0 < x < 00,

a similar argument using H 2 ( -x) in place of HI (x) yields an entire F 2 . If


F2(Z) ~f F2( -z) then J ~f FI + F2 E E and J()R(z)) > g()R(z)). 0
11
Analytic Continuation

11.1. Analytic Continuation of Series

11.1. a) In U, J(z) = zO::::=ozn)' = z (_1_)'


l-z (
z
l-z )
2' Thus

F(z) ~f z
(1 - z)2
is an analytic continuation of J from U along any curve in C \ {I}. Fur-
thermore, F is the only analytic continuation of J by virtue of the identity
theorem for holomorphic functions.
J(n) (a)
b) cn(a) = --,
n.
-, Rc(a) = la -11- D
11.2. Note that for z in U U 0 if (0, z) is a polygonal path lying in
U U 0 (which is automatically connected) then g(z) =
_1_ dw is
i(o,z) 1 - w
r
well-defined, independent of the choice of (O,z), and in H(UUO). If z E U,
then g(z) ~f L::=1 zn. Furthermore, g(z) E H(U U 0) and for z in U,
-
n z
J(z) = r g(w) dw. Thus
i(o,z) w
r
g(w) dw is an analytic continuation of J
i(o,z) w
from U along any curve in U U o. D
[Note sl1.1: For z in UUO, _1_ = e9 (z), i.e., g(z) is a branch
l-z
of In (_1_) or -g(z) is a branch of In(1 - z).]
l-z

11.3. If Izl < 1 then L::=1 Izn! I < I! Izl'


If p E Z, q E N, a = -,
defp
= re 271'<a
n 2: q, and z '
then zn. = r n, and so
I I

q
IJ(z)1 ~ 00 as r i 1. Thus each point e 271'ia is a singular point of J. Note
that {e 2 71'ia} aEQ is dense in T. D
11.4. a) Because 1 is a regular point of J, it follows that the result of
analytically continuing J is holomorphic in some 0 containing U U {I}. For
the holomorphic function g(z) ~f ~ (ZA + ZA+1) note that: i. g(l) = 1; ii.
. 1
If z E D(O, 1) \ {I} then 11 + zl < 2, whence Ig(z)1 = '2izAi'I(1 + z)1 < 1.

414
B. R. Gelbaum, Problems in Real and Complex Analysis
© Springer-Verlag New York, Inc. 1992
Solution 11.5 415

Thus g(D(a, 1)) c U U {I} C 0 and the Heine-Borel theorem implies


that for some positive e, g(D(a, 1 + e)) cO, i.e., D(a,1 + e) C g-I(O).
Since h(z) ~f log(z) is holomorphic in g-I(O), it follows that for some
{bn } nEN' h(z) = 2::=0 bmz m obtains in D(a, 1 + e)o. Direct calculation
shows that for k in N,
Pk (>'+I)Pk
Spk(Z) = L cn(g(z))n = L bmzm . (sILl)
n=O m=O

The right member of (sILl) converges in D(a, 1 + e)O, i.e., {spkhEN con-
verges in D(a, 1 + e)O as required.
b) If a E '][' and a is a regular point of I then 1 is a regular point of
I (az). Hence the argument in a) applies and thus {SPk (z)} kEN converges
in some D(a, 1 + e)o. In the current circumstances, {spkhEN = {sp}PEN'
Hence if a is a regular point of I then in some D(a,1 + £)0, {Sp(Z)}PEN
converges, i.e., in some D(a, 1 + e)O, 2:~=0 cnz n converges. This conclusion
is incompatible with the assumption that Rc = 1.
c) Since lim n-+_
oo ICnl~ = 1,
1
for some subsequence S ~f {cnkhEN'
nk+1 > 2nk and limk-+oo Icnk Ink = 1. By the same token, S contains
pairwise disjoint subsequences Sp ~f {cm(p,q)} qEZ+ ,p EN, such that for
I
each p, m(p, q + 1) > 2m(p, q) and limq-+ oo ICm(p,q) ",(~.q) = 1. Then a)
applies.
d) In c) let: i. hp(z) be 2:':0 Cm(p,q)Zm(p,q); ii. H t be 2:;:1 ep(t)hp;
iii. 9 be I - HI; iv. for t in {a,l], It be 9 + H t . Since the sequences
{Sp}PEN are pairwise disjoint, Ht(z) exists for all z in U and '][' is a natural
boundary for each H t . If '][' contains a regular point for It. some open arc
f t contained in '][' consists of regular points of It. Furthermore, f t may
be chosen so that its endpoints are rational multiples of 211". If there are
uncountably many such ft, two of their corresponding arcs, say f t and fs,
overlap. Hence ft - Is = H t - Hs is holomorphic on f t n fs whereas b)
implies that '][' is the natural boundary for H t - Hs. 0
[ Note 811.2: If nk+1 > 2nk then '][' is a natural boundary for
1
2:~=0 znk. However, limk-+oo le-v'nkl
nk = 1 whence

L e-v'nk zn
00

F(z) ~ k

n=O

and all its derivatives are represented by power series converging


uniformly on D(a, 1): FE Coo (D(a, 1),Q and yet '][' is a natural
boundary for F.)

11.5. a) If z tJ. '][' then for some positive e and each n in N, Iz -


e27rina > e, I
whence the series converges uniformly in every compact subset of 0 1 and
in every compact subset of O2 ,
416 11. Analytic Continuation: Solutions

b) For z in either of Oi, i = 1,2, direct calculation leads to the basic


equation:

(sl1.2)

If e27ria is a regular point it follows that limz--+l J (e 27ria z) exists whence


limz--+l J(z) cannot exist, i.e.,

{e27ria is a regular point} =? {I is a singular point} . (sl1.3)

From (sl1.2) and (sl1.3) it follows that if e 27ria is a regular point then e- 27ria
is a singular point; in other words, at least one of e±27ria is a singular point.
However, (sl1.2) and induction imply that if e 27ria is a regular point
then for each k in fiI, e-27rika is a singular point. Because {e-27rika} kEN
is dense in '][' (cf. 4.23), it follows that every point of'][' is singular; in
particular there emerges the contradictory conclusion that e27ria is itself a
singular point.
Thus e 27ria must be a singular point, in which case from (sl1.2) it
follows that the dense set {e27rika} kEN consists entirely of singular points,
i.e., '][' is a natural boundary for Jlo;, i = 1,2. 0
11.6. a) If z E 0 and n is large, then

z I 1 z 21z1
In(z-n) =n2 (~-1)
<-2'
n

whence the series converges. If z tf. 0, one term of the series is undefined.
b) The series converges uniformly on every compact subset of 0: J is
z 1 1
holomorphic in O. For k in fiI, k(z _ k) = k + z _ k and so

J(z) = -
Il L
+ -k + (
z def1
)= - + Fk(Z). (s11.4)
z-k nz-n z-k
nt-k

Because Fk is holomorphic in D(k, 1)°, it follows that P(f) = fiI and that
each pole of J has multiplicity one.
c) If k E fiI, Res(f, k) = 1. The residue theorem and the formula in
(sl1.4) imply the result. 0

11.7. Becau,., ,z , ~ (;z ) ~!2 (_1_


z -n 2 z-n
+ _1_), the argu-
z+n
n --1
n2
ments in Solutions 11.6a) - 11.6c) apply, mutatis mutandis. Note that
. 1
for c), If k E Z, Res(f, k) = "2. 0
Solutions 11.8-11.12 417

11.2. General Theory

11.8. For z near a, f(z) = L:'=o


f(:!(a) zn and the hypothesis implies
that the series converges for every z. Thus the series defines an entire
function F with the required properties. 0
11.9. The (Laurent) expansion L:'=-oo Cnz n converges in A(O, r : R)O
and defines the function F as required. 0
11.10. Cauchy's theorem implies that if F as described exists then

Conversely, if fll' f(z)zn dz == ° then for z in U, define F(z) to be


~
211"z
[ f(w) dw. For w in '[' and z in U, _1_ =
JlI' w - Z W - Z
~
W
f
n=O
(-=-)n.
W
As defined,

F E C(D(O, 1), C) and if r E [0, 1], ~zl=r F(z) dz = 0. Morera's theorem


applies. 0
11.11. Because fn ~ f on compact subsets of fh it follows that f E
H (0 1 ). But Vitali's theorem implies that for some F in H(O), fn ~ F on
compact subsets of 0 whence Fln
1 = f. 0
11.12. If a E [0,1) then for some positive r, D(a, r) c D(O, 2)°.
As can be seen from consideration of Figure sILl and the uniform

Figure sILl.
418 11. Analytic Continuation: Solutions

continuity of f on D( a, r),

r
J1z-al=r
fez) dz = r fez) dz + Jr fez) dz
Jr , r2

= lim ( r fez) dz + r fez) dZ) = O.


J
6-.0 Jr
r l(6) 2 (6)

Morera's theorem implies that f E H (D(a, r)O). o


11.13. Via repeated use of the Schwarz reflection principle, f may be
continued analytically first from QO to the union 0 1 of Q and the interiors
of the four squares ±1 + Q, ±i + Q, then to the union of O~ and the interiors
of the eight squares adjoining the four new squares, etc. 0
11.14. The Schwarz reflection principle may be applied, mutatis mutandis,
as in 11.13. 0
11.15. In D(I, r)O, fez) = 1::'=0 Cn(Z - l)n ~f fr(z). The hypothesis
implies Icol ~ 6. Thus g(z) ~f f(zl- ~) E H oo and if h(z) == ~ then
-Co z - Co
(1 - z)g(z) + f(z)h(z) == 1. 0
11.16. Let 1::'=0 cn(z - l)n be the power series representing f in
D (1, ~) 0. Then for each determination of (3(z) for In z in D (0, ~) 0,

L
00

fez) = Cn (e1nz - It ~f 9 0 (3(z).


n=O

Let 0 (D (1, ~) 0).


1 be {3 f may be continued analytically along
Because
each line L as described, there is an analytic continuation F of f from
D ( 1, ~) ° along any curve in O. The determination of In z may be chosen
so that {3 maps 0 bijectively (and hence biholomorphically) onto the strip
{w : -1r < <J(w) < 1r, -00 < ~(w) < oo} ~f O 2 , For z in 01,

LC -If LC (e
00 00

Fo{3(z) ~f F(w) = n (e/3(Z) = n W -1),


n=O n=O

which may be continued analytically throughout O2 • However, for k in


Z, F(w) = 1::'=ocn (e/3(Z)+2k11"i -It. The Schwarz reflection principle
implies that for some G in E, Gi n2= Gi2k?ri+n2 = F(w), k E Z. 0
11.17. Since e nl = 0, for each w in 0 1 let F(w) be f (e W ). It follows
that F E H (0 1 ), Note that D(a, r)O is simply connected and every branch
oflnz is in H(D(a,r)O). In other words, F(lnz) = fez) in D(a,r)o. 0
Solution 11.18 419

11.18. a) The formula in 10.45 shows that ~ E e and Z (~ ) = IE \ N,


whence r is meromorphic and p(r) = IE \ N.
1
b) Because m- Z = e- z1nm , the definitions of r(z) and Euler's constant
"y imply

c)

= _1_
m [
(1+-nI)Z+l] -'-! (I)Z]
1+-
m

z+ 1 [
r(z+ 1) lim lim n
r(z) m-too!! 1+ z: 1 . zm-t oo ! ! 1 + ~
z r m [(1 + .!.) (z + n)]
n
= z + 1 m~oo ! ! z+n+ 1

= z lim m + 1 = z.
m-too Z +m+ 1
d) h(z + 1) = g(z + l)r(z + 1) = g(z)zr(z) = zh(z).
e) Induction and (11.1) apply.
1
f) If rn(z) ~f n-zz II n
(1+ ~), then (sl1.5) implies

1 (( 1 ))
~.!...~ r n(z) exp ~1 m
n e-"(z
-z - In n = r(z)
, z
r(z) = lim r n(z) = lim n.n
n-too n-too z(z + 1) ... (z + n)
(s11.6)
420 11. Analytic Continuation: Solutions

g) Because r(1 - z) = -zr( -z) it follows that

1
r(z)r(1 _ z) =z II ( z2) .
1 - n2
nEZ\{O}

· h t memb er a b
However t h e rig ove'IS, accord'mg to 10.48, -
sin- .
7I"Z
0
71"

11.19. a) Note that the dominated convergence theorem implies

[00 e-tt z- I dt
10
= lim
n-t
r (1 _ !)n
00 10 n
t z- I dt

~f lim Qn(Z)
n-too

r:;
Integration by parts and induction imply Qn(z) = IIn(z) (cf. (sl1..6)).
Hence 11.18f) applies.

b) By definition, (Ii (x; Y) li(X)/i(Y) , i = 1,2. The Schwarz


inequality in its simplest version ("the geometric mean does not exceed the
arithmetic mean") implies

Owing to the continuity of the Ii, i = 1,2, their sum is log convex.
c) The integrand e-tt x- 1 in (11.2) is a log convex function of x. From
b) it follows that the Riemann sums that converge to r(x) are log convex
whence r(x) is log convex. 0
11.20. a) The series representing ( converges uniformly in every compact
10
subset of 0 1 , For z in 01, n-Zr(z) = 00 tz-1e- nt dt. It follows by summing
on n and (via Fubini's theorem) inverting the order of summation and
integration that

1 [00 e- I
((z) = r(z) 10 et -1 dt. (sl1. 7)

b) The equation r(z+l) = zr(z) permits a sequence of analytic contin-


uations ofr from 0 1 to (-1 + 0 1 ) \ {a, -I}, then to (-2 + OJ)\ {a, -1, -2},
etc., and ultimately to C \ (Z \ Z+).
For € positive, let LE be the contour described in Figure 811.2 and
for t in L" write t as Itle- i8 ~ re- i8 , -71" < () < 71".
Solution 11.20 (cont.) 421

Figure sl1.2.

When !R(t) < 0, the integral (11.2) is related to the (contour) integral

[ etr-zei()t dt
JL,
~f 1 L,
jet; z) dt ~f ¢(z, to).

For t ~f 0" + iT and z ~f x+ iy, Ij(t; z)1 = e(f . (0"2 + T2ft . e-()Y, and so
for b,c in JR, limal-ooJa~~ j(t;z)dt = 0. It follows that ¢(z, to) is to-free.
On the semicircle K" as to 1 0,

When !R( z) < 0, the limit of the integral on each of the straight lines in
L, \ K, is, by virtue of the dominated convergence theorem,

When !R(z) < 0, ¢(z, 0)


sin 1I"Z
= --r(l-z), 1 1
r( ) = - .
j
etC z dt. Because
11" z 211"2 Lo
the last integral represents an entire function, the representation is valid in
C.
The integral in (11.3) may be treated in a manner similar to that used
for the integral representing r(z) when !R(z) > 1. The conclusion is

((z) = r(1 -. z) [ ett z - 1 dt. (sl1.8)


211"2 JL, 1 - e t
422 11. Analytic Continuation: Solutions

The integral in (sl1.7) represents an entire function. Since both <: and r
are in H(n), (sl1.7) provides a continuation to C \ N, where r(1 - z) is
holomorphic. Because <: is holomorphic in n it follows that (sl1.7) provides
a continuation of <: to C \ {I}. 0
11.21. Note that 1 is a constant k iff k = 0 and that if I(z) = sinz then
1(2z) = 2/(z)f'(z).
If 1 is not constant, the formula 1(2z) = 2/(z)f'(z) defines F in
U:=12nu, Le., in C. The product rule for derivatives shows that so de-
fined, F is holomorphic throughout C. The identity theorem for holomor-
phic functions implies that F is completely determined by the values of 1
inU. 0
11.22. Note that 1 is a constant k iff k = 0 and if I(z) = tan z then
2/(z)
1(2z) = 1 - (f(z))2'
If F is an analytic continuation then F(z) =f. ±1 because I(z) =f. ±1.
Picard's theorem implies that any nonconstant continuation F must have
singularities.
Because for z in U, limn -+ oo 1 (2-nz) = 1(0), it follows that

1(0)((f(0))2 + 1) = 0, whence 1(0) = 0 or 1(0) = ±i.

The last two choices are excluded since otherwise, 1(0) = ~/~01.
Since 1 E H(U), (f(z))2 =f. 1 in (~u) c ~f K. Extend 1 by iterated
use of its functional equation so long as in each 2n K, n EN, (f (z ))2 =f. 1.
Since there is no entire continuation of I, there is a least natural number
Nl such that (f(Z))2 =f. 1 in 2NIK and for some (at most finitely many!) z
in 2Nl+ 1 K, (f(z))2 = LIt follows that for some sequence {NdkvN in N,
if Nk < n < Nk+l then as extended, (f(Z))2 =f. 1 in 2nK \ 2NkK but for
some (at most finitely many!) z in 2Nk+1K \ 2NkK, (f(z))2 = 1. Then F
is a meromorphic extension of I. 0
11.23. a) The series - L::=l ~ defines a function ¢ in H (D(O, 1)°) and
1
¢'(z) = ---. If 9 = e<!> then
l-z

g'(z) = g(z) = (zg)', g(z) = zg + C, g(z) = -C1 .


1- z - z

Hence = 1 - z and since e-<!>(O) = 1, C = 1, i.e., in U,


e-<!>(z) e-<!>(z) = 1- z
C
and in D(I, 1)°, e-<!>(l-z) = z. Hence I(z) = -¢(1 - z).
b) For a in 11' n 8D(I, 1), represent 1 as L::=o dn(z - a)n and thereby
provide an immediate analytic continuation of 1 from D(I, 1)° to D(a, 1)°.
Solutions 11.24-11.27 423

Any b in 11' can be reached in a finite number of such immediate analytic


continuations. There emerges the region 0 ~f D(O, 2)° \ {O} such that for
any c in C \ {O}, analytic continuation is possible from a point a in 0 n [0, c]
along [a, c]. 0
11.24. a) From 11.23a) it follows that (g(z))2 ~ (e! f
= z. A specific
form for 9 can be found by equating the coefficients in the equation

(g(z))' ~ (t. c,,(z _1)" r = 1 + (z -1).)


b) The procedure for 11.23b) may be followed, mutatis mutandis.
o
11.25. Because S ~ {p + iq : p, q E Q} is dense in C, each (g, N(a))
may be produced via a polygonal path (0, a) the vertices of which are in S.
There are only countably many such paths. 0
11.26. The argument of Solution 9.32 may be repeated, mutatis mu-
tandis. 0
11.27. If a E 0 and Ik (a) =I- 0, some branch 13k of In!k is holomorphic
in some D(a,ra) contained in O. Then in D(a,rat, Ikk(z) ~ ePk /3k(Z).
Regardless of the choice of 13k, l!k (z Wk is unambiguously defined. For each
such k, the application of 11.26 shows that sUPzEn 1!k(z)I Pk S M whence
n
I/(a)1 S L sup I/k(Z)IPk S L sup 1!k(z)IPk
fk (a)#O zEn k=l zEn
sup I/(z)1 S Mn.
zEn

[If Oc is compact and Ikk is continuous on OC then the basic maximum


modulus theorem implies sUPzEnc I/k(Z)I Pk is achieved on 80.] 0
[ Note sl1.3: The result in 11.27 is a generalized maximum
modulus principle for regular functions.]
12
Singularities

12.1. General Theory

12.1. If a is a pole then for all z in some D(a,r)O, some min N, and some
gin H (D(a, r)O), J(z) = (g(z\ m . Hence for some pin C[z) and some 9
z-a
in H (D(a, r)O), deg(p) < m and J(z) = (p(z\ m + g(z). Hence
z-a

ef ()
z = e~eg
~
z = e~G(z).
()def ~

~
Because G is holomorphic near a and e ~ has an essential singularity
at a, (ef(D(a,r)O)t = C and so a (j. P (e f ).
If a is an essential singularity of J then for any N(a), (f (N(a))t = C
whence (ef(N(a»t = C and thus a (j. P (e f ). 0
12.2. In C \ {O},

L
00

J(z) = cnz n
n=-l

L
00

g(O) ~f cne inll E C([O, 271"], 1R),


n=-l

and for n in Z, Cauchy's theorem implies

~ {21r g(O)e- inll dO = ~ (21r g(O)einll dO = Cn = C- n


271" 10 271" 10
=Oifn>l. 0

12.3. If the assertion is false then for some positive r and some a,

D(a,r)O c C\E

and so g(z) ~ J(z{ _ a E H (U \ ({an}nEN U {O})). Because

lim g(z) = O,n E N,


z~an

424

B. R. Gelbaum, Problems in Real and Complex Analysis


© Springer-Verlag New York, Inc. 1992
Solutions 12.4-12.7 425

each an is a removable singularity of g, 9 (an) = 0, and 9 E H (U \ {O}).


If zero is a removable singularity of 9 then the identity theorem for holo-
morphic functions implies 9 == 0, a contradiction. Thus zero is an isolated
essential singularity of g, whence (g(U\ {O})t = C. In particular, Igl is
unbounded in U and so a E E, a contradiction. 0
12.4. Since lef(z)1 = le!R(f(z»I ~ Iz _lal m' it follows that I(z - a)mef(z) I
is bounded near a, whence near a, ef(z) = (z - a)-m L:~=o en(z - a)n and
Co f:. o. If a is a pole of I then a is an isolated essential singularity of
ef , a contradiction of the preceding sentence. If a is an isolated essential
singularity of I then a is an isolated essential singularity of - I. But near
a and for some N(a) and a gin H(N(a)), e-f(z) = (z - a)mg(z), whence
lim z-+ a cf(z) = 0, in contradiction of Picard's theorem (applied to -f).
Hence a is a removable singularity of I. 0
12.5. a) When k ~ 2 and I(z) = z-k, zero is a pole of order not less than
two and if r > 0, Jr(f) = O.
b) If I(z) = e~ - ~ then zero is an essential singularity and Jr(f) = O.
z
L Cnz n then Jr(f) =
00

c) If C-l f:. 0 and I(z) = C~l + C-l· 0


n=O

12.6. By hypothesis for some {en}nEZ' if z ~f re it E (U\ {O}) then


I(z) = L:~=-oo cnz n . Thus

LII (re it ) 12 d.A2(rcost,rsint)

= lIfo L
-oo<m,n<oo
1<
1
-< ( [21<
Jo
CmCnrm+n+lei(m-n)t dr) dt

= lim 1 f: Icnl
<to <
1
-<
n=-oo
2 r2n+1 dr.

If n < 0 and Cn f:. 0 then, as E 1 0, 1<


1-<
Icn l2 r 2n+1 dr i 00, a contradiction.
o
12.7. a) From 2.72 it follows that the series converges in D(b, Ibl)°.
b) The argument of Solution 2.72 applies.
c) From a) it follows that each Ib may be continued analytically along
any curve in C\ {O}. If a:: E Z+ then b) implies Ib(Z) == zn. If a:: E (Z \ Z+)
then P (fb) = {O}. In all other instances, b) implies that no (g, D(O, r )0) is
an immediate analytic continuation of (fb, D(b, Ibl)°).
d) See b).
426 12. Singularities: Solutions

e) From b) it follows that Ifbl is bounded near zero. Hence zero is not
a pole. Picard's theorem implies zero is not an essential singularity. 0

12.2. Meromorphic Functions

12.8. The hypothesis implies that for some n in N, (P(f) n D(O, 1)) is a
finite set {Pn}:=l whence for some region fh containing D(O, 1),
N
g(z) ~ fez) II
il>pn (z) E H (n l ) .
n=l
Furthermore, g('Jl') c 'Jl' and thus g(D(O, 1)) c D(O, 1). It follows from 8.12
that 9 is a rational function whence so is f. 0
12.9. Since Coo is compact, P(f) is finite. Enumerate P(f) as {Pn}:=l
so that each pole of multiplicity J.t is listed J.t times. If
N
g(z) ~f fez) II (z - Pn)
n=l
is not a polynomial, 00 is an essential singularity of 9 and hence of f. 0
z - an
Because z _ bn = 1 + bzn _- ban . .f d ({} )C
12.10. n ' It follows that I z 'F bn nEN

then E:'=l Ib; ~ ~n I < 00, which assures the convergence of the infinite
product and the meromorphy of the function it represents. 0
[ Note s12.1: If an == bn , the conclusion is valid and vacuous.
If bn == -1 and an == -1 + 2"
1 then --b- II (z -
an) falls
. to converge
n n n nEN Z - n
when z = 0.]
°
12.11. Note that E A whence limz-+o zf(z) = f(I). Induction shows
that fez + n) = (z + n - 1)(z + n - 2)··· zf(z). Hence a), b), and c) are
resolved by the equations:
. . f(z+n+l)
hm (z + n)f(z)
z-+-n
= z-+-n
hm
(z + n - 1)(z + n - 2) ... z
f(l)
= (-1)(-2)··· (-n)" o
12.12. a) The hypothesis implies #(A) ~ No. If z E U \ A and S = 0
then D(O, Izl) n A is finite and there is a polygonal path (0, z) contained in
U \ A. If S ~ {aI, ... , an} then for some {TI, ... , Tn} contained in (0,1),
n
F~f U D(ak,Tk) C U
k=l
Solutions 12.13-12.16 427

and 0 ~ U \ F is a region. Now D(O, Izl) nOn A is finite and there is a


polygonal path (0, z) contained in 0 \ A.
b) The function G ~r !!:. has only removable singularities in U whence
9
gG~ k E M(U). 0
12.13. If, for some positive 6 and all z near Po, IJ(z) -bl ~ 6 then eachpn
is a removable singularity of g(z) ~ (J(z) - b)-l and so 9 may be defined
to be zero at each Pn. But then 9 == 0, a contradiction. 0

12.3. Mittag-Leffler, Runge, and Weierstraf3 Theorems

12.14. Because C is arcwise connected, components of open subsets of C


are open. In each component there is a z of the form P + iq, p, q E Q. 0
12.15. If the subspace R is not dense in C(K, C) then for some nonzero
F* in n-L, i.e., for some nonzero complex Borel measure J.L, and for every J
in R, IK J(z) dJ.L(z) = 0.
If Pn is in the component Cn of Coo \ K and Pn =F 00, then for some
·t·
POSl lve T,
D (p no T) eC
n an
d £or z ill
."...
'-'00
\K , ~oo
L..k=O
(z - Pn)k 1
k+l = - -
(W -Pn) W - Z
in K and the series converges uniformly on K. Since

g(z) ~r f dJ.L(w) E H (Coo \ K),


iK w-z
it follows that 9 = 0.
If Pn =00, a similar argument applies to the series _1_
w-z
="00

L- z
~+l.
k

1
k=O

In sum,
dJ.L(w) - .
°
- - = ill Coo \K.
K w-z
If J E H(O) then Cauchy's formula and Fubini's theorem imply that
for any cycle r contained in 0 \ K,

f J(z)dJ.L(z) = f 1. f J(w) dW} dJ.L(z)


{-2
iK iK 7r~ if' W - Z

= ~ f J(w) dw f dJ.L(z)
27r~ if' iK W - Z
=0,

a contradiction since J.L is nonzero. o


12.16. The complement 0 of the Cantor set Co is a region. Its comple-
ment is totally disconnected and of cardinality c. 0
428 12. Singularities: Solutions

12.17. a) For n in N, Kn ~f D(O, n) n {z : inf wllo Iz - wi ~ ~ } is

compact while Wn ~f D(O,n + 1)0 n {z : inf wllo Iz - wi> _1_} is


n+1
open and Kn C Wn C K~+1'
b) If K(C) 3 Ken then for some n in N, K c D(O,n) and

inf Iz -
zEK
wi > ~.
n
wllO

c) Because Un ~ Coo \ Kn ::> Coo \ n ~ F, 00 is in each unbounded


component of Un and each unbounded component of F. On the other hand,
if C is a bounded component of Un then by definition, for some z in Un
and some w in C \ n, zED ( w, ~) 0 while D ( w, ~) 0 C Coo \ Kn. Since

D ( w, ~) 0 is connected, z is in component of Coo \ Kn. In sum:

Each component of Un meets a component of F.


If a component C of F meets two components Cl, C 2 of Un then

is a connected proper superset of both C I and C2 , is contained in Un, and


denies the maximality of C I and C2 • 0
12.18. For {Kn} nEN' as in 12.17, 12.15 applies and provides a rational
function Rn with poles in S and such that If(z) - Rn(z)1 < ~ on Kn. If
n
K is a compact subset of n then for some n, K C Kn. 0
12.19. For {Kn} nEN as in 12.17, the sets

are finite and Pn(Z) ~f LaESn ra(Z) is a rational function holomorphic in an


open set containing K n - I . Thus 12.17 applies, yields a rational function
Rn such that IPn(z) - Rn(z)1 < 2: on Kn-l, and f ~f PI +L~=2 (Pn - Rn)
meets the requirements. 0
12.20. Each component C of n is a region. Hence 12.19 applies: for
some g ~ PI + L~=2 (Pn - R n ), peg) n C = S n C. Since C is not neces-
sarily simply connected, for a fixed in C, if ,*
is a rectifiable curve-image
connecting a to z in C \ S, then J-y g(w) dw is defined modulo 21Ti. Hence
G ~f exp (J-y g( u;; dW) is unambiguously defined and G (C \ S) c (C \ {O} ).
Solutions 12.21-12.22 429

Furthermore, if bE S n C, z E C \ S and z -+ b then G(z) -+ O. Its form


shows that G =f. 0 in C \ S. Thus S n C is not only a set of removable
singularities for G, but if G(S) ~ {O} then G E H(C).
The argument may be repeated for each component of n. 0
[ Note 812.2:
d f
If J E £ and Z(f) ~ {an}nEN then 7J' E M(C).
The Mittag-Leffler theorem combined with the argument in 12.20
yields the WeierstraB product representation for f. J

12.21. If nEZ, the principal part of J(z) ~f ~


sm 7rZ
near n is ( 1 )2'
z- n
Hence the Mittag-Leffler representation of J is, for some gin £,

J(z) = g(z) +L (z ~ n)2 ~f g(z) + h(z).


nEZ

Note that both J and h are periodic with period 1. Furthermore,

Isin 2 7r(x + iy)1 = ~ (cosh27rY - cos 27rx)

whence limlyl-+<Xl IJ(x + iy)1 = O. Finally, limlyl-><Xl Ih(x + iy)1 = 0, 9 is


entire, and 9 = J - h whence g(z) == o. 0
12.22. For J in 12.21, if z ~ Z, then J(z) = (-7r cot 7rz)'. On the
other hand, LnEZ\{O} (z~n +~) = LnEZ\{O}n(z~n) ~f
k(z) con-
verges uniformly on compact sets not meeting Z \ {O}. Hence termwise
differentiation is permissible and it follows that -k' = (-pcot 7rz)', i.e., for
some constant C, k(z) = 7r cot 7rZ + C.
2z
Furthermore, k(z) = -+ L
1 <Xl
2
z n=l z -n
2 whence k(z) = -k(-z). Because

7rcot7rZ = -7rcot7r(-z), it follows that C = O. 0


13
Harmonic Functions

13.1. Basic Properties

13.1. If 9 ~f u+iv, the chain rule for calculating partial derivatives applies
to h(u, v). Note that since 9 E H(O), the Cauchy-Riemann equations imply
that both u and v are harmonic in O. D
13.2. Direct calculation shows that if a ~f b + ic and for (x, y) in N(a),

v(x, y) d~ flY hx(x, t) dt - Jbr hy(t, c) dt


c

then h ~f u + iv E H(N(a». D
13.3. Riemann's (conformal) mapping theorem implies there is a biholo-
morphic bijection ¢ : U 1-+ O. Thus h 0 ¢ ~ 9 is harmonic in U. From
13.2 it follows that for some F in H(U), 9 = !R(F). If 1 ~f F 0 ¢-i then
1 E H(O) and h = !R(f). D
13.4. If Vi and V2 are harmonic conjugates of h then

h ~f h + iVj E H(O),j = 1,2, whence (It - h) (0) c ilR.

The open mapping theorem implies It - 12 is a constant, i.e., Vi - V2 is a


constant. D

13.5. Because L(O) contains a v such that h + iv ~f 1 E H(O), it follows


that 0 ~ 1(0) and so for some 9 ~f P + iq in H(O), 1 = 92 • D

rei(J }
e,t+-
.
13.6. Because PI. (0 - t)
R
= ~!R {
211".
R.(J' PJ;, is harmonic in
ret n
e't--
R
D(O, R)o. Hence if J-L is real and Iz - al < R then h is the real part of

I(z) ~f
1
_.
211"Z
1 0
271" R it +
e. z dJ-L (a + Reit )
Re,t - z
.

430

B. R. Gelbaum, Problems in Real and Complex Analysis


© Springer-Verlag New York, Inc. 1992
Solution 13.7 431

In the context of 7.33, X is 3, 9 is id, and if

m(z)
def
= 1
-2'
1 R
Reit .
d", (a + Re zt ) and

1
't
11'~ :s e' - z
) def
( z 1-. 1 d (R it)
n =2n :s Re zt. - z '" e

then m, n E H (D(a, R)O), and I(z) = m(z) - zn(z) E H (D(a, R)O). Thus
h is in L (D(a, R)O). When", is complex, it is a linear combination of real
measures. 0
13.7. a) Since h is the real part of a function in H(D(a,r)O), Cauchy's
formula yields the result.
b) i. For A ~f I(a), 1- 1 (A) ~f F n n is closed in n. For bin F n n,
there is a positive r such that D(b, r) C n. If D(b, r)O contains a c where
I(c) < A then for all t in some nonempty open interval J of [0,211'), and
for some positive E, 1 (b + Ib - cle it ) < A - E. On the other hand, since
1 E MVP(n),

A = I(b) =21 (r r
+
11' J J J[O,21r)V
1 (b + Ib - cleit ) dt)
1 EA(J)
< 211' ((A - E)A(J) + A(211' - A(J») = A - ~,

a contradiction. Hence D(b,r)O c F n n, i.e., F and F n n are open.


Because n is a region and F n n ~ 0, F n n = n.
[ Note s13.1: If 1 == 1 in U and 1 == 2 in 2 + U then 1 is
harmonic in the open nonregion S ~f Ul!J(2 + U). For all z in S,
1(2) ~ I(z). Nevertheless, 1 is not a constant.]
b) ii. The result in b)i applies to -I.
c) If hE MVP(n), D(a,R) C n, and 0 :::; r < R then from 13.6 it
follows that

(s13.1)

is harmonic. The properties of Pr (cf. 8.14) imply that: i) H is continuous


on D(a, R); ii) since h enjoys the MVP, so does H; iii)

Thus for z in D(a, R), H(z) = h(z). The right member of (s13.1) is Pois-
son's formula for h.
For the converse, cf. a). 0
432 13. Harmonic Functions: Solutions

13.8. The map

P: C(1', JR) 3 9

1-+ 2~ 121f Pr(f} - t)g (e it ) dt ~ peg) E L(U) n C(D(O, l),q


is a bijection. The maximum principle implies P(f)(a) :::; maxlzl=ll/(z)1
while the character of P implies that if I ~ 0 then P(f) ~ O. It follows that
J: C(1', JR) 3 11-+ P(f)(a) is a linear functional to which the theory of the
Daniell integral (cf. Section 1.2) is applicable. Thus for some nonnegative
measure /-ta, P(f)(a) = 121f I(z) d/-ta(z). If h E L(U) n C(D(O, 1), q then
the preceding equation applies to both !R(h) and ~(h). 0
[Note 813.2: A Dirichlet region 0 in C is a region such that for
every I in C(aO, JR) for some h in L(O)nC CO e , JR), an = I. If, to
hl
boot, oe is compact then for some a in oe, h(a) = maxzEnc h(z)
and the maximum principle implies a E a~. Thus the argument
and conclusion above apply to any Dirichlet region 0 such that
oe is compact.)
13.9. If h ~f U + iv then h2 = u 2 - v2 + i2uv and if {h, h2} c L(O) then
direct calculation shows that h~ + h~ = 0, i.e.,

Thus the Cauchy-Riemann equations obtain for h or h. o


13.10. If a E 0 then for some positive r, all z in D(a,'r)O, and some
g ~ p + iq in H (D(a, r)O),

1= e9 , III = eP , In III = p E L(D(a,rt)·


It follows that dlnl/(z)1 == 0 in O.
Another proof involves the examination of h ~ III = (u 2 + v 2)! and
a direct calculation of dh. 0
13.11. For I ~f u+iv, the observation III = v'1 ·7 and direct calculation
of I/lxx and I/lyy shows that if

8 1 ~f {(x, y) : ux(x, y) = uy(x, y) = vx(x, y) = vy(x, y) = O} n 0 and


82~f{(x,y): (u(x,y»2+(v(x,y»2=0}nO

then 8 1 U 8 2 = O. If I =1= 0 then 8 2 = 0, whence 8'1 #- 0 in which case I is


a constant on some nonempty D(a,r)O contained in O. 0
Solutions 13.12-13.14 433

13.2. Developments

13.12. If lal < R, Poisson's formula may be written

h(a) = 2-1
271" Co(R)
R2 -lal22 h(z) dO =
Iz - al 271"
2-1 Co(R)
~ (z + a) h(z) dO
z- a

= ~ [~1 (+ z h(() d(] ~f ~/(z).


271"Z Co(R) ( - z (

Thus I E H (D(O, R)O) and ~(f) = h. D


13.13. Schwarz's formula (cf. 13.12) implies that lR contains a sequence
{ kn }nEN such that If. Iz I < 1, In ()
Z = -.
1 W + Z un(w)
- -- - dw + zk
271"z Co(R) W - Z w
1 . n.

The uniform convergence of {un} nEN implies that the integrals converge
and since limn --+ oo In(O) exists, limn --+ oo kn ~ k exists. D
13.14. a) The Poisson formula for v implies

(re iO) = 1
-2
(111" + 1211" R2 2Rrsin(O - t) (it))
V
11" 2Rrcos-t
(0 ) U re dt

2- (r10
2
71" 0 +r-
= 2Rrsin(O - t)u (re it ) dt)
271" R2 + r2 - 2Rr cos(t - 0)

+-
1 (111" 2Rr sin(t - O)u (re i (t-11"») )
dt
271" 0 R2 + r2 + 2Rr cos(t - 0) .

Because

-d 1n (R2
dt
+ r 2 ± 2Rr cos(0))
t - =f2Rr sinet - 0)
= -=---'~..,..---:-=-,---;-'--=
R2 + r2 ± 2Rrcos(t - 0)

the condition Ihl :::; M and the equation

rI 2Rrsin(t - 0) Id r 2Rrsint
10 R2 + r2 ± 2Rrcos(t - 0) t = 10 R2 + r2 ± 2Rrcost dt
imply

Iv (re')
·0 1 M 2 2M R +r
:::; - (In(R+r) -In(R-r)2) = - In--.
71" 71" R-r

b) There is a biholomorphic bijection if> : U t---> {w : -1 < ~(w) < 1}.


If if> ~f P + iq then Ipi :::; 1 whereas SUPzEU Iq(z)1 = 00. Furthermore, any
two harmonic conjugates of p differ by a constant. D
434 13. Harmonic Functions: Solutions

13.15.
° R - Izl {Re it +
If z E D(a, R) then R + Izl ::; ~ Reit _ z
Z}R + Izl .
::; R _ Izl' Smce
h ;::: 0, the Schwarz formula (cf. 13.11) implies the result. 0
13.16. The existence of J ~ f027r In 11- eitl dt turns on the existence,
for f in (0,1), of the (improper) integral, f;
lIn 11 - eitll dt. However, for
small positive t,

lIn 11 - eit II = ~ lIn 12 - 2 cos til

::; ~ (In2+21Inlsin~1 D: ; 1~2 + Ilnl~11


and f; Ilnxl dx = -lim6!0 (xlnx - x) I~= f - finf. Hence J exists.
Next, if 0 < r < 0.5 and z = 1 + r, then z - w, as a function of w, is
in H(U) and is never zero in U, whence In Iz - wi E L(U) = MVP(U) and
1 {27r
so 211' 10 In Iz - eitl dt = In Izl·

0- l+r=z

Figure 813.1.
Furthermore, the geometry depicted in Figure 813.1 indicates that
if w lies on the arc AlB, then In Iw - 11 and In Iw - zl are negative but
Iw - 11 < Iw - zl, whence Ilnlw - 111 > Ilnlw - zll. In l' \ {AlB},
lIn Iw - zll is bounded. In sum, for w in l' and z in [1,1.5]' lIn Iw - zll is
dominated by an integrable function. Hence a passage to the limit as r ! 0
(and z -+ 1) is justified. Because limz-> 1 In Izl = 0, it follows that

1 127r . 1 127r .
lim -
dO 211' 0
In Iz - e't I dt = -
211' 0
In 11 - e't I dt = o. o

13.17. The basic idea is to replace f by a function F free of zeros in some


slightly larger D(a, R + f) so that In IFI E L (D(a, R + f)O).
Solutions 13.18-13.19 435

If some zeros of f are in D(O, R)O, the notation may be chosen so that
IZnl < R, 1 ~ n ~ m, and if m < N, IZnl = R, m + 1 ~ n ~ N. Then an F
as described is given by the formulre

F(z) ~ f(z) IT R2 - Znz ZnIT


n=l R (z - zn) n=m+l Z - Zn

IF(O) I =If(O)1 IT I~I'


If Zn = Re'°o and Z def
def
n
°o
= Ret, On,O E [0,211"), m +1~n ~ N, then

I -I {I11-
R 2 - Zn Z _
R(z - zn) -
1
ifl~n~m

eO-On I ifn+ 1 ~ m ~ N. (s13.2)

For some positive £, In IFI E L (D(O, R + £)0) and the mean value
property implies In IF(O) I = 2~ 127r In IF (Re iO ) I dO. Owing to (s13.2),
In IF (Re iO ) I = In If (Re iO ) 1- L::-=m+l ln 11 - eo-on I, whence by virtue of
13.16, J: 7r
In IF (Re iO ) I dO = J: 7r
In If (Re iO ) I dO. Note that in the desired

formula, L::-=m+1ln II~II = o. D


13.18. The Poisson integral applies to In IF(z)1 in Solution 13.17. D
I J:
13.19. a) If If (re iO ) ~ M < 00 then 7r In+ If (re iO ) dO ~ 211" In + M. I
b) It may be assumed that f(O) i:- O. Jensen's formula and the hypoth-
esis imply that for some M,

If(O)1 !! ILl ~ (1 io[27r


N
exp 211" In+ If (re iO ) I) dO

and 1 ~ n:-=l lanl ~ If(%,r N • Hence as first, r i 1 and then N i 00,

1 2: n:'=l lanl 2: If~)I. If :E:'=1 (1 -Ianl) = 00, the argument presented


in Solution 8.27b) may be applied in the present context and yields:

II lanl = II (1 -
00 00

(1 - lanD = 0,
n=l n=l
a contradiction. D
436 13. Harmonic Functions: Solutions

13.20. a) The dominated convergence theorem implies that for each a in


0, hn(a) = 21 r
JaD(a,r)
11"
hn (a+re i8 ) dO _ 21 r
11" JaD(a,r)
fa,r(a+re i9 ))dO.

It follows that h(z) = limn--+ oo hn(z) exists, h = h a.e., and h E MVP(O).


- def -

b) See a).
c 0 and
c) Harnack's inequality (cf. 13.15) implies that if D(a,r)
hn(a) i 00 then hn(z)ID(a,r) i 00, whence Loo ~ {z : hn(z) i oo} n 0 is
open. Harnack's inequality implies as well that if hn(a) S M < 00 then
for some constant K, hn(z)ID(a,r)S K. Hence h E MVP(D(a,r)O), i.e.,
h E L (D(a, r)O). It follows that (0 \ Loo) nO is open and since 0 is a
region either Loo = 0 or Loo = O. 0
14
Families of Functions

14.1. Sequences of Functions

14.1. Some subsequence {Ink hEN converges uniformly on compact sub-


sets of n. Thus each of the sequences 8 p ~f {I;:} kEN' p E Z+, converges
on compact subsets of n. Furthermore, 8 p +l C 8 p • The diagonal sequence
8 ~f {/;p}.
P
is a subset of 8 0 ,
pEN
0

14.2. If {an} liEN C U and lim n co an = 1, the requirements are met by


-+

{ -cI> an} nEN" 0


14.3. If n c U, Fn is normal. If n n(C \ U) -# 0, {zn} nEN fails to converge
somewhere in n. Thus Fn is normal iff n C u. 0
14.4. If k, n E N, and z = x + iy E IL then
Isin nz - sin(n + k)zl
= 2Iein"'e-ny (1 - eik"'e- ky ) - e-in"'eny (1 _ e-ikYeky) I ~f 21/ - III.
For large k, n, III is large while II II is small whence if n n TI_ -# 0, then n
contains compact sets (points!) on which {sin nz} nEN fails to converge. A
similar argument shows a similar result if n n TI+ -# 0. Because no region
is a subset of JR, Fn is normal for no region n. 0

14.5. If In(z) ~f E:=o cnmz m in U, then


r Iln(z)1 2 d>'2 = f: ~cnml:
lu
27f
m+
::;
m=O
r Iln(zW d>'2
lu
27f

and if r E [0,1),

lin (re it ) I ::; (f:


m=O
Icnml ) (J2m + 2) rm
J2m+2
1 1

: ; (t. ~~:: ) (t. 2 . (2m + 2)r 2m ) 2 ~f In . II.


Because lim -+ In =n co 0 and I I = d~ (1:: x 2) I",=r < 00, In ~ 0
on
compact subsets of U. o
437
B. R. Gelbaum, Problems in Real and Complex Analysis
© Springer-Verlag New York, Inc. 1992
438 14. Families of Functions: Solutions

14.2. General Families

14.6. If {fn}nEN is a lliloo-Cauchy sequence then limn -+ oo fn ~f f exists


and fEe (0, C). As the uniform limit of functions holomorphic in 0,
f E H(O). Cauchy's formula implies B(O, 1) is equicontinuous. 0
14.7. a) Since, for each r, Mp(rj·) is a norm for the vector space

{ fID(o,r) : f E H(U) },

it follows that II lip is a norm for HP.


b) Cauchy's formula and Holder's inequality imply that a II lip-Cauchy
sequence S ~f {fn} nEN confined to D(O, r), is a II lloo-Cauchy sequence.
Thus for a unique f in H(U), the elements of S converge uniformly to f
on compact subsets of U. Direct calculation shows that f E HP and that
IIfn - flip - °
as n - 00.
c) The argument in b) shows that ifT ~f {fn}nEN is a subset of B(O, 1)
then T confined to D(O,r) is a normal family. 0
1

14.8. a) Since M2(rj 1) = (L::=o Icn l2 r2n rand f E H2, it follows


that L::=olenI 2 < 00 and thus 1*(0) ~f L::=ocnein9 E L 2 ([0,271"],>.).
Furthermore, 111* - frll2 = L:~o lenl 2 • (1 - r2n). If Os rn i 1 then from
the dominated convergence theorem [applied in L1 (Z+,()] it follows that
frn IIJ21*.
b) The properties listed in 8.14 and Note 8.1 for the Poisson kernels
Pr imply that if 9 E L1 ([-71",71"],>') and grn(s) ~ D7r
g(t-s)Prn (s)ds then
gr n IIJ1 9 as rn i 1. The desired conclusion is somewhat stronger.
For a in [-71",71"], and t: in (0,71"),

gr(a) - g(a) = [: [g(a - t) - g(a)] Pr(S) ds

= [~€ + [€€ + 17r [g(a - t) - g(a)]Pr(s) ds


def
= h+I2 +h
Since max s E[-7r,-€j Pr(s) = max s E[€,1t"j Pr(s) = Pr(t:), reference to the in-
tegrals above leads to 1111 + 1131 s Pr(t:) (11g111 + 271"Ig(a) I). It follows that
°
lId + Ihl - as r - 1.
At this point, recourse to the Lebesgue set Lg of 9 is helpful. Thus for
J;
G(tj a) ~f Ig(a - s) - g(a)1 ds and R ~ 71"(1- r), integration by parts of
Solutions 14.9-14.10 439

the right member in (sI4.1) below leads to (sI4.2):

II I < RjE Ig(a - s) - g(a)1 ds (sI4.1)


2 - -E R2 + 4s 2
< (RG(t;a)) IE 8RjE sG(s;a) ds. (sI4.2)
- R2 + 4t 2 -E + -E (R2 + 4s2)2

If a E L9 and € is small, the right member of (sI4.2) is small uniformly with

respect to r.
Because H2 C HI, it follows that the results in the previous paragraph
may be applied to f. 0
[Note s14.1: The approach is parallel to that used in the proof
of Fejer's theorem, where, instead of the existence and proper-
ties of the Lebesgue set, the uniform continuity of the function in
question is exploited.
The fact that fr 1I-!1 f* permits the conclusion that for some
sequence {r n }O<rn<I' frn ~. f*. The use of Lf leads to the
stronger result: limrTl fr == f*.J

14.9. a) Because f* E L2 ([0, 21iJ, >'), then in the sense of II 112-cQnvergence,


f*(t) = E':=-oc:P
neint, whereas if 0:::; r < 1,

f (re it ) ~=
n
{en0 if n E Z+ .
otherwise

Since the Fourier coefficients are the values of continuous linear functionals,
n oo en "E.
it follows that lim -+ =
b) Since E':=-ooIYnl 2 < 00, for z in U, E':=oYnzn converges and de-
fines a function 9 in H(U). The orthogonality of the sequence {e int } nEZ
implies 9 E H2 and 14.6 implies g~ = {co otherwise
if n Z+.
n E Conse-
quently, g* == g. 0
14.10. If g(z) ~ E':=obnz n , f E F, and 0:::; r < 1 then

(sI4.3)

If 8 ~ {fn}nEN c Fthen (sI4.3), Cauchy's formula and the Arzela-Ascoli


theorem imply that 8 contains a subsequence 8 1 converging uniformly on
D (0, 1 - 2- 1 ). If there are subsequences 8 m such that

8m J 8 m +1> m = 1,2, ... ,M - 2,


440 14. Families of Functions: Solutions

and the elements of 8 m converge uniformly on D (0, 1 - 2- m ), then 8 M - 1


contains a subsequence 8M converging on D (0, 1 - 2- M ). Hence for some
diagonal sequence 8 ~ {Inl' In2 2 " " } ' Inl E 81. In22 E 8 2 , ••• , and 8
converges uniformly on compact sets of U. 0

14.11. L et ao be zero. If 9 def


= TIn I ~ then the maximum modulus
k=O ak
theorem implies g(U) C U. It follows that

1'(0) = {O if the multiplicity of ao is more than one


9'(0) TI~=l (-ak) otherwise .

Because g(O) = 0, Schwarz's lemma implies that if 9 is not a constant then


19'(0)1 < 1. Hence 11'(0)1 < TI~=llakl. Furthermore, if g(z) == ei8 then
11'(0)1 = TI~=llakl· 0
14.12. For r in [0,1), let F(r) be sup!E.FM(r; f). If F is normal and
F(r) = 00 then for some sequence 8 ~ {fn}nEN in F, M (r; In) 1 00
in which case 8 fails to contain a subsequence converging uniformly on
compact sets of U, a contradiction.
Hence Cauchy's formula shows that for k in Z+ and I in F,

1
1 (F(r))k
and so for all r in [0,1), SUP!E.F ICk (f)1 k :::; . It follows that
r

Conversely, if K :::; 1 then for each r in [0,1) and some positive fer),
(1 + f(r))r ~ p < 1 and for some ko and all I in F, if k > ko then
1
ICk(f)lk < 1 + fer). Hence if IE F and Izl :::; r,

Cauchy's formula implies Fis equicontinuous and the Arzela-Ascoli theorem


implies each sequence in F contains a subsequence converging uniformly on
D(O, r).
The argument at the end of Solution 14.10 applies. 0
1 l 8 1(w, z)/8z .
14.13. For w in n, N(w) = -.
def
21r~ 11'
I ( ) dz IS the number of zeros
w, z
of I(w, z) in U. As a Z+ -valued continuous function, N is a constant and
Solutions 14.14-14.18 441

N(O) = n. Thus for each w in n, U contains points, 9k,w, 1 :::; k :::; n, not
necessarily pairwise different and such that I (w, 9k,w) = O. 0
14.14. For each I in F and some 9, in H(U), I(z) = z3 9,(z). The
maximum modulus theorem implies 9,(U) C U. Cauchy's formula and
the Arzela..Ascoli theorem imply F is normal and hence for some 10 in F,
ko (~) 1= a. Thus if 10 (~) = ae it then e- it 10 (~) ~ F (~) = a

and F E F. Because 9,(U) C U, 19, (~) 1 :::; 1. It follows that a :::; ~.


Since z3 E F, a = ~. If 9, is not constant, the maximum modulus theorem

implies 19, (~) 1 < 1. Thus F(z) = Z3. 0


14.15. If S· n C i= 0 then I = {O} = 0 . E. That trivial case aside, for
some h in E, Z(h) = S and the order of the zero of hat Zn is qn. Hence if
I E I then k E E, i.e., I = h . E. 0

14.16. If I E F and 1(0) = a then CPa 0 I ~f 9 E F, 9(0) = 0, and


1'(0) - lal 2
Schwarz's lemma implies 19'(0)1 :::; 1. However, 9'(0) = (1- lal 2 ) • Hence

18~ ~2 (0) 1:::; 11'(0)1 :::; 1. Moreover, if lo(z) ~ z then 10 E F and

18~~~(0) 1= I~(O) = 1. o
14.17. If r E [0,1), I E F, and z E D(O, r) then Cauchy's formula implies
that I/(z)1 :::; 21 - 11 . Hence Cauchy's formula and the Arzela..Ascoli
7r -r
technique imply F is normal. 0
14.18. a) For each a in n and some positive r(a), D(a, r(a)) C n. If
I E F then Cauchy's formula implies

I/(a)1 :::; 10 II (a + r(a)e it ) I dt and


27r

r(a)
10
I/(a)1 sds = (r(a))21/(a)1 :::;
2
r
1D(a,r(a))
I/(z)1 d)..2:::; rI/(z)1 d)..2.
10.
2
Let M(a) be (r(a))2.
b) For some sequence K ~ {Km}mEN of compact sets, Km C K~+1
and n = UmEN Km. Hence any compact subset of n is a subset of some
member of K.
442 14. Families of Functions: Solutions

If 8 ~f Un} nEN C :F, the result in a) and the Heine-Borel theorem


imply that for some constant M m , Ifn(z)1 :=:; Mm,n E N, on Km. Cauchy's
formula and the Arzela-Ascoli theorem imply that 8 contains a subsequence
8 1 uniformly convergent on K 1 ; 8 1 contains a subsequence 8 2 uniformly
convergent on K 2 ; •••• Hence there is, by abuse of language, a diagonal
subsequence 8 00 uniformly convergent on every compact subset of O.
c) For the compact-open topology T of :F, let 0 be an open cover of
:F. If B is a countable base for the topology of C and K is the sequence in
b) then the set of all sets {f : f(K) C 0, K E K,O E B} is a countable
base for T. Thus it may be assumed that 0 is countable: 0 = {Up} pEN'
If for each P in N, Vp ~f
def
U: Up fails to cover :F, then for some sequence
-
G = {gp} PEN' gp ¢. Vp. But a) ensures that G contains a subsequence G
converging uniformly on each compact subset of 0 to some function 9 in
H(O). For each compact subset K of 0, iK Ig(z)1 d)..2 :=:; 1, whence 9 E :F.
If 9 E Upo then all but finitely many of the elements of G are in Upo ' hence
in Vpo ' a contradiction. D
[ Note sI4.2: The method of proof for c) above is essentially
that for proving the Heine-Borel theorem.]

14.19. a) Because J(t) = # (Z(f(·, t)) n U) (E Z+) and J(t) is a contin-


uous function of t, it follows that J(t) is a constant.
b) If K is a compact subset of U, the continuity of 9 implies that
N
the holomorphic functions FN(Z) ~f ~ L f (z, ;) 9 (;) ,N E N, are
n=1
uniformly bounded on K. Cauchy's formula and the Arzela-Ascoli theorem
imply a subsequence {FNm}mEN converges uniformly on K to F g • D

14.3. Defective Functions

14.20. The simple connectedness of 0 justifies each of the following


arguments.
a) Since f is {O}-defective, for some pin H(O), f = e21l'iP. Because f
is {I }-defective pis Z-defective.
b) Since p is {O }-defective, there is a q such that p = q2.
c) Since p - 1 is {O}-defective, there is an r such that p - 1 = r2.
d) Since q2 - r2 = 1 = (q - r)(q + r), q - r is {O}-defective whence for
some S, q - r = e S • D
14.21. In the notations of 14.20, f = _e1l'icosh2s. If s(z) E 8 then direct
calculation shows f (z) = 1. D
[ Note sI4.3: The technique used in Solutions 14.20, 14.21
may be viewed as producing from a {O, 1}-defective function new
functions that are 8-defective for various infinite sets 8.]
Solutions 14.22-14.24 443

14.22. If z ~f a + ib E S then z ± ~i are in S. Because

In(Jm + 1 + y'm) -In(y'm + Jm -1)


= In( v'2 + 1) < 1 if m =1
{
<In.Jm+1 ::;lnV3<1 ifm>1'
m-1

~ any z
t hen lor = x+
def.
~y and £,or some a =
def.
p+ ~r
. S 1
m 1 1 d
, X - P < '2 an

7r V3
Iy - rl < "4 < 2"' and so Iz - 81 < 1. 0

14.23. Integration implies I/(z)1 ::; RM in D(O, R). Because 9 ~ 1 -


is {O}-defective in D(O, R)O, for some h in H (D(O, R)o),

= (1'(0)
(h(z)) 2 def 1'(0)
1 - - z + ... )2 = 1 - - z + ... = g(z).
2, ,
RM
In D(O,R)O, l(h(z))21 ::; 1 + TYr whence

a2 2 11'(0)1 2 2 2 RM
1+ 41,1 2 R ::; 1 + 41,1 2 R ::; IIhll oo ::; 1+ TYr'

14.24. The sequence { M(1-~'I')}


2;k ' converges to zero and, by

kEZ+
hypothesis, M(O; 1') ?: 1. Let ko be the largest k such that

. TO = 21o then 0 < TO::; 1,ToM(1- To;I')?: 1 > 2M


Thus If TO (
1- 2;1'
TO )
.
k
For some b in D (0, 1 - TO), II'(b)1 = M (1 - TO; 1'). Then

g(z) ~ I(z + b) - I(b)


def 1 2
behaves as does
.
1m 14.23 when a = - , M
TO
= -.
TO o
444 14. Families of Functions: Solutions

14.25. For s as in 14.20, if Ibl ::::; r and s'(b) =I- 0 then

( ) ~f s(b+(1-r)z) ~f
gz - (l-r)s'(b) -Co+z+···

is such that g(U) contains no translate of D (0, (1 _ r~ ISI(b)l). Thus

14.23 implies Is'(b)1 < ~


1-r
which is valid a fortiori if s'(b) = o.
Since the value of s(O) is determined by the value of 1(0), it follows
that when z E D(O,r) then

16 16
Is(z) - s(O)I::::; 1- rr <1- r'
16 def
Is(z)1 < Is(O)1 + 1- r = <P 1 (f(0),r), and
II(z)1 < e1rcosh24>1(f(O),r) ~f<p(f(O), r). D

14.26. If I E Fa,b then g(z) ~f ~~:~ =~ is meromorphic and {O,oo}-


defective. As a meromorphic function, 9 is the quotient of two entire func-
tions h, k sharing no zeros: 9 = £. Because 9 is {O, 00 }-defective, it may
be assumed that both hand k are {O}-defective. Hence 9 is entire and
{O, 00 }-defective. Picard's theorem implies 9 is a constant, whence I is a
constant. Any family of constant functions is spherically normal. D
14.27. Since B is simply connected, it follows that # (C \ B) > 1. Because
the elements of Fare (C \ B)-defective, 14.26 implies that F is spherically
normal. D
14.28. A Mobius transformation 9 maps I1+ onto B ~f {z : 3?(z) < O}
and g(lR) = {z : 3?(z) = O}. By abuse of language, gog ~f 1i consists of
functions holomorphic in n and mapping n into B. If lEg then lego! is I
bounded by one, whence Cauchy's formula and the Arzela-Ascoli technique
imply e goF is normal, i.e., for some real constants a, b, {ea!+b : I E F}
is normal. Hence {e a ! : I E F} is normal. If a ~ 0 then F is normal; if
a < 0, F is spherically normal. D
[ Note sI4.4: In the discussion above, I1+ may be replaced by
any half-space (determined by aline).J

14.4. Bergman's Kernel Functions

14.29. a) If Un} nEN C SJ is a II 112-Cauchy sequence then for some I in


L2 (n, A2), IIln - 1112 -+ 0 as n -+ 00. It follows that In ~ I as n -+ 00.
Solutions 14.30-14.31 445

If a E 0 and 9 E Sj then for some positive r,

in D(a, r). Thus

(s14.4)

Hence the map L2 (0, >'2) 3 9 f--> g(a) E C is a bounded linear functional.
It follows that In(a) -+ I(a) as n -+ 00. The argument shows also that on
D(a, r), In ~ I. It follows that I E H(O) whence I E Sj.
b) If a E K and 0 < 'T/ < 8 then D(a,'T/)O cO and if Z E D(a,'T/)O,

=L
00 I(n)( )
I(z) - - Ia_(z - at and
n.

t 1" I~ f(~~a) l'


n= O

IIflll;, rne,n' r drdO

00 I/(n)( )1 2 2n+2
> 7r '"' a > 7r'TI21/(a)12.
- f;:o _'T/_
(n!)2 n+1 - "

1 2
Thus (7r8 2 ) may serve for M K ,

c) If {In}nEN C F, the inequality in (s14.4), the argument in a),


Cauchy's formula, and the Arzela-Ascoli theorem imply that F is normal.
o
14.30. If z E 0 6 then 14.29b) implies

o
14.31. If I ~f E;;=1 cn¢n E MN then 1 ~ II/II~ E;;=11¢n(t)1 2 and the
criterion for equality in Schwarz's inequality implies that for some constant
-- 1
c, en = c¢n(t). Thus C = N 2 and then J.L = ,;c. 0
En=ll¢n(t)1
446 14. Families of Functions: Solutions

[ Note 814.5: In Solution 14.31, if r is the distance of t from


an then from 14.29 it follows that E:=11<Pn(t)1 2 ::; 1r~2.J
14.32. a) The convergence of the series representing K(z, w) is assured
by 14.31 and Note 814.5. The Schwarz inequality applies.
b) Because K(w,w) = E:=11<Pn(w)1 2, the argument in Solution
14.31 implies K(w, w) > O.
c) For the sequence {IN ~ E:=l (f, <Pn) <Pn } NEN'

and the dominated convergence theorem applies. D


def K(z, w) 2 1
14.33. If g(z,w) = K(w,w) then g(w,w) = 1 and IIgll2 = K(w,w)
If h(z, w) ~ I(z) - g(z, w) then for fixed w, h E Sj and h(w, w) = O.
Since 0 = h(w, w) = E:=l f3n<Pn(w), direct calculation of the sequence
{ f3n ~f (h, <Pn)} shows that
nEN

IIIII~ = K(~, w) + IIhll~ = Ilgll~ + Ilhll~


whence 111112 > IIgl12 unless h = O. Hence min {111112 : IE Sj, I(w) = I}
is achieved iff 1= g. In particular, K(z, w) is independent of the choice of
the orthonormal system {<Pn} nEN' D
14.34. Direct calculation shows that

(f(z,w))n-l 81(z,w) }
{ <Pn(z) ~f ~ 8z
Y; Rn
nEN
1
is a complete orthonormal system for Sj, <Pl(W) = y'rrR' and <Pn(w) =0
if n 2: 2. Because K(z,w) is independent of the choice of a complete
81(z,w)
orthonormal system, K (z, w) = -1r-"'~-==Z2:;-- D
15
Convexity Theorems

15.1. Thorin's Theorem

15.1. a) If f is monotonely increasing resp. decreasing then

sup f(x) = f(b) resp. sup f(x) = f(a).


"'E[a,b] "'E[a,b]

If f is convex then the result 3.8 implies the conclusion.


b) For example, In x satisfies the hypothesis but In x is strictly concave.
o
15.2. If ¢ is convex then

¢(tx + (1 - t)y) - At ~ t¢(x) + (1 - t)¢(y) - At


= t(¢(x) - A) + (1 - t)¢(y)
~ max {¢(x) - A, ¢(y)}.

Conversely, for all A in JR,

sup ¢(tx + (1 - t)y) - At ~ max {¢(x) - A, ¢(y)}


tE[O,I]

iff

¢(tx + (1 - t)y) ~ max {¢(x) - A + At, ¢(y) + At}


or
¢(tx + (1 - t)y) ~ max {¢(x) - A(1 - t), ¢(y) + At}.

Hence ¢(tx + (1 - t)y) ~ ¢(x) - A(1 - t) if

max {¢(x) - A(1 - t), ¢(y) + At} = ¢(x) - A(1 - t)

and ¢(tx + (1 - t)y) ~ ¢(y) + At if


max {¢(x) - A(1 - t), ¢(y) + At} = ¢(y) + At.

In each case when A = ¢(x) - ¢(y),

¢(tx + (1 - t)y) ~ t¢(x) + (1 - t)¢(y). D


447
B. R. Gelbaum, Problems in Real and Complex Analysis
© Springer-Verlag New York, Inc. 1992
448 15. Convexity Theorems: Solutions

15.3. If t E [0, 1] then

¢)..(tx + (1- t)y) ~ t¢)..(x) + (1- t)¢)..(y)


~ t¢(x) + (1 - t)¢(y). D

°
15.4. If E > then for some b(E) in B and for all d in D, f(b(E))+E
whence, since 9 is right-continuous, if 8 > then ° > f(d)

g(f(b(E)) + 8) ~ go f(d),
g(f(b(E)) + 8) ~ sup 9 0 f(d),
dED
go f(b(E))) ~ sup 9 0 f(d), and
dED
sup 9 0 f(b) ~ sup 9 0 f(d). D
bEB dED

15.5. As formulated, the result is equivalent to the statement that for all
A, the maximum principle in [0,1] relative to {O, 1} obtains for

f (tx1 + (1 - t)xo, Y) - At.


From 15.2 it follows that for each y, f (x, y) is a convex function of x. Then
15.3 applies. D
15.6. Since L is linear, L- 1 (C) is convex. Then 15.5 and 15.3 apply.
D
15.2. Applications

15.7. For each A in JR, e)..z f(z) ~f F(z) conforms to the hypotheses for
f. The maximum modulus principle as discussed in 9.32 implies, in the
present context,

M(x; F) ~ max (e)..a M(a; f), e)"b M(b; I)) and


InM(x; F) = AX + InM(x; I) ~ max(Aa + InM(a; f), Ab + InM(b; I)).

As in the proof of Thorin's theorem (15.6), if

, - InM(b;f) -lnM(a;l).
1\ - b ' 1.e., 1·f'I\a + I n M( a,·f) -- 1\'b + I n M(b·f)
"
a-
then direct calculation shows that for t in [0, 1],

InM(ta + (1 - t)b; I) ~ tlnM(a; I) + (1- t) InM(b; I). D


Solutions 15.8-15.10 449

15.8. For n as in 15.7 and a suitable real 0:, the map cp : nc 3 z 1-+ eaz
carries nc onto A(a, r : R). Hence f 0 cp conforms to the hypotheses of
15.7. D
[ Note s15.1: It is the method of proof of Thorin's theorem
rather than the theorem itself that resolves 15.7 and hence 15.8.
In [Th] the author attributes his attack on the general theorem
to Hadamard's original approach!
For In M (Xj f) resp. In M (rj f), one may substitute go M (Xj f)
resp. go M(rj f) when 9 is a monotonely increasing function con-
tinuous on the right. A notion of the strength of 15.7 and 15.8
and of the last remark can be derived from the following consid-
erations.
The maximum modulus theorem implies merely

M(xj f) :::; max (M(a j f), M(bj f))


resp.
M(tj f) :::; max (M(rj f), M(Rj f)) .
These inequalities do not imply the convexity of either M (Xj f)
or M(rj f)j a fortiori, they do not imply any of the convexity
properties oflnM(xj f), InM(tj f), goM(xj f), or goM(rj f) (cf.
3.12).
The appearance of In r in 15.8 is occasioned by the map cp.]

def "9
15.9. a) If 0 :::; rl < r2 < r then for some z = r2 e' 2,

~
-1 L.....
n
If (rl e ni): :I : ; -n1 L.....
2k .. ~ If (r2e
P a.!uU
n ei92 )IP

k=l k=l

(cf. Note 11.3, Note sl1.3, and 11.27). The inequality persists as
n~oo.

b) Thorin's method of proof as exhibited in 15.7 and as used in 15.8


applies. D
15.10. Write
~f a i<!>j ~f f3 i,Pk
Xj - rj e ,Yk - Ske
o :::; CPj, 'lj;k < 271", 1 :::; j :::; m, 1 :::; k :::; n.
For A1, A2 real and fixed and (0:0, ,80) in Q, owing to 15.5 and 15.6, it
suffices to prove that the logarithm of
M (0:0 + Alt, (Jo + A2t)
L L ajkrjO+A1t S~O+A2t ei(<!>j+..pk)
m n
def sup
(x,y,p,cr)ES j=l k=l
450 15. Convexity Theorems: Solutions

is a convex function of t on (0,00). If the real variable t is replaced by the


complex variable t + iu then each <Pj resp. 1/lk is translated by U>'1 In Tj resp.
u>'2lnsk and

m n
'"' '"' ao+).t(t+i'u) .Bo+).2(t+iu) i(<!>j+.p,,)
sup L..J L..J ajkTj sk e
(x,y,p,u)ES j=1 k=1

= M (ao + >'It, f30 + >'2t) .


If all the~iables save t and u are fixed;.jhe three-lines theorem is appli-
cable to M (ao + >'It,/30 + >'2t). Thus InM (ao + >'It,f30 + >'2t) is a convex
function oft on (0,00). 0
Bibliography

The numbers in parentheses following the entries refer to the pages in this
text where the citations occur.

Ar - Arens, R., Note on convergence in topology, Mathematics Magazine,


23 (1950), 229 - 234 (154).
Be - Berberian, S. K., Lectures in junctional analysis and operator theory,
Springer-Verlag, New York, 1974 (368).
Berl - Bergman, S., dissertation, Philosophische Fakultat der Berliner
Universitat, 1921 (130).
Ber2 - , Uber die Entwicklung der harmonischen Funktionen der
Ebene und des Raumes nach Orthogonalfunktionen, Mathematische An-
nalen, 86 (1922), 238 - 271 (130).
Bes - Besicovitch, A. S., On the definition and value of the area of surface,
Quarterly Journal of Mathematics, 16 (1945), 86-102 (228).
BOll - Bourbaki, N., Topologie generale, Livre III, Chapitre I, Chapitre
II, Hermann & Cie., Paris, 1940 (21).
Cohl- Cohen, P. J., Factorization in group algebras, Duke Mathematical
Journal, 26 (1959), 199 - 205 (321).
Coh2 - , The independence of the continuum hypothesis, Pro-
ceedings of the National Acadademy of Sciences, 50 (1963), 1143 - 1148;
The independence of the continuum hypothesis, Proceedings of the National
Acadademy of Sciences, 51, (1964), 105 - 110 (273).
Fin - Fine, N., On the Walsh junctions, Transactions of the American
Mathematical Society, 65 (1949), 372 - 414 (68).
GeK - Gelbaum, B. R. and Kalisch, G. K., Measure in semigroups, Cana-
dian Journal of Mathematics, 4 (1952), 396 - 406 (246).
GKO - , and Olmsted, J. M. H., On the embedding of
topological semigroups and integral domains, Proceedings of the American
Mathematical Society, 2 (1951), 807 - 821 (247).
GeO - and Olmsted, J. M. H., Theorems and counterexamples
in mathematics, Springer-Verlag, New York, 1990 (154, 199,203, 214, 221,
228,353).
GiJe - Gilman, L. and Jerison, M., Rings of continuous junctions, D. van
Nostrand Company, Inc., New York, 1960 (16).
Halm - Halmos, P. R., Measure theory, D. van Nostrand Company, Inc.,
New York, 1950 (244).
HeR - Hewitt, E. and Ross, K. A., Abstract harmonic analysis, Second
Edition, 2 vols., Springer-Verlag, New York, 1979 (54, 244, 321).

451
452 Bibliography

HeS - Hewitt, E. and Stromberg, K., Real and abstmct analysis, Springer-
Verlag, New York, 1965 (48).
Hi - Hille, E., Analytic function theory, I,II, Ginn and Company, Boston,
1962 (130).
KacS - Kaczmarz, S. and Steinhaus, H., Theorie der Orthogonal Reihen,
Monografje Matematyczne, Warszawa, 1951 (68).
Kak - Kakutani, S., Ueber die Metrisation der topologischen Gruppen,
Proceedings of the Imperial Academy of Japan, 12 (1936), 82 (191).
Ko - Kolmogoroff, A., Grundbegriffe der Wahrscheinlichkeitsrechnung,
Chelsea Publishing Company, New York, 1946 (67).
Ko - Kothe, G., Topological vector spaces I, Springer-Verlag, New York,
1969 (349).
La - Landau, E., Darstellung und Begriindung einiger neuerer Ergeb-
nisse der Funktionentheorie, Chelsea Publishing Company, New York, 1946
(129).
Loo - Loomis, L. H., An introduction to abstmct harmonic analysis, D.
van Nostrand Company, Inc., New York, 1953 (54, 346, 368).
Me - Mendelson, E., Introduction to mathematical logic, D. van Nostrand
Company, Inc., New York, 1964 (243).
NM - Neumann, J. von and Morgenstern, 0., Theory of games and eco-
nomic behavior, Princeton University Press, Princeton, 1947 (132).
Rin - Rinow, W., Lehrbuch der Topologie, VEB Deutscher Verlag der
Wissenschaften, Berlin, 1975 (154).
RoDi - Roelcke, W.and Dierolf, S., Uniform structures on topological
groups and their quotients, McGraw-Hill, Inc., New York, 1981 (16).
Rud - Rudin, W., Real and complex analysis, Third edition, McGraw-Hill,
Inc., New York, 1987 (48, 368).
Sch - Schaefer, H. H., Topological vector spaces, Springer-Verlag, New
York, 1970 (349).
Sier - Sierpinski, W., Sur une propriete des series qui ne sont pas ab-
solument convergentes, Bulletin International de l' Academie Polonaise des
Sciences et des Lettres, Classe des Sciences Mathematiques et Naturelles,
Cracovie [Cracow] 149 (1911), 149 - 158 (273).
SiW - Singer, 1. M. and Wermer, J., Derivations on commutative Banach
algebms, Mathematische Annalen, 129 (1955), 260 - 264 (368).
Sol - Solovay, R., A model of set theory in which every set is Lebesgue-
measumble, Annals of Mathematics, 92 (1970), 1 - 56 (243).
Sz - Szpilrajn, E., 0 mierzalnosci i warunku Baire'a [On measumbility and
the condition of Baire], Comptes Rendus du I Congres des mathematiciens
des Pays Slaves, Varsovie (1929), 299ff. (146).
Th - Thorin, G. 0., Convexity theorems, Seminaire Mathematique de
L'Universite de Lund, Tome 9, 1948 (449).
Bibliography 453

Tu - Tukey, J., Convergence and uniformity in topology, Princeton Uni-


versity Press, Princeton, 1940 (21).
Wal- Walsh, J. L., A closed set of normal orthogonal/unctions, American
Journal of Mathematics, 55 (1923), 5 - 24 (68).
We! - Weil, A., Sur les espaces a structure uniforme et sur la topologie
generale, Hermann & Cie., Paris, 1937 (16, 21, 68).
We2 - , L'integration dans les groupes topologiques et ses appli-
cations, Hermann & Cie., Paris, 1940 (54, 135, 246).
Wey - Weyl, H., Uber die Gleichverteilung von Zahlen mod. Eins, Math-
ematische Annalen, 77 (1916), 313 - 15 (43).
Wi - Widder, D. V., The Laplace transform, Princeton University Press,
Princeton, 1946 (251).
Zy - Zygmund, A., Trigonometric series, 2 vols., Cambridge University
Press, Cambridge, 1988 (135).
Symbol List

The notation a.b. d indicates Chapter a, Section b, page d among the


Problems; similarly s a.b. c indicates Chapter a, Section b, page c
among the Solutions No, 1.1. 4

A, 1.1. 6
AO, 2.1. 12
A, 1.2.9
A, 2.1. 14: the closure of A
A(a, r : R), 9.3. 107
A(E), 1.1. 4
AC([O, a), C), 3.1. 35: the set of absolutely continuous functions in lR[O,a)
AC([a, b], lR), 5.1. 58: in lR[a,b] , the set of absolutely continuous functions
a.e. (JL), 5.1. 56
A-free, 4.1. 42: of a number, that it is independent of A
A-invariant, 6.4. 89
A(r), 8.2. 104
arg(z), 9.2. 391: any number () such that z = Izle i9
A(U), 6.4. 90
A ~ B, 5.1. 48: with respect to the relevant measure A/:::,B is a null set
A/:::,B, 5.1. 48

~, 2.1. 14
B-module, s 6.1. 321
B(O, 1), 6.1. 77
B(O,r)O, 3.1. 34
B 1 (E)), 2.3. 27
B n , s 5.1. 258
BV([O, 1], C), 3.1. 35: the set offunctions in C[O,l] and of bounded variation
BV([a, b], lR), 5.1. 58: in lR[a,b] , the set of functions of bounded variation

c, 1.1. 4
C, 1.1. 3
C-algebra, 6.4. 89: an algebra that is a C-module
Coo, 7.1. 96
CH, s 5.1. 273
Ck (lRn,c m), Coo (lRn,c m), 3.1. 30
C(X, Y), 1.1. 4
(C([O, 1],C))*, 2.3. 25

455
456 Symbol List

Co, 2.1. 16
Co (X, q, 1.1. 4
CooO(X, q, 1.1. 4
Cont(f), 2.3. 34: the set where (the function) I is continuous
Conv(A), 6.3. 86

D(O, 1), s 2.3. 185


D(a,r), 2.3. 26, 7.1. 95
deg(p), 2.2. 21
det(M), 7.3. 99
diam, 4.1. 41
dim(M), 6.2. 82
Discont(f), 3.1. 36: the set where (the function) I is discontinuous
dkf (xo), 3.1. 30
D N , 5.1. 54
def . ¢>(x + h) - ¢>(x)
D±¢>, s 3.1. 196: e.g., D+¢>(x) = hmh!O h
d(x, K), s 1.1. 143: inf {d(x, y) : y E K}

C, 2.1. 20, 7.1. 95


E(f) = In Iw) dP(w), 5.2. 66
E(P) - (E(f»2, 5.1. 52
cp , 6.1 77
EW, 6.2. 81
E(z,p), 10.3. 112
Eo(f, a), 1.2. 9
(E, I II) 2.3. 24

I[a], 2.3. 26, 3.1. 31


I[a], 3.1. 31
I-fixed point, 8.2. 105: a point x left fixed by I, i.e., I(x) = x
I-image, s 3.1. 202: for an I : X f---+ Y, I(X)
I{n}, 2.3. 24
F N , 5.1. 54
In ~. I, 5.1. 50: ¢>n converges to I almost everywhere (a.e.), i.e., except
for x in some null set, limn --+ oo In (x) = I(x)
In m~as I, 5.1. 51: In converges to I in measure, i.e., if E > 0 then
limn --+ oo JL ({ x : I/n(x) - l(x)1 > E}) = 0
F, 2.1. 14
Fn, 2.1. 15
(f,O), 11.1. 115
I ~ g, 5.1. 50: except on a null set, I = g
1/\0, I V 0, 3.1. 31
I /\ g, I V g, 1.2. 7
Fen 2.3. 25: the l:nion of a countable set of closed sets
Symbol List 457

g, 2.3. 29
(G; FI, ... ,Fn)-convex, 15.0. 132
g(f), 2.1. 33
G / H, 4.2. 45: for a group G and a subgroup H, the set of left or the set of
right cosets of H
G : H, 4.2. 45: for a group G and a subgroup H, # (G / H)
Gfj , 2.1. 20: the intersection of a countable set of open sets

lHI, 1.1. 3
H(O), 6.4. 90

id, 3.1. 30
im(f), 1.1. 4
Ind-y(a), s 9.3. 392
llR' 1.1. 3

I n , s 6.3. 359

ker(T), 6.3. 84
K(f), 10.3. 113
K (fn), 5.1. 64
lK-Hamel basis, 6.1. 71, 6.3. 84: a Hamel basis with respect to coefficients
from the field lK
lK-maximallinearly independent, 6.1. 71
K -< f -< U, 2.1. 17, 4.1. 43: when K(X) 3 K CUE O(X) then
0::; f(x) ::; 1, f(x)IK= 1, supp(f) C U
K -< g, s 4.1. 238: K -< 9 -< X

ceg, 2.3.29
L g , s 14.2. 438
limn_co En, 1.1. 4
limx=af(x), limx=af(x), 2.1. 16
Lip(a), 3.1. 31
In, 11.1. 115
L(r), 8.2. 104
L u , 1.2. 9
LuI, 1.2. 10
L(O), 13.1. 124
LRN, s 5.1. 252
lsc, 2.1. 16
Ll, 1.2. 10
U(X, /-L), 2.3. 24
£(-Y), 4.1. 41: for a metric space (X, d) and a curve, : [0, 1] ~ X,
sup {L;;=1 d(g (tn) , 9(tn-I)) : 0 = to < tl < ... < tN = 1 }
458 Symbol List

M, 1.2. 8, 5.1. 58
Mat mn , s 3.1. 207
max{f,g}, 3.1. 31
M'R.}, 2.3. 29
mid(a, b, c), 1.2. 7
mid(f,g,h)(x), 1.2. 7
min{f,g}, 3.1. 31
M n1 ,n2, ... ,nk' 1.1. 6
Mo, 7.1. 97
m(r; I), 7.1. 97
M(r; I), 7.1. 97
(M)w· , s 6.3. 349: the weak* closure of M
M u , 6.1. 70
MVP(O), 13.1. 124

N(A), 2.1. 12
n ~ n', 2.1. 14

OSCN(f), 6.1. 72, S 5.1. 275

P, 5.1. 53, S 13.2. 432


P, 3.1. 35
IFI, 3.1. 35: for a finite partition P, the maximum of the measures of the
constituents of P
Pa(f), 12.1. 121
P(f), 7.3. 99

QL, 2.3. 29

JR, 1.1. 3
i:, 1.2. 7
Res(f, a) 12.1. 121
JR+, 1.1. 3, 2.2 22, s 4.2. 246
JRd, s 4.2. 245: JR in its discrete topology
R(E), 1.1. 4
(JR, SA, A), s 5.1. 251

S, 7.1. 95
S+,1.1.3
se, 7.1. 95
5, 1.2. 11
se, 2.3. 28
S-defeetive, 14.3. 129
S(f), 7.3. 99
sgn(z), 2.2. 22
Symbol List 459

SL(2, C), 7.3. 99


SN(J, x), 5.1. 54
span(S), 5.3. 67, 6.3. 83
supp(g), s 6.1. 309: the support of (the function) 9
SIx], 3.1. 31
S,a, 1.1. 4
S,a (IR n ), 1.1. 5
A~, A~, 5.3. 67, 6.1. 71

T*, 6.1. 70
IITII, 2.3. 24: sup { IITxll wx E B(O, I)}
To, 7.3. 98
Tab, 7.3. 98
Tabcd, 7.3. 96
79,2.3.29
7VS, 2.3. 24

U(n, C), s 4.2. 245: the group of n x n unitary matrices


usc, 2.1. 16
Uxx = (Uij)~j:l' s 5.1. 279

(V, II 11), 6.1. 71


Var(F), 5.1. 52
Var(J)n, 5.2. 66
Var[O,l],P(J), 3.1. 35
var[O,x] (J), 3.1. 35
[V, W], [V, W]e, [V], (V]a, 6.1. 70

{x}, 4.1. 41
[x], 4.1. 41
#(X), 1.1. 4
(X, d), 1.1. 4, 2.1. 15
x' -section, 5.1. 49

Il, 1.1. 3
Z(J), 7.3. 97
ZF, s 4.2. 243
zP, 7.1. 97

r(z), 10.3. 114

f(x + h) - f(x)
{
D.h, 6.2. 79: ("2 hh)
f xe''''' - f(x)
if f E C'll'
h
460 Symbol List

an, 3.1. 36: for I in G(R., R.), x ~ 2n (f (x + 2- n) - I(x»


"(*, 2.1. 12, 7.1. 95
1£*, 1£*, 1.2. 11, 4.2. 40
11£1, 4.1. 40
I£f(r), 10.3. 112
I£f : [a, b), 5.1. 64
1£ * >., 5.1. 61
1£1 «: 1£2, 5.1. 48
1£1 ..L 1£2, 5.1. 48

lIf(r), 10.3. 112

aA(E), aR(E), 1.1. 4


aN(f, x), 5.1. 54
a (V, V*), a (V*, V), 6.1. 70

T, 2.3. 27

v, 10.3. 112

¢Hx), ¢~(x), 3.1. 33


(Po:(z), 8.2. 105, s 7.1. 374

n, 1.1. 4, s 1.1. 141


w(f), 10.3. 113

6:), Ee, 6.3. 83, 6.3. 88, s 6.3. 350


as, 2.1. 15
oooG, 9.3. 109
X1'erXp 2.1. 13
6., 2.1. 15, 21
6. (p,q,r), 7.1. 98
AWB, 1.1. 3: AU B when An B = 0
l:JnENAn' 4.1. 40: UnEN An when the summands are pairwise disjoint
V-closed, 4.1. 44
A-closed, 4.1. 43
1, 1.1. 6, 1.2. 9
f, 3.2. 37: line integral
In ~. I, 5.1. 50: In converges to I a.e., i.e., except on a null set
In m~as I, 4.1. 44: In converges to I in measure, i.e., if € > 0,
limn --+ oo 1£ {x : I/n(x) - l(x)1 ~ €} = 0
Symbol List 461

fn ~ f, 2.3. 25: fn converges uniformly to f, i.e., if E > 0 then for some


N(E) and all x, Ifn(x) - f(x)1 < E when n > N(E)
Xn ~ X, S 6.1. 308: Xn converges weakly to x, i.e., for every x* in the dual
space, limn -+ oo (xn' x*) = 0
fn IIJr f, 6.1. 73: fn converges in p-norm to f, i.e., if E > 0 then for some
N(E), IIfn - flip < E if n > N(E)
Glossary jIndex

The notation a.b. d indicates Chapter a, Section b, page d among the


Problems; similarly s a.b. c indicates Chapter a, Section b, page c among
the Solutions.

ABEL, N.H. (lemma), 2.2. 22


Abel summation, s 3.1. 217, 218, s 5.1. 274
absolutely continuous, 3.1. 35, 5.1. 64: of an f in c[a,bj, that if f > 0,
for some positive 6, if a S ai < bi < ai+1 S b,1 SiS n and
I:~=1 Ibi - ail < 6, then I:~=1 If (b i ) - f (ai)1 < f.
- - - - - - , (measure), 5.1. 48
absorbing, 6.1. 70
adhere, adherence, adherent, 2.1. 15
adjoint, 6.1. 70, 6.2. 79, 6.3. 86
ALAOGLU, L. theorem, s 6.3.351: For a Banach space E, B(O, 1) in E* is
a (E*, E)-compact.
algebra, 14.2. 129: a ring R that is a module over a field lK.
algebra resp. a-algebra (of sets), 1.1. 4: a set of sets closed with respect to
the formation of complements and finite resp. countable unions.
algebraic number, 1.1. 3: a zero of a polynomial in Q[z].
algebraic variety, s 3.3. 207: the set of common zeros of a finite set of
polynomials in lR[z].
A-measurable, 1.2. 9
analytic continuation, 11.1. 115
analytic (set), 1.2. 7: for a set E of sets, an element of A(E).
anharmonic ratio, 7.1. 96
annulus (A(a,r: R)), 9.3.107
antipodal, 2.3. 25, 3.2. 39: of a pair of points a ± x on 8B(z, r).
approximate identity, 6.1. 76, s 5.1. 285: for a topological algebra A, a net
n : A :3 >. t---> n). E A such that for all a in A, limA n). . a = a.
arcwise connected, s 12.3. 427: of a set S in a topological space X, that
for each pair {a, b} contained in X, and some curve

, : [0,1] :3 t t---> ,(t) E S,

,(0) = a and ,(1) = b.


463
464 Glossary jIndex

area theorem, s 8.1. 386: For f in 0 1 (JR n , JR n ), the Jacobian transformation,


J(x) ~f ::' and a measurable subset A of JR n , if f is injective on A
then >'n(f(A)) = fA f(x) IdetJ(x)I d>'n(x).
ARZELA, C.-AscoLI, G. theorem, s 2.1. 161 et alibi: For a compact met-
ric space X, a uniformly bounded equicontinuous set of functions in
O(X, JR) is II lloo-compact.
atom, 4.1. 40
auteomorphism, 2.1. 19: a homeomorphic self-map.
autojection, 2.2. 22: a bijective self-map.
automorphism, 3.1. 30: a bijective homomorphism.
Axiom of Choice, s 4.2. 243: If A is a set and each S).., >. E A is a set then
there is a set consisting of precisely one element from each S)...

BAIRE, R. category theorem, s 2.2. 169, s 2.3. 179: The intersection of


countably many dense open subsets of a complete metric space is dense.
BANACH, S., 5.1. 59, et alibi
- algebra, 6.4. 89, s 6.1 321: a C-algebra that is a Banach space (A, II II)
such that IIx, yll ~ IIxll . Ilyll·
- space, 6.3. 84: a complete normed vector space.
- STEINHAUS, H. theorem, s 6.1. 308: If E, F are Banach spaces, S is of
the second category in E, A C [E, FJ, and sUPTEA IIT(x) II < 00 for
every x in S then SUPTEA IITII < 00.
base (for a topology), 2.1. 12
- (at a point), 2.1. 13
-(for a uniformity), 2.1. 15
basic neighborhood, 2.1. 13, s 2.1. 157
basis, 6.1. 71
BENDIXSON, 1.,2.1. 166, 2.3. 186
BERGMAN, S. kernel function, 14.4. 130
BERNSTEIN, S. polynomials, 5.1. 258 - 259
BESSEL, F. W. inequality, 5.1. 251: For an orthonormal set {4>)..hEA in a
Hilbert space 5) and an x in 5), L)..EA I(x, 4>)..) 12 ~ Ilx11 2.
biholomorphic bijection, s 13.1. 430: for two regions 0 1 , O2 contained in C,
in H (01) a bijection 4> such that 4> (Od = O2 and 4>-1 E H (0 2 ),
bijection, 5.1. 62: a surjective injection.
bijective, 2.1. 20: of a map 4> : X t--+ Y, that 4> is one-one and 4>(X) = Y.
binary marker, 5.1. 50
biorthogonal, 6.3. 86: for a Banach space E, of a set {x).., X~hEA in Ex E*,
{Idef
that ( x).., x lL ) = {j)..1L = 0
if >. = Il
h . .
ot erWlse
BIRKHOFF, G. D. ergodic theorem, 5.3. 69, s 3.1. 192
BLASCHKE, W. product, 8.2. 105
Glossary jIndex 465

BLOCH, A. theorem, 8.2. 129


block, 3.1. 31
BOCHNER, S. measurable, s 6.3.358: for a measure situation (X, 5, J.L) and
a Banach space (E, II II), of a function I : X I-t E that it is a.e. the
II II-limit of a sequence of simple (E-valued) functions.
BOREL, E., 1.1. 5 et alibi
- -CANTELLI, F. P., 5.1. 50
- measure, 1.1. 5: : for a topological space X, a measure on the a-ring
S~(X) generated by F(X).
- set, 4.1. 40: in a topological space X, an element in the a-ring S.a(X)
generated by O(X).
boundary, 2.1. 15, 9.3. 109
bounded, 6.1. 70
- approximate identity, 6.3. 90: an approximate identity n such that for
some M and all >., IIn.\1I ~ M.
- convergence theorem, s 2.3. 183: If {In} nEN is a sequence of uniformly
bounded functions in Ll(X, J.L) and In ~. I, then I E Ll(X, J.L) and
Ix
limn-+ oo Ix
In (X) dJ.L(x) = I(x) dJ.L(x).
bounded variation, 3.1. 35: A function in JR[a,bj is of bounded variation iff
for some M and each n in N,

branch, 11.1. 115


bridging function, s 6.1. 313: for an interval [a, b] and a positive €, in
Coo (JR, JR) a function such that [a, b] -< I -< (a - €, a + €).
BROUWER, L. E. J. fixed point theorem, 3.3. 39
- invariance of domain theorem, s 3.3. 216: If V E 0 (JR n ) and

I :V I-t W C JRn

is a homeomorphism then W E 0 (JR n ).

c
CANTELLI, F. P., 5.1. 50
CANTOR, G., 2.1. 16, et alibi
- -Bendixson theorem, cf. BENDIXSON, I.
--LEBESGUE, H. theorem, 5.1. 51: IfE~=oancosnO+bnsinnO converges
a.e. then limn-->oo an = limn-->oo bn = o.
- -like set, 2.1. 16
- (the) function, 2.1. 18
- (the) set, 2.1. 16
466 Glossary/Index

CARATHEODORY, C. measurable, 1.2. 11


cardinality, 1.1. 4: The Axiom of Choice implies every set X can be well-
ordered. When X and Y are well-ordered and there is an order- pre-
serving injection T : X 1--+ Y, by definition, X -< Y. For a given set
Y, Y ~f {X : X is well-ordered and X -< Y} is well-ordered with re-
spect to -< and #(Y) ~ infY.
Cartesian product, 1.1. 3, 2.1. 13: for a set {XY}-YEr of sets, the set of all
= ( ... , x-Y' . .. ) ,x-y E X-y.
vectors x def
category, 2.3. 24, 29: a) a class C ~ {A} of objects; b) a class of pairwise dis-
joint sets [A, Bj, A, BEe of morphisms; c) an associative composition
(I, g) E [A, Bj x [B, ejl--+ go f E [A, ej; d) in each [A, Aj an identity
lA such that for f in [A, Bj and 9 in [e, Aj, f 0 lA = f, lA 0 9 = g.
CAUCHY, A. L. DE, 2.1. 16 et alibi
- complete, 2.2. 16
- completion, 2.1. 16: for a uniformity situation (X, U), the set of equiva-
lence classes of Cauchy nets.
- formula, s 7.3. 377 et alibi: If f E H(O), a E 0, r is a cycle, and
Indr(b) = 0 for all b in C \ 0 then

f
(k) k!
(a)· Indr(a) = -2'
7rZ
i (
r z-a
f(z)
)k+l dz.

- -net, 2.1. 16
- -Riemann equations, s 8.1. 381: If f ~f U + iv E H(O) then

- theorem, s 9.3. 394, et alibi:If f E H(O), a E 0, r is a cycle, and

lrrf(z) dz = O.
1.
Indr(b) = 0 for all b in C \ 0 then -2
7rZ
central index, 10.3. 112
chain (of indices), 3.1. 31
character, 6.1. 72
characteristic function, 1.1. 4
circled, 6.1. 70
closed (with respect to ... ), 1.1. 6, 2.1. 13
- ball, 2.1. 16
- curve, 2.1. 12
- graph theorem, s 6.3. 347: If E, F are Banach spaces and T : E 1--+ F is
linear (not necessarily continuous) then T E [E, Fj iff the graph of T
is closed (in E x F).
- half-line, s 3.2. 224
- interval, 1.1. 3
- set, 2.1. 14
Glossary/Index 467

closure, 2.1. 12, 7.1. 95


coarser, 2.1. 14
cofinal, 2.1. 14
compact, 1.1. 4: of a set K in a topological space, that every open cover
contains a finite sub cover .
- -open,14.2. 127
complete measure situation, 1.2. 11
- metric space, 2.1. 16, 4.1. 42: a metric space (X, d) in which every
Cauchy net converges.
- orthonormal set, 6.2. 81: in a Hilbert space .fj, an orthonormal set 8
such that 81- = o.
-ly regular, 2.2. 21: of a topological space, that for every closed set F and
every x in X \ F, for some f in C(X, 1R), f(x) = 0 and f(F) = {1}
({x}-<1-f-<X\F).
completion (of a a-ring), 1.2. 11
complex measure, 1.1. 4
component, 5.1. 60: a maximal connected subset of a set.
concave (function), 4.1. 41
conditionally convergent, 5.1. 50: of a series 2:::=1 an, that for some per-
mutation 7r of N, 2:::=1 a,..(n) diverges.
conformal, 10.2. 110: of an f in H(O), that f is biholomorphic.
conjugate bilinear, 6.3. 79
connected, 2.1. 14
continuous, 2.1. 12
- on the right, 15.1. 133: of an f in X IR , that lime!O f(x + €) = f(x).
Continuum Hypothesis, s 5.1. 273: 2~'" = ~o+l.
contraction, contractive, 2.1. 19
converge, 2.1. 14
convex function (map), 3.1. 30
- hull, 2.3. 25: for a set 8 (in a vector space V), the intersection of all
convex supersets of 8.
- set, 2.3. 25: in a vector space V, a set 8 such that

{{ x, Y E 8} 1\ {o ::; t ::; 1}} =? {tx + (1 - t)y E 8} .


convolution, 6.1. 72
coset representative, 4.2. 45: an element of an equivalence class determined
by, e.g., a subgroup of a group, an ideal in a ring, etc.
- space, 2.3. 29: the set of cosets of, e.g., a subgroup of a group, etc.
cover, 2.1. 21: for a set X, in 2x a subset 8 such that UAES A:::> X.
count ably additive, 1.1. 4: of a set function CJl, that

L CJl (An).
00

CJl (UnENAn) =
n=1
468 Glossary/Index

count ably determined, 2.1. 13


- subadditive, 4.1. 40
counting measure, 4.1. 40
cross ratio, 7.1. 96
curve, 2.1. 12
- -image, 2.1. 12
cycle, s12.3. 427: for a set bih~i~n of curves, a functional denoted Ir and
defined according to: Ir(f) ~f Ir I(z) dz ~f L~=l 1'"1; I(z) dz.
cyclic group, s 2.3. 189: a group consisting of powers of a single element.
cylinder, 2.1. 13

DANIELL, P. J. functional, 1.2. 9


- measurable, 1.2. 8
- -STONE, M. R. extension 1.2. 7
degree, 2.3. 23: for a polynomial p(x) ~f L~=o akxk in which an '" 0, the
integer n.
dense, 1.2. 11: of a subset S of a topological space X, that S = X.
- in itself, 2.1. 14
derivation, 6.4. 90, s 6.4. 368
derivative, 3.1. 30
DIAMOND, R., s 5.1. 289
diagonal sequence, s 2.1. 161, s 14.2. 440
differential, 3.1. 30, 6.3. 84
dilation, s 7.1. 373: an expansive map between metric spaces.
DINI, U. theorem, s 5.1. 265: If X is compact, {gn}nEN C C(X,IR), and
gn L 0 then Ilgnlloo L O.
directed downward, 4.1. 43
- set, 2.1. 13
- upward, 4.1. 44
direct sum, 6.3. 83, 6.3. 88, s 6.3. 350
DIRICHLET, P. G. L. region, s 13.2. 432
- kernel, 5.1. 54
discrete, 2.1. 12
diset, 2.1. 13
distinguished (index), 3.1. 31
dominated convergence theorem, s 1.2. 147: For (X, S, JL), {In} nEZ+ con-
tained in L 1 (X,JL), Ifni ::; /10/, and In ~. g, then 9 E L 1 (X,JL) and
limn->ooIx Ix
In (x) dJL(x) = g(x) dJL(x).
d-separable, 5.2. 66: of a metric space (X, d), that it is separable in the
d-induced topology.
dual group, 6.1. 72: for a group G, the set of homomorphisms Q: : G 1-+ ']I'
forming a grvup with respect to pointwise multiplication.
Glossary/Index 469

dual pair s 6.3 349: a pair {V, W} of topological vector spaces such that:
a) W C V·, V c W· j b) if v is fixed in V and (v, w) = 0 for each w in
F resp. if w is fixed in W and (v, w) = 0 for each v in V then v = 0
resp. w = o.
- space, 2.3. 24, 6.1. 70
dyadic rational number, s 2.1. 160
- space, 2.1. 18

E
EGOROV, D. F. theorem, 5.1. 53, s 5.1. 251: For a totally finite (X,5,JL)
and a sequence {fn}nEN in M, if In ~. I and € > 0 then for some E
in 5, JL(E) < € and In ~ I on X\E.
eigenvalue, 6.2. 82: for a vector space V and a T in [V], a number A such
that for some nonzero vector v in V, T(v) = AV.
entire, 7.1. 95: of an I in ee, that I E H(C).
epimorphism, 4.2. 46: in a category C, a morphism A I-t B such that
A I-t B ~ C = A I-t B ~ C iff x = Yj when morphisms are homomor-
phisms, a surjective homomorphism.
equicontinuous, s 3.1. 204: of a set S of maps between uniform spaces (X, U)
and (Y, V) that if V E V then for some U in U and all I in S,

{(a,b) E U} =? {(f(a),/(b» E V}.

equidistributed, 4.1. 43
equivalence class, 1.1. 4: for ail equivalence relation R on a set S, a subset
E that is R-saturated, i.e.,

{{x E S} /\ {y E E} /\ {(x,y) E R}} =? {x E E}.

- relation, 2.1. 14: for a set S, a relation R that is reflexive, i.e.,

{x E S} =? {(x, x) E R},

symmetric, i.e., {(x, y) E R} =? {(y, x) E R}, and transItive, i.e.,

{{(x, y) E R} /\ {(y, z) E R}} =? {(x, z) E R}.

equivalent (functions), 6.1. 71


ergodic theorem (pointwise), 5.3. 69, s 3.1. 192
- - (mean), 5.3. 69
essential supremum, 2.3. 24
essentially bounded, 2.3. 24: of a function I that 11/1100 < 00.
- equal (nets), 2.1. 14
470 Glossary jIndex

Euclidean topology, 2.1. 20: for en, the topology induced by the (Euclidean)
norm: II (XI, ... ,xn)11 ~f J~~=1IxkI2.
EULER, L. constant, 10.3. 114
- formula, s 5.1. 261, s 8.1. 385: eix = cos x + isinx.
even, 3.1. 35: of a function 1 in eC , that 1 ( - z) = 1 (z); of a permutation
1r in Sn, the set of permutations of {I, 2, ... ,n}, that

i-j
IT . . = 1.
. . 1r(z) -1r(J)
-<3

event, 5.2. 66
eventually, 2.1. 14
expansion, expansive, 2.1. 19
expected value, 5.2. 66
exponent of convergence, 10.3. 112
- - divergence, 10.3. 112
extended complex plane, 7.1. 96
- real number system, 1.2. 7
extension, 6.3. 87: for a map 1 defined on a subset S of a set X, a map F
defined on X and such that Fls= 1.
extreme point, 2.3. 25

F
FATOU, P. lemma, 4.1. 41, s 3.1. 203, s 5.1. 249
FEJER, L. kernel, 5.1. 54
- theorem, 5.1. 267: If 1 E c('lr, C) and then Fn * 1 ~ 1; if 1 E Ll('lr, C)
then limN--+oo IlFn * 1 - 1111 = o.
FERMAT, P. DE conjecture, s 10.2. 401
filter, 2.1. 14
- base, 2.1. 14
- generated (by), 2.1. 14
finer (filter), 2.1. 14
finite Borel partition, s 4.1. 233
- cylinder, s 6.3. 359
- intersection property, s 2.1. 154, s 4.2. 247: for a set S of sets, if
n FES F = 0 then for a finite subset S' of S, nFES' F = 0.
- rectangle, 5.1. 49
- (X, S, It), 1.1. 5
finitely additive, 4.1. 42: of a set function ¢, that

K
¢(Uf=lEk) = L¢(Ek),K E N.
k=l
Glossary /Index 471

finitely determined, 2.1. 13


fixed point, 7.1. 98, s 3.1. 200
FOURIER, J. coefficient, 5.1. 54
- series, 5.1. 54, s 10.2. 401
- -STIELTJES, T. J. series, 5.1. 55
- transform, 5.1. 54, 6.1. 72, s 6.1. 317: for a locally compact abelian group
G and (G,S,f,L), the map Ll(G,f,L):3 f 1---+ fGf(x)(O'.,x)df,L(x) ~f 1(0'.).
fractional part, 4.1. 41
frequently, 2.1. 14
FUBINI, G. theorem, s 3.1. 207: If {fn}nEN is a sequence of monotonely
increasing functions on [0,1] and E:'=l fn ~f S is finite then on (0,1),
• t
s I eXlS s a.e. and s I ='",,00 I'
L..m=l n'
- -TONELLI, L. theorem, 5.1. 49, s 5.1. 268: (Fubini) If (X,S,f,L) and
(Y, T, v) are measure situations,

f: X x Y 1---+ 1R+

is integrable with respect to f,L x v then for almost every x resp. y the
restricted map

(When x is held constant) fCx) : Y 1---+ IR

resp.
(when y is held constant) fCy) : X 1---+ IR
is measurable and

IxXyf(x,Y)d(f,L x v) = Ix ([fcx)(Y)dV) df,L

= [ (Ix f CY ) (x) df,L) dv.

(Tonelli) If f is nonnegative and measurable, if (X, 5, f,L) and (Y, T, v)


are a-finite, and if either of the iterated integrals

is finite then f is integrable (whence, by Fubini's theorem, the two


iterated integrals are equal and their common value is

r
lxxy
f(x, y) d(f,L x v).

function lattice, 1.2. 7,8


- element, 11.1. 115
472 Glossary/Index

Fundamental Theorem of Algebra, s 9.3. 394: If p E C[z] then for some a


in C, p(a) = O.
- - - Calculus, s 3.1. 203, s 3.1. 215, s 5.1. 289: If J E LI(JR,).) and
F(x) ~f J~oo J(t) dt then F' exists a.e. and F' :;;, J.

G
Gamma function, 10.3. 114, 11.2. 119
Gauilian plane, 7.1. 95: the representation of Cas JR2 .
GELFAND, I. M.-FoURIER, J. transform, 6.1. 72
GELLES, G., s 5.1. 289
gen~ralized nilpotent, 6.4. 90
generated group, 4.2. 46: for a subset S of a group G, the intersection of
the set of all subgroups containing S.
gradient, 6.2. 79
GRAM, J. P.-SCHMIDT, E. biorthogonalization process, s 6.3. 348: for a
Banach space E and its dual E*, the homolog, based on the Hahn-
Banach theorem, of the algorithpJ. used to produce an orthonormal set
in a Hilbert space 5), q.v.
- orthonormalization process, s 3.2. 223, s 6.2. 336: for a linearly indepen-
dent sequence {xn}nEN contained in a Hilbert space 5), the algorithm:

def Xl
YI = IIxIl1
Yn ~f Xn - L~:: (Xn' Yk) Xk ,n E N \ {I}.
IIXn - L~:: (Xn,yk)Xkll
graph, 2.1. 19, 5.1. 54, 6.2. 82
greatest integer in, 4.1. 41
GREEN, G. theorem, s 6.2.332
GUTZMER, A. coefficient estimate, s 7.3. 378, s 10.3. 405: If

E Cnz
00

J(z) ~f n E H (D(O, Rt)


n=O

HAAR, A. measurable, measure, sets, 4.2. 44, 45


HADAMARD, J. determinant estimate, 3.2. 39
- factorization theorem, 10.3. 114
- gap theorem, 11.1. 116
Glossary/Index 473

Hadamard three-circlesfthree-lines theorems, 15.2. 132, 134


HAHN, H. decomposition, s 5.1. 249: for a signed (X, S, /-L), a partition
{A+, A-} such that for every E in S, A± nEE S and /-L (E n A+) 2: 0
resp. /-L (E n A-) ~ O.
- -BANACH, S. theorem, 15.1. 132, et alibi: If M is a subspace of a locally
convex vector space X and x tf. M then for some x* in M.l.., (x, x*) = 1.
half-line, 11.2. 118: for some a, b in e, the set

{z : z = a + t(b - a), 0 ~ t < oo}.

- -open cube, interval 1.1. 3


- -space, 10.2. 111
HAMEL, G. basis, 6.3. 84, s 4.2. 244: for a vector space V over a field lK, a
maximal linearly independent subset.
HARDY, G. H. class, 14.2. 127
harmonic conjugate, 13.1. 124
- function, 13.1. 124
HARNACK, A. inequality, 13.2. 125
- theorem, 13.2. 126
HAUSDORFF, F. maximality principle, 2.1. 14: In a poset, every chain, i.e.,
linearly ordered subset, is contained in a maximal chain.
- space resp. topology, 2.1. 15, 2.2. 20, 21: a topological space X resp.
topology such that if x =I- y then for some N(x), N(y), N(x)nN(y) = 0.
- -YOUNG, G. C. and YOUNG, W. H. theorem, 15.2. 135
HEINE, E.-Borel, E. theorem, s 3.1. 194, s 14.3. 442: If, for every infinite
subset S of a metric space X, S· =I- 0 then X is compact.
HELLY, E. (selection) theorem, s 5.1. 251: If {¢n}nEN C BV([O, 1],~),
sUPnENvar(¢n) < 00, and sUPnENI¢n(O)1 < 00 then for some subse-
quence {¢nkhEN' and all x in [0,1], ¢(x) ~ limk-+<Xl¢nk exists and
f E BV([O, 1], ~).
Hessian, s 3.1. 209
HILBERT, D. space, 5.3. 69, 6.2. 79
hit-index, s 2.1. 157
HOLDER, 0., 15.1. 132, et alibi.
holomorphic, 6.4. 90, 7.1. 95: of a function f in Co, that f'(z) exists for z
in n.
homotopic to a constant, 7.1. 95: of a map f : X 1---+ Y, that for some
t-continuous map F : X x [0,1] :3 (x, t) 1---+ F(x, t) E Y, F(x,O) = f(x)
and F(x, 1) == Yo E Y.

ideal, 2.3. 24, 6.4. 89: in a ring R, a proper subset I such that RI U IRe I.
idempotent, 6.3. 85: in a ring R, an element x such that x 2 = X.
474 Glossary jlndex

identity modulo (an ideal), 6.4. 89: for an ideal I in a ring R, an element u
such that for x in R, ux - X,xu - x E I.
- theorem (for holomorphic functions), s 8.1. 382, s 9.3. 396, s 11.2. 422:
If I,g E H(n) and for some subset S of n, S· n n #- 0 and 118= gl8
then I = g.
immediate analytic continuation, 11.1. 115
improperly Riemann integrable, 5.1. 54: of a function I in 1R1R, that I is Rie-
mann integrable on each interval [a, b] and that lima-+-oo
b-+oo
I(x) dxJ:
exists.
inclusion map, 3.2. 38: for a subset X of a set Y, the map X '3 x 1--+ X E Y.
independent (events), 5.2. 66
index (of a curve), s 9.2. 392
indivisible, s 1.1. 140
injective, 2.1. 20, 8.2. 104: of a map I : X 1--+ Y, that I is one-one.
inner measure, 1.2. 146, s 4.2. 243: for (X, S, J.t), the set function

J.t* : 2x '3 E 1--+ sup {J.t(B) : BE S,B c E}.

inner product, 6.2. 79


inner regular, 4.1. 40
integers, 1.1. 3
interior, 1.1. 3, 2.1. 12
intermediate value property, 3.1. 31
interval, 1.1. 3
- function, 5.1. 64, s 6.3. 362
inverse function theorem, s 7.3. 377: If I E H(n), a E n, and I'(a) #- 0,
then on some N(a), I is injective and 1- 1 E H(f(N(a))).
involution, 6.3. 84 - 85
irrational numbers, 1.1. 3
isolated essential singularity, 7.3. 99: for a function in H (A(a, r : R)O) the
point a that is neither a removable singularity nor a pole.
- point, 2.1. 14
isometric circle, 7.1. 98
isometry, 2.1. 19, 6.3. 84

Jacobian, s 6.1. 322, s 8.1. 386: for a self-map I ~f (/1, ... , In) of IRn, the
. J def
matrIx =
(aax.
Ii ) n .. .
3 ',3=1
JENSEN, J. L. W. V. inequality (for convex functions), 4.1. 41
- - (for holomorphic functions), 13.2. 125
Glossary/Index 475

K
KAKUTANI, S., s 2.3. 191
K-cylinder, s 6.3. 359
k-ary marker, 5.1. 50
k-ary representations, 5.1. 49
kernel, 5.1. 54
KOLMOGOROV, A. N. theorem (for topological vector spaces), 6.4. 89
KREIN, M.-MILMAN, D. theorem, s 6.1. 329: If K is a compact subset
of a topological vector space, then K is the closed convex hull of its
extreme points.
KRONECKER, L. function, s 6.1. 320
- lemma, 2.3. 24

LANDAU, E., 14.3. 129


LAPLACE, DE, P. S. map (Laplacian), 6.2. 79, 13.1. 124
lattice, 1.2. 7: a poset L such that if x, y E L then x 1\ y, x V Y E L.
LAURENT, P. A. expansion, 12.1. 121, s 11.2. 417
LEBESGUE, H. decomposition, 5.1. 48
- monotone convergence theorem, s 1.2. 147: For (X, 5,1"), if {fn}nEN is
a monotonely increasing sequence of nonnegative functions in M then
limn -+ oo f n ~f f E M and, by abuse of language,

Ix fn{x) dJL{x) i Ix f{x) dJL{x).

- -RADON, J.-NIKODYM, O. theorem, 5.1. 48


- set, s 14.2. 438: for f in L1 (lR, A),

[A{lR\LJ) =0).
left-continuous, 2.3. 26
- -invariant (Haar) measure, 4.2. 44
LEIBNIZ, VON, G. W. rule, s 6.4. 367: For a derivation D,

Dn{xy) = L Dk(x)Dl{y).
k+l=n

length (Euclidean), 3.1. 30


476 Glossary jlndex

length of a curve, 3.1. 36, 4.1. 41, s 3.1. 212: for a metric space X and a
curve'Y: [0,1]1-+ X,

lexicographic partial order, 7.1. 95


limit point, 2.1. 14
LINDELOF, E. space, 6.2. 345: a topological space for which every open
cover contains a countable sub cover .
linear programming, 15.1. 132: the process of minimizing a linear form
defined on the intersection of finitely many half- spaces.
- combination, 5.3. 67: for a finite subset {x!, ... ,xn } of a vector space
V over a field lK, L:~=l akxk, ak E lK.
LIOUVILLE, J. theorem, s 10.1. 398: A bounded entire function is a con-
stant.
LIPSCHITZ, R. constant, 5.1. 64, s 5.1. 291: for an I in RIR and a positive
a, a K (if one exists) such that I/(x) - l(y)1 :::; Klx _ YI<>·
living (on a set S), s 1.1. 141: of a measure JL in (X,S,JL), that S E 5 and
JL(A) = 0 if A E 5 and An S = 0.
locally bounded, 6.1. 70
- compact, 1.1. 4: of a topological space X, that there is a neighborhood
base consisting of compact sets.
- convex, 6.1. 70, s 6.3. 349
- injective, s 10.2. 402: of a map I : X 1-+ Y of a topological space X,
that for each x in X I is injective on some N(x).
log convex, 11.2. 119: of a function I in RIR, that lnl is convex.
lower semicontinuous, 2.1. 16
- triangular matrix, s 3.2. 223: a matrix (aij)~j:l such that aij = 0 if
i < j.

maximum modulus principle, 9.3. 109, s 6.4. 365, s 7.3. 379


- principle (in D relative to B), 15.1. 133
mean value property, 13.1. 124
- - theorem, s 2.2. 169: If I is continuous on [a, b] and differentiable on
(a, b) then for some c in (a, b), I(b) - I(a) = f'(c)(b - a).
measurable partition, 4.1. 40, 6.1. 72, s 6.1. 306
measure situation, 1.1. 4
- -isomorphic, 5.2. 66: of two measure situations (X, 5, JL) and (Y, T, v),
that for some bijection R : 51-+ T, JL(A) == v(R(A)),

R(A \ B) = R(A) \ R(B), and R (UnEN


An) = U
nEN
R (An) .
Glossary/Index 477

meromorphic, 11.2. 117, 12.1. 121, s 8.2. 387


mesh (of a partition), s 3.1. 214: the supremum of the measures of the
ingredient sets of the partition.
metric density theorem, s 5.1. 266, s 5.1. 279: For (JR, SA, A), if A E SA,

A(A n (x - E, X + E)) ~. {IOn A .


2E 0 off A

- space, 1.1. 4, 2.1. 15: a set X and a map (the metric)

d: X x X 3 (x,y) 1-+ d(x,y) E [0,(0)

such that: a) d(x, y) = 0 iff x = Yi b) d(y, z) ~ d(x, y) + d(x, z).


- topological group, 2.3. 29: a topological group with a metric-induced
topology.
metrizable, 2.1. 19: of a topological space, that its topology can be metric-
induced.
metrized, 2.1. 19: of a topological space, that it is endowed with a metric.
middle function, 1.2. 7
- number, 1.2. 7
midpoint convexity, s 3.1. 199
minimal base, 2.1. 18
- measurable cover, 5.1. 52
minimum modulus theorem, s 10.2. 400: If J E H(U) and J(z) ¥- 0 when
Izl < r < 1 then minlzl~r IJ(z)1 is achieved only on {z : Izl = r}
unless J is a constant.
MINKOWSKI, H. functional, 6.1. 70
MOBIUS, A. F. transformation, 7.1. 96
MITTAG-LEFFLER, G. theorem, 12.3. 123, s 10.2. 399, s 12.3. 429
modular function, 4.2. 46
- ideal, 6.4. 89
module, 6.1. 71: an abelian group over a ring
modulo 1 addition, 5.1. 53: addition in the quotient group JR/Z.
- a null set, s 5.1. 252: For measurable sets A, B, A = B modulo a null
set (A == B) iff (A \ B) U (B \ A) is a null set.
- column/row operations, s 3.1. 193, For two matrices A, B, A = B modulo
column/row operations iff B is the result of applying such operations
to A.
modulus of continuity, 3.1. 31
monodromy theorem, s 10.2. 402: If D(a,r)O en, n is simply connected,
and J E H (D( a, r)O), then the result of an analytic continuation of J
along any curve "( from a to some b in n is independent of the choice
of "(.
monotone class 1.1. 4
478 Glossary/Index

monotone class of functions, 1.2. 8: a class of functions closed with respect


to the formation of limits of monotone sequences.
monotone convergence theorem, 1.2. 10 (cf. Lebesgue monotone conver-
gence theorem).
monotonely increasing, 5.1. 63: of a function f in ]RIR, that if a < b then
f(a) S f(b).
MORERA, G. theorem, s 11.2. 417, 418: If f E C(D(O,r)O,C) and, for
every closed rectifiable curve 'Y : [0,1] ....... D(O, r)O, J-y f(z) dz = 0 then
f E H(D(O,r)O).
morphism, 2.3. 24: for a category C and objects A, B in C, an element of
[A,B].
multiplicity, 7.3. 99, s 10.1. 403, s 12.3. 426: of a zero resp. pole a of
an f in H (A (a, r : R)O), the number m Res-m in the Laurent series
L:=m cn(z - a)n representation of f.
mutually singular, 5.1. 48

natural boundary, 11.1 116


natural numbers, 1.1. 3
neighborhood, 2.1. 12
net, 2.1. 14
--corresponding to a filter, 2.1. 15
NEUMANN, VON, J., 5.3. 69, 15.1. 132
nonatomic, 4.1. 40, 5.1. 53: of a measure situation (X, S, JL), that for every
E in S, if JL(E) > 0 then E contains an F such that JL(E) > JL(F) > O.
nonnegative (linear functional), 1.2. 8
nonoverlapping, s 5.1. 269: of a pair (1, J) of (n-dimensional) intervals that
(1 n J)O = 0.
norm, 2.3. 24,6.1. 71: for a vector space V, a map (the norm)

II II : V 3 x . . . . Ilxll E [0,00)
such that a) 0 S IIxll, b) IIxll = 0 iff x = 0, and c) IIx+yll S IIxll+IIYII·
- -basis, 6.1. 71
normal (family of functions), 14.2. 127
- (subgroup), s 2.3. 189: that the subgroup, say N of a group G, is, for
all x in G, invariant under conjugation: N = x-I N x.
- (topology), s 6.2. 345: that disjoint closed sets are contained in disjoint
neighborhoods.
normed vector space, 2.3. 24, 6.1. 71, 6.2. 78, 6.3. 85: a vector space endowed
with a norm, q.v.
nowhere dense, s 2.1. 160, s 6.2. 345: of a subset S of a topological space
that (8) ° = 0.
null set, 1.2. 11, 4.2. 45
Glossary/Index 479

o
object, 2.3. 29
odd, 2.3. 25, s 6.2.336: of an 1 in ]RR, that I(-x) = -/(x).
open ball, 2.1. 16, 3.1. 34, s 2.3. 191
- cover, 2.2. 21: for a topological space X, a set {U>.hEA of open sets such
that U>'EA U>. = X.
- interval, 1.1. 3
- map, 2.1. 12
- mapping theorem (for topological vector spaces), s 6.3. 350, 363: For
a pair (E, F) of Banach spaces, if T E [E, F]e and U E O(E) then
T(U) E O(F)j (for holomorphic functions), 9.3. 109, s 9.1. 390, s 13.1.
430: If 8 is an open subset of a region 0 in C and 1 E H(O) then 1(8)
is open.
- set, 2.1. 12
operator norm, 2.3. 24 - 25, 6.2. 82: for a pair (E, F) of normed vector
spaces the map II II : [E, F] 3 T 1-+ sup {IIT(x)11 : IIxll ~ 1} ~f IITII.
order (of a group), s 2.3. 189: for the group G, #(G).
- (of growth), 10.3. 112
orthogonal(ity), s 8.1. 384: of a subset {cPAhEA of a Hilbert space 5), that

{'\ =J JL} ::} {(cP>., cP~) = O}.


orthogonal projection, 6.2. 81, s 6.2. 345: for a Hilbert space 5), in [5)], a
self-adjoint element P such that p 2 = P.
orthonormal, 5.1. 51, 6.2. 81: of a subset {cP>.hEA of a Hilbert space 5),
that (cP>., cP,..,) = D>.w
- basis, s 5.1. 279: for a Hilbert space 5) an orthonormal system that is a
Schauder basis.
OSTROWSKI, A. theorem, 11.1. 116
outer measure, 1.2. 11, 4.1. 40,41, s 4.2. 243: a nonnegative and count ably
subadditive set function JL* such that

{A C B} ::} {JL*(A) ~ JL*(B)} and JL*(0) = o.


- regular (measure), 4.1. 40

paracompact, s 6.2. 345: of a topological space X, that for every open


cover {U>.hEA' there is an open cover {V~} ~EM such that: a) each V,..,
is contained in some U>.j b) some neighborhood of each point x meets
only finitely many Vw
480 Glossary/Index

PARSEVAL(-DESCHENES), M. A. equation, s 3.2.222, s 6.2.335: If {xAhEA


is a complete orthonormal system in a Hilbert space .fj and y E .fj then
lIyl12 = EAEA l(y,xA)1 2.
partially ordered set, 2.1. 13
partition, 3.1. 35: for an interval [xo, Xn) a set

for a set X, a finite or countably infinite set {Enh<n<P<oo such that


X = l:.h~n<pEn' - -
- of unity, 6.1. 72
p-dimensional Hausdorff measure, 4.1. 41
perfect, 2.1. 14, 20, 5.1. 56: of a subset S of a topological space, that it is
closed and self-dense.
period, 6.3. 85, s 12.3. 428: of an operation T, that for some n in N, Tn = id.
periodic, s 12.3. 428: for a group G, an x in G, and a set S, of an I in SG,
that for all y in G, I(xy) = I(y).
PICARD, E. (great) theorem, 14.3 130, s 9. 394, s 10.2. 400, et alibi:
If I E H (A(a, r : R)O) and a is an essential singularity of I then
# (C \ (f (A (a, r : R)O))) :::; 1.
PLANCHEREL, M. theorem, s 5.1. 287, s 6.1. 320: If G is a locally com-
pact abelian group then the Gelfand-Fourier transform confined to
LI( G, f./,) n L2( G, f./,) is II 112- preserving.
POINCARE, H. theorem, 11.2. 119
point of condensation, s 9.3. 392: for a subset S of a topological space, a
point e such that for every N(e), # (N(e) n S) > ~o.
pointwise ergodic theorem, 5.3. 69
POISSON, S. D. formula, s 13.1. 431, s 13.2. 433
- JENSEN, J. L. W. V. formula, 13.2. 126
- kernel, 8.1. 103
pole, 7.3. 99, s 8.2. 387
polynomial, 2.3. 26, 3.1. 31, 5.1. 58
poset, 2.1. 13
positive definite, 6.2. 79, s 6.3. 361: of a conjugate bilinear form B(x, y),
that B(x, x) 2:: 0 and B(x, x) = 0 iff x = O.
- homogeneous, s 1.2. 149
- measure, 1.1. 4
principal ideal, 2.3. 24, 28, 14.3. 129
- part, 12.1. 121, s 10.2. 399
principle of the argument, s 9.2. 391: If I E M(n) and, for the closed curve
"I : [0,1] ...... n \ (Z(f) U P(f», Indl'(e) = 0 for all e in C \ n then for
the closed curve r : [0,1) 3 t ...... 1 0 "I(t),

Indr(O) = I: Indl'(a) - I: Indl' (b).


aEZ(f) bEP(f)
Glossary/Index 481

principles of the maximum and minimum, 13.1. 124 - 125,


product measure, 5.1. 48
- topology, 2.1. 13, 18
projection (in a Cartesian product), 5.1. 49
- (in a Hilbert space), 6.2. 81, s 6.2. 345

quasinorm, 6.1. 70
quaternion, 1.1. 3
quotient group, s 2.3. 189: for a group G and a normal subgroup N, the
coset space G / N regarded as a group.
- norm, 6.3. 84
- obj~ct, 2.3. 29: In a category C, two epimorphisms A 1---+ B i , i = 1,2 are
equivalent iff there are morphisms Bl 1---+ B2 and B2 1---+ Bl such that
A 1---+ Bl 1---+ B2 = A 1---+ B2 and A 1---+ B2 1---+ Bl = A 1---+ B 1 • A quotient
object is an equivalence class of epimorphisms.
- space, 4.2. 45: a quotient object in a category of algebraic objects and
homomorphisms.
- topology, 6.3. 83

RADEMACHER, H. functions, 5.3. 68


radial, 6.1. 70
- limit function, 8.1. 102
radical, 6.4. 90, s 6.4. 368
radius, 2.1. 16
- of convergence, 7.3. 99: for a series 2:::'0 enzn, 1 1 .

limn --+ oo len In


RADON, J.-NIKODYM, O. derivative, 5.1. 48, s 4.1. 230
random variable, 5.2. 66
range, 2.1. 17: of a map f: X 1---+ Y, f(X).
real algebraic number, 1.1. 3
- number, 1.1. 3
- rational number, 1.1. 3
- irrational number, 1.1. 3
rectifiable, 2.1. 20: for a metric space (X, d), of a curve 'Y : [0,1] 3 t 1---+ X,
that

refinement, 2.1. 14, 21


482 Glossary/Index

refines, 2.1. 14
reflection, 7.1. 97
reflexive Banach space, 6.3. 84
- relation: a relation, q.v., R such that for all x, (x, x) E R.
region, 6.4. 90, 7.1. 95: a nonempty connected open subset of a topological
space.
regular function, 11.2. 120
- ideal, 6.4. 89
- map, 1.1. 6
- measure, 4.1. 40
- point, 11.1. 115
- space, s 6.2. 345: a topological space in which: a) points are closed; b)
for each pair consisting of a point x and a closed set F disjoint from
{x}, some N(x) and some N(F) are disjoint.
relation, 2.1. 14: for a set 8, a subset R of 8 2 •
removable singularity, 9.3. 107, 12.3. 121, 122, s 12.2. 425: for an f in
H (A (a, 0: R)O), the point a if sup {If(z)1 : z E A(a, 0 : R)O } < 00.
residue, 12.1. 121
- theorem, s 11.2. 416: If f E H (A (a, 0: R)O) and 0 < r < R then
Res(f, a) = -2 1.r
1U J1z1=r
f(z) dz.
RIEMANN, B.-LEBESGUE, H. lemma, s 6.1. 317: If G is a locally compact
abelian group and f E Ll(G,J.L) then E 0 0 1 (G, c).
--STIELTJES, T. J. integral, 7.1. 95: for (g,1) in

BV([O, 1], R) x 0([0,1], R),

and the (signed) measure engendered by the interval function

v: [a, b) 1--4 9 (b - 0) - g(a),

J;f(x) dv(x).
- sums, s 11.2. 420: the approximating sums used in the definition of the
Riemann integral.
RIEsz, F. lemmata, 3.1. 32
- -HAUSDORFF, F.-YOUNG, G. C.-YOUNG, W. H. theorem, 15.2. 135
- representation theorem, s 2.3. 183, s 5.1. 262, 264, s 6.4. 366: If X is a
locally compact Hausdorff space and F E (00 (X, C))* then for some
outer regular Borel measure 1", F(f) = Jx f(x) dJ.L(x).
RIEsz, M. convexity theorem, 15.2. 134, 135
right-continuous, 3.1. 35, 10.3. 112: of an f in RIR, that

f(x) = limf(y) d~f f(x +0).


y.J.x
Glossary/Index 483

ring (algebraic), 2.3. 24, 3.1. 31: a set endowed with two binary associative
operations + and '; + is commutative and· is +-distributive.
- (of sets), 1.1. 4, 5.1. 50: for a set X, in 2x , a subset closed with respect
to the formation of set differences and finite unions.
rotation-invariant, 5.1. 60: of an 1 on lRn resp. 2Rn , that for each rotation
T of lRn , I(T(x)) = I(x) resp. I(T(A)) = I(A).
ROUCHl~, E. theorem, s 7.2. 376, s 8.1. 384: If

I,g E H (D(O, 1t), r E (0,1), and I/(z)llco(r» Ig(z)IICo(r)

then # (Z(f) n D(O, r)O) = # (Z(f + g) n D(O, r)O).


row operations, s 3.1. 193: for a matrix (aij)7,'j~l: a) row interchange; b)
multiplication of a row by a scalar; c) addition of two rows.
RUNGE, C. theorem, 12.3. 123
running water lemma, s 3.1. 192

s
SCHAUDER, J. basis, 6.1. 71,6.3. 86
SCHOTTKY, F. theorem, 14.4. 130
SCHWARZ, H. A. formula, 13.2. 125
- lemma, s 8.1. 382, 386, 389: If

1 E H(U), 1(0) = 0, and I(U) c U

then for z in A(O, 0: 1)°,

either I/(z)1 < Izl or I(z) == ei8 z,f) E [0, 21r).

- reflection principle, s 8.1. 382: If 0 c U, 1 E H(O), for some arc,

A dcl{
= e'~ : ° ~ a < f) < b < 21r } C 8(0),

1 E C(O U A, q, 1(0) c D(a, r), and B ~ I(A) c 8(f(0)) is an arc


on 8(D(a, r), then for all z in 0, f* (zP) ~f (f(z))P defines an analytic
continuation of 1 across A.
second category, 5.1. 61: of a subset S of a topological space, that S is not
the countable union of nowhere dense sets.
section, 5.1. 49, s 4.2. 243
self-dense, 2.1. 14
- kernel, 2.1. 14, s 2.3. 186
self-map, s 8.2.389: a map 1 : X ~ X.
semicontinuous, 2.3. 28: of an 1 in lRx , that 1 is both upper semicontinuous
and lower semicontinuous.
484 Glossary/Index

semigroup, 4.2. 47: a set 8 and a map 8 2 :3 (x, y) 1--+ xy E 8 such that
(xy)z = x(yz).
- with a cancellation law, 4.2. 47
seminorm, 6.1. 70
separable, 2.1. 19, 20, 4.1. 42: of a topological space, that there is a count-
able base for its topology.
separated, 2.1. 14
separating, 2.3. 178: of a function algebra A, that if x =j:. y then for some f
in A, f(x) = 0 = 1 - fey).
set function, 4.1. 40: for a set X, a function defined on 2x.
a-ring, 1.1. 4: a ring of sets closed under the formation of countable unions.
a-compact, 4.2. 46: of a topological space X, that it is the union of count-
ably many compact sets.
- -finite, 1.1. 5
signed measure, 1.1 4
simple arc, s 9.2. 390: in a topological space X, an injective continuous
"f: [0,1)1--+ X.
- closed curve, 2.1. 12
- function, 4.1. 41, s 5.1. 290: a linear combination of characteristic func-
tions.
- pole, 12.1. 121: for a function f, an a such that the Laurent series for f
is E~=-l cn(z - a)n.
simply connected, 7.1. 95, 9.1. 106, s 10.2. 399
singular part, 12.1. 121
- point, 11.1. 116
singularity, 7.3. 99, 12.1. 121
span (noun), 5.3. 67: for a set 8 in a vector space, the set of all linear
combinations of vectors in 8.
- (verb), 5.3. 67
spherically normal, 14.4. 127: of a set F of functions holomorphic in some
region n, that if {fn}nEN C F then some subsequence {fnkhEN' re-
garded as a sequence of maps of n into Coo, converges at each point of
n.
step-function, 10.3. 113, s 4.1. 237: a linear combination of characteristic
functions of intervals.
stereographic projection, 7.1. 96
STIELTJES, T. J. measure, s 5.1. 263: for a gin BV(IR,IR), the (signed)
measure generated by the interval function

1/ : 21R :3 [a, b) 1--+ g(b - 0) - g(a).

STIRLING, J. formula, s 10.3. 407: and some (): (0,00):3 x 1--+ (0,1),

hm
. 1
rex) 1~=1.
X--->OO X X -"2 e-X (271")"2 e 12x
Glossary/Index 485

STONE, M. H., 1.2. 7


- -WEIERSTRASS, K. theorem, 2.3. 176: For a compact Hausdorff space
X, C(X, JR) is the II lloo-closure of each separating subalgebra.
strict local maximum, 3.1. 36: for a topological space X and an f in JRx , a
b ~ f(a) such that for all x in some N(a) \ {a}, f(x) < b.
stronger, -est (topology), 2.1. 12
subadditive, 1.2. 10, s 4.1. 225: of a functional F, that

F(J + g) ::; F(f) + F(g).


subcover, 2.2. 21: a cover that is a subset of a cover.
subharmonic, 15.1. 133
subordinate, 6.1. 72
subspace, 5.3. 67: for a vector space, a subset that is also a vector space.
subsum, 2.2. 22
superadditive, s 4.1. 225
support (of a function), 1.1. 4: for a topological space X and an f in eX,
the set
{x : f(x) i= o}.
- (of a measure), 5.1. 48
supporting line, 3.1. 30, s 3.1. 196
surjection, 2.1. 18, s 2.1. 156: a surjective map.
surjective, 6.2. 82, s 2.1. 156: a map f : X ~ Y such that f(X) = Y.
SUSLIN, M. [SOUSLIN, M.], 1.2. 7
symmetric difference, 5.1. 48
- relation: a relation, q.v., R such that {(x, y) E R} :::::} {(y, x) E R}.
SZPILRAJN, E., s 1.2. 146

T
ternary. 5.1. 50
the Cantor function, 2.1. 18
the Cantor set Co, 2.1. 16
the Cantor-like set CO" 2.1. 16
the k-ary representation, 5.1. 50
theory of games, 15.1. 132: the mathematical formulation and study of the
concept of a game.
thick, s 4.4. 243, for a (X, 5,1-"), of a subset S, that for each E in 5,

I-".(E \ S) = O.
THORIN, G. O. theorem, 15.1. 132, s 15.1. 447 - 448
TIETZE, H. extension theorem, s 3.1. 214: If X is a normal topological
space, F E F(X), and f E C(F, JR) then for some gin C(X, JR), glF= f.
486 Glossary/Index

TOEPLITZ, 0. matrix, 2.2. 23


topological field, 2.3. 24, 3.1. 30, 6.1. 70: a field topologized so that the
binary operations are jointly continuous and inversion is continuous.
- group, 2.1. 15,2.3. 29
- semigroup, 4.2. 47
- vector space, 3.1. 30, 6.1. 70
totally bounded, s 6.3. 353: of a metric space (X, d), that for each positive
r, X is the union of finitely many sets of diameter not exceeding r.
- disconnected, 3.1. 36: of a subset S of a topological space X, that S
contains no connected subsets save 0 and singleton sets {x}.
- finite, 1.1. 5
- O'-finite, 1.1. 5
transcendental, 10.2. 110
transfinite induction (principle), s 1.1. 140 - 141: H a statement S about
ordinal numbers a is true when a = 1, and if the truth of S for all
ordinal numbers a less than /3 implies the truth of S for /3, then S is
true.
transitive, 2.1. 13: of a relation, q.v., R, that

{{(x, y) E R}" {(y, z) E R}} '* {(x, z) E R}.


translate, 2.3. 26
translation-invariant, 2.3. 27, 6.2. 81, s 2.3. 183, s 4.1. 230: of an f defined
on a group G resp. 2G , that for each x in G,

f(xy) == f(y) resp. f(xA) == f(A).

triangle inequality, s 3.2. 223


trivial topology, 2.1. 12
type (of its order), 10.3. 112

u
ultrafilter, 2.1. 14
uniform boundedness principle, s 2.3.187, s 4.1. 236, s 6.1. 316: the Banach-
Steinhaus theorem.
uniformity situation, 2.1. 15
uniformly continuous, 2.2. 21
- convex, 6.1. 71
- integrable, 5.1. 49
unimodular, 4.2. 46
unitary, 5.3. 69, 6.2. 79
unit ball, 6.1. 77: in a normed vector space,

{x : IIxll ~ I} ~ B(O,I).
Glossary/Index 487

unit sphere, 3.1. 35


univalent, s 10.2. 400: for a region n in C, of an I in H(n), that I is
injective.
upper semicontinuous, 2.1. 16, 2.3. 26
URYSOHN, P. lemma, 4.1. 43, s 2.3. 188: If X is locally compact and
K(X) 3 K CUE O(X), then for some I in C(X, ~), A -< I -< U.

v
VANDERMONDE, A. T. s 2.2.172
vanishing at infinity, 1.1. 4: for a locally compact topological space X, of
an I in C(X,q, that if f > 0 then for some K(f) in K(X), II(x)1 < f
when x fj. K(f).
variance, 5.2. 66
variation, 3.1. 35: for an I in ~[xo,xnl and a partition P,

the sum:
N
L II (xn) - I (xn-dl ~f var[Xo,xn],plj
n=1

when P is the set of all partitions of [xo, Xn] then

VITALI, G. theorem, s 9.3. 392, 396, s 11.2. 417: If {fn}nEN C H(n),


limn -+ oo In(z) ~ I(z) exists for all z in an S such that 0 :F S· c n,
and Iln(z)1 ::; M < 00 in n, then I E H(n).

w
WALSH, J. L. functions, 5.3. 68
weak basis, 6.1. 71
weaker, -est, 2.1. 12
weakly*, s 6.1. 309
WEIERSTRASS, K. approximation theorem, s 2.2. 172, s 2.3. 176, 5.1. 258:
The set ~[xJI[O, 1] is II 1100 dense in C([O, 1], IR).
WeierstraB-CAsoRATI, F. theorem, s 10.2. 399: If a is an isolated essential
singularity of I then (f (N (a) \ {a} nc = C.
- factorization (product) theorem, 10.3. 112, s 10.1. 398
- M-test, s 7.3.377: If an > Iln(x)l,n E Nand 2::'=1 an < 00 then
2::'=1 In (x) converges uniformly.
weight, 2.1. 18
488 Glossary/Index

WElL, A., topology, s 4.2.247: For a group G associated with some (G, 5, v)
in which v is translation-invariant, and the functions

TE(X) ~f v(xEf:::,E),

the set {{x : TE(X) < f}}E>O,EES is a base of neighborhoods at the


identity of G and so topologized, G is a topological group. The Cauchy
completion of G is locally compact.
well-ordered, 1.1. 4: of a poset (8, -<) such that for each pair (x, y) in 8 2 ,
exactly one of x -< y, y -< x, and x = y obtains, that if A c 8 then for
some x in A and all y in A \ {x}, x -< y.
Well-ordering Axiom, 2.1. 14: Every set can be well-ordered.
WEYL, H. equidistribution theorem, 4.1. 43
WIENER, N.-TAUBER, A. theorem, s 6.2. 346: For a locally compact abelian
group G and an f in Ll(G, {t), the span of the translates of f is I 111-
1
dense in L 1 ( G, {t) iff never vanishes.
winding number, s 9.2. 391: for a rectifiable closed curve, : [0,1] 1-+ C
and a point a in C \ ,*, the index, Ind')' (a ); (Any closed curve can be
approximated by rectifiable closed curves and thus the winding number
is definable for an arbitrary closed curve.)
WIRTINGER, W. inequality, 3.2. 38

YOUNG, G. C.-YOUNG, W. H. theorem, 15.2. 135

z
ZERMELO, E.-FRAENKEL, A. A. axiom system for set theory, 4.2. 243
zero object, 2.3. 29: in a category C, an object N such that for every object
A in the category, #[A, N] = #[N, A] = 1.
zero-one law, 5.3. 67
Zeta function, 11.2. 118
ZORN, M. lemma, 2.1. 14: If every linearly ordered subset of a poset (r,-<)
has an upper bound then some, in r is maximal, i.e., , -< " obtains
for no element " different from ,.
Problem Books in Mathematics (continued)

Demography Through Problems


by Nathan Keyfitz and John A. Beekman

Theorems and Problems in Functional Analysis


by A.A. Kirillov and A.D. Gvishiani

Problem-Solving Through Problems


by Loren C. Larson

A Problem Seminar
by Donald J. Newman

Exercises in Number Theory


by D.P. Parent

You might also like